Sie sind auf Seite 1von 197

SEARCHES AND SEIZURES

SYLLABUS
1. CONSTITUTIONAL LAW; BILL OF RIGHTS; GUARANTEE AGAINST UNREASONABLE
SEARCHES AND SEIZURES; PRONOUNCEMENT OF UNITED STATES FEDERAL
SUPREME COURT AND STATE APPELLATE COURTS, DOCTRINAL IN THIS
JURISDICTION. — Our present constitutional provision on the guarantee against unreasonable
search and seizure had its origin in the 1935 Charter which was in turn derived almost verbatim
from the Fourth Amendment to the United States Constitution. As such, the Court may turn to
the pronouncements of the United States Federal Supreme Court and State Appellate Courts
which are considered doctrinal in this jurisdiction. 2. REMEDIAL LAW; EVIDENCE;
ADMISSIBILITY; EXCLUSIONARY RULE ON EVIDENCE OBTAINED IN VIOLATION OF THE
GUARANTEE AGAINST UNREASONABLE SEARCHES AND SEIZURES. — In a number of
cases, the Court strictly adhered to the exclusionary rule and has struck down the admissibility
of evidence obtained in violation of the constitutional safeguard against unreasonable searches
and seizures. (Bache & Co., (Phil.), Inc., v . Ruiz, 37 SCRA 823 [1971]; Lim v . Ponce de Leon,
66 SCRA 299 [1975]; People v . Burgos, 144 SCRA 1 [1986]; Roan v . Gonzales, 145 SCRA
687 [1987]; See also Salazar v . Hon. Achacoso, et al., GR No. 81510, March 14, 1990). 3.
CONSTITUTIONAL LAW; BILL OF RIGHTS; LIBERTIES MAY BE INVOKED ONLY AGAINST
THE STATE, NOT UPON PRIVATE INDIVIDUALS. — In the absence of governmental
interference, the liberties guaranteed by the Constitution cannot be invoked against the State.
As this Court held in Villanueva v . Querubin (48 SCRA 345 [1972]: 1. T his c o n s tit u tio n al
rig h t (against unreasonable search and seizure) r e f e r s t o t h e im m u nit y o f o n e ' s p e r
s o n, w h e t h e r citiz e n o r alie n, f r o m in t e r f e r e n c e b y g o v e r n m e n t, included in
which is his residence, his papers, and other possessions . . . That the Bill of Rights embodied
in the Constitution is not meant to be invoked against acts of private individuals finds support in
the deliberations of the Constitutional Commission. The constitutional proscription against
unlawful searches and seizures therefore applies as a restraint directed only against the
government and its agencies tasked with the enforcement of the law. Thus, it could only be
invoked against the State to whom the restraint against arbitrary and unreasonable exercise of
power is imposed. 4. ID.; ID.; ID.; GUARANTEE AGAINST UNREASONABLE SEARCH AND
SEIZURE; CASE AT BAR. — The contraband in the case at bar having come into possession of
the Government without the latter transgressing appellant's rights against unreasonable search
and seizure, the Court sees no cogent reason why the same should not be admitted against him
in the prosecution of the offense charged. If the search is made upon the
CD Technologies Asia, Inc. © 2016 cdasiaonline.com
request of law enforcers, a warrant must generally be first secured if it is to pass the test of
constitutionality. However, if the search is made at the behest or initiative of the proprietor of a
private establishment for its own and private purposes, as in the case at bar, and without the
intervention of police authorities, the right against unreasonable search and seizure cannot be
invoked for only the act of private individual, not the law enforcers, is involved. In sum, the
protection against unreasonable searches and seizures cannot be extended to acts committed
by private individuals so as to bring it within the ambit of alleged unlawful intrusion by the
government. 5. ID.; ID.; ID.; ID.; MERE PRESENCE OF NBI AGENTS TO OBSERVE AND
LOOK AT WHICH IS IN PLAIN SIGHT IS NOT A SEARCH. — The mere presence of the NBI
agents did not convert the reasonable search effected by Reyes into a warrantless search and
seizure proscribed by the Constitution. Merely to observe and look at that which is in plain sight
is not a search. Having observed that which is open, where no trespass has been committed in
aid thereof, is not search (Chadwick v . State, 429 SW2d 135). Where the contraband articles
are identified without a trespass on the part of the arresting officer, there is not the search that is
prohibited by the constitution (US v . Lee 274 US 559, 71 L.Ed. 1202 [1927]; Ker v . State of
California 374 US 23, 10 L. Ed. 2d. 726 [1963]; Moore v . State, 429 SW2d 122 [1968]). 6. ID.;
CONSTITUTION; DOES NOT GOVERN RELATIONSHIP BETWEEN INDIVIDUALS. — The
constitution, in laying down the principles of the government and fundamental liberties of the
people, does not govern relationships between individuals. 7. REMEDIAL LAW; EVIDENCE;
ADMISSIBILITY; EVIDENCE PROCURED BY INDIVIDUALS EFFECTED THROUGH PRIVATE
SEIZURE, ADMISSIBLE. — Similarly, the admissibility of the evidence procured by an
individual effected through private seizure equally applies, in p a ri p a s s u , to the alleged
violation, non-governmental as it is, of appellant's constitutional rights to privacy and
communication. 8. ID.; ID.; CREDIBILITY; SELF-SERVING DENIALS, DESERVE NO WEIGHT
IN LAW. — Denials, if unsubstantiated by clear and convincing evidence, are negative self-
serving evidence which deserve no weight in law and cannot be given greater evidentiary
weight than the testimony of credible witnesses who testify on affirmative matters (People v.
Esquillo, 171 SCRA 571 [1989]; People v s . Sariol, 174 SCRA 237 [1989]). 9. ID.; ID.; ID.;
REQUISITE FOR EVIDENCE TO BE BELIEVED. — Evidence, to be believed, must not only
proceed from the mouth of a credible witness, but it must be credible in itself such as the
common experience and observation of mankind can approve as probable under the
circumstances. 10. ID.; ID.; BURDEN OF PROOF AND PRESUMPTIONS; THINGS WHICH A
PERSON POSSESSES ARE PRESUMED OWNED BY HIM; CASE AT BAR. — As records
further show, appellant did not even bother to ask Michael's full name, his complete address or
passport number. Furthermore, if indeed, the German national was the owner of the
merchandise, appellant should have so indicated in the contract of shipment. On the contrary,
appellant signed the contract as the owner and shipper thereof giving more weight to the
presumption that things which a person possesses, or exercises acts of ownership over, are
owned by him (Sec. 5 [j], Rule 131). At this point, appellant is therefore estopped to claim
otherwise.

G.R. No. 81561 January 18, 1991

PEOPLE OF THE PHILIPPINES, plaintiff-appellee


vs.
ANDRE MARTI, accused-appellant.

The Solicitor General for plaintiff-appellee.


Reynaldo B. Tatoy and Abelardo E. Rogacion for accused-appellant.

BIDIN, J.:
This is an appeal from a decision * rendered by the Special Criminal Court of Manila (Regional
Trial Court, Branch XLIX) convicting accused-appellant of violation of Section 21 (b), Article IV
in relation to Section 4, Article 11 and Section 2 (e) (i), Article 1 of Republic Act 6425, as
amended, otherwise known as the Dangerous Drugs Act.

The facts as summarized in the brief of the prosecution are as follows:

On August 14, 1987, between 10:00 and 11:00 a.m., the appellant and his common-law
wife, Shirley Reyes, went to the booth of the "Manila Packing and Export Forwarders" in
the Pistang Pilipino Complex, Ermita, Manila, carrying with them four (4) gift wrapped
packages. Anita Reyes (the proprietress and no relation to Shirley Reyes) attended to
them. The appellant informed Anita Reyes that he was sending the packages to a friend
in Zurich, Switzerland. Appellant filled up the contract necessary for the transaction,
writing therein his name, passport number, the date of shipment and the name and
address of the consignee, namely, "WALTER FIERZ, Mattacketr II, 8052 Zurich,
Switzerland" (Decision, p. 6)

Anita Reyes then asked the appellant if she could examine and inspect the packages.
Appellant, however, refused, assuring her that the packages simply contained books,
cigars, and gloves and were gifts to his friend in Zurich. In view of appellant's
representation, Anita Reyes no longer insisted on inspecting the packages. The four (4)
packages were then placed inside a brown corrugated box one by two feet in size (1' x
2'). Styro-foam was placed at the bottom and on top of the packages before the box was
sealed with masking tape, thus making the box ready for shipment (Decision, p. 8).

Before delivery of appellant's box to the Bureau of Customs and/or Bureau of Posts, Mr.
Job Reyes (proprietor) and husband of Anita (Reyes), following standard operating
procedure, opened the boxes for final inspection. When he opened appellant's box, a
peculiar odor emitted therefrom. His curiousity aroused, he squeezed one of the bundles
allegedly containing gloves and felt dried leaves inside. Opening one of the bundles, he
pulled out a cellophane wrapper protruding from the opening of one of the gloves. He
made an opening on one of the cellophane wrappers and took several grams of the
contents thereof (tsn, pp. 29-30, October 6, 1987; Emphasis supplied).

Job Reyes forthwith prepared a letter reporting the shipment to the NBI and requesting a
laboratory examination of the samples he extracted from the cellophane wrapper (tsn,
pp. 5-6, October 6, 1987).

He brought the letter and a sample of appellant's shipment to the Narcotics Section of
the National Bureau of Investigation (NBI), at about 1:30 o'clock in the afternoon of that
date, i.e., August 14, 1987. He was interviewed by the Chief of Narcotics Section. Job
Reyes informed the NBI that the rest of the shipment was still in his office. Therefore,
Job Reyes and three (3) NBI agents, and a photographer, went to the Reyes' office at
Ermita, Manila (tsn, p. 30, October 6, 1987).

Job Reyes brought out the box in which appellant's packages were placed and, in the
presence of the NBI agents, opened the top flaps, removed the styro-foam and took out
the cellophane wrappers from inside the gloves. Dried marijuana leaves were found to
have been contained inside the cellophane wrappers (tsn, p. 38, October 6, 1987;
Emphasis supplied).
The package which allegedly contained books was likewise opened by Job Reyes. He
discovered that the package contained bricks or cake-like dried marijuana leaves. The
package which allegedly contained tabacalera cigars was also opened. It turned out that
dried marijuana leaves were neatly stocked underneath the cigars (tsn, p. 39, October 6,
1987).

The NBI agents made an inventory and took charge of the box and of the contents
thereof, after signing a "Receipt" acknowledging custody of the said effects (tsn, pp. 2-3,
October 7, 1987).

Thereupon, the NBI agents tried to locate appellant but to no avail. Appellant's stated address in
his passport being the Manila Central Post Office, the agents requested assistance from the
latter's Chief Security. On August 27, 1987, appellant, while claiming his mail at the Central Post
Office, was invited by the NBI to shed light on the attempted shipment of the seized dried
leaves. On the same day the Narcotics Section of the NBI submitted the dried leaves to the
Forensic Chemistry Section for laboratory examination. It turned out that the dried leaves were
marijuana flowering tops as certified by the forensic chemist. (Appellee's Brief, pp. 9-11, Rollo,
pp. 132-134).

Thereafter, an Information was filed against appellant for violation of RA 6425, otherwise known
as the Dangerous Drugs Act.

After trial, the court a quo rendered the assailed decision.

In this appeal, accused/appellant assigns the following errors, to wit:

THE LOWER COURT ERRED IN ADMITTING IN EVIDENCE THE ILLEGALLY


SEARCHED AND SEIZED OBJECTS CONTAINED IN THE FOUR PARCELS.

THE LOWER COURT ERRED IN CONVICTING APPELLANT DESPITE THE


UNDISPUTED FACT THAT HIS RIGHTS UNDER THE CONSTITUTION WHILE
UNDER CUSTODIAL PROCEEDINGS WERE NOT OBSERVED.

THE LOWER COURT ERRED IN NOT GIVING CREDENCE TO THE EXPLANATION


OF THE APPELLANT ON HOW THE FOUR PARCELS CAME INTO HIS POSSESSION
(Appellant's Brief, p. 1; Rollo, p. 55)

1. Appellant contends that the evidence subject of the imputed offense had been obtained in
violation of his constitutional rights against unreasonable search and seizure and privacy of
communication (Sec. 2 and 3, Art. III, Constitution) and therefore argues that the same should
be held inadmissible in evidence (Sec. 3 (2), Art. III).

Sections 2 and 3, Article III of the Constitution provide:

Sec. 2. The right of the people to be secure in their persons, houses, papers and effects
against unreasonable searches and seizures of whatever nature and for any purpose
shall be inviolable, and no search warrant or warrant of arrest shall issue except upon
probable cause to be determined personally by the judge after examination under oath
or affirmation of the complainant and the witnesses he may produce, and particularly
describing the place to be searched and the persons or things to be seized.
Sec. 3. (1) The privacy of communication and correspondence shall be inviolable except
upon lawful order of the court, or when public safety or order requires otherwise as
prescribed by law.

(2) Any evidence obtained in violation of this or the preceding section shall be
inadmissible for any purpose in any proceeding.

Our present constitutional provision on the guarantee against unreasonable search and seizure
had its origin in the 1935 Charter which, worded as follows:

The right of the people to be secure in their persons, houses, papers and effects against
unreasonable searches and seizures shall not be violated, and no warrants shall issue
but upon probable cause, to be determined by the judge after examination under oath or
affirmation of the complainant and the witnesses he may produce, and particularly
describing the place to be searched, and the persons or things to be seized. (Sec. 1 [3],
Article III)

was in turn derived almost verbatim from the Fourth Amendment ** to the United States
Constitution. As such, the Court may turn to the pronouncements of the United States Federal
Supreme Court and State Appellate Courts which are considered doctrinal in this jurisdiction.

Thus, following the exclusionary rule laid down in Mapp v. Ohio by the US Federal Supreme
Court (367 US 643, 81 S.Ct. 1684, 6 L.Ed. 1081 [1961]), this Court, in Stonehill v. Diokno (20
SCRA 383 [1967]), declared as inadmissible any evidence obtained by virtue of a defective
search and seizure warrant, abandoning in the process the ruling earlier adopted in Moncado v.
People's Court (80 Phil. 1 [1948]) wherein the admissibility of evidence was not affected by the
illegality of its seizure. The 1973 Charter (Sec. 4 [2], Art. IV) constitutionalized
the Stonehill ruling and is carried over up to the present with the advent of the 1987
Constitution.

In a number of cases, the Court strictly adhered to the exclusionary rule and has struck down
the admissibility of evidence obtained in violation of the constitutional safeguard against
unreasonable searches and seizures. (Bache & Co., (Phil.), Inc., v. Ruiz, 37 SCRA 823 [1971];
Lim v. Ponce de Leon, 66 SCRA 299 [1975]; People v. Burgos, 144 SCRA 1 [1986]; Roan v.
Gonzales, 145 SCRA 687 [1987]; See also Salazar v. Hon. Achacoso, et al., GR No. 81510,
March 14, 1990).

It must be noted, however, that in all those cases adverted to, the evidence so obtained were
invariably procured by the State acting through the medium of its law enforcers or other
authorized government agencies.

On the other hand, the case at bar assumes a peculiar character since the evidence sought to
be excluded was primarily discovered and obtained by a private person, acting in a private
capacity and without the intervention and participation of State authorities. Under the
circumstances, can accused/appellant validly claim that his constitutional right against
unreasonable searches and seizure has been violated? Stated otherwise, may an act of a
private individual, allegedly in violation of appellant's constitutional rights, be invoked against the
State?
We hold in the negative. In the absence of governmental interference, the liberties guaranteed
by the Constitution cannot be invoked against the State.

As this Court held in Villanueva v. Querubin (48 SCRA 345 [1972]:

1. This constitutional right (against unreasonable search and seizure) refers to the
immunity of one's person, whether citizen or alien, from interference by government,
included in which is his residence, his papers, and other possessions. . . .

. . . There the state, however powerful, does not as such have the access except under
the circumstances above noted, for in the traditional formulation, his house, however
humble, is his castle. Thus is outlawed any unwarranted intrusion by government, which
is called upon to refrain from any invasion of his dwelling and to respect the privacies of
his life. . . . (Cf. Schermerber v. California, 384 US 757 [1966] and Boyd v. United
States, 116 US 616 [1886]; Emphasis supplied).

In Burdeau v. McDowell (256 US 465 (1921), 41 S Ct. 547; 65 L.Ed. 1048), the Court there in
construing the right against unreasonable searches and seizures declared that:

(t)he Fourth Amendment gives protection against unlawful searches and seizures, and
as shown in previous cases, its protection applies to governmental action. Its origin and
history clearly show that it was intended as a restraint upon the activities of sovereign
authority, and was not intended to be a limitation upon other than governmental
agencies; as against such authority it was the purpose of the Fourth Amendment to
secure the citizen in the right of unmolested occupation of his dwelling and the
possession of his property, subject to the right of seizure by process duly served.

The above ruling was reiterated in State v. Bryan (457 P.2d 661 [1968]) where a parking
attendant who searched the automobile to ascertain the owner thereof found marijuana instead,
without the knowledge and participation of police authorities, was declared admissible in
prosecution for illegal possession of narcotics.

And again in the 1969 case of Walker v. State (429 S.W.2d 121), it was held that the search and
seizure clauses are restraints upon the government and its agents, not upon private individuals
(citing People v. Potter, 240 Cal. App.2d 621, 49 Cap. Rptr, 892 (1966); State v. Brown, Mo.,
391 S.W.2d 903 (1965); State v. Olsen, Or., 317 P.2d 938 (1957).

Likewise appropos is the case of Bernas v. US (373 F.2d 517 (1967). The Court there said:

The search of which appellant complains, however, was made by a private citizen — the
owner of a motel in which appellant stayed overnight and in which he left behind a travel
case containing the evidence*** complained of. The search was made on the motel
owner's own initiative. Because of it, he became suspicious, called the local police,
informed them of the bag's contents, and made it available to the authorities.

The fourth amendment and the case law applying it do not require exclusion of evidence
obtained through a search by a private citizen. Rather, the amendment only proscribes
governmental action."
The contraband in the case at bar having come into possession of the Government without the
latter transgressing appellant's rights against unreasonable search and seizure, the Court sees
no cogent reason why the same should not be admitted against him in the prosecution of the
offense charged.

Appellant, however, would like this court to believe that NBI agents made an illegal search and
seizure of the evidence later on used in prosecuting the case which resulted in his conviction.

The postulate advanced by accused/appellant needs to be clarified in two days. In both


instances, the argument stands to fall on its own weight, or the lack of it.

First, the factual considerations of the case at bar readily foreclose the proposition that NBI
agents conducted an illegal search and seizure of the prohibited merchandise. Records of the
case clearly indicate that it was Mr. Job Reyes, the proprietor of the forwarding agency, who
made search/inspection of the packages. Said inspection was reasonable and a standard
operating procedure on the part of Mr. Reyes as a precautionary measure before delivery of
packages to the Bureau of Customs or the Bureau of Posts (TSN, October 6 & 7, 1987, pp. 15-
18; pp. 7-8; Original Records, pp. 119-122; 167-168).

It will be recalled that after Reyes opened the box containing the illicit cargo, he took samples of
the same to the NBI and later summoned the agents to his place of business. Thereafter, he
opened the parcel containing the rest of the shipment and entrusted the care and custody
thereof to the NBI agents. Clearly, the NBI agents made no search and seizure, much less an
illegal one, contrary to the postulate of accused/appellant.

Second, the mere presence of the NBI agents did not convert the reasonable search effected by
Reyes into a warrantless search and seizure proscribed by the Constitution. Merely to observe
and look at that which is in plain sight is not a search. Having observed that which is open,
where no trespass has been committed in aid thereof, is not search (Chadwick v. State, 429
SW2d 135). Where the contraband articles are identified without a trespass on the part of the
arresting officer, there is not the search that is prohibited by the constitution (US v. Lee 274 US
559, 71 L.Ed. 1202 [1927]; Ker v. State of California 374 US 23, 10 L.Ed.2d. 726 [1963]; Moore
v. State, 429 SW2d 122 [1968]).

In Gandy v. Watkins (237 F. Supp. 266 [1964]), it was likewise held that where the property was
taken into custody of the police at the specific request of the manager and where the search
was initially made by the owner there is no unreasonable search and seizure within the
constitutional meaning of the term.

That the Bill of Rights embodied in the Constitution is not meant to be invoked against acts of
private individuals finds support in the deliberations of the Constitutional Commission. True, the
liberties guaranteed by the fundamental law of the land must always be subject to protection.
But protection against whom? Commissioner Bernas in his sponsorship speech in the Bill of
Rights answers the query which he himself posed, as follows:

First, the general reflections. The protection of fundamental liberties in the essence of
constitutional democracy. Protection against whom? Protection against the state. The
Bill of Rights governs the relationship between the individual and the state. Its concern is
not the relation between individuals, between a private individual and other individuals.
What the Bill of Rights does is to declare some forbidden zones in the private sphere
inaccessible to any power holder. (Sponsorship Speech of Commissioner Bernas ,
Record of the Constitutional Commission, Vol. 1, p. 674; July 17, 1986; Emphasis
supplied)

The constitutional proscription against unlawful searches and seizures therefore applies as a
restraint directed only against the government and its agencies tasked with the enforcement of
the law. Thus, it could only be invoked against the State to whom the restraint against arbitrary
and unreasonable exercise of power is imposed.

If the search is made upon the request of law enforcers, a warrant must generally be first
secured if it is to pass the test of constitutionality. However, if the search is made at the behest
or initiative of the proprietor of a private establishment for its own and private purposes, as in
the case at bar, and without the intervention of police authorities, the right against unreasonable
search and seizure cannot be invoked for only the act of private individual, not the law
enforcers, is involved. In sum, the protection against unreasonable searches and seizures
cannot be extended to acts committed by private individuals so as to bring it within the ambit of
alleged unlawful intrusion by the government.

Appellant argues, however, that since the provisions of the 1935 Constitution has been modified
by the present phraseology found in the 1987 Charter, expressly declaring as inadmissible any
evidence obtained in violation of the constitutional prohibition against illegal search and seizure,
it matters not whether the evidence was procured by police authorities or private individuals
(Appellant's Brief, p. 8, Rollo, p. 62).

The argument is untenable. For one thing, the constitution, in laying down the principles of the
government and fundamental liberties of the people, does not govern relationships between
individuals. Moreover, it must be emphasized that the modifications introduced in the 1987
Constitution (re: Sec. 2, Art. III) relate to the issuance of either a search warrant or warrant of
arrest vis-a-vis the responsibility of the judge in the issuance thereof (See Soliven v. Makasiar,
167 SCRA 393 [1988]; Circular No. 13 [October 1, 1985] and Circular No. 12 [June 30, 1987].
The modifications introduced deviate in no manner as to whom the restriction or inhibition
against unreasonable search and seizure is directed against. The restraint stayed with the State
and did not shift to anyone else.

Corolarilly, alleged violations against unreasonable search and seizure may only be invoked
against the State by an individual unjustly traduced by the exercise of sovereign authority. To
agree with appellant that an act of a private individual in violation of the Bill of Rights should also
be construed as an act of the State would result in serious legal complications and an absurd
interpretation of the constitution.

Similarly, the admissibility of the evidence procured by an individual effected through private
seizure equally applies, in pari passu, to the alleged violation, non-governmental as it is, of
appellant's constitutional rights to privacy and communication.

2. In his second assignment of error, appellant contends that the lower court erred in convicting
him despite the undisputed fact that his rights under the constitution while under custodial
investigation were not observed.

Again, the contention is without merit, We have carefully examined the records of the case and
found nothing to indicate, as an "undisputed fact", that appellant was not informed of his
constitutional rights or that he gave statements without the assistance of counsel. The law
enforcers testified that accused/appellant was informed of his constitutional rights. It is
presumed that they have regularly performed their duties (See. 5(m), Rule 131) and their
testimonies should be given full faith and credence, there being no evidence to the contrary.
What is clear from the records, on the other hand, is that appellant refused to give any written
statement while under investigation as testified by Atty. Lastimoso of the NBI, Thus:

Fiscal Formoso:

You said that you investigated Mr. and Mrs. Job Reyes. What about the accused here,
did you investigate the accused together with the girl?

WITNESS:

Yes, we have interviewed the accused together with the girl but the accused availed of
his constitutional right not to give any written statement, sir. (TSN, October 8, 1987, p.
62; Original Records, p. 240)

The above testimony of the witness for the prosecution was not contradicted by the defense on
cross-examination. As borne out by the records, neither was there any proof by the defense that
appellant gave uncounselled confession while being investigated. What is more, we
have examined the assailed judgment of the trial court and nowhere is there any reference
made to the testimony of appellant while under custodial investigation which was utilized in the
finding of conviction. Appellant's second assignment of error is therefore misplaced.

3. Coming now to appellant's third assignment of error, appellant would like us to believe that he
was not the owner of the packages which contained prohibited drugs but rather a certain
Michael, a German national, whom appellant met in a pub along Ermita, Manila: that in the
course of their 30-minute conversation, Michael requested him to ship the packages and gave
him P2,000.00 for the cost of the shipment since the German national was about to leave the
country the next day (October 15, 1987, TSN, pp. 2-10).

Rather than give the appearance of veracity, we find appellant's disclaimer as incredulous, self-
serving and contrary to human experience. It can easily be fabricated. An acquaintance with a
complete stranger struck in half an hour could not have pushed a man to entrust the shipment of
four (4) parcels and shell out P2,000.00 for the purpose and for appellant to readily accede to
comply with the undertaking without first ascertaining its contents. As stated by the trial court,
"(a) person would not simply entrust contraband and of considerable value at that as the
marijuana flowering tops, and the cash amount of P2,000.00 to a complete stranger like the
Accused. The Accused, on the other hand, would not simply accept such undertaking to take
custody of the packages and ship the same from a complete stranger on his mere say-so"
(Decision, p. 19, Rollo, p. 91). As to why he readily agreed to do the errand, appellant failed to
explain. Denials, if unsubstantiated by clear and convincing evidence, are negative self-serving
evidence which deserve no weight in law and cannot be given greater evidentiary weight than
the testimony of credible witnesses who testify on affirmative matters (People v. Esquillo, 171
SCRA 571 [1989]; People vs. Sariol, 174 SCRA 237 [1989]).

Appellant's bare denial is even made more suspect considering that, as per records of the
Interpol, he was previously convicted of possession of hashish by the Kleve Court in the Federal
Republic of Germany on January 1, 1982 and that the consignee of the frustrated shipment,
Walter Fierz, also a Swiss national, was likewise convicted for drug abuse and is just about an
hour's drive from appellant's residence in Zurich, Switzerland (TSN, October 8, 1987, p. 66;
Original Records, p. 244; Decision, p. 21; Rollo, p. 93).

Evidence to be believed, must not only proceed from the mouth of a credible witness, but it must
be credible in itself such as the common experience and observation of mankind can approve
as probable under the circumstances (People v. Alto, 26 SCRA 342 [1968], citing Daggers v.
Van Dyke, 37 N.J. Eg. 130; see also People v. Sarda, 172 SCRA 651 [1989]; People v. Sunga,
123 SCRA 327 [1983]); Castañares v. CA, 92 SCRA 567 [1979]). As records further show,
appellant did not even bother to ask Michael's full name, his complete address or passport
number. Furthermore, if indeed, the German national was the owner of the merchandise,
appellant should have so indicated in the contract of shipment (Exh. "B", Original Records, p.
40). On the contrary, appellant signed the contract as the owner and shipper thereof giving
more weight to the presumption that things which a person possesses, or exercises acts of
ownership over, are owned by him (Sec. 5 [j], Rule 131). At this point, appellant is therefore
estopped to claim otherwise.

Premises considered, we see no error committed by the trial court in rendering the assailed
judgment.

WHEREFORE, the judgment of conviction finding appellant guilty beyond reasonable doubt of
the crime charged is hereby AFFIRMED. No costs.

SO ORDERED.

G.R. No. L-19550 June 19, 1967

HARRY S. STONEHILL, ROBERT P. BROOKS, JOHN J. BROOKS and KARL


BECK, petitioners,
vs.
HON. JOSE W. DIOKNO, in his capacity as SECRETARY OF JUSTICE; JOSE LUKBAN, in
his capacity as Acting Director, National Bureau of Investigation; SPECIAL
PROSECUTORS PEDRO D. CENZON, EFREN I. PLANA and MANUEL VILLAREAL, JR. and
ASST. FISCAL MANASES G. REYES; JUDGE AMADO ROAN, Municipal Court of Manila;
JUDGE ROMAN CANSINO, Municipal Court of Manila; JUDGE HERMOGENES CALUAG,
Court of First Instance of Rizal-Quezon City Branch, and JUDGE DAMIAN JIMENEZ,
Municipal Court of Quezon City, respondents.

Paredes, Poblador, Cruz and Nazareno and Meer, Meer and Meer and Juan T. David for
petitioners.
Office of the Solicitor General Arturo A. Alafriz, Assistant Solicitor General Pacifico P. de Castro,
Assistant Solicitor General Frine C. Zaballero, Solicitor Camilo D. Quiason and Solicitor C.
Padua for respondents.

SYLLABUS

1. CONSTITUTIONAL LAW; SEARCH AND SEIZURE; WHO MAY CONTEST LEGALITY


THEREOF CASE AT BAR. — It is well settled that the legality of a seizure can be contested
only by the party whose rights have been impaired thereby (Lewis v s . U.S., 6 F . 2d. 22) and
that the objection to an unlawful search and seizure is purely personal and cannot be availed of
by third parties (In. re Dooley, 48 F . 2d. 121: Rouda v s . U.S., 10 F . 2d. 916; Lusco v s . U.S.,
287 F . 69; Ganci v s . U.S., 287 F , 60; Moriz v s . U.S., 26 F . 2d. 444). Consequently,
petitioner in the case at bar may not validly object to the use in evidence against them of the
document, papers, and things seized from the ofces and premises of the corporation adverted
to, since the right to object to the admission of said papers in evidence belongs exclusively to
the corporations, to whom the seized effects belong, and may not be invoked by the corporate
ofcers in proceedings against them in their individual capacity U.S., v s . Gaas, 17 F. 2d. 997;
People v s . Rubio, 57 Phil., 384). 2. ID.; ID.; REQUISITES FOR ISSUANCE OF SEARCH
WARRANT. — Two points must be stressed in connection with this constitutional mandate,
namely: (1) that no warrant issue but upon probable cause, to be determined by the judge in the
manner set forth in said provision; and (2) that the warrant shall particularly describe the things
to be seized. None of these requirements has been complied with in the contested warrants.
Indeed, the same were issued upon applications stating that the natural and juridical persons
therein named had committed a "violation of Central Bank Laws, Tariff and Customs Laws,
Internal Revenue (Code) and Revised Penal Code." In other words, no specic offense had been
alleged in said applications. The averments thereof with respect to the offense committed were
abstract. As a consequence, it was impossible for the judges who issued the

CD Technologies Asia, Inc. © 2016 cdasiaonline.com

warrants to have found the existence of probable cause, for the same presupposes the
introduction of competent proof that the party against whom it is sought has performed particular
acts, or committed specic omissions, violating a given provision of our criminal laws. As a matter
of fact, the applications involved in the case at bar do not allege any specic acts performed by
herein petitioners. It would be a legal heresy, of the highest order, to convict anybody of a
"violation of Central Bank Laws, Tariff and Customs Laws, Internal Revenue (Code) and
Revised Penal Code", — as alleged in aforementioned applications — without reference to any
determine provision of said laws or coders. 3. ID.; ID.; ID.; GENERAL WARRANTS ARE
OUTLAWED BY THE CONSTITUTION. — To uphold the validity of the warrants in question,
would be to wipe out completely one of the most fundamental rights guaranteed in our
Constitution, for it would place the sanctity of the domicile and the privacy of communication and
correspondence at the mercy of the victims, caprice or passion of peace ofcers. This is precisely
the evil sought to be remedied by the constitutional provision Sec. 1, par. 3 Art. III, Const.) — to
outlaw the socalled general warrants. It is not difcult to imagine what would happen, in times of
keen political strife, when the party in power feels that the minority is likely to wrest it, even
though by legal means. Such is the seriousness of the irregularities committed in connection
with the disputed search warrants, that this Court deemed it t to amend Section 3 of Rule 122 of
the former Rules of Court, by providing in its counterpart, under the Revised Rules of Court
(Sec. 3, Rule 126) that "a search warrant shall not issue but upon probable cause in connection
with one specic offense." Not satised with this qualication, the Court added thereto paragraph,
directing that "no search warrant shall issue for more than one specific offense." 4. ID.; ID.; ID.;
ID.; CASE AT BAR. — The grave violation of the Constitution made in the application for the
contested search warrants was compounded by the description therein made of the effects to
be searched for and seized, to wit: "Books of accounts, Financial records, vouchers, journals,
correspondence, receipts, ledgers, portfolios, credit journals, typewriters, and other documents
and/or papers, showing all business transactions including disbursement receipts, balance
sheets and related prot and loss statements." Thus, the warrants authorized the search for and
seizure of records pertaining to all b u sin e s s t r a n s a c tio n s petitioners herein, regardless
of whether the transaction were legal or illegal. The warrants sanctioned the seizure of all
records of the petitioners and the aforementioned corporations, whatever their nature, thus
openly contravening the explicit command of our Bill of Rights — that the things to be seized be
p a r tic ula rly d e s c rib e d — as well as tending to defeat its major objective: the elimination
of g e n e r al warrants. 5. ID.; ID.; ID.; NON-EXCLUSIONARY RULE CONTRAVENES THE
CONSTITUTIONAL PROHIBITIONS AGAINST UNREASONABLE SEARCH AND SEIZURES.
— Indeed, the nonexclusionary rule is contrary, not only to the letter, but also to the spirit of the
constitutional injunction against unreasonable searches and seizures. To be sure, if the
applicant for a search warrant has competent evidence to establish probable cause of the
commission of a given crime by the party against whom the warrant is intended, then there is no
reason why the applicant should not comply with the requirements of the fundamental law. Upon
the other hand, if he has no such competent evidence, then it is not possible for the Judge to nd
that there is probable cause and only possible for the Judge to nd that there is probable cause
and hence, no justication for the issuance of the warrant. The only possible explanation (not
justication) for its issuance is the necessity of shing evidence of the commission of crime. But
when this shing expedition is indicative of the absence of evidence to establish a probable
cause.

CD Technologies Asia, Inc. © 2016 cdasiaonline.com

6. ID.; ID.; ID.; ID.; PROSECUTION OF THOSE WHO SECURE ILLEGAL SEARCH WARRANT
OR MAKE UNREASONABLE SEARCH OR SEIZURE IS NO EXCUSE. — The theory that the
criminal prosecution of those who secure an illegal search warrant and/or make unreasonable
searches or seizures would sufce to protect the constitutional guarantee under consideration,
overlooks the fact that violations thereof are, in general, committed by agents of the party in
power, for certainly, those belonging to the minority could not possibly abuse a power they do
not have. Regardless of the handicap under which the minority usually but understandably nds
itself in prosecuting agents of the majority, one must not lose sight of the fact that the
psychological and moral effect of the possibility of securing their conviction, is watered down by
the pardoning power of the party for whose benefit the illegality had been committed. 7. ID.; ID.;
ID.; MONCADO DOCTRINE ABANDONED. — The doctrine adopted in the Moncado case must
be, as it is hereby, abandoned; the warrants for the search of 3 residences of petitioners, as
specied in the Resolution of June 29, 1962, are null and void; the searches and seizures therein
made are illegal.

CONCEPCION, C.J.:

Upon application of the officers of the government named on the margin1 — hereinafter referred
to as Respondents-Prosecutors — several judges2 — hereinafter referred to as Respondents-
Judges — issued, on different dates,3 a total of 42 search warrants against petitioners
herein4 and/or the corporations of which they were officers,5 directed to the any peace officer, to
search the persons above-named and/or the premises of their offices, warehouses and/or
residences, and to seize and take possession of the following personal property to wit:

Books of accounts, financial records, vouchers, correspondence, receipts, ledgers,


journals, portfolios, credit journals, typewriters, and other documents and/or papers
showing all business transactions including disbursements receipts, balance sheets and
profit and loss statements and Bobbins (cigarette wrappers).

as "the subject of the offense; stolen or embezzled and proceeds or fruits of the offense," or
"used or intended to be used as the means of committing the offense," which is described in the
applications adverted to above as "violation of Central Bank Laws, Tariff and Customs Laws,
Internal Revenue (Code) and the Revised Penal Code."
Alleging that the aforementioned search warrants are null and void, as contravening the
Constitution and the Rules of Court — because, inter alia: (1) they do not describe with
particularity the documents, books and things to be seized; (2) cash money, not mentioned in
the warrants, were actually seized; (3) the warrants were issued to fish evidence against the
aforementioned petitioners in deportation cases filed against them; (4) the searches and
seizures were made in an illegal manner; and (5) the documents, papers and cash money
seized were not delivered to the courts that issued the warrants, to be disposed of in
accordance with law — on March 20, 1962, said petitioners filed with the Supreme Court this
original action for certiorari, prohibition, mandamus and injunction, and prayed that, pending
final disposition of the present case, a writ of preliminary injunction be issued restraining
Respondents-Prosecutors, their agents and /or representatives from using the effects seized as
aforementioned or any copies thereof, in the deportation cases already adverted to, and that, in
due course, thereafter, decision be rendered quashing the contested search warrants and
declaring the same null and void, and commanding the respondents, their agents or
representatives to return to petitioners herein, in accordance with Section 3, Rule 67, of the
Rules of Court, the documents, papers, things and cash moneys seized or confiscated under
the search warrants in question.

In their answer, respondents-prosecutors alleged, 6 (1) that the contested search warrants are
valid and have been issued in accordance with law; (2) that the defects of said warrants, if any,
were cured by petitioners' consent; and (3) that, in any event, the effects seized are admissible
in evidence against herein petitioners, regardless of the alleged illegality of the aforementioned
searches and seizures.

On March 22, 1962, this Court issued the writ of preliminary injunction prayed for in the petition.
However, by resolution dated June 29, 1962, the writ was partially lifted or dissolved, insofar as
the papers, documents and things seized from the offices of the corporations above mentioned
are concerned; but, the injunction was maintained as regards the papers, documents and things
found and seized in the residences of petitioners herein.7

Thus, the documents, papers, and things seized under the alleged authority of the warrants in
question may be split into two (2) major groups, namely: (a) those found and seized in the
offices of the aforementioned corporations, and (b) those found and seized in the residences of
petitioners herein.

As regards the first group, we hold that petitioners herein have no cause of action to assail the
legality of the contested warrants and of the seizures made in pursuance thereof, for the simple
reason that said corporations have their respective personalities, separate and distinct from the
personality of herein petitioners, regardless of the amount of shares of stock or of the interest of
each of them in said corporations, and whatever the offices they hold therein may be.8 Indeed, it
is well settled that the legality of a seizure can be contested only by the party whose rights have
been impaired thereby,9 and that the objection to an unlawful search and seizure is purely
personal and cannot be availed of by third parties. 10 Consequently, petitioners herein may not
validly object to the use in evidence against them of the documents, papers and things seized
from the offices and premises of the corporations adverted to above, since the right to object to
the admission of said papers in evidence belongs exclusively to the corporations, to whom the
seized effects belong, and may not be invoked by the corporate officers in proceedings against
them in their individual capacity. 11 Indeed, it has been held:
. . . that the Government's action in gaining possession of papers belonging to
the corporation did not relate to nor did it affect the personal defendants. If these papers
were unlawfully seized and thereby the constitutional rights of or any one were invaded,
they were the rights of the corporation and not the rights of the other defendants. Next, it
is clear that a question of the lawfulness of a seizure can be raised only by one whose
rights have been invaded. Certainly, such a seizure, if unlawful, could not affect the
constitutional rights of defendants whose property had not been seized or the privacy of
whose homes had not been disturbed; nor could they claim for themselves the benefits
of the Fourth Amendment, when its violation, if any, was with reference to the rights
of another. Remus vs. United States (C.C.A.)291 F. 501, 511. It follows, therefore, that
the question of the admissibility of the evidence based on an alleged unlawful search
and seizure does not extend to the personal defendants but
embraces only the corporation whose property was taken. . . . (A Guckenheimer & Bros.
Co. vs. United States, [1925] 3 F. 2d. 786, 789, Emphasis supplied.)

With respect to the documents, papers and things seized in the residences of petitioners herein,
the aforementioned resolution of June 29, 1962, lifted the writ of preliminary injunction
previously issued by this Court, 12 thereby, in effect, restraining herein Respondents-
Prosecutors from using them in evidence against petitioners herein.

In connection with said documents, papers and things, two (2) important questions need be
settled, namely: (1) whether the search warrants in question, and the searches and seizures
made under the authority thereof, are valid or not, and (2) if the answer to the preceding
question is in the negative, whether said documents, papers and things may be used in
evidence against petitioners herein.1äwphï1.ñët

Petitioners maintain that the aforementioned search warrants are in the nature of general
warrants and that accordingly, the seizures effected upon the authority there of are null and
void. In this connection, the Constitution 13 provides:

The right of the people to be secure in their persons, houses, papers, and effects against
unreasonable searches and seizures shall not be violated, and no warrants shall issue
but upon probable cause, to be determined by the judge after examination under oath or
affirmation of the complainant and the witnesses he may produce, and particularly
describing the place to be searched, and the persons or things to be seized.

Two points must be stressed in connection with this constitutional mandate, namely: (1) that no
warrant shall issue but upon probable cause, to be determined by the judge in the manner set
forth in said provision; and (2) that the warrant shall particularly describe the things to be seized.

None of these requirements has been complied with in the contested warrants. Indeed, the
same were issued upon applications stating that the natural and juridical person therein named
had committed a "violation of Central Ban Laws, Tariff and Customs Laws, Internal Revenue
(Code) and Revised Penal Code." In other words, no specific offense had been alleged in said
applications. The averments thereof with respect to the offense committed were abstract. As a
consequence, it was impossible for the judges who issued the warrants to have found the
existence of probable cause, for the same presupposes the introduction of competent proof that
the party against whom it is sought has performed particular acts, or
committed specific omissions, violating a given provision of our criminal laws. As a matter of
fact, the applications involved in this case do not allege any specific acts performed by herein
petitioners. It would be the legal heresy, of the highest order, to convict anybody of a "violation
of Central Bank Laws, Tariff and Customs Laws, Internal Revenue (Code) and Revised Penal
Code," — as alleged in the aforementioned applications — without reference to any determinate
provision of said laws or

To uphold the validity of the warrants in question would be to wipe out completely one of the
most fundamental rights guaranteed in our Constitution, for it would place the sanctity of the
domicile and the privacy of communication and correspondence at the mercy of the whims
caprice or passion of peace officers. This is precisely the evil sought to be remedied by the
constitutional provision above quoted — to outlaw the so-called general warrants. It is not
difficult to imagine what would happen, in times of keen political strife, when the party in power
feels that the minority is likely to wrest it, even though by legal means.

Such is the seriousness of the irregularities committed in connection with the disputed search
warrants, that this Court deemed it fit to amend Section 3 of Rule 122 of the former Rules of
Court 14 by providing in its counterpart, under the Revised Rules of Court 15 that "a search
warrant shall not issue but upon probable cause in connection with one specific offense." Not
satisfied with this qualification, the Court added thereto a paragraph, directing that "no search
warrant shall issue for more than one specific offense."

The grave violation of the Constitution made in the application for the contested search warrants
was compounded by the description therein made of the effects to be searched for and seized,
to wit:

Books of accounts, financial records, vouchers, journals, correspondence, receipts,


ledgers, portfolios, credit journals, typewriters, and other documents and/or papers
showing all business transactions including disbursement receipts, balance sheets and
related profit and loss statements.

Thus, the warrants authorized the search for and seizure of records pertaining to all business
transactions of petitioners herein, regardless of whether the transactions were legal or illegal.
The warrants sanctioned the seizure of all records of the petitioners and the aforementioned
corporations, whatever their nature, thus openly contravening the explicit command of our Bill of
Rights — that the things to be seized be particularly described — as well as tending to defeat its
major objective: the elimination of general warrants.

Relying upon Moncado vs. People's Court (80 Phil. 1), Respondents-Prosecutors maintain that,
even if the searches and seizures under consideration were unconstitutional, the documents,
papers and things thus seized are admissible in evidence against petitioners herein. Upon
mature deliberation, however, we are unanimously of the opinion that the position taken in the
Moncado case must be abandoned. Said position was in line with the American common law
rule, that the criminal should not be allowed to go free merely "because the constable has
blundered," 16 upon the theory that the constitutional prohibition against unreasonable searches
and seizures is protected by means other than the exclusion of evidence unlawfully
obtained, 17 such as the common-law action for damages against the searching officer, against
the party who procured the issuance of the search warrant and against those assisting in the
execution of an illegal search, their criminal punishment, resistance, without liability to an
unlawful seizure, and such other legal remedies as may be provided by other laws.
However, most common law jurisdictions have already given up this approach and eventually
adopted the exclusionary rule, realizing that this is the only practical means of enforcing the
constitutional injunction against unreasonable searches and seizures. In the language of Judge
Learned Hand:

As we understand it, the reason for the exclusion of evidence competent as such, which
has been unlawfully acquired, is that exclusion is the only practical way of enforcing the
constitutional privilege. In earlier times the action of trespass against the offending
official may have been protection enough; but that is true no longer. Only in case the
prosecution which itself controls the seizing officials, knows that it cannot profit by their
wrong will that wrong be repressed.18

In fact, over thirty (30) years before, the Federal Supreme Court had already declared:

If letters and private documents can thus be seized and held and used in evidence
against a citizen accused of an offense, the protection of the 4th Amendment, declaring
his rights to be secure against such searches and seizures, is of no value, and, so far as
those thus placed are concerned, might as well be stricken from the Constitution. The
efforts of the courts and their officials to bring the guilty to punishment, praiseworthy as
they are, are not to be aided by the sacrifice of those great principles established by
years of endeavor and suffering which have resulted in their embodiment in the
fundamental law of the land.19

This view was, not only reiterated, but, also, broadened in subsequent decisions on the same
Federal Court. 20 After reviewing previous decisions thereon, said Court held, in Mapp vs.
Ohio (supra.):

. . . Today we once again examine the Wolf's constitutional documentation of the right of
privacy free from unreasonable state intrusion, and after its dozen years on our books,
are led by it to close the only courtroom door remaining open to evidence secured by
official lawlessness in flagrant abuse of that basic right, reserved to all persons as a
specific guarantee against that very same unlawful conduct. We hold that all evidence
obtained by searches and seizures in violation of the Constitution is, by that same
authority, inadmissible in a State.

Since the Fourth Amendment's right of privacy has been declared enforceable against
the States through the Due Process Clause of the Fourteenth, it is enforceable against
them by the same sanction of exclusion as it used against the Federal Government.
Were it otherwise, then just as without the Weeks rule the assurance against
unreasonable federal searches and seizures would be "a form of words," valueless and
underserving of mention in a perpetual charter of inestimable human liberties, so
too, without that rule the freedom from state invasions of privacy would be so ephemeral
and so neatly severed from its conceptual nexus with the freedom from all brutish means
of coercing evidence as not to permit this Court's high regard as a freedom "implicit in
the concept of ordered liberty." At the time that the Court held in Wolf that the
amendment was applicable to the States through the Due Process Clause, the cases of
this Court as we have seen, had steadfastly held that as to federal officers the Fourth
Amendment included the exclusion of the evidence seized in violation of its provisions.
Even Wolf "stoutly adhered" to that proposition. The right to when conceded operatively
enforceable against the States, was not susceptible of destruction by avulsion of the
sanction upon which its protection and enjoyment had always been deemed dependent
under the Boyd, Weeks and Silverthorne Cases. Therefore, in extending the substantive
protections of due process to all constitutionally unreasonable searches — state or
federal — it was logically and constitutionally necessarily that the exclusion doctrine —
an essential part of the right to privacy — be also insisted upon as an essential
ingredient of the right newly recognized by the Wolf Case. In short, the admission of the
new constitutional Right by Wolf could not tolerate denial of its most important
constitutional privilege, namely, the exclusion of the evidence which an accused had
been forced to give by reason of the unlawful seizure. To hold otherwise is to grant the
right but in reality to withhold its privilege and enjoyment. Only last year the Court itself
recognized that the purpose of the exclusionary rule to "is to deter — to compel respect
for the constitutional guaranty in the only effectively available way — by removing the
incentive to disregard it" . . . .

The ignoble shortcut to conviction left open to the State tends to destroy the entire
system of constitutional restraints on which the liberties of the people rest. Having once
recognized that the right to privacy embodied in the Fourth Amendment is enforceable
against the States, and that the right to be secure against rude invasions of privacy by
state officers is, therefore constitutional in origin, we can no longer permit that right to
remain an empty promise. Because it is enforceable in the same manner and to like
effect as other basic rights secured by its Due Process Clause, we can no longer permit
it to be revocable at the whim of any police officer who, in the name of law enforcement
itself, chooses to suspend its enjoyment. Our decision, founded on reason and truth,
gives to the individual no more than that which the Constitution guarantees him to the
police officer no less than that to which honest law enforcement is entitled, and, to the
courts, that judicial integrity so necessary in the true administration of justice. (emphasis
ours.)

Indeed, the non-exclusionary rule is contrary, not only to the letter, but also, to the spirit of the
constitutional injunction against unreasonable searches and seizures. To be sure, if the
applicant for a search warrant has competent evidence to establish probable cause of the
commission of a given crime by the party against whom the warrant is intended, then there is no
reason why the applicant should not comply with the requirements of the fundamental law. Upon
the other hand, if he has no such competent evidence, then it is not possible for the Judge to
find that there is probable cause, and, hence, no justification for the issuance of the warrant.
The only possible explanation (not justification) for its issuance is the necessity
of fishing evidence of the commission of a crime. But, then, this fishing expedition is indicative of
the absence of evidence to establish a probable cause.

Moreover, the theory that the criminal prosecution of those who secure an illegal search warrant
and/or make unreasonable searches or seizures would suffice to protect the constitutional
guarantee under consideration, overlooks the fact that violations thereof are, in general,
committed By agents of the party in power, for, certainly, those belonging to the minority could
not possibly abuse a power they do not have. Regardless of the handicap under which the
minority usually — but, understandably — finds itself in prosecuting agents of the majority, one
must not lose sight of the fact that the psychological and moral effect of the possibility 21 of
securing their conviction, is watered down by the pardoning power of the party for whose benefit
the illegality had been committed.
In their Motion for Reconsideration and Amendment of the Resolution of this Court dated June
29, 1962, petitioners allege that Rooms Nos. 81 and 91 of Carmen Apartments, House No.
2008, Dewey Boulevard, House No. 1436, Colorado Street, and Room No. 304 of the Army-
Navy Club, should be included among the premises considered in said Resolution as
residences of herein petitioners, Harry S. Stonehill, Robert P. Brook, John J. Brooks and Karl
Beck, respectively, and that, furthermore, the records, papers and other effects seized in the
offices of the corporations above referred to include personal belongings of said petitioners and
other effects under their exclusive possession and control, for the exclusion of which they have
a standing under the latest rulings of the federal courts of federal courts of the United States. 22

We note, however, that petitioners' theory, regarding their alleged possession of and control
over the aforementioned records, papers and effects, and the alleged "personal" nature thereof,
has Been Advanced, not in their petition or amended petition herein, but in the Motion for
Reconsideration and Amendment of the Resolution of June 29, 1962. In other words, said
theory would appear to be readjustment of that followed in said petitions, to suit the approach
intimated in the Resolution sought to be reconsidered and amended. Then, too, some of the
affidavits or copies of alleged affidavits attached to said motion for reconsideration, or submitted
in support thereof, contain either inconsistent allegations, or allegations inconsistent with the
theory now advanced by petitioners herein.

Upon the other hand, we are not satisfied that the allegations of said petitions said motion for
reconsideration, and the contents of the aforementioned affidavits and other papers submitted in
support of said motion, have sufficiently established the facts or conditions contemplated in the
cases relied upon by the petitioners; to warrant application of the views therein expressed,
should we agree thereto. At any rate, we do not deem it necessary to express our opinion
thereon, it being best to leave the matter open for determination in appropriate cases in the
future.

We hold, therefore, that the doctrine adopted in the Moncado case must be, as it is hereby,
abandoned; that the warrants for the search of three (3) residences of herein petitioners, as
specified in the Resolution of June 29, 1962, are null and void; that the searches and seizures
therein made are illegal; that the writ of preliminary injunction heretofore issued, in connection
with the documents, papers and other effects thus seized in said residences of herein
petitioners is hereby made permanent; that the writs prayed for are granted, insofar as the
documents, papers and other effects so seized in the aforementioned residences are
concerned; that the aforementioned motion for Reconsideration and Amendment should be, as
it is hereby, denied; and that the petition herein is dismissed and the writs prayed for denied, as
regards the documents, papers and other effects seized in the twenty-nine (29) places, offices
and other premises enumerated in the same Resolution, without special pronouncement as to
costs.

It is so ordered.

Reyes, J.B.L., Dizon, Makalintal, Bengzon, J.P., Zaldivar and Sanchez, JJ., concur.

[A.M. No. RTJ-93-964. February 28, 1996.]


LEOVIGILDO U. MANTARING LEOVIGILDO U. MANTARING, c o m p lain a n t , v s v s .
JUDGE MANUEL A. . JUDGE MANUEL A. ROMAN, JR., RTC, Branch 42, Pinamalayan,
Oriental Mindoro; and ROMAN, JR., RTC, Branch 42, Pinamalayan, Oriental Mindoro; and
JUDGE IRENEO B. MOLATO, MTC, Bongabon, Oriental Mindoro JUDGE IRENEO B.
MOLATO, MTC, Bongabon, Oriental Mindoro, r e s p o n d e n t s .
SYLLABUS SYLLABUS
1. REMEDIAL LAW; NEW TRIAL; REMEDY AVAILABLE TO PARTIES DENIED OF A FAIR
AND IMPARTIAL TRIAL. — There is a remedy available to the party seeking the disqualification
of the judge. If he is denied a fair and impartial trial, caused by the judge's bias or prejudice, he
can ask for a new trial in the interest of justice which will be granted if that is really the case. 2.
ID.; CRIMINAL PROCEDURE; SEARCH WARRANT; WARRANT OF ARREST; BASIS FOR
ISSUANCE. — The issuance of a search warrant and of a warrant of arrest requires the
showing of probabilities as to different facts. In the case of search warrants, the determination is
based on the finding that (1) the articles to be seized are connected to a criminal activity and (2)
they are found in the place to be searched. It is not necessary that a particular person be
implicated. On the other hand, in arrest cases, the determination of probable cause is based on
a finding that a crime has been committed and that the person to be arrested has committed it.
3. ID.; ID.; WARRANT OF ARREST; REQUIREMENTS FOR ISSUANCE. — It is now settled
that in issuing warrants of arrest in preliminary investigations, the investigating judge must: (a)
have examined in writing and under oath the complainant and his witnesses by searching
questions and answers; (b) be satisfied that probable cause exists; and (c) that there is a need
to place the respondent under immediate custody in order not to frustrate the ends of justice.
MENDOZA, J.:

Respondent Judge Ireneo B. Molato is the presiding judge of the Municipal Trial Court of
Bongabon, Oriental Mindoro. On January 7, 1993, an administrative complaint was filed against
him and Judge Manuel A. Roman, Jr., presiding judge of the Regional Trial Court of
Pinamalayan, Oriental Mindoro, Branch 42, by Leovigildo U. Mantaring, Sr., who charged them
with conduct unbecoming of members of the judiciary. On February 21, 1994, after the parties
had filed their respective pleadings and supporting documents, this Court dismissed the
complaint against the two for lack of merit. The motion for reconsideration filed by complainant
was subsequently denied.

What is before us now is the Supplemental Complaint filed by Leovigildo U. Mantaring, Sr.
against Judge Ireneo B. Molato, which charges him with harassment. It is alleged that because
of the filing of the first complaint against him, respondent Judge Ireneo B. Molato should have
inhibited himself from conducting the preliminary investigation of a criminal case considering
that the respondents in that case were complainant and his son. Instead, it is alleged, he took
cognizance of the case and ordered the arrest of complainant and his son, Leovigildo
Mantaring, Jr., out of hatred and revenge for them because of the filing of the first case by the
complainant.

The Supplemental Complaint was referred to the Office of the Court Administrator which, in a
Memorandum dated 25 November 1994, recommended the dismissal of the case for lack of
merit. Nonetheless, the Court required the respondent Judge Ireneo B. Molato to comment.
In his Comment dated July 6, 1995, respondent judge denies the allegations against him. He
avers that on the application by SPO4 Pacifico L. Fradejas, he issued a search warrant which
resulted in the seizure from a certain Joel Gamo of a home-made gun, a hand grenade, five live
ammunitions for Cal. 38 and three live ammunitions for 12 gauge shotgun; that on August 25,
1993, a complaint for Illegal Possession of Firearms and Ammunition was filed against Joel
Gamo in which the herein complainant Leovigildo, Sr. and his son, Leovigildo, Jr., were
included; that finding that the house in which the firearms and ammunition had been found was
owned by complainant and his son, he concluded that there was probable cause to believe that
complainant and his son were guilty of illegal possession of firearms and ammunition and
accordingly ordered their arrest. Respondent judge claims that he inhibited himself from the
case after he was ordered by the Executive Judge, RTC, Branch 41, Pinamalayan Oriental
Mindoro.

In his Reply complainant contends that as the search warrant was issued only against Joel
Gamo and Mantaring, Jr. it was wrong for respondent judge to find probable cause against him
on the theory that, as owners of the house in which the firearms and ammunition were found,
they had constructive possession of the same. He likewise contends that respondent judge did
not inhibit himself until after the preliminary examination was terminated and the warrant of
arrest issued, and only after complainant had filed a petition for inhibition which the Executive
Judge found to be well taken.

On October 16, 1995, this case was referred to the OCA for reevaluation, report and
recommendation. On January 12, 1996, the OCA submitted a Memorandum, recommending
dismissal of the supplemental complaint for lack of merit, for the following reasons:

(1) It is erroneous for herein complainant to equate the application for the issuance of search
warrant with the institution and prosecution of criminal action in a trial court. (Malaloan vs. Court
of Appeals, 232 SCRA 249) Complainant cannot insist that since his name was not included in
the search warrant, the house designated to be searched did not belong to him, and that he was
not present at the preliminary investigation of witnesses preparatory to the issuance of the
questioned warrant of arrest, there was no basis for respondent judge to order his arrest.

(2) No taint of irregularity attended the issuance by respondent judge of the warrant of arrest
against complainant and his son. Neither was the charge that the warrant of arrest was issued
by respondent judge in the spirit of anger, hatred or harassment purposes substantiated.

To begin with, it cannot be contended that complainant Leovigildo Mantaring, Sr. could not be
proceeded against simply because he was not included in the search warrant issued against
Gamo and Leovigildo Mantaring, Jr., who is apparently his son. The determination of probable
cause in preliminary investigations is based solely on the evidence presented by the
complainant, regardless of whether or not the respondent in that case is named in the
proceedings for a search warrant. As correctly pointed out by, the OCA,1 the issuance of a
search warrant and of a warrant of arrest requires the showing of probabilities as to different
facts. In the case of search warrants, the determination is based on the finding that (1) the
articles to be seized are connected to a criminal activity and (2) they are found in the place to be
searched. It is not necessary that a particular person be implicated. On the other hand, in arrest
cases, the determination of probable cause is based on a finding that a crime has been
committed and that the person to be arrested has committed it.

In this case, the arrest of herein complainant and his son, together with Joel Gamo, was ordered
on the basis of respondent's finding that the place from where the guns and ammunition were
seized belonged to complainant Leovigildo Mantaring, Sr. and the testimonies of witnesses
presented by SPO4 Fradejas. Of course complainant denies that the house in which the
firearms and ammunition were found belonged to him and claims that at the time of the search
he was in Manila. The provincial prosecutor subsequently dismissed the case against
complainant on precisely these grounds, i.e., that the house did not belong to complainant and
he was in Manila at the time the search and seizure were conducted. But to say this is not to
say that respondent acted arbitrarily or that he abused his powers so as to give ground for
administrative disciplinary action against him. It is only to say that he committed an error of
judgment for which complainant's remedy is judicial.

What we think requires serious consideration is the contention by the complainant that
respondent judge should have inhibited himself from conducting the preliminary investigation of
the criminal case, considering that the respondent was the present complainant, who had earlier
filed an administrative case against the judge and another one.

We are not unmindful of the cases in which it was stated that the mere filing of an administrative
case against a judge by one of the parties before him is not a ground for disqualifying him from
hearing a case.2 An examination of these cases reveals, however, that the administrative cases
were filed during the pendency of the cases, and it is evident that the administrative cases were
filed only to force the judge to inhibit himself from the consideration of the case before him. As
this Court held, if on every occasion the party apparently aggrieved were allowed to stop the
proceedings in order to await the final decision on the desired disqualification, or demand the
immediate inhibition of the judge on the basis alone of his being so charged, many cases would
have to be kept pending or perhaps there would not be enough judges left to handle all the
cases pending in all the courts.3 On the other hand, there is a remedy available to the party
seeking the disqualification of the judge. If he is denied a fair and impartial trial, caused by the
judge's bias or prejudice, he can ask for a new trial in the interest of justice which will be granted
if that is really the case. 4

But, in the case at bar, an administrative complaint against respondent and Judge Manuel A.
Roman, Jr. had previously been filed and it was paramount that respondent was free from any
appearance of bias against, or hostility toward, the complainant. The impression could not be
helped that his action in that case was dictated by a spirt of revenge against complainant for the
latter's having filed an administrative disciplinary action against the judge. The situation called
for sedulous regard on his part for the principle that a party is entitled to nothing less than the
cold neutrality of an impartial judge.

This circumstance should have underscored for respondent the need of steering clear of the
case because he might be perceived, rightly or wrongly, to be susceptible to bias and partiality.
For his judgment must not be tainted by even the slightest suspicion of improbity or
preconceived interest in order to preserve at all times the faith and confidence in courts of
justice by any party to the litigation.5

Indeed prudence should have made respondent judge heed the admonition that "a spotless
dispensation of justice requires not only that the decision rendered be intrinsically fair but that
the judge rendering it must at all times maintain the appearance of fairness and impartiality."6

Moreover, we think it was improper for respondent judge to have issued the warrants of arrest
against complainant and his son without any finding that it was necessary to place them in
immediate custody in order to prevent a frustration of justice. It is now settled7 that in issuing
warrants of arrest in preliminary investigations, the investigating judge must:

(a) have examined in writing and under oath the complainant and his witnesses by searching
questions and answers;

(b) be satisfied that probable cause exists; and

(c) that there is a need to place the respondent under immediate custody in order not to
frustrate the ends of justice.

In this case, respondent judge justified the issuance of the warrant of arrest on the following
ground:

In view of the above considerations [referring to the antecedent facts], it is the honest belief and
finding of the Court that there is sufficient probable cause that the crime of Illegal Possession of
Firearm and Ammunition was committed and that the named three (3) accused Joel Gamo,
Leovigildo Mantaring, Sr. and Leovigildo Mantaring, Jr. are the ones probably guilty thereof for
which reason Warrant of Arrest was issued by undersigned against them.
He thus ordered the issuance of warrant of arrest solely on his finding of probable cause, totally
omitting to consider the third requirement that there must be a need to place the respondent
under immediate custody "in order not to frustrate the ends of justice."

The framers of the Constitution confined the determination of probable cause as basis for the
issuance of warrants of arrest and search warrants to judges the better to secure the people
against unreasonable searches and seizures. Respondent judge failed to live up to this
expectation by refusing to inhibit himself even when his very impartiality was in question and
worse by issuing a warrant of arrest without determining whether or not it was justified by the
need to prevent a frustration of the ends of justice. Parenthetically, the records show that the
criminal complaints against herein complainant and his son were eventually dismissed by the
Provincial Prosecutor, but not without the following parting words:

It cannot be gainsaid that respondents Mantarings were greatly prejudiced and suffered
damages as a consequence of their inclusion in the criminal complaint. The unfortunate incident
could have been avoided had the Honorable Municipal Trial Judge exercised the necessary
prudence and judicial perpecuity [sic] expected of an impartial Judge in the conduct of
preliminary investigation before issuance of warrant of arrest.

WHEREFORE, respondent judge Ireneo B. Molato is REPRIMANDED and WARNED that


commission of similar acts in the future will be dealt with more severely. All other charges are
dismissed for lack of merit.

SO ORDERED.

G.R. No. 82585 November 14, 1988

MAXIMO V. SOLIVEN, ANTONIO V. ROCES, FREDERICK K. AGCAOLI, and GODOFREDO


L. MANZANAS, petitioners,
vs.
THE HON. RAMON P. MAKASIAR, Presiding Judge of the Regional Trial Court of Manila,
Branch 35, UNDERSECRETARY SILVESTRE BELLO III, of the Department of Justice,
LUIS C. VICTOR, THE CITY FISCAL OF MANILA and PRESIDENT CORAZON C.
AQUINO, respondents.

G.R. No. 82827 November 14, 1988

LUIS D. BELTRAN, petitioner,


vs.
THE HON. RAMON P. MAKASIAR, Presiding Judge of Branch 35 of the Regional Trial
Court, at Manila, THE HON. LUIS VICTOR, CITY FISCAL OF MANILA, PEOPLE OF THE
PHILIPPINES, SUPERINTENDENT OF THE WESTERN POLICE DISTRICT, and THE
MEMBERS OF THE PROCESS SERVING UNIT AT THE REGIONAL TRIAL COURT OF
MANILA, respondents.

G.R. No. 83979 November 14, 1988.

LUIS D. BELTRAN, petitioner,


vs.
EXECUTIVE SECRETARY CATALINO MACARAIG, SECRETARY OF JUSTICE SEDFREY
ORDOÑEZ, UNDERSECRETARY OF JUSTICE SILVESTRE BELLO III, THE CITY FISCAL
OF MANILA JESUS F. GUERRERO, and JUDGE RAMON P. MAKASIAR, Presiding Judge
of Branch 35 of the Regional Trial Court, at Manila, respondents.

Angara, Abello, Concepcion, Regala and Cruz for petitioners in G.R. No. 82585.

Perfecto V. Fernandez, Jose P. Fernandez and Cristobal P. Fernandez for petitioner in G.R.
Nos. 82827 and 83979.

RESOLUTION

PER CURIAM:

In these consolidated cases, three principal issues were raised: (1) whether or not petitioners
were denied due process when informations for libel were filed against them although the
finding of the existence of a prima facie case was still under review by the Secretary of Justice
and, subsequently, by the President; (2) whether or not the constitutional rights of Beltran were
violated when respondent RTC judge issued a warrant for his arrest without personally
examining the complainant and the witnesses, if any, to determine probable cause; and (3)
whether or not the President of the Philippines, under the Constitution, may initiate criminal
proceedings against the petitioners through the filing of a complaint-affidavit.

Subsequent events have rendered the first issue moot and academic. On March 30, 1988, the
Secretary of Justice denied petitioners' motion for reconsideration and upheld the resolution of
the Undersecretary of Justice sustaining the City Fiscal's finding of a prima facie case against
petitioners. A second motion for reconsideration filed by petitioner Beltran was denied by the
Secretary of Justice on April 7, 1988. On appeal, the President, through the Executive
Secretary, affirmed the resolution of the Secretary of Justice on May 2, 1988. The motion for
reconsideration was denied by the Executive Secretary on May 16, 1988. With these
developments, petitioners' contention that they have been denied the administrative remedies
available under the law has lost factual support.

It may also be added that with respect to petitioner Beltran, the allegation of denial of due
process of law in the preliminary investigation is negated by the fact that instead of submitting
his counter- affidavits, he filed a "Motion to Declare Proceedings Closed," in effect waiving his
right to refute the complaint by filing counter-affidavits. Due process of law does not require that
the respondent in a criminal case actually file his counter-affidavits before the preliminary
investigation is deemed completed. All that is required is that the respondent be given the
opportunity to submit counter-affidavits if he is so minded.
The second issue, raised by petitioner Beltran, calls for an interpretation of the constitutional
provision on the issuance of warrants of arrest. The pertinent provision reads:

Art. III, Sec. 2. The right of the people to be secure in their persons, houses,
papers and effects against unreasonable searches and seizures of whatever
nature and for any purpose shall be inviolable, and no search warrant or warrant
of arrest shall issue except upon probable cause to be determined personally by
the judge after examination nder oath or affirmation of the complainant and the
witnesses he may produce, and particularly describing the place to be searched
and the persons or things to be seized.

The addition of the word "personally" after the word "determined" and the deletion of the grant of
authority by the 1973 Constitution to issue warrants to "other responsible officers as may be
authorized by law," has apparently convinced petitioner Beltran that the Constitution now
requires the judge to personally examine the complainant and his witnesses in his determination
of probable cause for the issuance of warrants of arrest. This is not an accurate interpretation.

What the Constitution underscores is the exclusive and personal responsibility of the issuing
judge to satisfy himself of the existence of probable cause. In satisfying himself of the existence
of probable cause for the issuance of a warrant of arrest, the judge is not required to personally
examine the complainant and his witnesses. Following established doctrine and procedure, he
shall: (1) personally evaluate the report and the supporting documents submitted by the fiscal
regarding the existence of probable cause and, on the basis thereof, issue a warrant of arrest;
or (2) if on the basis thereof he finds no probable cause, he may disregard the fiscal's report
and require the submission of supporting affidavits of witnesses to aid him in arriving at a
conclusion as to the existence of probable cause.

Sound policy dictates this procedure, otherwise judges would be unduly laden with the
preliminary examination and investigation of criminal complaints instead of concentrating on
hearing and deciding cases filed before their courts.

On June 30, 1987, the Supreme Court unanimously adopted Circular No. 12, setting down
guidelines for the issuance of warrants of arrest. The procedure therein provided is reiterated
and clarified in this resolution.

It has not been shown that respondent judge has deviated from the prescribed procedure. Thus,
with regard to the issuance of the warrants of arrest, a finding of grave abuse of discretion
amounting to lack or excess of jurisdiction cannot be sustained.

Anent the third issue, petitioner Beltran argues that "the reasons which necessitate presidential
immunity from suit impose a correlative disability to file suit." He contends that if criminal
proceedings ensue by virtue of the President's filing of her complaint-affidavit, she may
subsequently have to be a witness for the prosecution, bringing her under the trial court's
jurisdiction. This, continues Beltran, would in an indirect way defeat her privilege of immunity
from suit, as by testifying on the witness stand, she would be exposing herself to possible
contempt of court or perjury.

The rationale for the grant to the President of the privilege of immunity from suit is to assure the
exercise of Presidential duties and functions free from any hindrance or distraction, considering
that being the Chief Executive of the Government is a job that, aside from requiring all of the
office holder's time, also demands undivided attention.

But this privilege of immunity from suit, pertains to the President by virtue of the office and may
be invoked only by the holder of the office; not by any other person in the President's behalf.
Thus, an accused in a criminal case in which the President is complainant cannot raise the
presidential privilege as a defense to prevent the case from proceeding against such accused.

Moreover, there is nothing in our laws that would prevent the President from waiving the
privilege. Thus, if so minded the President may shed the protection afforded by the privilege and
submit to the court's jurisdiction. The choice of whether to exercise the privilege or to waive it is
solely the President's prerogative. It is a decision that cannot be assumed and imposed by any
other person.

As regards the contention of petitioner Beltran that he could not be held liable for libel because
of the privileged character or the publication, the Court reiterates that it is not a trier of facts and
that such a defense is best left to the trial court to appreciate after receiving the evidence of the
parties.

As to petitioner Beltran's claim that to allow the libel case to proceed would produce a "chilling
effect" on press freedom, the Court finds no basis at this stage to rule on the point.

The petitions fail to establish that public respondents, through their separate acts, gravely
abused their discretion as to amount to lack of jurisdiction. Hence, the writs of certiorari and
prohibition prayed for cannot issue.

WHEREFORE, finding no grave abuse of discretion amounting to excess or lack of jurisdiction


on the part of the public respondents, the Court Resolved to DISMISS the petitions in G. R. Nos.
82585, 82827 and 83979. The Order to maintain the status quo contained in the Resolution of
the Court en banc dated April 7, 1988 and reiterated in the Resolution dated April 26, 1988 is
LIFTED.

[G.R. No. 81756. October 21, 1991.]


NICOMEDES SILVA @ "Comedes", MARLON SILVA @ "Tama" and NICOMEDES SILVA @
"Comedes", MARLON SILVA @ "Tama" and ANTONIETA SILVA ANTONIETA SILVA, p e titio
ners,
v s . v s . THE HONORABLE PRESIDING THE HONORABLE PRESIDING JUDGE,
REGIONAL TRIAL COURT OF NEGROS ORIENTAL, BRANCH JUDGE, REGIONAL TRIAL
COURT OF NEGROS ORIENTAL, BRANCH XXXIII, DUMAGUETE CITY XXXIII,
DUMAGUETE CITY, r e s p o n d e n t .
M a r c elo G . Flo r e s for petitioners.
SYLLABUS SYLLABUS
1. CONSTITUTIONAL LAW; BILL OF RIGHTS; RIGHT AGAINST UNLAWFUL SEARCH AND
SEIZURE; PURPOSE. — The purpose of the constitutional provision against unlawful searches
and seizures is to prevent violations of private security in person and property, and unlawful
invasion of the sanctity of the home, by officers of the law acting under legislative or judicial
sanction, and to give remedy against such usurpations when attempted. (Alvero vs. Dizon, 76
Phil. 637 [1946]). 2. ID.; ID.; ID.; SEARCH WARRANT; REQUISITES FOR ISSUANCE
THEREOF. — Based on Section 2, Article III of the 1987 Constitution and Sections 3 and 4,
Rule 126 of the Rules of Court, the judge must, before issuing a search warrant, determine
whether there is probable cause by examining the complainant and witnesses through
searching questions and answers. 3. ID.; ID.; ID.; ID.; ID.; PROBABLE CAUSE; DEFINED. — In
the case of P r u d e n t e v s . D a y rit , G.R. No. 82870, December 14, 1989, 180 SCRA 69,
767 this Court defined "probable cause" as follows: "The 'probable cause' for a valid search
warrant, has been defined 'as such facts and circumstances which would lead a reasonably
discreet and prudent man to believe that an offense has been committed, and that objects
sought in connection with the offense are in the place sought to be searched'. This probable
cause must be shown to be within the personal knowledge of the complainant or the witnesses
he may produce and not based on mere hearsay." 4. ID.; ID.; ID.; ID.; ID.; DUTY OF JUDGE TO
PERSONALLY EXAMINE THE APPLICANT AND THE WITNESSES; EFFECT OF FAILURE
TO COMPLY. — In issuing a search warrant, the judge must strictly comply with the
constitutional and statutory requirement that he must determine the existence of probable cause
by personally examining the applicant and his witnesses in the form of searching questions and
answers. His failure to comply with this requirement constitutes grave abuse of discretion. As
declared in M a r c elo v s . D e G u z m a n , G.R. No. L-29077, June 29, 1982, 114 SCRA 667,
"the capricious disregard by the judge in not complying with the requirements before issuance of
search warrants constitutes abuse of discretion". 5. ID.; ID.; ID.; ID.; SEIZURE OF PROPERTY
NOT SPECIFIED IN THE WARRANT; RULE. — The officers implementing the search warrant
clearly abused their authority when they seized the money of Antonieta Silva. This is highly
irregular considering that Antonieta Silva was not even named as one of the respondents, that
the warrant did not indicate the seizure of money but only of marijuana leaves, cigarettes and
joints, and that the search
CD Technologies Asia, Inc. © 2016 cdasiaonline.com
warrant was issued for the seizure of personal property (a) subject of the offense and (b) used
or intended to be used as means of committing an offense and NOT for personal property stolen
or embezzled or other proceeds of fruits of the offense.

FERNAN, C.J.:

In this special civil action for certiorari, petitioners seek the nullification of Search Warrant No. 1
issued by respondent Judge as well as the return of the money in the amount of P1,231.00
seized from petitioner Antonieta Silva.

The antecedent facts are as follows:

On June 13, 1986, M/Sgt. Ranulfo Villamor, Jr., as chief of the PC Narcom Detachment in
Dumaguete City, Negros Oriental, filed an "Application for Search Warrant" with the Regional
Trial Court, Branch XXXIII, Dumaguete City against petitioners Nicomedes Silva and Marlon
Silva. 1 This application was accompanied by a "Deposition of Witness" executed by Pfc. Arthur
M. Alcoran and Pat. Leon T. Quindo, also dated June 13, 1986. 2
On the same day. Judge Nickarter A. Ontal, then Presiding Judge of the Regional Trial Court, Branch XXXIII, Dumaguete City, pursuant to the said
"Application for Search Warrant" and "Deposition of Witness", issued Search Warrant No. 1, directing the aforesaid police officers to search the room of
Marlon Silva in the residence of Nicomedes Silva for violation of Republic Act No. 6425, otherwise known as the Dangerous Drugs Act of 1972. as
amended. Pertinent portions of Search Warrant No. 1 read as follows:

It appearing to the satisfaction of the undersigned after examining oath (sic) MSGT. Ranulfo T. Villamor, Jr. and his witnesses (sic) Pfc.
Arthur M. Alcoran and Pat. Leon T. Quindo that there is probable cause to believe that possession and control of Marijuana dried leaves,
cigarettes, joint has been committed or is about to be committed and that there are good and sufficient reasons to believe that marijuana
dried leaves, cigarettes, joint has in possession and/or control at Tama's Room (Rgt. side lst Floor) located at Nono-Limbaga Drive, Tanjay,
Neg. Or. which is/are:

X (Subject of the offense stated above

(Stolen or embezzled or other proceeds of fruits of the offense;

X (Used or intended to be used as means of committing an offense.

You are hereby commanded to make an immediate search at any time of the day (night) of the room of Tama Silva residence of his father
Comedes Silva to open (sic) aparadors, lockers, cabinets, cartoons, containers, forthwith seize and take possession of the following
property Marijuana dried leaves, cigarettes, joint and bring the said property to the undersigned to be dealt with as the law directs. 3

In the course of the search, the serving officers also seized money belonging to Antonieta Silva in the amount of P1,231.40.

On June 16, 1986, Antonieta Silva filed a motion for the return of the said amount on the grounds that the search warrant only authorized the serving
officers to seize marijuana dried leaves, cigarettes and joint, and that said officers failed or refused to make a return of the said search warrant in gross
violation of Section 11, Rule 126 of the Rules of Court. 4

Acting on said motion, Judge Ontal issued an Order dated July 1, 1986, stating that the court "holds in abeyance the disposition of the said amount of
P1,231.40 pending the filing of appropriate charges in connection with the search warrant." 5

On July 28, 1987, petitioners filed a motion to quash Search Warrant No. 1 on the grounds that (1) it was issued on the sole basis of a mimeographed
"Application for Search Warrant" and "Deposition of Witness", which were accomplished by merely filling in the blanks and (2) the judge failed to
personally examine the complainant and witnesses by searching questions and answers in violation of Section 3, Rule 126 of the Rules of Court. 6

On August 11, 1987, respondent trial court, through Judge Eugenio M. Cruz, who, by then, had replaced retired Judge Ontal, issued an Order denying
the motion for lack of merit, finding the requisites necessary for the issuance of a valid search warrant duly complied with. 7

A motion for reconsideration dated September 1, 1987 filed by petitioners was likewise denied by Judge Cruz in an order dated October 19, 1987.
Hence, this special civil action for certiorari.

Petitioners allege that the issuance of Search Warrant No. 1 was tainted with illegality and that respondent Judge should be viewed to have acted
without or in excess of jurisdiction, or committed grave abuse of discretion amounting to lack of jurisdiction when he issued the Order dated August 11,
1987, denying their motion to quash Search Warrant No, 1.

We rule for petitioners.

Section 2, Article III (Bill of Rights) of the 1987 Constitution guarantees the right to personal liberty and security of homes against unreasonable
searches and seizures. This section provides:

Sec. 2. The right of the people to be secure in their persons, houses, papers, and effects against unreasonable searches and seizures of
whatever nature and for any purpose shall be inviolable, and no search warrant or warrant of arrest shall issue except upon probable cause
to be determined personally by the judge after examination under oath or affirmation of the complainant and the witnesses he may produce,
and particularly describing the place to be searched and the persons or things to be seized.

The purpose of the constitutional provision against unlawful searches and seizures is to prevent violations of private security in person and property,
and unlawful invasion of the sanctity of the home, by officers of the law acting under legislative or judicial sanction, and to give remedy against such
usurpations when attempted. 8

Thus, Sections 3 and 4, Rule 126 of the Rules of Court provide for the requisites for the issuance of a search warrant, to wit:

SEC. 3. Requisite for issuing search warrant. — A search warrant shall not issue but upon probable cause in connection with one specific
offense to be determined personally by the judge after examination under oath or affirmation of the complainant and the witnesses he may
produce, and particularly describing the place to be searched and the things to be seized.

SEC. 4. Examination of complainant; record. — The judge must, before issuing the warrant, personally examine in the form of searching
questions and answers, in writing and under oath the complainant and any witnesses he may produce on facts personally known to them
and attach to the record their sworn statements together with any affidavits submitted.

Based on the aforecited constitutional and statutory provisions, the judge must, before issuing a search warrant, determine whether there is probable
cause by examining the complainant and witnesses through searching questions and answers.

In the case of Prudente vs. Dayrit, G.R. No. 82870, December 14, 1989, 180 SCRA 69, 767 this Court defined "probable cause" as follows:

The "probable cause" for a valid search warrant, has been defined "as such facts and circumstances which would lead a reasonably
discreet and prudent man to believe that an offense has been committed, and that objects sought in connection with the offense are in the
place sought to be searched". This probable cause must be shown to be within the personal knowledge of the complainant or the witnesses
he may produce and not based on mere hearsay.

In the case at bar, we have carefully examined the questioned search warrant as well as the "Application for Search Warrant" and "Deposition of
Witness", and found that Judge Ontal failed to comply with the legal requirement that he must examine the applicant and his witnesses in the form of
searching questions and answers in order to determine the existence of probable cause. The joint "Deposition of Witness" executed by Pfc. Alcoran
and Pat. Quindo, which was submitted together with the "Application for Search Warrant" contained, for the most part suggestive questions answerable
by merely placing "yes" or "no" in the blanks provided thereon. In fact there were only four (4) questions asked, to wit:
Q Do you personally know M/Sgt. Ranulfo Villamor, Jr. the applicant for a search warrant?

A Yes, sir.

Q Do you have personal knowledge that the said premises subject of the offense stated above, and other proceeds of fruit of the offense,
used or obtain (sic) or intended to be used as means of committing an offense?

A Yes, sir.

Q Do you know personally who is/are the person who has/have the property in his/their possession and control?

A Yes, sir.

Q How did you know all this (sic) things?

A Through discreet surveillance. 9

The above deposition did not only contain leading questions but it was also very broad. The questions propounded to the witnesses were in fact, not
probing but were merely routinary. The deposition was already mimeogragphed and all that the witnesses had to do was fill in their answers on the
blanks provided.

In the case of Nolasco vs. Paño, G.R. No. 69803, October 8, 1985, 139 SCRA 152, 163, this Court held:

The "probable cause" required to justify the issuance of a search warrant comprehends such facts and circumstances as will induce a
cautious man to rely upon them and act in pursuant thereof. Of the 8 questions asked, the 1st, 2nd and 4th pertain to identity. The 3rd and
5th are leading not searching questions. The 6th, 7th and 8th refer to the description of the personalities to be seized, which is identical to
that in the Search Warrant and suffers from the same lack of particularity. The examination conducted was general in nature and merely
repetitious of the deposition of said witness. Mere generalization will not suffice and does not satisfy the requirements or probable cause
upon which a warrant may issue.

Likewise, in the Prudente case cited earlier, this Court declared the search warrant issued as invalid due to the failure of the judge to examine the
witness in the form of searching questions and answers. Pertinent portion of the decision reads:

Moreover, a perusal of the deposition of P/Lt. Florencio Angeles shows that it was too brief and short. Respondent Judge did not examine
him "in the form of searching questions and answers". On the contrary, the questions asked were leading as they called for a simple "yes"
or "no" answer. As held in Quintero vs. NBI, "the questions propounded by respondent Executive Judge to the applicant's witness' are not
sufficiently searching to establish probable cause. Asking of leading questions to the deponent in an application for search warrant, and
conducting of examination in a general manner, would not satisfy the requirements for issuance of a valid search warrant. 10
Thus, in issuing a search warrant, the judge must strictly comply with the constitutional and statutory requirement that he must determine the existence
of probable cause by personally examining the applicant and his witnesses in the form of searching questions and answers. His failure to comply with
this requirement constitutes grave abuse of discretion. As declared in Marcelo vs. De Guzman, G.R. No. L-29077, June 29, 1982, 114 SCRA 657, "the
capricious disregard by the judge in not complying with the requirements before issuance of search warrants constitutes abuse of discretion".

The officers implementing the search warrant clearly abused their authority when they seized the money of Antonieta Silva. This is highly irregular
considering that Antonieta Silva was not even named as one of the respondents, that the warrant did not indicate the seizure of money but only of
marijuana leaves, cigarettes and joints, and that the search warrant was issued for the seizure of personal property (a) subject of the offense and (b)
used or intended to be used as means of committing an offense and NOT for personal property stolen or embezzled or other proceeds of fruits of the
offense. Thus, the then presiding Judge Ontal likewise abused his discretion when he rejected the motion of petitioner Antonieta Silva seeking the
return of her seized money.

WHEREFORE, the petition is granted. Search Warrant No. 1 is hereby declared null and void. Respondent Judge of the Regional Trial Court of Negros
Oriental, Branch XXXIII is directed to order the return to petitioner Antonieta Silva of the amount of P1,231.40 which had earlier been seized from her
by virtue of the illegal search warrant. This decision is immediately executory. No costs.

SO ORDERED.

[G.R. No. L-22196. June 30, 1967.]


ESTEBAN MORANO, CHAN SAU WAH and FU YAN FUN ESTEBAN MORANO, CHAN SAU
WAH and FU YAN FUN, p e titio n e r s a p p ella n t s ,
v s . v s . HON. MARTINIANO VIVO, in his capacity as Acting HON. MARTINIANO VIVO, in his
capacity as Acting Commissioner of Immigration Commissioner of Immigration, r e s p o n d e n
t- a p p ella n t .
E n g r a cio F a b r e L a w O f fic e for petitioners-appellants. S olicit o r G e n e r al A r t u r o A
. Ala f riz, and S olicit o r A . M . A m o r e s for respondent-appellant.
SYLLABUS SYLLABUS
1. CITIZENSHIP; MARRIAGE OF AN ALIEN WOMAN TO A FILIPINO CITIZEN; EFFECT
THEREOF; CASE AT BAR. — In a series of cases, this Court has declared that the marriage of
an alien woman to a Filipino citizen does not ip s o f a c t o make her a Filipino citizen. She must
satisfactorily show that she has all the qualifications and none of the disqualifications required
by the Naturalization Law. Considering that in the additional stipulation of facts of July 3, 1963,
petitioners admit that Chan-Sau Wah is not possessed of all the qualifications required by the
Naturalization Law there can be no doubt that petitioner Chan Sau Wah did not become a
Filipino citizen. 2. DEPORTATION PROCEEDINGS; SECTION I(3) ARTICLE III OF THE
CONSTITUTION NOT APPLICABLE. — The constitutional guarantee set forth in Section 1(3),
Article III of the Constitution, requiring that the issue of probable cause be determined by a
judge, does not extend to deportation proceedings (Tu Chuan Hai v s . Commissioner of
Immigration, 55 Off. Gaz., No. 28, pp. 681-683). 3. ID.; RULE AS TO TEMPORARY VISITORS
UPON EXPIRATION OF PERIOD OF STAY. — The law is to the effect that temporary visitors
who do not depart upon the expiration of the period of stay granted' them are subject to
deportation by the Commissioner of Immigration, for having violated the limitation or condition
under which they were admitted as non-immigrants (Immigration Law, Sec. 37 (a), subsection
(7); C.A. 613, as amended. 4. ID.; ID.; CHANGE OF STATUS; REQUISITES THEREFOR. —
This Court in a number of cases has ruled, and consistently too, that an alien admitted as a
temporary visitor cannot change his or her status without first departing from the country and
complying with the requirements of Section 9 of the Immigration Act. Thus, the marriage of a
temporary alien visitor to a Filipino does not ip s o f a c t o make her a permanent resident in
this country. 5. ID.; CITIZENSHIP; MEANING OF "CHILD" IN SECTION 15 PARAGRAPH 3,
COMMONWEALTH ACT 473. — The word c hild we are certain means legitimate child, not a
step-child. Thus, when the Constitution provides that "those whose fathers are citizens of the
Philippines are citizens thereof" the fundamental charter intends "those" to apply to legitimate
children. In another case, the term "minor children" or "minor child" in Section 15 of the Revised
Naturalization Law was interpreted to refer only to legitimate children of Filipino citizens.
CD Technologies Asia, Inc. © 2016 cdasiaonline.com
6. ID.; BONDS; RULE REQUIRING OFFICIAL APPROVAL THEREOF MERELY DIRECTORY.
— The provision requiring official approval of a bond is merely directory. "Irregularity or entire
failure in this respect does not affect the validity of the bond." (9 C.J., p. 25). 7. ID.; ID.; ID.;
ESTOPPEL ON EQUITABLE CONSIDERATIONS. — Equitable consideration will stop
petitioners from pleading invalidity of the bond. They offered that bond to enable them to enter
and stay in this country. They enjoyed benefits therefrom. They cannot "in law and good
conscience be allowed to reap the fruits" of that bond, and then jettison the same. They are
"precluded from attacking the validity" of such bond.

G.R. No. L-22196 June 30, 1967

ESTEBAN MORANO, CHAN SAU WAH and FU YAN FUN, petitioners-appellants,


vs.
HON. MARTINIANO VIVO in his capacity as Acting Commissioner of
Immigration, respondent-appellant.

Engracio Fabre Law Office for petitioners-appellants.


Office of the Solicitor General Arturo A. Alafriz and Solicitor A. M. Amores for respondent-
appellant.

SANCHEZ, J.:

Chan Sau Wah, a Chinese citizen born in Fukien, China on January 6, 1932, arrived in the
Philippines on November 23, 1961 to visit her cousin, Samuel Lee Malaps. She left in mainland
China two of her children by a first marriage: Fu Tse Haw and Fu Yan Kai With her was Fu Yan
Fun, her minor son also by the first marriage, born in Hongkong on September 11, 1957.

Chan Sau Wah and her minor son Fu Yan Fun were permitted only into the Philippines under a
temporary visitor's visa for two (2) months and after they posted a cash bond of P4,000.00.

On January 24, 1962, Chan Sau Wah married Esteban Morano, a native-born Filipino citizen.
Born to this union on September 16, 1962 was Esteban Morano, Jr.

To prolong their stay in the Philippines, Chan Sau Wah and Fu Yan Fun obtained several
extensions. The last extension expired on September 10, 1962.1äwphï1.ñët
In a letter dated August 31, 1962, the Commissioner of Immigration ordered Chan Sau Wah and
her son, Fu Yan Fun, to leave the country on or before September 10, 1962 with a warning that
upon failure so to do, he will issue a warrant for their arrest and will cause the confiscation of
their bond.

Instead of leaving the country, on September 10, 1962, Chan Sau Wah (with her husband
Esteban Morano) and Fu Yan Fun petitioned the Court of First Instance of Manila
for mandamus to compel the Commissioner of Immigration to cancel petitioners' Alien
Certificates of Registration; prohibition to stop the Commissioner from issuing a warrant for their
arrest, and preliminary injunction to restrain the Commissioner from confiscating their cash bond
and from issuing warrants of arrest pending resolution of this case.1 The trial court, on
November 3, 1962, issued the writ of preliminary injunction prayed for, upon a P2,000-bond.
After trial and the stipulations of facts filed by the parties, the Court of First Instance rendered
judgment, viz:

IN VIEW OF ALL THE FOREGOING, judgment is hereby rendered as follows:

(a) Granting this petition for Mandamus and Prohibition with respect to petitioner CHAN
SAU WAH, who is hereby declared a citizen of the Philippines; ordering the respondent
to cancel her Alien Certificate of Registration and other immigration papers, upon the
payment of proper dues; and declaring the preliminary injunction with respect to her
permanent, prohibiting the respondent, his representatives or subordinates from
arresting and/or deporting said petitioner;

(b) Dismissing this petition with respect to petitioner FU YAN FUN, and dissolving the
writ of preliminary injunction issued herein, restraining the respondent, his
representatives or subordinates from arresting and/or deporting said petitioner;

(c) Authorizing respondent Commissioner to forfeit the bond filed by herein petitioners
CHAN SAU WAH and FU YAN FUN in the amount of P4,000.00; and

(d) Denying, for lack of merit, the prayer to declare Sec. 37 (a) of the Philippine
Immigration Act of 1940 unconstitutional;

Without pronouncement, as to costs.

Petitioners and respondent Commissioner both appealed.

We will deal with the claims of both appellants in their proper sequence.

1. The Solicitor General's brief assails the trial court's declaration that Chan Sau Wah is a
citizen of the Philippines. The court a quo took the position that "Chan Sau Wah became, by
virtue of, and upon, her marriage to Esteban Morano, a natural-born Filipino, a Filipino citizen.2

Placed to the fore is paragraph 1, Section 15 of Commonwealth Act 473 [Revised Naturalization
Act], which reads:
Sec. 15. Effect of the naturalization on wife children. — Any woman who is now or may
hereafter be married to a citizen of the Philippines, and who might herself be lawfully
naturalized shall be deemed a citizen of the Philippines.

To apply this provision, two requisites must concur: (a) valid marriage of an alien woman to a
citizen of the Philippines and (b) the alien woman herself might be lawfully naturalized.

We may concede that the first requisite has been properly met. The validity of the marriage is
presumed.

But can the same be said of the second requisite? This question by all means is not new. In a
series of cases, this Court has declared that the marriage of an alien woman to a Filipino citizen
does not ipso facto make her a Filipino citizen. She must satisfactorily show that she has all the
qualifications and none of the disqualifications required by the Naturalization Law.3 Ly Giok Ha
alias Wy Giok Ha et al. vs. Emilio Galang, L-21332, March 18, 1966,* clearly writes down the
philosophy behind the rule in the following expressive language, viz:

Reflection will reveal why this must be so. The qualifications prescribed under section 2
of the Naturalization Act, and the disqualifications enumerated in its section 4, are not
mutually exclusive; and if all that were to be required is that the wife of a Filipino be not
disqualified under section 4, the result might well be that citizenship would be conferred
upon persons in violation of the policy of the statute. For example, section 4 disqualifies
only —

"(c) Polygamists or believers in the practice of polygamy; and

(b) Persons convicted of crimes involving moral turpitude,"

so that a blackmailer, or a maintainer of gambling or bawdy houses, not previously


convicted by a competent court, would not be thereby disqualified; still it is certain that
the law did not intend such a person to, be admitted as a citizen in view of the
requirement of section 2 that an applicant for citizenship "must be of good moral
character."

Similarly, the citizen's wife might be a convinced believer in racial supremacy, in


government by certain selected classes, in the right to vote exclusively by certain
"herrenvolk," and thus disbelieve in the principles underlying the Philippine Constitution;
yet she would not be disqualified under section 4, as long as she is not "opposed to
organized government," nor affiliated to groups "upholding or teaching doctrines
opposing all organized governments," nor "defending or teaching the necessity or
propriety of violence, personal assault or assassination for the success or predominance
of their ideas." Et sic de caeteris.

Upon the principle of selective citizenship, we cannot afford to depart from the wise precept
affirmed and reaffirmed in the cases heretofore noted.

In the additional stipulation of facts of July 3, 1963, petitioners admit that Chan Sau Wah is not
possessed of all the qualifications required by the Naturalization Law.
Because of all these we are left under no doubt that petitioner Chan Sau Wah did not become a
Filipino citizen.

2. Squarely put in issue by petitioners is the constitutionality of Section 37 (a) of the Immigration
Act of 1940, which reads:

Sec. 37. (a) The following aliens shall be arrested upon the warrant of the Commissioner
of Immigration or of any other officer designated by him for the purpose and deported
upon the warrant of the Commissioner of Immigration after a determination by the Board
of Commissioners of the existence of the ground for deportation as charged against the
alien:

xxx xxx xxx

(7) Any alien who remains in the Philippines in violation of any limitation or condition
under which he was admitted as a nonimmigrant.

Petitioners argue that the legal precept just quoted trenches upon the constitutional mandate in
Section 1 (3), Article III [Bill of Rights] of the Constitution, to wit:

(3) The right of the people to be secure in their persons, houses, papers, and effects
against unreasonable searches and seizures shall not be violated, and no warrants shall
issue but upon probable cause, to be determined by the judge after examination under
oath or affirmation of the complainant and the witnesses he may produce, and
particularly describing the place to be searched, and the persons or things to be seized.

They say that the Constitution limits to judges the authority to issue warrants of arrest and that
the legislative delegation of such power to the Commissioner of Immigration is thus violative of
the Bill of Rights.

Section 1 (3), Article III of the Constitution, we perceive, does not require judicial intervention in
the execution of a final order of deportation issued in accordance with law. The constitutional
limitation contemplates an order of arrest in the exercise of judicial power4 as a step preliminary
or incidental to prosecution or proceedings for a given offense or administrative action, not as a
measure indispensable to carry out a valid decision by a competent official, such as a legal
order of deportation, issued by the Commissioner of Immigration, in pursuance of a valid
legislation.

The following from American Jurisprudence,5 is illuminating:

It is thoroughly established that Congress has power to order the deportation of aliens
whose presence in the country it deems hurtful. Owing to the nature of the proceeding,
the deportation of an alien who is found in this country in violation of law is not a
deprivation of liberty without due process of law. This is so, although the inquiry
devolves upon executive officers, and their findings of fact, after a fair though summary
hearing, are made conclusive.

xxx xxx xxx


The determination of the propriety of deportation is not a prosecution for, or a conviction
of, crime; nor is the deportation a punishment, even though the facts underlying the
decision may constitute a crime under local law. The proceeding is in effect simply a
refusal by the government to harbor persons whom it does not want. The coincidence of
local penal law with the policy of Congress is purely accidental, and, though supported
by the same facts, a criminal prosecution and a proceeding for deportation are separate
and independent.

In consequence, the constitutional guarantee set forth in Section 1 (3), Article III of the
Constitution aforesaid, requiring that the issue of probable cause be determined by a judge,
does not extend to deportation proceedings.6

The view we here express finds support in the discussions during the constitutional convention.
The convention recognized, as sanctioned by due process, possibilities and cases of
deprivation of liberty, other than by order of a competent court.7

Indeed, the power to deport or expel aliens is an attribute of sovereignty. Such power is planted
on the "accepted maxim of international law, that every sovereign nation has the power, as
inherent in sovereignty, and essential to self-preservation, to forbid the entrance of foreigners
within its dominions."8 So it is, that this Court once aptly remarked that there can be no
controversy on the fact that where aliens are admitted as temporary visitors, "the law is to the
effect that temporary visitors who do not depart upon the expiration of the period of stay granted
them are subject to deportation by the Commissioner of Immigration, for having violated the
limitation or condition under which they were admitted as non-immigrants (Immigration Law,
Sec. 37 (a), subsection (7); C.A. 613, as amended)."9

And, in a case directly in point, where the power of the Commissioner to issue warrants of
arrest was challenged as unconstitutional, because "such power is only vested in a judge by
Section 1, paragraph 3, Article III of our Constitution," this Court declared —

This argument overlooks the fact that the stay of appellant Ng Hua To as temporary
visitor is subject to certain contractual stipulations as contained in the cash bond put up
by him, among them, that in case of breach the Commissioner may require the
recommitment of the person in whose favor the bond has been filed. The Commissioner
did nothing but to enforce such condition. Such a step is necessary to enable the
Commissioner to prepare the ground for his deportation under section 37 (a) of
Commonwealth Act 613. A contrary interpretation would render such power nugatory to
the detriment of the State.10

It is in this context that we rule that Section 37 (a) of the Immigration Act of 1940 is not
constitutionally proscribed.

3. A sequel to the questions just discussed is the second error set forth in the government's
brief. The Solicitor General balks at the lower court's ruling that petitioner Chan Sau Wah is
entitled to permanent residence in the Philippines without first complying with the requirements
of Sections 9 and 13 of the Immigration Act of 1940, as amended by Republic Act 503.

We first go to the law, viz:

SEC. 9 [last paragraph]


An alien who is admitted as a nonimmigrant cannot remain in the Philippines
permanently. To obtain permanent admission, a nonimmigrant alien must depart
voluntarily to some foreign country and procure from the appropriate Philippine consul
the proper visa and thereafter undergo examination by the officers of the Bureau of
Immigration at a Philippine port of entry for determination of his admissibility in
accordance with the requirements of this Act.

SEC. 13. Under the conditions set forth in this Act there may be admitted into the
Philippines immigrants, termed "quota immigrants" not in excess of fifty (50) of any one
nationality or without nationality for any one calendar year, except that the following
immigrants, termed "nonquota immigrants," maybe admitted without regard to such
numerical limitations.

The corresponding Philippine Consular representative abroad shall investigate and


certify the eligibility of a quota immigrant previous to his admission into the Philippines.
Qualified and desirable aliens who are in the Philippines under temporary stay may be
admitted within the quota, subject to the provisions of the last paragraph of section 9 of
this Act.

(a) The wife or the husband or the unmarried child under twenty-one years of age of a
Philippine citizen, if accompanying or following to join such citizen;

(b) A child of alien parents born during the temporary visit abroad of the mother, the
mother having been previously lawfully admitted into the Philippine for permanent
residence, if the child is accompanying or coming to join a parent and applies for
admission within five years from the date of its birth;

Concededly, Chan Sau Wah entered the Philippines on a tourist-temporary visitor's visa. She is
a non-immigrant. Under Section 13 just quoted, she may therefore be admitted if she were a
qualified and desirable alien and subject to the provisions of the last paragraph of Section 9.
Therefore, first, she must depart voluntarily to some foreign country; second, she must procure
from the appropriate consul the proper visa; and third, she must thereafter undergo examination
by the officials of the Bureau of Immigration at the port of entry for determination of her
admissibility in accordance with the requirements of the immigration Act.

This Court in a number of cases has ruled, and consistently too, that an alien admitted as a
temporary visitor cannot change his or her status without first departing from the country and
complying with the requirements of Section 9 of the Immigration Act. 11

The gravamen of petitioners' argument is that Chan Sau Wah has, since her entry, married in
Manila a native-born Filipino, Esteban Morano. It will not particularly help analysis for petitioners
to appeal to family solidarity in an effort to thwart her deportation. Chan Sau Wah, seemingly is
not one who has a high regard for such solidarity. Proof: She left two of her children by the first
marriage, both minors, in the care of neighbors in Fukien, China.

Then, the wording of the statute heretofore adverted to is a forbidding obstacle which will
prevent this Court from writing into the law an additional provision that marriage of a temporary
alien visitor to a Filipino would ipso facto make her a permanent resident in his country. This is a
field closed to judicial action. No breadth of discretion is allowed us. We cannot insulate her
from the State's power of deportation.
Really, it would be an easy matter for an alien woman to enter the Philippines as a temporary
visitor, go through a mock marriage, but actually live with another man as husband and wife,
and thereby skirt the provisions of our immigration law. Also, a woman of undesirable character
may enter this country, ply a pernicious trade, marry a Filipino, and again throw overboard
Sections 9 and 13 of the Act. Such a flanking movement, we are confident, is impermissible.

Recently we confirmed the rule that an alien wife of a Filipino may not stay permanently without
first departing from the Philippines. Reason: Discourage entry under false pretenses. 12

The ruling of the trial court on this score should be reversed.

4. It is petitioners' turn to point as error the dismissal of the petition for mandamus and
prohibition with respect to petitioner Fu Yan Fun.

Petitioners' line of thought is this: Fu Yan Fun follows the citizenship of his mother. They cite
Section 15, paragraph 3, Commonwealth Act 473, which says that:

A foreign-born minor child, if dwelling in the Philippines at the time of the naturalization
of the parent, shall automatically become a Philippine citizen. . . .

Petitioners' position is based on the assumption that Chan Sau Wah, the mother, is a Filipino
citizen. We have held that she is not. At best, Fu Yan Fun is a step-son of Esteban Morano,
husband of Chan Sau Wah. A step-son is not a foreign-born child of the step-father.
The word child, we are certain, means legitimate child, not a step-child. We are not wanting in
precedents. Thus, when the Constitution provides that "[t]hose whose fathers are citizens of the
Philippines" are citizens thereof, 13 the fundamental charter intends "those" to apply to legitimate
children. 14 In another case, the term "minor children" or "minor child" in Section 15 of the
Revised Naturalization Law refers only to legitimate children of Filipino citizens. This Court, thru
Mr. Chief Justice Roberto Concepcion, there said: 15

It is claimed that the phrases "minor children" and "minor child," used in these
provisions, include adopted children. The argument is predicated upon the theory that an
adopted child is, for all intents and purposes, a legitimate child. Whenever, the word
"children" or "child" is used in statutes, it is generally understood, however, to refer to
legitimate children, unless the context of the law and its spirit indicate clearly the
contrary. Thus, for instance, when the Constitution provides that "those whose fathers
are citizens of the Philippines," and "those whose mothers are citizens of the Philippines"
who shall elect Philippine citizenship upon reaching the age of majority, are citizens of
the Philippines (Article IV, Section 1, subdivisions [3] and [4]), our fundamental law
clearly refers to legitimate children (Chiongbian vs. De Leon, 46 Off. Gaz., 3652-3654;
Serra v. Republic, L-4223, May 12, 1952).

At any rate, Fu Yan Fun entered the Philippines as a temporary visitor. The status of a
temporary visitor cannot be converted into, that of a permanent resident, as we have heretofore
held, without first complying with Section 9 of the Immigration Law.

5. Petitioners finally aver that the lower court erred in authorizing respondent Commissioner to
forfeit the bond filed by petitioners Chan Sau Wah and Fu Yan Fun in the amount of P4,000.00.
Here is petitioners' posture. They enjoyed their stay in the Philippines upon a bond. Now they
come to court and say that as the prescribed form of this bond was not expressly approved by
the Secretary of Justice in accordance with Section 3 of Commonwealth Act 613, which reads

SEC. 3. . . . He [Commissioner of Immigration] shall issue, subject to the approval of the


Department Head, such rules and regulations and prescribes such forms of bond,
reports, and other papers, and shall issue from time to time such instruction, not
inconsistent with law, as he shall deem best calculated to carry out the provisions of the
immigration laws. . . .

that bond is void.

Reasons there are which prevent us from giving our imprimatur to this argument.

The provision requiring official approval of a bond is merely directory. "Irregularity or entire
failure in this respect does not affect the validity of the bond. 16 The reason for the rule, is found
in 9 C.J., p. 26 (footnote), which reads:

(a) Reason for rule. — "Statutes requiring bonds to be approved by certain officials are not for
the purpose of protecting the obligors in the bond, but are aimed to protect the public, to insure
their solvency, and to create evidence of an unimpeachable character of the fact of their
execution. When they are executed for a legal purpose, before a proper tribunal, and are in fact
accepted and approved by the officer or body, whose duty it was to approve them, it could serve
no useful purpose of the law to hold them invalid, to release all the obligors thereon, and to
defeat every purpose of its execution, simply because the fact of approval was not indorsed
precisely as had been directed by the Legislature." American Book Co. vs. Wells, 83 SW 622,
627, 26 Ky L-1159. (emphasis supplied)

And another. This bond was accepted by the government. It had been there. The form of the
bond here used is of long continued usage. If the government did not question the form of the
bond at all, then we must assume that it counted with the Secretary's approval. For the
presumption is that official duty has been legally performed.

Surely enough, equitable considerations will stop petitioners from pleading invalidity of the bond.
They offered that bond to enable them to enter and stay in this country. They enjoyed benefits
therefrom. They cannot, "in law, and good conscience, be allowed to reap the fruits" of that
bond, and then jettison the same. They are "precluded from attacking the validity" of such
bond. 17

Actually, to petitioners the bond was good while they sought entry into the Philippines; they
offered it as security for the undertaking; that they "will actually depart from the Philippines"
when their term of stay expires. Now that the bond is being confiscated because they
overstayed, they make an about-face and say that such bond is null and void. They shall not
profit from this inconsistent position. Their bond should be confiscated.

Conformably to the foregoing, the judgment under review is hereby modified as follows:

(1) The portion thereof which reads:


(a) Granting their petition for Mandamus and Prohibition with respect to petitioner CHAN
SAU WAH, who is hereby declared a citizen of the Philippines; ordering the respondent
to cancel her Alien Certificate of Registration and other immigration papers, upon the
payment of proper dues; and declaring preliminary injunction with respect to her
permanent, prohibiting the respondent, his representatives or subordinates from
arresting and/or deporting said petitioner;

is hereby reversed: and, in consequence —

The petition for mandamus and prohibition with respect to petitioner Chan Sau Wah is hereby
denied; and the judgment declaring her a citizen of the Philippines, directing respondent to
cancel her Alien Certificate of Registration and other immigration papers, and declaring the
preliminary injunction with respect to her permanent, are all hereby set aside; and

(2) In all other respects, the decision appealed from is hereby affirmed.

No costs. So ordered.

G.R. No. 82544 June 28, 1988

IN THE MATTER OF THE PETITION FOR HABEAS CORPUS OF: ANDREW HARVEY,
JOHN SHERMAN and ADRIAAN VAN DEL ELSHOUT, petitioners,
vs.
HONORABLE COMMISSIONER MIRIAM DEFENSOR SANTIAGO, COMMISSION ON
IMMIGRATION AND DEPORTATION, respondent.

MELENCIO-HERRERA, J.:

A petition for Habeas Corpus.

Petitioners Andrew Harvey and John Sherman, 52 and 72 years, respectively, are both
American nationals residing at Pagsanjan, Laguna, while Adriaan Van Elshout, 58 years old, is
a Dutch citizen also residing at Pagsanjan, Laguna.

The case stems from the apprehension of petitioners on 27 February 1988 from their respective
residences by agents of the Commission on Immigration and Deportation (CID) by virtue of
Mission Orders issued by respondent Commissioner Miriam Defensor Santiago of the CID.
Petitioners are presently detained at the CID Detention Center.

Petitioners were among the twenty-two (22) suspected alien pedophiles who were apprehended
after three months of close surveillance by CID agents in Pagsanjan, Laguna. Two (2) days
after apprehension, or on 29 February 1988, seventeen (17) of the twenty-two (22) arrested
aliens opted for self-deportation and have left the country. One was released for lack of
evidence; another was charged not for being a pedophile but for working without a valid working
visa. Thus, of the original twenty two (22), only the three petitioners have chosen to face
deportation.
Seized during petitioners apprehension were rolls of photo negatives and photos of the
suspected child prostitutes shown in salacious poses as well as boys and girls engaged in the
sex act. There were also posters and other literature advertising the child prostitutes.

The "Operation Report," on Andrew Harvey and Richard Sherman dated 29 February 1988
stated:

xxx xxx xxx

ANDREW MARK HARVEY was found together with two young boys.

RICHARD SHERMAN was found with two naked boys inside his room.

In respect of Van Den Elshout the "After Mission Report," dated 27 February 1988 read in part:

Noted:

There were two (2) children ages 14 & 16 which subject readily
accepted having been in his care and live-in for quite sometime.

On 4 March 1988, deportation proceedings were instituted against petitioners for being
undesirable aliens under Section 69 of the Revised Administrative Code (Deportation Case No.
88-13). The "Charge Sheet" read inter alia:

Wherefore, this Office charges the respondents for deportation, as undesirable


aliens, in that: they, being pedophiles, are inimical to public morals, public health
and public safety as provided in Section 69 of the Revised Administrative Code.

On 7 March 1988, Warrants of Arrest were issued by respondent against petitioners for violation
of Sections 37, 45 and 46 of the Immigration Act and Section 69 of the Revised Administrative
Code On the same date, the Board of Special Inquiry III commenced trial against petitioners.

On 14 March 1988, petitioners filed an Urgent Petition for Release Under Bond alleging that
their health was being seriously affected by their continuous detention. Upon recommendation
of the Board of Commissioners for their provisional release, respondent ordered the CID doctor
to examine petitioners, who certified that petitioners were healthy.

On 22 March 1988, petitioners filed a Petition for Bail which, however, respondent denied
considering the certification by the CID physician that petitioners were healthy. To avoid
congestion, respondent ordered petitioners' transfer to the CID detention cell at Fort Bonifacio,
but the transfer was deferred pending trial due to the difficulty of transporting them to and from
the CID where trial was on-going.

On 4 April 1988 petitioner Andrew Harvey filed a Manifestation/Motion stating that he had
"finally agreed to a self-deportation" and praying that he be "provisionally released for at least
15 days and placed under the custody of Atty. Asinas before he voluntarily departs the country."
On 7 April 1988, the Board of Special Inquiry — III allowed provisional release of five (5) days
only under certain conditions. However, it appears that on the same date that the aforesaid
Manifestation/ Motion was filed, Harvey and his co-petitioners had already filed the present
petition.

On 4 April 1988, as heretofore stated, petitioners availed of this Petition for a Writ of Habeas
Corpus. A Return of the Writ was filed by the Solicitor General and the Court heard the case on
oral argument on 20 April 1988. A Traverse to the Writ was presented by petitioners to which a
Reply was filed by the Solicitor General.

Petitioners question the validity of their detention on the following grounds:

1) There is no provision in the Philippine Immigration Act of 1940 nor under Section 69 of the
Revised Administrative Code, which legally clothes the Commissioner with any authority to
arrest and detain petitioners pending determination of the existence of a probable cause leading
to an administrative investigation.

2) Respondent violated Section 2, Article III of the 1987 Constitution prohibiting unreasonable
searches and seizures since the CID agents were not clothed with valid Warrants of arrest,
search and seizure as required by the said provision.

3) Mere confidential information made to the CID agents and their suspicion of the activities of
petitioners that they are pedophiles, coupled with their association with other suspected
pedophiles, are not valid legal grounds for their arrest and detention unless they are caught in
the act. They further allege that being a pedophile is not punishable by any Philippine Law nor is
it a crime to be a pedophile.

We reject petitioners' contentions and uphold respondent's official acts ably defended by the
Solicitor General.

There can be no question that the right against unreasonable searches and seizures
guaranteed by Article III, Section 2 of the 1987 Constitution, is available to all persons, including
aliens, whether accused of crime or not (Moncado vs. People's Court, 80 Phil. 1 [1948]. One of
the constitutional requirements of a valid search warrant or warrant of arrest is that it must be
based upon probable cause. Probable cause has been defined as referring to "such facts and
circumstances antecedent to the issuance of the warrant that in themselves are sufficient to
induce a cautious man to rely on them and act in pursuance thereof." (People vs. Syjuco 64
Phil. 667 [1937]; Alverez vs. CFI, 64 Phil. 33 [1937]).

The 1985 Rules on Criminal Procedure also provide that an arrest wit a warrant may be effected
by a peace officer or even a private person (1) when such person has committed, actually
committing, or is attempting to commit an offense in his presence; and (2) when an offense has,
in fact, been committed and he has personal knowledge of facts indicating that the person to be
arrested has committed it (Rule 113, Section 5).

In this case, the arrest of petitioners was based on probable cause determined after close
surveillance for three (3) months during which period their activities were monitored. The
existence of probable cause justified the arrest and the seizure of the photo negatives,
photographs and posters without warrant (See Papa vs. Mago, L-27360, February 28, 1968,22
SCRA 857; People vs. Court of First Instance of Rizal, L-41686, November 17, 1980, 101 SCRA
86, cited in CRUZ, Constitutional Law, 1987 ed., p. 143). Those articles were seized as an
incident to a lawful arrest and, are therefore, admissible in evidence (Section 12, Rule 126,1985
Rules on criminal Procedure).

But even assuming arguendo that the arrest of petitioners was not valid at its inception, the
records show that formal deportation charges have been filed against them, as undesirable
aliens, on 4 March 1988. Warrants of arrest were issued against them on 7 March 1988 "for
violation of Section 37, 45 and 46 of the Immigration Act and Section 69 of the Administrative
Code." A hearing is presently being conducted by a Board of Special Inquiry. The restraint
against their persons, therefore, has become legal. The Writ has served its purpose. The
process of the law is being followed (Cruz vs. Montoya, L-39823, February 25, 1975, 62 SCRA
543). "were a person's detention was later made by virtue of a judicial order in relation to
criminal cases subsequently filed against the detainee, his petition for hebeas corpus becomes
moot and academic" (Beltran vs. Garcia, L-49014, April 30, 1979, 89 SCRA 717). "It is a
fumdamental rule that a writ of habeas corpus will not be granted when the confinement is or
has become legal, although such confinement was illegal at the beginning" (Matsura vs. Director
of Prisons, 77 Phil. 1050 [1947]).

That petitioners were not "caught in the act" does not make their arrest illegal. Petitioners were
found with young boys in their respective rooms, the ones with John Sherman being naked.
Under those circumstances the CID agents had reasonable grounds to believe that petitioners
had committed "pedophilia" defined as "psychosexual perversion involving children" (Kraft-
Ebbing Psychopatia Sexualis p. 555; Paraphilia (or unusual sexual activity) in which children are
the preferred sexual object" (Webster's Third New International Dictionary, 1971 ed., p. 1665)
[Solicitor General's Return of the Writ, on p. 101. While not a crime under the Revised Penal
Code, it is behavior offensive to public morals and violative of the declared policy of the State to
promote and protect the physical, moral, spiritual, and social well-being of our youth (Article II,
Section 13, 1987 Constitution).

At any rate, the filing by petitioners of a petition to be released on bail should be considered as
a waiver of any irregularity attending their arrest and estops them from questioning its validity
(Callanta v. Villanueva, L-24646 & L-24674, June 20, 1977, 77 SCRA 377; Bagcal vs. Villaraza,
L-61770, January 31, 1983, 120 SCRA 525).

The deportation charges instituted by respondent Commissioner are in accordance with Section
37(a) of the Philippine Immigration Act of 1940, in relation to Section 69 of the Revised
Administrative Code. Section 37(a) provides in part:

(a) The following aliens shall be arrested upon the warrant of the Commissioner
of Immigration and Deportation or any other officer designated by him for the
purpose and deported upon the warrant of the Commissioner of Immigration and
Deportation after a determination by the Board of Commissioners of the
existence of the ground for deportation as charged against the alien;

xxx xxx xxx

The foregoing provision should be construed in its entirety in view of the summary and
indivisible nature of a deportation proceeding, otherwise, the very purpose of deportation
proceeding would be defeated.
Section 37(a) is not constitutionally proscribed (Morano vs. Vivo, L-22196, June 30, 1967, 20
SCRA 562). The specific constraints in both the 1935 1 and 1987 2 Constitutions, which are
substantially Identical, contemplate prosecutions essentially criminal in nature. Deportation
proceedings, on the other hand, are administrative in character. An order of deportation is never
construed as a punishment. It is preventive, not a penal process. It need not be conducted
strictly in accordance with ordinary Court proceedings.

It is of course well-settled that deportation proceedings do not constitute a


criminal action. The order of deportation is not a punishment, (Maliler vs. Eby,
264 U.S., 32), it being merely the return to his country of an alien who has broken
the conditions upon which he could continue to reside within our borders (U.S.
vs. De los Santos, 33 Phil., 397). The deportation proceedings are administrative
in character, (Kessler vs. Stracker 307 U.S., 22) summary in nature, and need
not be conducted strictly in accordance with the ordinary court proceedings
(Murdock vs. Clark, 53 F. [2d], 155). It is essential, however, that the warrant of
arrest shall give the alien sufficient information about the charges against him,
relating the facts relied upon. (U.S. vs. Uhl 211 F., 628.) It is also essential that
he be given a fair hearing with the assistance of counsel, if he so desires, before
unprejudiced investigators (Strench vs. Pedaris, 55 F. [2d], 597; Ex parte Jew
You On, 16 F. [2d], 153). However, all the strict rules of evidence governing
judicial controversies do not need to be observed; only such as are fumdamental
and essential like the right of cross-examination. (U.S. vs. Hughes, 104 F. [2d],
14; Murdock vs. Clark, 53 F. [2d], 155.) Hearsay evidence may even be admitted,
provided the alien is given the opportunity to explain or rebut it (Morrell vs. Baker,
270 F., 577; Sercerchi vs. Ward, 27 F. Supp., 437). (Lao Tang Bun vs. Fabre 81
Phil. 682 [1948]).

The ruling in Vivo vs. Montesa (G. R. No. 24576, July 29, 1968, 24 SCRA 155) that "the
issuance of warrants of arrest by the Commissioner of Immigration, solely for purposes of
investigation and before a final order of deportation is issued, conflicts with paragraph 3, Section
I of Article III of the Constitution" (referring to the 1935 Constitution) 3 is not invocable herein.
Respondent Commissioner's Warrant of Arrest issued on 7 March 1988 did not order petitioners
to appear and show cause why they should not be deported. They were issued specifically "for
violation of Sections 37, 45 and 46 of the Immigration Act and Section 69 of the Revised
Administrative Code." Before that, deportation proceedings had been commenced against them
as undesirable aliens on 4 March 1988 and the arrest was a step preliminary to their possible
deportation.

Section 37 of the Immigration Law, which empowers the Commissioner of


Immigration to issue warrants for the arrest of overstaying aliens is constitutional.
The arrest is a stop preliminary to the deportation of the aliens who had violated
the condition of their stay in this country. (Morano vs. Vivo, L-22196, June 30,
1967, 20 SCRA 562).

To rule otherwise would be to render the authority given the Commissioner nugatory to the
detriment of the State.

The pertinent provision of Commonwealth Act No. 613, as amended, which gives
authority to the Commissioner of Immigration to order the arrest of an alien
temporary visitor preparatory to his deportation for failure to put up new bonds
required for the stay, is not unconstitutional.

xxx xxx xxx

... Such a step is necessary to enable the Commissioner to prepare the ground
for his deportation under Section 37[al of Commonwealth Act 613. A contrary
interpretation would render such power nugatory to the detriment of the State.
(Ng Hua To vs. Galang, G. R. No. 10145, February 29, 1964, 10 SCRA 411).

"The requirement of probable cause, to be determined by a Judge, does not extend to


deportation proceedings." (Morano vs. Vivo, supra, citing Tiu Chun Hai vs. Commissioner, infra).
There need be no "truncated" recourse to both judicial and administrative warrants in a single
deportation proceedings.

The foregoing does not deviate from the ruling in Qua Chee Gan vs. Deportation Board (G. R.
No. 10280, September 30, 1963, 9 SCRA 27 [1963]) reiterated in Vivo vs. Montesa, supra, that
"under the express terms of our Constitution (the 1935 Constitution), it is therefore even doubtful
whether the arrest of an individual may be ordered by any authority other than a judge if the
purpose is merely to determine the existence of a probable cause, leading to an administrative
investigation." For, as heretofore stated, probable cause had already been shown to exist
before the warrants of arrest were issued.

What is essential is that there should be a specific charge against the alien intended to be
arrested and deported, that a fair hearing be conducted (Section 37[c]) with the assistance of
counsel, if desired, and that the charge be substantiated by competent evidence. Thus, Section
69 of the Revised Administrative Code explicitly provides:

Sec. 69. Deportation of subject of foreign power. A subject of a foreign power


residing in the Philippines shall not be deported, expelled, or excluded from said
Islands or repatriated to his own country by the President of the Philippines
except upon prior investigation, conducted by said Executive or his authorized
agent, of the ground upon which such action is contemplated. In such a case the
person concerned shall be informed of the charge or charges against him and he
shall be allowed not less than 3 days for the preparation of his defense. He shall
also have the right to be heard by himself or counsel, to produce witnesses in his
own behalf, and to cross-examine the opposing witnesses.

The denial by respondent Commissioner of petitioners' release on bail, also challenged by


them, was in order because in deportation proceedings, the right to bail is not a matter of right
but a matter of discretion on the part of the Commissioner of Immigration and Deportation.
Thus, Section 37(e) of the Philippine Immigration Act of 1940 provides that "any alien under
arrest in a deportation proceeding may be released under bond or under such other conditions
as may be imposed by the Commissioner of Immigration." The use of the word "may" in said
provision indicates that the grant of bail is merely permissive and not mandatory on the part of
the Commissioner. The exercise of the power is wholly discretionary (Ong Hee Sang vs.
Commissioner of Immigration, L-9700, February 28,1962, 4 SCRA 442). "Neither the
Constitution nor Section 69 of the Revised Administrative Code guarantees the right of aliens
facing deportation to provisional liberty on bail." (Tiu Chun Hai et al vs. Deportation Board, 104
Phil. 949 [1958]). As deportation proceedings do not partake of the nature of a criminal action,
the constitutional guarantee to bail may not be invoked by aliens in said proceedings (Ong Hee
Sang vs. Commissioner of Immigration, supra).

Every sovereign power has the inherent power to exclude aliens from its territory upon such
grounds as it may deem proper for its self-preservation or public interest (Lao Tan Bun vs.
Fabre 81 Phil. 682 [1948]). The power to deport aliens is an act of State, an act done by or
under the authority of the sovereign power (In re McCulloch Dick, 38 Phil. 41 [1918]). It is a
police measure against undesirable aliens whose continued presence in the country is found to
be injurious to the public good and the domestic tranquility of the people (Forbes vs. Chuoco
Tiaco et al., 16 Phil. 534 [1910]). Particularly so in this case where the State has expressly
committed itself to defend the tight of children to assistance and special protection from all
forms of neglect, abuse, cruelty, exploitation, and other conditions prejudicial to their
development (Article XV, Section 3[2]). Respondent Commissioner of Immigration and
Deportation, in instituting deportation proceedings against petitioners, acted in the interests of
the State.

WHEREFORE, the Petition is dismissed and the Writ of Habeas Corpus is hereby denied.

SO ORDERED.

G.R. No. 81510 March 14, 1990

HORTENCIA SALAZAR, petitioner,


vs.
HON. TOMAS D. ACHACOSO, in his capacity as Administrator of the Philippine Overseas
Employment Administration, and FERDIE MARQUEZ, respondents.

Gutierrez & Alo Law Offices for petitioner.

SARMIENTO, J.:

This concerns the validity of the power of the Secretary of Labor to issue warrants of arrest and
seizure under Article 38 of the Labor Code, prohibiting illegal recruitment.

The facts are as follows:

xxx xxx xxx

1. On October 21, 1987, Rosalie Tesoro of 177 Tupaz Street, Leveriza, Pasay
City, in a sworn statement filed with the Philippine Overseas Employment
Administration (POEA for brevity) charged petitioner Hortencia Salazar, viz:

04. T: Ano ba ang dahilan at ikaw ngayon ay narito


at
nagbibigay ng salaysay.
S: Upang ireklamo sa dahilan ang aking PECC Card ay
ayaw ibigay sa akin ng dati kong manager. — Horty
Salazar — 615 R.O. Santos, Mandaluyong, Mla.

05. T: Kailan at saan naganap and ginawang


panloloko sa
iyo ng tao/mga taong inireklamo mo?

S. Sa bahay ni Horty Salazar.

06. T: Paano naman naganap ang pangyayari?

S. Pagkagaling ko sa Japan ipinatawag niya ako.


Kinuha
ang PECC Card ko at sinabing hahanapan ako ng
booking sa Japan. Mag 9 month's na ako sa Phils.
ay
hindi pa niya ako napa-alis. So lumipat ako ng
ibang
company pero ayaw niyang ibigay and PECC Card
ko.

2. On November 3, 1987, public respondent Atty. Ferdinand Marquez to whom


said complaint was assigned, sent to the petitioner the following telegram:

YOU ARE HEREBY DIRECTED TO APPEAR BEFORE FERDIE


MARQUEZ POEA ANTI ILLEGAL RECRUITMENT UNIT 6TH
FLR. POEA BLDG. EDSA COR. ORTIGAS AVE.
MANDALUYONG MM ON NOVEMBER 6, 1987 AT 10 AM RE
CASE FILED AGAINST YOU. FAIL NOT UNDER PENALTY OF
LAW.

4. On the same day, having ascertained that the petitioner had no license to
operate a recruitment agency, public respondent Administrator Tomas D.
Achacoso issued his challenged CLOSURE AND SEIZURE ORDER NO. 1205
which reads:

HORTY SALAZAR
No. 615 R.O. Santos St.
Mandaluyong, Metro Manila

Pursuant to the powers vested in me under Presidential Decree No. 1920 and
Executive Order No. 1022, I hereby order the CLOSURE of your recruitment
agency being operated at No. 615 R.O. Santos St., Mandaluyong, Metro Manila
and the seizure of the documents and paraphernalia being used or intended to
be used as the means of committing illegal recruitment, it having verified that you
have —
(1) No valid license or authority from the Department of Labor and
Employment to recruit and deploy workers for overseas
employment;

(2) Committed/are committing acts prohibited under Article 34 of


the New Labor Code in relation to Article 38 of the same code.

This ORDER is without prejudice to your criminal prosecution


under existing laws.

Done in the City of Manila, this 3th day of November, 1987.

5. On January 26, 1988 POEA Director on Licensing and Regulation Atty.


Estelita B. Espiritu issued an office order designating respondents Atty. Marquez,
Atty. Jovencio Abara and Atty. Ernesto Vistro as members of a team tasked to
implement Closure and Seizure Order No. 1205. Doing so, the group assisted by
Mandaluyong policemen and mediamen Lito Castillo of the People's Journal and
Ernie Baluyot of News Today proceeded to the residence of the petitioner at 615
R.O. Santos St., Mandaluyong, Metro Manila. There it was found that petitioner
was operating Hannalie Dance Studio. Before entering the place, the team
served said Closure and Seizure order on a certain Mrs. Flora Salazar who
voluntarily allowed them entry into the premises. Mrs. Flora Salazar informed the
team that Hannalie Dance Studio was accredited with Moreman Development
(Phil.). However, when required to show credentials, she was unable to produce
any. Inside the studio, the team chanced upon twelve talent performers —
practicing a dance number and saw about twenty more waiting outside, The team
confiscated assorted costumes which were duly receipted for by Mrs. Asuncion
Maguelan and witnessed by Mrs. Flora Salazar.

6. On January 28, 1988, petitioner filed with POEA the following letter:

Gentlemen:

On behalf of Ms. Horty Salazar of 615 R.O. Santos, Mandaluyong, Metro Manila,
we respectfully request that the personal properties seized at her residence last
January 26, 1988 be immediately returned on the ground that said seizure was
contrary to law and against the will of the owner thereof. Among our reasons are
the following:

1. Our client has not been given any prior notice or hearing, hence
the Closure and Seizure Order No. 1205 dated November 3, 1987
violates "due process of law" guaranteed under Sec. 1, Art. III, of
the Philippine Constitution.

2. Your acts also violate Sec. 2, Art. III of the Philippine


Constitution which guarantees right of the people "to be secure in
their persons, houses, papers, and effects against unreasonable
searches and seizures of whatever nature and for any purpose."
3. The premises invaded by your Mr. Ferdi Marquez and five (5)
others (including 2 policemen) are the private residence of the
Salazar family, and the entry, search as well as the seizure of the
personal properties belonging to our client were without her
consent and were done with unreasonable force and intimidation,
together with grave abuse of the color of authority, and constitute
robbery and violation of domicile under Arts. 293 and 128 of the
Revised Penal Code.

Unless said personal properties worth around TEN THOUSAND


PESOS (P10,000.00) in all (and which were already due for
shipment to Japan) are returned within twenty-four (24) hours from
your receipt hereof, we shall feel free to take all legal action, civil
and criminal, to protect our client's interests.

We trust that you will give due attention to these important


matters.

7. On February 2, 1988, before POEA could answer the letter, petitioner filed the
instant petition; on even date, POEA filed a criminal complaint against her with
the Pasig Provincial Fiscal, docketed as IS-88-836.1

On February 2, 1988, the petitioner filed this suit for prohibition. Although the acts sought to be
barred are already fait accompli, thereby making prohibition too late, we consider the petition as
one for certiorari in view of the grave public interest involved.

The Court finds that a lone issue confronts it: May the Philippine Overseas Employment
Administration (or the Secretary of Labor) validly issue warrants of search and seizure (or
arrest) under Article 38 of the Labor Code? It is also an issue squarely raised by the petitioner
for the Court's resolution.

Under the new Constitution, which states:

. . . no search warrant or warrant of arrest shall issue except upon probable


cause to be determined personally by the judge after examination under oath or
affirmation of the complainant and the witnesses he may produce, and
particularly describing the place to be searched and the persons or things to be
seized. 2

it is only a judge who may issue warrants of search and arrest. 3 In one case, it was declared
that mayors may not exercise this power:

xxx xxx xxx

But it must be emphasized here and now that what has just been described is the
state of the law as it was in September, 1985. The law has since been altered.
No longer does the mayor have at this time the power to conduct preliminary
investigations, much less issue orders of arrest. Section 143 of the Local
Government Code, conferring this power on the mayor has been abrogated,
rendered functus officio by the 1987 Constitution which took effect on February 2,
1987, the date of its ratification by the Filipino people. Section 2, Article III of the
1987 Constitution pertinently provides that "no search warrant or warrant of
arrest shall issue except upon probable cause to be determined personally by the
judge after examination under oath or affirmation of the complainant and the
witnesses he may produce, and particularly describing the place to be searched
and the person or things to be seized." The constitutional proscription has
thereby been manifested that thenceforth, the function of determining probable
cause and issuing, on the basis thereof, warrants of arrest or search warrants,
may be validly exercised only by judges, this being evidenced by the elimination
in the present Constitution of the phrase, "such other responsible officer as may
be authorized by law" found in the counterpart provision of said 1973
Constitution, who, aside from judges, might conduct preliminary investigations
and issue warrants of arrest or search warrants. 4

Neither may it be done by a mere prosecuting body:

We agree that the Presidential Anti-Dollar Salting Task Force exercises, or was
meant to exercise, prosecutorial powers, and on that ground, it cannot be said to
be a neutral and detached "judge" to determine the existence of probable cause
for purposes of arrest or search. Unlike a magistrate, a prosecutor is naturally
interested in the success of his case. Although his office "is to see that justice is
done and not necessarily to secure the conviction of the person accused," he
stands, invariably, as the accused's adversary and his accuser. To permit him to
issue search warrants and indeed, warrants of arrest, is to make him both judge
and jury in his own right, when he is neither. That makes, to our mind and to that
extent, Presidential Decree No. 1936 as amended by Presidential Decree No.
2002, unconstitutional. 5

Section 38, paragraph (c), of the Labor Code, as now written, was entered as an amendment by
Presidential Decrees Nos. 1920 and 2018 of the late President Ferdinand Marcos, to
Presidential Decree No. 1693, in the exercise of his legislative powers under Amendment No. 6
of the 1973 Constitution. Under the latter, the then Minister of Labor merely exercised
recommendatory powers:

(c) The Minister of Labor or his duly authorized representative shall have the
power to recommend the arrest and detention of any person engaged in illegal
recruitment. 6

On May 1, 1984, Mr. Marcos promulgated Presidential Decree No. 1920, with the avowed
purpose of giving more teeth to the campaign against illegal recruitment. The Decree gave the
Minister of Labor arrest and closure powers:

(b) The Minister of Labor and Employment shall have the power to cause the
arrest and detention of such non-licensee or non-holder of authority if after
proper investigation it is determined that his activities constitute a danger to
national security and public order or will lead to further exploitation of job-
seekers. The Minister shall order the closure of companies, establishment and
entities found to be engaged in the recruitment of workers for overseas
employment, without having been licensed or authorized to do so. 7
On January 26, 1986, he, Mr. Marcos, promulgated Presidential Decree No. 2018, giving the
Labor Minister search and seizure powers as well:

(c) The Minister of Labor and Employment or his duly authorized representatives
shall have the power to cause the arrest and detention of such non-licensee or
non-holder of authority if after investigation it is determined that his activities
constitute a danger to national security and public order or will lead to further
exploitation of job-seekers. The Minister shall order the search of the office or
premises and seizure of documents, paraphernalia, properties and other
implements used in illegal recruitment activities and the closure of companies,
establishment and entities found to be engaged in the recruitment of workers for
overseas employment, without having been licensed or authorized to do so. 8

The above has now been etched as Article 38, paragraph (c) of the Labor Code.

The decrees in question, it is well to note, stand as the dying vestiges of authoritarian rule in its
twilight moments.

We reiterate that the Secretary of Labor, not being a judge, may no longer issue search or
arrest warrants. Hence, the authorities must go through the judicial process. To that extent, we
declare Article 38, paragraph (c), of the Labor Code, unconstitutional and of no force and effect.

The Solicitor General's reliance on the case of Morano v. Vivo 9 is not well-taken. Vivo involved
a deportation case, governed by Section 69 of the defunct Revised Administrative Code and by
Section 37 of the Immigration Law. We have ruled that in deportation cases, an arrest (of an
undesirable alien) ordered by the President or his duly authorized representatives, in order to
carry out a final decision of deportation is valid. 10 It is valid, however, because of the
recognized supremacy of the Executive in matters involving foreign affairs. We have held: 11

xxx xxx xxx

The State has the inherent power to deport undesirable aliens (Chuoco Tiaco vs.
Forbes, 228 U.S. 549, 57 L. Ed. 960, 40 Phil. 1122, 1125). That power may be
exercised by the Chief Executive "when he deems such action necessary for the
peace and domestic tranquility of the nation." Justice Johnson's opinion is that
when the Chief Executive finds that there are aliens whose continued presence
in the country is injurious to the public interest, "he may, even in the absence of
express law, deport them". (Forbes vs. Chuoco Tiaco and Crossfield, 16 Phil.
534, 568, 569; In re McCulloch Dick, 38 Phil. 41).

The right of a country to expel or deport aliens because their continued presence
is detrimental to public welfare is absolute and unqualified (Tiu Chun Hai and Go
Tam vs. Commissioner of Immigration and the Director of NBI, 104 Phil. 949,
956). 12

The power of the President to order the arrest of aliens for deportation is, obviously,
exceptional. It (the power to order arrests) can not be made to extend to other cases, like the
one at bar. Under the Constitution, it is the sole domain of the courts.
Moreover, the search and seizure order in question, assuming, ex gratia argumenti, that it was
validly issued, is clearly in the nature of a general warrant:

Pursuant to the powers vested in me under Presidential Decree No. 1920 and
Executive Order No. 1022, I hereby order the CLOSURE of your recruitment
agency being operated at No. 615 R.O. Santos St., Mandaluyong, Metro Manila
and the seizure of the documents and paraphernalia being used or intended to
be used as the means of committing illegal recruitment, it having verified that you
have —

(1) No valid license or authority from the Department of Labor and


Employment to recruit and deploy workers for overseas
employment;

(2) Committed/are committing acts prohibited under Article 34 of


the New Labor Code in relation to Article 38 of the same code.

This ORDER is without prejudice to your criminal prosecution under existing


laws. 13

We have held that a warrant must identify clearly the things to be seized, otherwise, it is null and
void, thus:

xxx xxx xxx

Another factor which makes the search warrants under consideration


constitutionally objectionable is that they are in the nature of general warrants.
The search warrants describe the articles sought to be seized in this wise:

1) All printing equipment, paraphernalia, paper, ink, photo


equipment, typewriters, cabinets, tables, communications/
recording equipment, tape recorders, dictaphone and the like used
and/or connected in the printing of the "WE FORUM" newspaper
and any and all documents/communications, letters and facsimile
of prints related to the "WE FORUM" newspaper.

2) Subversive documents, pamphlets, leaflets, books, and other


publications to promote the objectives and purposes of the
subversive organizations known as Movement for Free
Philippines, Light-a-Fire Movement and April 6 Movement; and

3) Motor vehicles used in the distribution/circulation of the "WE


FORUM" and other subversive materials and propaganda, more
particularly,

1) Toyota-Corolla, colored yellow with Plate No. NKA 892;

2) DATSUN, pick-up colored white with Plate No. NKV 969;


3) A delivery truck with Plate No. NBS 542;

4) TOYOTA-TAMARAW, colored white with Plate No. PBP 665;


and

5) TOYOTA Hi-Lux, pick-up truck with Plate No. NGV 472 with
marking "Bagong Silang."

In Stanford v. State of Texas, the search warrant which authorized the search for
"books, records, pamphlets, cards, receipts, lists, memoranda, pictures,
recordings and other written instruments concerning the Communist Parties of
Texas, and the operations of the Community Party in Texas," was declared void
by the U.S. Supreme Court for being too general. In like manner, directions to
"seize any evidence in connection with the violation of SDC 13-3703 or
otherwise" have been held too general, and that portion of a search warrant
which authorized the seizure of any "paraphernalia which could be used to
violate Sec. 54-197 of the Connecticut General Statutes (the statute dealing with
the crime of conspiracy)" was held to be a general warrant, and therefore invalid.
The description of the articles sought to be seized under the search warrants in
question cannot be characterized differently.

In the Stanford case, the U.S. Supreme court calls to mind a notable chapter in
English history; the era of disaccord between the Tudor Government and the
English Press, when "Officers of the Crown were given roving commissions to
search where they pleased in order to suppress and destroy the literature of
dissent both Catholic and Puritan." Reference herein to such historical episode
would not be relevant for it is not the policy of our government to suppress any
newspaper or publication that speaks with "the voice of non-conformity" but
poses no clear and imminent danger to state security. 14

For the guidance of the bench and the bar, we reaffirm the following principles:

1. Under Article III, Section 2, of the l987 Constitution, it is only judges, and no
other, who may issue warrants of arrest and search:

2. The exception is in cases of deportation of illegal and undesirable aliens,


whom the President or the Commissioner of Immigration may order arrested,
following a final order of deportation, for the purpose of deportation.

WHEREFORE, the petition is GRANTED. Article 38, paragraph (c) of the Labor Code is
declared UNCONSTITUTIONAL and null and void. The respondents are ORDERED to return all
materials seized as a result of the implementation of Search and Seizure Order No. 1205.

No costs.

SO ORDERED.

G.R. No. L-45358 January 29, 1937


NARCISO ALVAREZ, petitioner,
vs.
THE COURT OF FIRST INSTANCE OF TAYABAS and THE ANTI-USURY BOARD,
respondents.

Godofredo Reyes for petitioner.


Adolfo N. Feliciano for respondents Anti-Usury Board.
No appearance for other respondent.

IMPERIAL, J.:

The petitioner asks that the warrant of June 3, 1936, issued by the Court of First Instance of
Tayabas, ordering the search of his house and the seizure, at any time of the day or night, of
certain accounting books, documents and papers belonging to him in his residence situated in
Infanta, Province of Tayabas, as well as the order of a later date, authorizing the agents of the
Anti-Usury Board to retain the articles seized, be declared illegal and set aside, and prays that
all the articles in question be returned to him.

On the date above-mentioned, the chief of the secret service of the Anti-Usury Board, of the
Department of Justice, presented to Judge Eduardo Gutierrez David then presiding over the
Court of First Instance of Tayabas, an affidavit alleging that according to reliable information, the
petitioner kept in his house in Infanta, Tayabas, books, documents, receipts, lists, chits and
other papers used by him in connection with his activities as a money-lender charging usurious
rates of interest in violation of the law. In his oath at the and of the affidavit, the chief of the
secret service stated that his answers to the questions were correct to the best of his knowledge
and belief. He did not swear to the truth of his statements upon his own knowledge of the facts
but upon the information received by him from a reliable person. Upon the affidavit in question
the Judge, on said date, issued the warrant which is the subject matter of the petition, ordering
the search of the petitioner's house at nay time of the day or night, the seizure of the books and
documents above-mentioned and the immediate delivery thereof to him to be disposed of in
accordance with the law. With said warrant, several agents of the Anti-Usury Board entered the
petitioner's store and residence at seven o'clock on the night of June 4, 1936, and seized and
took possession of the following articles: internal revenue licenses for the years 1933 to 1936,
one ledger, two journals, two cashbooks, nine order books, four notebooks, four checks stubs,
two memorandums, three bankbooks, two contracts, four stubs, forty-eight stubs of purchases
of copra, two inventories, two bundles of bills of lading, one bundle of credit receipts, one bundle
of stubs of purchases of copra, two packages of correspondence, one receipt book belonging to
Luis Fernandez, fourteen bundles of invoices and other papers many documents and loan
contracts with security and promissory notes, 504 chits, promissory notes and stubs of used
checks of the Hongkong & Shanghai Banking Corporation. The search for and a seizure of said
articles were made with the opposition of the petitioner who stated his protest below the
inventories on the ground that the agents seized even the originals of the documents. As the
articles had not been brought immediately to the judge who issued the search warrant, the
petitioner, through his attorney, filed a motion on June 8, 1936, praying that the agent Emilio L.
Siongco, or any other agent, be ordered immediately to deposit all the seized articles in the
office of the clerk of court and that said agent be declared guilty of contempt for having
disobeyed the order of the court. On said date the court issued an order directing Emilio L.
Siongco to deposit all the articles seized within twenty-four hours from the receipt of notice
thereof and giving him a period of five (5) days within which to show cause why he should not
be punished for contempt of court. On June 10th, Attorney Arsenio Rodriguez, representing the
Anti-Usury Board, filed a motion praying that the order of the 8th of said month be set aside and
that the Anti-Usury Board be authorized to retain the articles seized for a period of thirty (30)
days for the necessary investigation. The attorney for the petitioner, on June 20th, filed another
motion alleging that, notwithstanding the order of the 8th of said month, the officials of the Anti-
Usury Board had failed to deposit the articles seized by them and praying that a search warrant
be issued, that the sheriff be ordered to take all the articles into his custody and deposit of the
Anti-Usury Board be punished for contempt of court. Said attorney, on June 24th, filed an ex
parte petition alleging that while agent Emilio L. Siongco had deposited some documents and
papers in the office of the clerk of court, he had so far failed to file an inventory duly verified by
oath of all the documents seized by him, to return the search warrant together with the affidavit
it presented in support thereof, or to present the report of the proceedings taken by him; and
prayed that said agent be directed to filed the documents in question immediately. On the 25th
of said month the court issued an order requiring agent Emilio L. Siongco forthwith to file the
search warrant and the affidavit in the court, together with the proceedings taken by him, and to
present an inventory duly verified by oath of all the articles seized. On July 2d of said year, the
attorney for the petitioner filed another petition alleging that the search warrant issue was illegal
and that it had nit yet been returned to date together with the proceedings taken in connection
therewith, and praying that said warrant be cancelled, that an order be issued directing the
return of all the articles seized to the petitioner, that the agent who seized them be declared
guilty of contempt of court, and that charges be filed against him for abuse of authority. On
September 10, 1936, the court issued an order holding: that the search warrant was obtained
and issued in accordance with the law, that it had been duly complied with and, consequently,
should not be cancelled, and that agent Emilio L. Siongco did not commit any contempt of court
and must, therefore, be exonerated, and ordering the chief of the Anti-Usury Board in Manila to
show case, if any, within the unextendible period of two (2) days from the date of notice of said
order, why all the articles seized appearing in the inventory, Exhibit 1, should not be returned to
the petitioner. The assistant chief of the Anti-Usury Board of the Department of Justice filed a
motion praying, for the reasons stated therein, that the articles seized be ordered retained for
the purpose of conducting an investigation of the violation of the Anti-Usury Law committed by
the petitioner. In view of the opposition of the attorney for the petitioner, the court, on September
25th, issued an order requiring the Anti-Usury Board to specify the time needed by it to examine
the documents and papers seized and which of them should be retained, granting it a period of
five (5) days for said purpose. On the 30th of said month the assistant chief of the Anti-Usury
Board filed a motion praying that he be granted ten (10) days to comply with the order of
September 25th and that the clerk of court be ordered to return to him all the documents and
papers together with the inventory thereof. The court, in an order of October 2d of said year,
granted him the additional period of ten(10) days and ordered the clerk of court to send him a
copy of the inventory. On October 10th, said official again filed another motion alleging that he
needed sixty (60) days to examine the documents and papers seized, which are designated on
pages 1 to 4 of the inventory by Nos. 5, 1016, 23, 25, 26, 27, 30, 31, 34, 36, 37, 38, 39, 40, 41,
42, 43 and 45, and praying that he be granted said period of sixty (60) days. In an order of
October 16th, the court granted him the period of sixty (60) days to investigate said nineteen
(19) documents. The petitioner alleges, and it is not denied by the respondents, that these
nineteen (19)documents continue in the possession of the court, the rest having been returned
to said petitioner.

I. A search warrant is an order in writing, issued in the name of the People of the Philippine
Islands, signed by a judge or a justice of the peace, and directed to a peace officer,
commanding him to search for personal property and bring it before the court (section 95,
General Orders. No. 58, as amended by section 6 of Act No. 2886). Of all the rights of a citizen,
few are of greater importance or more essential to his peace and happiness than the right of
personal security, and that involves the exemption of his private affairs, books, and papers from
the inspection and scrutiny of others (In re Pacific Railways Commission, 32 Fed., 241;
Interstate Commerce Commission vs Brimson, 38 Law. ed., 1047; Broyd vs. U. S., 29 Law. ed.,
746; Caroll vs. U. S., 69 Law. ed., 543, 549). While the power to search and seize is necessary
to the public welfare, still it must be exercised and the law enforced without transgressing the
constitutional rights or citizen, for the enforcement of no statue is of sufficient importance to
justify indifference to the basis principles of government (People vs. Elias, 147 N. E., 472).

II. As the protection of the citizen and the maintenance of his constitutional right is one of the
highest duties and privileges of the court, these constitutional guaranties should be given a
liberal construction or a strict construction in favor of the individual, to prevent stealthy
encroachment upon, or gradual depreciation on, the rights secured by them(State vs. Custer
County, 198 Pac., 362; State vs. McDaniel, 231 Pac., 965; 237 Pac., 373). Since the proceeding
is a drastic one, it is the general rule that statutes authorizing searches and seizure or search
warrants must be strictly construed (Rose vs. St. Clair, 28 Fed., [2d], 189; Leonard vs. U. S., 6
Fed. [2d], 353; Perry vs. U. S. 14 Fed. [2d],88; Cofer vs. State, 118 So., 613).

III. The petitioner claims that the search warrant issued by the court is illegal because it has
been based upon the affidavit of agent Mariano G. Almeda in whose oath he declared that he
had no personal knowledge of the facts which were to serve as a basis for the issuance of the
warrant but that he had knowledge thereof through mere information secured from a person
whom he considered reliable. To the question "What are your reason for applying for this search
warrant", appearing in the affidavit, the agent answered: "It has been reported to me by a
person whom I consider to be reliable that there are being kept in said premises, books,
documents, receipts, lists, chits, and other papers used by him in connection with his activities
as a money-lender, charging a usurious rate of interest, in violation of the law" and in attesting
the truth of his statements contained in the affidavit, the said agent states that he found them to
be correct and true to the best of his knowledge and belief.
Section 1, paragraph 3, of Article III of the Constitution, relative to the bill of rights, provides that
"The right of the people to be secure in their persons, houses, papers, and effects against
unreasonable searches and seizures shall not be violated, and no warrants shall issue but upon
probable cause, to be determined by the judge after examination under oath or affirmation of the
complainant and the witnesses he may produce, and particularly describing the place top be
searched, and the persons or things to be seized." Section 97 of General Orders, No. 58
provides that "A search warrant shall not issue except for probable cause and upon application
supported by oath particularly describing the place to be searched and the person or thing to be
seized." It will be noted that both provisions require that there be not only probable cause before
the issuance of a search warrant but that the search warrant must be based upon an application
supported by oath of the applicant ands the witnesses he may produce. In its broadest sense,
an oath includes any form of attestation by which a party signifies that he is bound in
conscience to perform an act faithfully and truthfully; and it is sometimes defined asan outward
pledge given by the person taking it that his attestation or promise is made under an immediate
sense of his responsibility to God (Bouvier's Law Dictionary; State vs. Jackson, 137 N. W.,
1034; In re Sage, 24 Oh. Cir. Ct. [N. S.], 7; Pumphery vs. State, 122 N. W., 19; Priest vs. State,
6 N. W., 468; State vs. Jones, 154 Pac., 378; Atwood vs. State, 111 So., 865). The oath
required must refer to the truth of the facts within the personal knowledge of the petitioner or his
witnesses, because the purpose thereof is to convince the committing magistrate, not the
individual making the affidavit and seeking the issuance of the warrant, of the existence of
probable cause (U. S. vs. Tureaud, 20 Fed., 621; U. S. vs. Michalski, 265 Fed., 8349; U. S. vs.
Pitotto, 267 Fed., 603; U. S. vs. Lai Chew, 298 Fed., 652). The true test of sufficiency of an
affidavit to warrant issuance of a search warrant is whether it has been drawn in such a manner
that perjury could be charged thereon and affiant be held liable for damages caused (State vs.
Roosevelt Country 20th Jud. Dis. Ct., 244 Pac., 280; State vs. Quartier, 236 Pac., 746).

It will likewise be noted that section 1, paragraph 3, of Article III of the Constitution prohibits
unreasonable searches and seizure. Unreasonable searches and seizures are a menace
against which the constitutional guarantee afford full protection. The term "unreasonable search
and seizure" is not defined in the Constitution or in General Orders No. 58, and it is said to have
no fixed, absolute or unchangeable meaning, although the term has been defined in general
language. All illegal searches and seizure are unreasonable while lawful ones are reasonable.
What constitutes a reasonable or unreasonable search or seizure in any particular case is
purely a judicial question, determinable from a consideration of the circumstances involved,
including the purpose of the search, the presence or absence or probable cause, the manner in
which the search and seizure was made, the place or thing searched, and the character of the
articles procured (Go-Bart Importing Co. vs. U. S. 75 Law. ed., 374; Peru vs. U. S., 4 Fed., [2d],
881;U. S. vs. Vatune, 292 Fed., 497; Angelo vs. U. S. 70 Law, ed., 145; Lambert vs. U. S. 282
Fed., 413; U. S. vs. Bateman, 278 Fed., 231; Mason vs. Rollins, 16 Fed. Cas. [No. 9252], 2
Biss., 99).

In view of the foregoing and under the above-cited authorities, it appears that the affidavit, which
served as the exclusive basis of the search warrant, is insufficient and fatally defective by
reason of the manner in which the oath was made, and therefore, it is hereby held that the
search warrant in question and the subsequent seizure of the books, documents and other
papers are illegal and do not in any way warrant the deprivation to which the petitioner was
subjected.

IV. Another ground alleged by the petitioner in asking that the search warrant be declared illegal
and cancelled is that it was not supported by other affidavits aside from that made by the
applicant. In other words, it is contended that the search warrant cannot be issued unless it be
supported by affidavits made by the applicant and the witnesses to be presented necessity by
him. Section 1, paragraph 3, of Article III of the Constitution provides that no warrants shall
issue but upon probable cause, to be determined by the judge after examination under oath or
affirmation of the complainant and the witnesses he may produce. Section 98 of General
Orders, No. 58 provides that the judge or justice must, before issuing the warrant, examine
under oath the complainant and any witnesses he may produce and take their depositions in
writing. It is the practice in this jurisdiction to attach the affidavit of at least the applicant or
complainant to the application. It is admitted that the judge who issued the search warrant in this
case, relied exclusively upon the affidavit made by agent Mariano G. Almeda and that he did not
require nor take the deposition of any other witness. Neither the Constitution nor General
Orders. No. 58 provides that it is of imperative necessity to take the deposition of the witnesses
to be presented by the applicant or complainant in addition to the affidavit of the latter. The
purpose of both in requiring the presentation of depositions is nothing more than to satisfy the
committing magistrate of the existence of probable cause. Therefore, if the affidavit of the
applicant or complainant is sufficient, the judge may dispense with that of other witnesses.
Inasmuch as the affidavit of the agent in this case was insufficient because his knowledge of the
facts was not personal but merely hearsay, it is the duty of the judge to require the affidavit of
one or more witnesses for the purpose of determining the existence of probable cause to
warrant the issuance of the search warrant. When the affidavit of the applicant of the complaint
contains sufficient facts within his personal and direct knowledge, it is sufficient if the judge is
satisfied that there exist probable cause; when the applicant's knowledge of the facts is mere
hearsay, the affidavit of one or more witnesses having a personal knowledge of the fact is
necessary. We conclude, therefore, that the warrant issued is likewise illegal because it was
based only on the affidavit of the agent who had no personal knowledge of the facts.

V. The petitioner alleged as another ground for the declaration of the illegality of the search
warrant and the cancellation thereof, the fact that it authorized its execution at night. Section
101 of General Orders, No. 58 authorizes that the search be made at night when it is positively
asserted in the affidavits that the property is on the person or in the place ordered to be
searched. As we have declared the affidavits insufficient and the warrant issued exclusively
upon it illegal, our conclusion is that the contention is equally well founded and that the search
could not legally be made at night.

VI. One of the grounds alleged by the petitioner in support of his contention that the warrant was
issued illegally is the lack of an adequate description of the books and documents to be seized.
Section 1, paragraphs 3, of Article III of the Constitution, and section 97 of General Orders, No.
58 provide that the affidavit to be presented, which shall serve as the basis for determining
whether probable cause exist and whether the warrant should be issued, must contain a
particular description of the place to be searched and the person or thing to be seized. These
provisions are mandatory and must be strictly complied with (Munch vs. U. S., 24 Fed. [2d], 518;
U. S. vs. Boyd, 1 Fed. [2d], 1019; U. S. vs. Carlson, 292 Fed., 463; U. S. vs. Borkowski, 268
Fed., 408; In re Tri-State Coal & Coke Co., 253 Fed., 605; People vs. Mayen, 188 Cal., 237;
People vs. Kahn, 256 Ill. App., 4125); but where, by the nature of the goods to be seized, their
description must be rather generally, it is not required that a technical description be given, as
this would mean that no warrant could issue (People vs. Rubio, 57 Phil., 284; People vs. Kahn,
supra). The only description of the articles given in the affidavit presented to the judge was as
follows: "that there are being kept in said premises books, documents, receipts, lists, chits and
other papers used by him in connection with his activities as money-lender, charging a usurious
rate of interest, in violation of the law." Taking into consideration the nature of the article so
described, it is clear that no other more adequate and detailed description could have been
given, particularly because it is difficult to give a particular description of the contents thereof.
The description so made substantially complies with the legal provisions because the officer of
the law who executed the warrant was thereby placed in a position enabling him to identify the
articles, which he did.

VII. The last ground alleged by the petitioner, in support of his claim that the search warrant was
obtained illegally, is that the articles were seized in order that the Anti-Usury Board might
provide itself with evidence to be used by it in the criminal case or cases which might be filed
against him for violation of the Anti-usury Law. At the hearing of the incidents of the case raised
before the court it clearly appeared that the books and documents had really been seized to
enable the Anti-Usury Board to conduct an investigation and later use all or some of the articles
in question as evidence against the petitioner in the criminal cases that may be filed against
him. The seizure of books and documents by means of a search warrant, for the purpose of
using them as evidence in a criminal case against the person in whose possession they were
found, is unconstitutional because it makes the warrant unreasonable, and it is equivalent to a
violation of the constitutional provision prohibiting the compulsion of an accused to testify
against himself (Uy Kheytin vs. Villareal, 42 Phil,, 886; Brady vs. U. S., 266 U. S., 620;
Temperani vs. U. S., 299 Fed., 365; U. S. vs. Madden, 297 Fed., 679; Boyd vs. U. S.,116 U. S.,
116; Caroll vs. U. S., 267 U. S., 132). Therefore, it appearing that at least nineteen of the
documents in question were seized for the purpose of using them as evidence against the
petitioner in the criminal proceeding or proceedings for violation against him, we hold that the
search warrant issued is illegal and that the documents should be returned to him.

The Anti-Usury Board insinuates in its answer that the petitioner cannot now question the
validity of the search warrant or the proceedings had subsequent to the issuance thereof,
because he has waived his constitutional rights in proposing a compromise whereby he agreed
to pay a fine of P200 for the purpose of evading the criminal proceeding or proceedings. We are
of the opinion that there was no such waiver, first, because the petitioner has emphatically
denied the offer of compromise and, second, because if there was a compromise it reffered but
to the institution of criminal proceedings fro violation of the Anti-Usury Law. The waiver would
have been a good defense for the respondents had the petitioner voluntarily consented to the
search and seizure of the articles in question, but such was not the case because the petitioner
protested from the beginning and stated his protest in writing in the insufficient inventory
furnished him by the agents.

Said board alleges as another defense that the remedy sought by the petitioner does not lie
because he can appeal from the orders which prejudiced him and are the subject matter of his
petition. Section 222 of the Code of Civil Procedure in fact provides that mandamus will not
issue when there is another plain, speedy and adequate remedy in the ordinary course of law.
We are of the opinion, however, that an appeal from said orders would have to lapse before he
recovers possession of the documents and before the rights, of which he has been unlawfully
deprived, are restored to him (Fajardo vs. Llorente, 6 Phil., 426; Manotoc vs. McMicking and
Trinidad, 10 Phil., 119; Cruz Herrera de Lukban vs. McMicking, 14 Phil., 641; Lamb vs. Phipps,
22 Phil., 456).

Summarizing the foregoing conclusions, we hold:

1. That the provisions of the Constitution and General Orders, No. 58, relative to search and
seizure, should be given a liberal construction in favor of the individual in order to maintain the
constitutional guaranties whole and in their full force;

2. That since the provisions in question are drastic in their form and fundamentally restrict the
enjoyment of the ownership, possession and use of the personal property of the individual, they
should be strictly construed;

3. That the search and seizure made are illegal for the following reasons: (a) Because the
warrant was based solely upon the affidavit of the petitioner who had no personal knowledge of
the facts of probable cause, and (b) because the warrant was issued for the sole purpose of
seizing evidence which would later be used in the criminal proceedings that might be instituted
against the petitioner, for violation of the Anti-Usury Law;

4. That as the warrant had been issued unreasonably, and as it does not appear positively in
the affidavit that the articles were in the possession of the petitioner and in the place indicated,
neither could the search and seizure be made at night;

5. That although it is not mandatory to present affidavits of witnesses to corroborate the


applicant or a complainant in cases where the latter has personal knowledge of the facts, when
the applicant's or complainant's knowledge of the facts is merely hearsay, it is the duty of the
judge to require affidavits of other witnesses so that he may determine whether probable cause
exists;
6. That a detailed description of the person and place to be searched and the articles to be
seized is necessary, but whereby, by the nature of the articles to be seized, their description
must be rather general, but is not required that a technical description be given, as this would
mean that no warrant could issue;

7. That the petitioner did not waive his constitutional rights because the offer of compromise or
settlement attributed to him, does not mean, if so made, that he voluntarily tolerated the search
and seizure; and

8. That an appeal from the orders questioned by the petitioner, if taken by him, would not be an
effective, speedy or adequate remedy in the ordinary course of law, and, consequently, the
petition for mandamus filed by him, lies.

For the foregoing considerations, the search warrant and the seizure of June 3, 1936, and the
orders of the respondent court authorizing the relation of the books and documents, are
declared illegal and are set aside, and it is ordered that the judge presiding over the Court of
First Instance of Tayabas direct the immediate return to the petitioner of the nineteen (19)
documents designated on pages 1 to 4 of the inventory by Nos. 5, 10, 16, 23, 25,26, 27, 30, 31,
34, 36, 37, 38, 39, 40, 41, 42, 43 and 45, without special pronouncement as to costs. So
ordered.
[G.R. No. 50720. March 26, 1984.]

SORIANO MATA, Petitioner, v. HON. JOSEPHINE K. BAYONA, in her capacity as Presiding


Judge of the City Court of Ormoc, BERNARDO GOLES and REYNALDO MAYOTE,
Respondents.

Valeriano R. Ocubillo for Petitioner.

The Solicitor General for Respondents.

SYLLABUS
1. CONSTITUTIONAL LAW; BILL OF RIGHTS; RIGHT AGAINST UNLAWFUL SEARCH AND
SEIZURE; REQUISITES FOR ISSUANCE OF SEARCH WARRANT. — Under the Constitution
"no search warrant shall issue but upon probable cause to be determined by the Judge or such
other responsible officer as may be authorized by law after examination under oath or
affirmation of the complainant and the witnesses he may produce." More emphatic and detailed
is the implementing rule of the constitutional injunction, Section 4 of Rule 126 which provides
that the judge must before issuing the warrant personally examine on oath or affirmation the
complainant and any witnesses he may produce and take their depositions in writing, and attach
them to the record, in addition to any affidavits presented to him.

2. ID.; ID.; ID.; ID.; INSUFFICIENCY OF AFFIDAVITS OF COMPLAINANT AND HIS


WITNESSES IN THE CASE AT BAR. — Before issuing a search warrant, the examining Judge
has to take depositions in writing of the complainant and the witnesses he may produce and to
attach them to the record. Such written deposition is necessary in order that the Judge may be
able to properly determine the existence or non-existence of the probable cause, and to hold
liable for perjury the person giving it if it will be found later that his declarations are false. Mere
affidavits of the complainant and his witnesses are thus not sufficient.

3. ID.; ID.; ID.; ID.; NO "DEPOSITION IN WRITING" ATTACHED TO RECORDS OF CASE IN


CASE AT BAR. — The judge’s insistence that she examined the complainants under oath has
become dubious by petitioner’s claim that at the particular time when he examined all the
relevant papers connected with the issuance of the questioned search warrant, after he
demanded the same from the lower court since they were not attached to the records, he did
not find any certification at the back of the joint affidavit of the complainants. Before he filed his
motion to quash the search warrant and for the return of the articles seized, he was furnished,
upon his request, certified true copies of the said affidavits by the Clerk of Court but which
certified true copies do not bear any certification at the back. Petitioner likewise claims that his
xerox copy of the said joint affidavit obtained at the outset of this case does not show also the
certification of respondent judge. This doubt becomes more confirmed by respondent Judge’s
own admission, while insisting that she did examine thoroughly the applicants, that "she did not
take the deposition of Mayote and Goles because to have done so would be to hold a judicial
proceeding which will be open and public", such that, according to her, the persons subject of
the intended raid will just disappear and move his illegal operations somewhere else. Could it be
that the certification was made belatedly to cure the defect of the warrant? Be that as it may,
there was no "deposition in writing" attached to the records of the case in palpable disregard of
the statutory prohibition heretofore quoted.

4. ID.; ID.; ID.; ID.; DEPOSITIONS, HOW TAKEN. — The searching questions propounded to
the applicants of the search warrant and his witnesses must depend to a large extent upon the
discretion of the Judge just as long as the answers establish a reasonable ground to believe the
commission of a specific offense and that the applicant is one authorized by law, and said
answers particularly describe with certainty the place to be searched and the persons or things
to be seized. The examination or investigation which must be under oath may not be in public. It
may even be held in the secrecy of his chambers. Far more important is that the examination or
investigation is not merely routinary but one that is thorough and elicit the required information.
To repeat, it must be under oath and must be in writing.

5. ID.; ID.; ID.; ID.; MUST BE STRICTLY COMPLIED WITH; CASE AT BAR. — Nothing can
justify the issuance of the search warrant but the fulfillment of the legal requisites. Thus, in
issuing a search warrant the Judge must strictly comply with the requirements of the
Constitution and the statutory provisions. In the case at bar, the search warrant is tainted with
illegality by the failure of the Judge to conform with essential requisites of taking the depositions
in writing and attaching them to record, rendering the search warrant invalid.

6. ID.; ID.; ID.; ALTHOUGH ILLEGAL, THINGS SEIZED CANNOT BE RETURNED; CASE AT
BAR. — While the search warrant is illegal, the return of the things seized cannot be ordered. In
Castro v. Pabalan (70 SCRA 478), it was held that the illegality of the search warrant does not
call for the return of the things seized, the possession of which is prohibited.

DECISION

DE CASTRO, J.:

The validity of the search warrant issued by respondent Judge (not reappointed) is challenged
by petitioner for its alleged failure to comply with the requisites of the Constitution and the Rules
of Court.

Specifically, the contention is that the search warrant issued by respondent Judge was based
merely on the application for search warrant and a joint affidavit of private respondents which
were wrongfully it is alleged subscribed, and sworn to before the Clerk of Court of respondent
Judge. Furthermore, there was allegedly a failure on the part of respondent Judge to attach the
necessary papers pertinent to the issuance of the search warrant to the records of Criminal
Case No. 4298-CC wherein petitioner is accused under PD 810, as amended by PD 1306, the
information against him alleging that Soriano Mata offered, took and arranged bets on the Jai
Alai game by "selling illegal tickets known as ‘Masiao tickets’ without any authority from the
Philippine Jai Alai & Amusement Corporation or from the government authorities concerned." 1
Petitioner claims that during the hearing of the case, he discovered that nowhere from the
records of the said case could be found the search warrant and other pertinent papers
connected to the issuance of the same, so that he had to inquire from the City Fiscal its
whereabouts, and to which inquiry respondent Judge replied, "it is with the court." The Judge
then handed the records to the Fiscal who attached them to the records.chanrobles.com : virtual
law library

This led petitioner to file a motion to quash and annul the search warrant and for the return of
the articles seized, citing and invoking, among others, Section 4 of Rule 126 of the Revised
Rules of Court. The motion was denied by respondent Judge on March 1, 1979, stating that the
court has made a thorough investigation and examination under oath of Bernardo U. Goles and
Reynaldo T. Mayote, members of the Intelligence Section of 352nd PC Co./Police District II INP;
that in fact the court made a certification to that effect; and that the fact that documents relating
to the search warrant were not attached immediately to the record of the criminal case is of no
moment, considering that the rule does not specify when these documents are to be attached to
the records. 2 Petitioner’s motion for reconsideration of the aforesaid order having been denied,
he came to this Court, with the instant petition, praying, among others, that this Court declare
the search warrant to be invalid and all the articles confiscated under such warrant as
inadmissible as evidence in the case, or in any proceedings on the matter.

We hold that the search warrant is tainted with illegality for being violative of the Constitution
and the Rules of Court.

Under the Constitution "no search warrant shall issue but upon probable cause to be
determined by the Judge or such other responsible officer as may be authorized by law after
examination under oath or affirmation of the complainant and the witnesses he may produce."
More emphatic and detailed is the implementing rule of the constitutional injunction, Section 4 of
Rule 126 which provides that the judge must before issuing the warrant personally examine on
oath or affirmation the complainant and any witnesses he may produce and take their
depositions in writing, and attach them to the record, in addition to any affidavits presented to
him.

Mere affidavits of the complainant and his witnesses are thus not sufficient. The examining
Judge has to take depositions in writing of the complainant and the witnesses he may produce
and to attach them to the record. Such written deposition is necessary in order that the Judge
may be able to properly determine the existence or non-existence of the probable cause, to hold
liable for perjury the person giving it if it will be found later that his declarations are false.

We, therefore, hold that the search warrant is tainted with illegality by the failure of the Judge to
conform with the essential requisites of taking the depositions in writing and attaching them to
the record, rendering the search warrant invalid.chanroblesvirtualawlibrary
The judge’s insistence that she examined the complainants under oath has become dubious by
petitioner’s claim that at the particular time when he examined all the relevant papers connected
with the issuance of the questioned search warrant, after he demanded the same from the lower
court since they were not attached to the records, he did not find any certification at the back of
the joint affidavit of the complainants. As stated earlier, before he filed his motion to quash the
search warrant and for the return of the articles seized, he was furnished, upon his request,
certified true copies of the said affidavits by the Clerk of Court but which certified true copies do
not bear any certification at the back. Petitioner likewise claims that his xerox copy of the said
joint affidavit obtained at the outset of this case does not show also the certification of
respondent judge. This doubt becomes more confirmed by respondent Judge’s own admission,
while insisting that she did examine thoroughly the applicants, that "she did not take the
deposition of Mayote and Goles because to have done so would be to hold a judicial proceeding
which will be open and public", 3 such that, according to her, the persons subject of the
intended raid will just disappear and move his illegal operations somewhere else.

Could it be that the certification was made belatedly to cure the defect of the warrant? Be that
as it may, there was no "deposition in writing" attached to the records of the case in palpable
disregard of the statutory prohibition heretofore quoted.

Respondent Judge impresses this Court that the urgency to stop the illegal gambling that lures
every man, woman and child, and even the lowliest laborer who could hardly make both ends
meet justifies her action. She claims that in order to abate the proliferation of this illegal
"masiao" lottery, she thought it more prudent not to conduct the taking of deposition which is
done usually and publicly in the court room.

Two points must be made clear. The term "depositions" is sometimes used in a broad sense to
describe any written statement verified by oath; but in its more technical and appropriate sense
the meaning of the word is limited to written testimony of a witness given in the course of a
judicial proceeding in advance of the trial or hearing upon oral examination. 4 A deposition is the
testimony of a witness, put or taken in writing, under oath or affirmation before a commissioner,
examiner or other judicial officer, in answer to interlocutory and cross interlocutory, and usually
subscribed by the witnesses. 5 The searching questions propounded to the applicants of the
search warrant and his witnesses must depend to a large extent upon the discretion of the
Judge just as long as the answers establish a reasonable ground to believe the commission of a
specific offense and that the applicant is one authorized by law, and said answers particularly
describe with certainty the place to be searched and the persons or things to be seized. The
examination or investigation which must be under oath may not be in public. It may even be
held in the secrecy of his chambers. Far more important is that the examination or investigation
is not merely routinary but one that is thorough and elicit the required information. To repeat, it
must be under oath and must be in writing.cralawnad
The other point is that nothing can justify the issuance of the search warrant but the fulfillment of
the legal requisites. It might be well to point out what has been said in Asian Surety & Insurance
Co., Inc. v. Herrera:jgc:chanrobles.com.ph

"It has been said that of all the rights of a citizen, few are of greater importance or more
essential to his peace and happiness than the right of personal security, and that involves the
exemption of his private affairs, books, and papers from inspection and scrutiny of others. While
the power to search and seize is necessary to the public welfare, still it must be exercised and
the law enforced without transgressing the constitutional rights of the citizens, for the
enforcement of no statute is of sufficient importance to justify indifference to the basic principles
of government." 6

Thus, in issuing a search warrant the Judge must strictly comply with the requirements of the
Constitution and the statutory provisions. A liberal construction should be given in favor of the
individual to prevent stealthy encroachment upon, or gradual depreciation of the rights secured
by the Constitution. 7 No presumption of regularity are to be invoked in aid of the process when
an officer undertakes to justify it. 8

While We hold that the search warrant is illegal, the return of the things seized cannot be
ordered. In Castro v. Pabalan, 9 it was held that the illegality of the search warrant does not call
for the return of the things seized, the possession of which is prohibited.

WHEREFORE, the writ of certiorari is granted and the order of March 1, 1979 denying the
motion to annul the search warrant as well as the order of March 21, 1979 denying the motion
for reconsideration are hereby reversed, the search warrant, being declared herein as illegal.
Notwithstanding such illegality, the things seized under such warrant, such as stock of "masiao"
tickets; "masiao" issue tickets; bet money; control pad or "masiao" numbers; stamping pad with
rubber stamp marked Ormoc City Jai-Alai," cannot be returned as sought by petitioner. No
costs.

SO ORDERED.
G.R. No. 109633 July 20, 1994

THE PEOPLE OF THE PHILIPPINES, plaintiff-appellee,


vs.
NORMANDO DEL ROSARIO Y LOPEZ, accused-appellant.
The Solicitor General for plaintiff-appellee.

Topacio and Topacio for accused-appellants.

MELO, J.:

Normando del Rosario was charged before Branch 17 of the Regional Trial Court of the Fourth
Judicial Region stationed in Cavite City with Illegal Possession of Firearm and Ammunitions in
Criminal Case No. 236-91 and Illegal Sale of Regulated Drugs in Criminal Case No. 237-91,
under two informations reading, respectively, as follows:

Criminal Case No. 236-91

That on or about September 4, 1991, in the City of Cavite, Republic of the Philippines and within
the jurisdiction of this Honorable Court, the above-named accused, without legal authority, did,
then and there, willfully, unlawfully, feloniously and knowingly have in his possession and
control a homemade (paltik)caliber .22 revolver with three (3) live ammunition.

Contrary to law.

Criminal Case No. 237-91

That on or about September 4, 1991, in the City of Cavite, Republic of the Philippines and within
the jurisdiction of this Honorable Court, the above-named accused, without legal authority, did,
then and there, willfully, unlawfully, feloniously and knowingly sell to a poseur buyer an
aluminum foil containing Methamphetamine Hydrochloride also known as "Shabu", a regulated
drug.

Contrary to law.

(pp. 20-21, Rollo.)


Upon arraignment, accused-appellant pleaded not guilty to both charges, and after joint trial of
the two cases, the court a quo rendered a decision, the dispositive portion of which reads:

WHEREFORE, in view of the foregoing, the Court finds the accused Normando del Rosario y
Lopez guilty beyond reasonable doubt in the above-entitled cases and he is hereby sentenced
to undergo imprisonment: in Crim. Case No. 236-91 for Violation of P.D. 1866 of Seventeen (17)
years, Four (4) months and One (1) day of reclusion temporal, as minimum to Twenty (20) years
of reclusion temporal, as maximum and in Crim. Case No. 237-91 for a violation of Section 15,
Article III of Republic Act 6425, as amended of life imprisonment and to pay a fine of
P30,000.00, without subsidiary imprisonment in case of insolvency and to pay the costs in both
cases.

The shabu, the One Hundred Peso bill and other paraphernalia are hereby ordered confiscated
in favor of the government.

(pp. 28-29, Rollo.)

From said decision, the instant appeal has been interposed.

The prosecution's version of the case, as set forth in appellee's brief, is as follows:

Upon application of SPO3 Raymundo Untiveros of the Philippine National Police (PNP) of
Cavite City, Regional Trial Court Judge Arturo de Guia issued in the morning of September 4,
1991 a search warrant
(Exh. T, p. 50, Rec.— Crim. Case No. 237-91) authorizing the search and seizure of an
"undetermined quantity of Methamphetamine Hydrochloride commonly known as shabu and its
paraphernalias" in the premises of appellant's house located at 828 R. Basa St., San Roque,
Cavite City. However, the search warrant was not implemented immediately due to the lack of
police personnel to form the raiding team (pp. 4, 7, tsn., Feb. 4, 1992).

At about 9 o'clock in the evening of that day, a raiding team was finally organized. SPO3
Untiveros headed the raiding team with PO3 Rogelio Francisco, SPO1 Eduardo Novero, SPO3
Reynaldo de la Cruz, PO1 Carlito Barbuco, PO3 Onrubio and SPO2 Villegas as members
(pp. 5, 10, tsn., Feb. 4, 1992; p. 7, tsn., Dec. 11, 1991).
In the final briefing of the raiding team at the police station, it was agreed upon that PO1
Venerando Luna will buy shabu from appellant and after his return from appellant's house, the
raiding team will implement the search warrant (p. 10, tsn., Feb. 4, 1992; pp. 17-18, tsn., Dec.
11, 1991). A marked money consisting of a P100 bill bearing serial no. PQ 329406
(Exh. P, p. 51, Rec.) was given by the Station Commander to PO1 Luna and entered in the
police logbook (p. 12, Feb. 4, 1992). PO1 Luna with a companion proceeded to appellant's
house to implement the search warrant. Barangay Capt. Maigue, Norma del Rosario and
appellant witnessed the search at appellant's house (p. 10, tsn., Dec. 11, 1991). SPO3 de la
Cruz and PO3 Francisco found a black canister containing shabu, an aluminum foil, a paltik .22
caliber (Exh. O) atop the TV set, three used ammunitions in a cup and three wallets (Exhs. Q,
R, S), one containing the marked money (Exh. P; pp. 11-12, tsn., Dec. 11, 1992). SPO1 Novero
found inside a show box aluminum foils, napkins and a burner (p. 9, tsn., March 11, 1992).
SPO3 de la Cruz turned over the wallet containing the marked money to PO3 Onrubio (p. 8, 32,
tsn., Jan. 7, 1992). The seized items were photographed thereat by Fred Agana and then turned
over to PO3 Onrubio (pp. 8, 32, tsn., Jan. 7, 1992). SPO3 Untiveros issued receipts (Exhs. V, V-
1, pp. 53-54, Rec.) for the seized items with Barangay Capt. Maigue and appellant's sister
Norma as signing witnesses. He also made a return (Exh. U, p. 52, Rec.) of the seized items to
the court (pp. 11-155, tsn., Feb. 18, 1992.).

At police station, the seized items were taped and initialed by SPO3 de la Cruz (p. 33, tsn., Jan.
7, 1992). The next day, SPO4 Pilapil, through PO1 Barbuco, forwarded to NBI Forensic Chemist
Mary Ann Aranas for laboratory analysis the aluminum foil (Exhs. A, J, pp. 37, 46, Rec.)
containing suspected shabu bought by PO1 Luna from appellant in the
buy-bust operation as well as the aluminum foils (Exhs. G, K, pp. 43, 47, Rec.) containing
suspected marijuana which were confiscated by virtue of the search warrant.

The findings of NBI Forensic Chemist Aranas disclosed that all the specimen submitted to her
for laboratory analysis by SPO1 Pilapil, thru PO1 Barbuco, gave positive results for
Methamphetamine Hydrochloride (pp. 2-9, tsn., Dec. 3, 1991; Exh. B, C, H, I, pp. 38, 39, 44, 45,
Rec.).

(pp. 102-105, Rollo.)

Carefully evaluating the evidence on record, we believe that the prosecution has failed to prove
the guilt of accused-appellant. Much is to be desired in the manner the police authorities
effected the arrest of accused-appellant and the same observation may be made with regard to
the way the prosecution conducted its case.

Foremost among the inadequacies of the prosecution is its failure to call to the witness stand
PO1 Venerando Luna, the alleged poseur-buyer. There is, thus, a total absence of evidence to
establish the purported sale of shabu by accused-appellant to Venerando Luna, the supposed
poseur-buyer. The omission to present the poseur-buyer casts serious doubts that an illegal
sale of a dangerous drug actually took place.

The trial court gave much weight to the testimonies of the police members of the buy-bust
operation. However, the prosecution did not present as witness the supposed poseur-buyer.
Such omission casts serious doubt on appellant's guilt because without the testimony of the
poseur-buyer, there is no convincing evidence to show that appellant sold marijuana. The
testimonies of the rest of the buy-bust operation are hearsay in view of the fact that the poseur-
buyer, was never presented at the trial. There was even no testimony that when the accused-
appellant handed the stuff to the poseur-buyer that the latter in turn handed the marked money.
The failure of the prosecution to present the alleged buyer of the marijuana was a fatal flaw in
the case against the accused.

(People vs. Fulgarillas, 212 SCRA 76, 80 [1992])

The testimony of prosecution witness PO3 Rogelio Francisco that Veneracion Luna, the alleged
Poseur-buyer, bought shabu from accused-appellant was derived solely from what Luna
supposedly told him (pp. 19-20, tsn., December 11, 1991) and, therefore, is patently hearsay
evidence, without any evidentiary weight whatsoever. Likewise, the statements of prosecution
witnesses Policemen Reynaldo de la Cruz, Raymundo Untiveros, and Eduardo Novera, Jr. as to
the alleged sale of shabu are hearsay, without weight, as all of them were not present during the
alleged sale.

According to the version of the prosecution, during the alleged buy-bust operation, accused-
appellant handed over to Veneracion Luna, the alleged poseur-buyer, a quantity of shabu, and
Luna in turn paid accused-appellant a marked P100 bill and then returned to the police station
and informed the raiding team that he had already bought the shabu from accused-appellant.
Thereupon, the raiding team proceeded to the house of accused-appellant to implement the
search warrant. The version of the prosecution is highly incredible. The record is devoid of any
reason why the police officers did not make any attempt to arrest accused-appellant at the time
he allegedly sold the shabu to Veneracion Luna who was accompanied by another police
officer. That was the opportune moment to arrest accused-appellant. The version foisted by the
prosecution upon this Court is contrary to human experience in the ordinary course of human
conduct. The usual procedure in a buy-bust operation is for the police officers to arrest the
pusher of drugs at the very moment he hands over the dangerous drug to the poseur-buyer.
That is the very reason why such a police operation is called a "buy-bust" operation. The police
poseur-buyer "buys" dangerous drugs from the pusher and "busts" (arrests) him the moment the
pusher hands over the drug to the police officer.
We thus entertain serious doubts that the shabu contained in a small canister was actually
seized or confiscated at the residence of accused-appellant. In consequence, the manner the
police officers conducted the subsequent and much-delayed search is highly irregular. Upon
bargaining into the residence of accused-appellant, the police officers found him lying down and
they immediately arrested and detained him in the living room while they searched the other
parts of the house. Although they fetched two persons to witness the search, the witnesses
were called in only after the policemen had already entered accused-appellant's residence (pp.
22-23, tsn, December 11, 1991), and, therefore, the policemen had more than ample time to
plant the shabu. Corollary to the constitutional precept that, in all criminal prosecutions, the
accused shall be presumed innocent until the contrary is proved (Sec. 14(2), Article III,
Constitution of the Republic of the Philippines) is the rule that in order to convict an accused the
circumstances of the case must exclude all and each and every hypothesis consistent with his
innocence (People vs. Tanchoco; 76 Phil. 463 [1946]; People vs. Constante, 12 SCRA 653
[1964]; People vs. Jara, 144 SCRA 516 [1986]). The facts of the case do not rule out the
hypothesis that accused- appellant is innocent.

At any rate, accused-appellant cannot be convicted of possession of the shabu contained in a


canister and allegedly seized at his house, for the charge against him was for selling shabu with
the information alleging that the "accused, without legal authority did . . . sell to a poseur buyer
an aluminum foil containing Methamphetamine Hydrochloride . . ." Sale is totally different from
possession. Article 1458 of the Civil Code defines sale as a contract whereby "one of the
contracting parties obligates himself to transfer the ownership of and to deliver a determine
thing, and the other to pay therefor a price certain in money or its equivalent", while "possession
is the holding of a thing or the enjoyment of a right" as defined by Article 523 of the Civil Code.
Accused-appellant cannot be convicted of a crime which is not charged in the information for to
do so would deny him the due process of law (People vs. Despavellador, 1 SCRA 205 [1961];
People vs. Mori, 55 SCRA 382 [1974]).

Neither can accused-appellant be convicted of illegal possession of firearm and ammunition.


The search warrant implemented by the raiding party authorized only the search and seizure of
". . . the described quantity of Methamphetamine Hydrochloride commonly known as shabu and
its paraphernalia" (Exh. O, p. 50, original record). Thus, the raiding party was authorized to
seize only shabu and paraphernalia for the use thereof and no other. A search warrant is not a
sweeping authority empowering a raiding party to undertake a finishing expedition to seize and
confiscate any and all kinds of evidence or articles relating to a crime. The Constitution itself
(Section 2, Article III) and the Rules of Court (Section 3, Rule 126) specifically mandate that the
search warrant must particularly describe the things to be seized. Thus, the search warrant was
no authority for the police officers to seize the firearm which was not mentioned, much less
described with particularity, in the search warrant. Neither may it be maintained that the gun
was seized in the course of an arrest, for as earlier observed, accused-appellant's arrest was far
from regular and legal. Said firearm, having been illegally seized, the same is not admissible in
evidence (Stonehill vs. Diokno, 20 SCRA 383 [1967]). The Constitution expressly ordains the
exclusion in evidence of illegally seized articles.
Any evidence obtained in violation of this or the preceding section shall be inadmissible for any
purpose in any proceeding.

(Section 3[2], Article III, Constitution of the Republic of the Philippines).

With the exclusion in evidence of the illegally seized firearm, there is, therefore, a total absence
of evidence to support the charge of illegal possession of firearm, against accused-appellant.

The same may be said of the charge of illegal possession of ammunition.

WHEREFORE, the decision appealed from is hereby REVERSED and accused-appellant is


hereby ACQUITTED in Criminal Case No. 236-91 and Criminal Case No. 237-91.

The immediate release of accused-appellant is hereby ordered unless there exists a pending
valid cause against him.

The shabu, the marked P100 bill, firearm, and ammunition are hereby ordered confiscated in
favor of the government.

SO ORDERED.
G.R. No. 95847-48. March 10, 1993.

PEOPLE OF THE PHILIPPINES, plaintiff-appellee, vs. GABRIEL GERENTE y BULLO,


accused-appellant.

The Solicitor General for plaintiff-appellee.

Public Attorney's Office for accused-appellant.

SYLLABUS
1. REMEDIAL LAW; CRIMINAL PROCEDURE; ARREST WITHOUT WARRANT; LAWFUL
WHEN ARRESTING OFFICER HAS PERSONAL KNOWLEDGE THAT THE PERSON TO BE
ARRESTED HAS COMMITTED THE CRIME; CASE AT BAR. — The policemen arrested
Gerente only some three (3) hours after Gerente and his companions had killed Blace. They
saw Blace dead in the hospital and when they inspected the scene of the crime, they found the
instruments of death: a piece of wood and a concrete hollow block which the killers had used to
bludgeon him to death. The eye-witness, Edna Edwina Reyes, reported the happening to the
policemen and pinpointed her neighbor, Gerente, as one of the killers. Under those
circumstances, since the policemen had personal knowledge of the violent death of Blace and
of facts indicating that Gerente and two others had killed him, they could lawfully arrest Gerente
without a warrant. If they had postponed his arrest until they could obtain a warrant, he would
have fled the law as his two companions did.

2. ID.; ID.; SEARCH AND SEIZURE; VALID EVEN WITHOUT A WARRANT WHEN MADE AS
AN INCIDENT TO LAWFUL ARREST; RATIONALE. — The search conducted on Gerente's
person was likewise lawful because it was made as an incident to a valid arrest. This is in
accordance with Section 12, Rule 126 of the Revised Rules of Court which provides: "Section
12. Search incident to lawful arrest. — A person lawfully arrested may be searched for
dangerous weapons or anything which may be used as proof of the commission of an offense,
without a search warrant." The frisk and search of appellant's person upon his arrest was a
permissible precautionary measure of arresting officers to protect themselves, for the person
who is about to be arrested may be armed and might attack them unless he is first disarmed. In
Adams vs. Williams, 47 U.S. 143, cited in Justice Isagani A. Cruz's Constitutional Law, 1991
Edition, p. 150, it was ruled that "the individual being arrested may be frisked for concealed
weapons that may be used against the arresting officer and all unlawful articles found his
person, or within his immediate control may be seized."

3. CRIMINAL LAW; CONSPIRACY; LIABILITY OF CONSPIRATORS; RULE; CASE AT BAR. —


There is no merit in appellant's allegation that the trial court erred in convicting him of having
conspired and cooperated with Fredo and Totoy Echigoren to kill Blace despite the testimony of
Dr. Valentin Bernales that the fracture on the back of the victim's skull could have been inflicted
by one person only. what Dr. Bernales stated was a mere possibility that only one person
dropped the concrete hollow block on the head of the victim, smashing it. That circumstance,
even if true, does not absolve the other two co-conspirators in the murder of Blace for when
there is a conspiracy to commit a crime, the act of one conspirator is the act of all. The
conspiracy was proven by the eyewitness-testimony of Edna Edwina Reyes, that she overheard
the appellant and his companions conspire to kill Blace, that acting in concert, they attacked
their victim with a piece of wood and a hollow block and caused his death. "When there is no
evidence indicating that the principal witness for the prosecution was moved by improper
motive, the presumption is that he was not so moved and his testimony is entitled to full faith
and credit" (People vs. Belibet, 199 SCRA 587, 588). Hence, the trial court did not err in giving
full credit to Edna Reyes' testimony.
4. ID.; CIVIL INDEMNITY FOR DEATH; INCREASED TO P50,000.00. — The Solicitor General
correctly pointed out in the appellee's brief that the award of P30,000.00 as civil indemnity for
the death of Clarito Blace should be increased to P50,000.00 in accordance with our ruling in
People vs. Sison, 189 SCRA 643.

DECISION

GRIÑO-AQUINO, J p:

This is an appeal from the decision of the Regional Trial Court of Valenzuela, Metro Manila,
Branch 172, which found the appellant guilty of Violation of Section 8 of Republic Act 6425
(Dangerous Drugs Act of 1972) and sentenced him to suffer the penalty of imprisonment for a
term of twelve (12) years and one (1) day, as minimum, to twenty (20) years, as maximum; and
also found him guilty of Murder for which crime he was sentenced to suffer the penalty of
reclusion perpetua. The dispositive portion of the appealed decision reads:

"WHEREFORE, in view of the foregoing the Court finds the accused Gabriel Gerente in Criminal
Case No. 10255-V-90 guilty beyond reasonable doubt of Violation of Section 8 of R.A. 6425 and
hereby sentences him to suffer the penalty of imprisonment of twelve years and one day as
minimum to twenty years as maximum, and a fine of twelve thousand, without subsidiary
imprisonment in case of insolvency, and to pay the costs.

"In Criminal Case No. 10256-V-90, the Court finds the accused Gabriel Gerente guilty beyond
reasonable doubt of the crime of Murder, and there by (sic) no aggravating circumstances nor
mitigating circumstances, is hereby sentenced to suffer the penalty of reclusion perpetua; to
indemnify the heirs of the victim in the sum of P30,000.00, and in the amount of P17,609.00 as
funeral expenses, without subsidiary imprisonment in case of insolvency, and to pay the costs.
The accused Gabriel Gerente shall be credited with the full term of his preventive
imprisonment." (p. 25, Rollo.)

Appellant Gabriel Gerente y Bullo was charged with Violation of Section 8, Art. II of R.A. 6425,
which was docketed as Criminal Case No. 10255-V-90 of the Regional Trial Court of
Valenzuela, Metro Manila. The Information reads:

"That on or about the 30th day of April, 1990, in the municipality of Valenzuela, Metro Manila,
Philippines, and within the jurisdiction of this Honorable Court, the above-named accused,
without justification, did then and there wilfully, unlawfully and feloniously have in his possession
and control dried flowering tops wrapped in foil with markings and place in a transparent plastic
bag which are considered prohibited drugs." (p. 2, Rollo.)
The same accused, together with Totoy and Fredo Echigoren who are both at large, was
charged with Murder in Criminal Case No. 10256-V-90 in an information of the same date and
signed by the same Assistant Provincial Prosecutor, as follows:

"That on or about the 30th day of April, 1990, in the municipality of Valenzuela, Metro Manila,
Philippines, and within the jurisdiction of this Honorable Court, the above-named accused
together with two (2) others who are still at large and against whom the preliminary investigation
has not yet been terminated by the Office of the Provincial Prosecutor of Bulacan, conspiring,
confederating together and mutually helping one another, armed with a piece of wood and
hallow (sic) block and with intent to kill one Clarito B. Blace, did then and there wilfully,
unlawfully and feloniously, with evident premeditation and treachery, attack, assault and hit with
the said piece of wood and hollow block the said Clarito B. Blace, hitting the latter on the
different parts of his body, thereby inflicting serious physical injuries which directly caused the
death of the said victim." (p. 3, Rollo.)

Edna Edwina Reyes testified that at about 7:00 a.m. of April 30, 1990, appellant Gabriel
Gerente, together with Fredo Echigoren and Totoy Echigoren, started drinking liquor and
smoking marijuana in the house of the appellant which is about six (6) meters away from the
house of the prosecution witness who was in her house on that day. She overheard the three
men talking about their intention to kill Clarito Blace. She testified that she heard Fredo
Echigoren saying, "Gabriel, papatayin natin si Clarito Blace," and Totoy Echigoren allegedly
seconded Fredo's suggestion saying: "Papatayin natin 'yan mamaya." Appellant allegedly
agreed: "Sigue, papatayin natin mamaya." (pp. 3-4, tsn, August 24, 1990.)

Fredo and Totoy Echigoren and Gerente carried out their plan to kill Clarito Blace at about 2:00
p.m. of the same day. The prosecution witness, Edna Edwina Reyes, testified that she
witnessed the killing. Fredo Echigoren struck the first blow against Clarito Blace, followed by
Totoy Echigoren and Gabriel Gerente who hit him twice with a piece of wood in the head and
when he fell, Totoy Echigoren dropped a hollow block on the victim's head. Thereafter, the three
men dragged Blace to a place behind the house of Gerente.

At about 4:00 p.m. of the same day, Patrolman Jaime Urrutia of the Valenzuela Police Station
received a report from the Palo Police Detachment about a mauling incident. He went to the
Valenzuela District Hospital where the victim was brought. He was informed by the hospital
officials that the victim died on arrival. The cause of death was massive fracture of the skull
caused by a hard and heavy object. Right away, Patrolman Urrutia, together with Police
Corporal Romeo Lima and Patrolman Alex Umali, proceeded to Paseo de Blas where the
mauling incident took place. There they found a piece of wood with blood stains, a hollow block
and two roaches of marijuana. They were informed by the prosecution witness, Edna Edwina
Reyes, that she saw the killing and she pointed to Gabriel Gerente as one of the three men who
killed Clarito.
The policemen proceeded to the house of the appellant who was then sleeping. They told him to
come out of the house and they introduced themselves as policemen. Patrolman Urrutia frisked
appellant and found a coin purse in his pocket which contained dried leaves wrapped in
cigarette foil. The dried leaves were sent to the National Bureau of Investigation for
examination. The Forensic Chemist found them to be marijuana.

Only the appellant, Gabriel Gerente, was apprehended by the police. The other suspects, Fredo
and Totoy Echigoren, are still at large.

On May 2, 1990, two separate informations were filed by Assistant Provincial Prosecutor
Benjamin Caraig against him for Violation of Section 8, Art. II, of R.A. 6425, and for Murder.

When arraigned on May 16, 1990, the appellant pleaded not guilty to both charges. A joint trial
of the two cases was held. On September 24, 1990, the trial court rendered a decision
convicting him of Violation of Section 8 of R.A. 6425 and of Murder.

In this appeal of the appellant, the following errors are ascribed to the trial court:

1. the court a quo gravely erred in admitting the marijuana leaves adduced in evidence by the
prosecution; and

2. the court a quo gravely erred in convicting the accused-appellant of the crimes charged
despite the absence of evidence required to prove his guilt beyond reasonable doubt.

The appellant contends that the trial court erred in admitting the marijuana leaves as evidence
in violation of his constitutional right not to be subjected to illegal search and seizure, for the
dried marijuana leaves were seized from him in the course of a warrantless arrest by the police
officers. We do not agree.

The search of appellant's person and the seizure of the marijuana leaves in his possession
were valid because they were incident to a lawful warrantless arrest.

Paragraphs (a) and (b), Section 5, Rule 113 of the Revised Rules of Court provide:
'SECTION 5. Arrest without warrant; when lawful. — A peace officer or a private person may,
without a warrant, arrest a person:

"(a) When, in his presence, the person to be arrested has committed, is actually committing, or
is attempting to commit an offense;"

"(b) When an offense has in fact just been committed, and he has personal knowledge of facts
indicating that the person to be arrested has committed it; . . .'

The policemen arrested Gerente only some three (3) hours after Gerente and his companions
had killed Blace. They saw Blace dead in the hospital and when they inspected the scene of the
crime, they found the instruments of death: a piece of wood and a concrete hollow block which
the killers had used to bludgeon him to death. The eye-witness, Edna Edwina Reyes, reported
the happening to the policemen and pinpointed her neighbor, Gerente, as one of the killers.
Under those circumstances, since the policemen had personal knowledge of the violent death of
Blace and of facts indicating that Gerente and two others had killed him, they could lawfully
arrest Gerente without a warrant. If they had postponed his arrest until they could obtain a
warrant, he would have fled the law as his two companions did.

In Umil vs. Ramos, 187 SCRA 311, the arrest of the accused without a warrant was effected
one (1) day after he had shot to death two Capcom soldiers. The arrest was held lawful by this
Court upon the rationale stated by us in People vs. Malasugui, 63 Phil. 221, 228, thus:

"To hold that no criminal can, in any case, be arrested and searched for the evidence and
tokens of his crime without a warrant, would be to leave society, to a large extent, at the mercy
of the shrewdest, the most expert, and the most depraved of criminals, facilitating their escape
in many instances."

The search conducted on Gerente's person was likewise lawful because it was made as an
incident to a valid arrest. This is in accordance with Section 12, Rule 126 of the Revised Rules
of Court which provides:

"SECTION 12. Search incident to lawful arrest. — A person lawfully arrested may be searched
for dangerous weapons or anything which may be used as proof of the commission of an
offense, without a search warrant."

The frisk and search of appellant's person upon his arrest was a permissible precautionary
measure of arresting officers to protect themselves, for the person who is about to be arrested
may be armed and might attack them unless he is first disarmed. In Adams vs. Williams, 47
U.S. 143, cited in Justice Isagani A. Cruz's Constitutional Law, 1991 Edition, p. 150, it was ruled
that "the individual being arrested may be frisked for concealed weapons that may be used
against the arresting officer and all unlawful articles found in his person, or within his immediate
control may be seized."

There is no merit in appellant's allegation that the trial court erred in convicting him of having
conspired and cooperated with Fredo and Totoy Echigoren to kill Blace despite the testimony of
Dr. Valentin Bernales that the fracture on the back of the victim's skull could have been inflicted
by one person only.

What Dr. Bernales stated was a mere possibility that only one person dropped the concrete
hollow block on the head of the victim, smashing it. That circumstance, even if true, does not
absolve the other two co-conspirators in the murder of Blace for when there is a conspiracy to
commit a crime, the act of one conspirator is the act of all. The conspiracy was proven by the
eyewitness-testimony of Edna Edwina Reyes, that she overheard the appellant and his
companions conspire to kill Blace, that acting in concert, they attacked their victim with a piece
of wood and a hollow block and caused his death. "When there is no evidence indicating that
the principal witness for the prosecution was moved by improper motive, the presumption is that
he was not so moved and his testimony is entitled to full faith and credit" (People vs. Belibet,
199 SCRA 587, 588). Hence, the trial court did not err in giving full credit to Edna Reyes'
testimony.

Appellant's failure to escape (because he was very drunk) is no indicium of his innocence.

The Solicitor General correctly pointed out in the appellee's brief that the award of P30,000.00
as civil indemnity for the death of Clarito Blace should be increased to P50,000.00 in
accordance with our ruling in People vs. Sison, 189 SCRA 643.

WHEREFORE, the appealed decision is hereby AFFIRMED, with modification of the civil
indemnity awarded to the heirs of the victim, Clarito Blace, which is hereby increased to
P50,000.00.

SO ORDERED.

UMIL vs. RAMOS

PER CURIAM:
The are eight (8) petitioners for habeas corpus filed before the Court, which have been
consolidated because of the similarity of issues raised, praying for the issuance of the writ
of habeas corpus, ordering the respective respondents to produce the bodies of the persons
named therein and to explain why they should not be set at liberty without further delay.

In their respective Returns, the respondents uniformly assert that the privilege of the writ
of habeas corpus is not available to the petitioners as they have been legally arrested and are
detained by virtue of valid informations filed in court against them.

The petitioners counter that their detention is unlawful as their arrests were made without
warrant and, that no preliminary investigation was first conducted, so that the informations filed
against them are null and void.

The Court has carefully reviewed the contentions of the parties in their respective pleadings,
and it finds that the persons detained have not been illegally arrested nor arbitrarily deprived of
their constitutional right to liberty, and that the circumstances attending these cases do not
warrant their release on habeas corpus.

The arrest of a person without a warrant of arrest or previous complaint is recognized in law.
The occasions or instances when such an arrest may be effected are clearly spelled out in
Section 5, Rule 113 of the Rules of Court, as amended, which provides:

Sec. 5. Arrest without warrant; when lawful. — A peace officer or a private


person may, without a warrant, arrest a person:

(a) When, in his presence, the person to be arrested has committed, is actually
committing, or is attempting to commit an offense;

(b) When an offense has in fact just been committed, and he has personal
knowledge of facts indicating that the person to be arrested has committed it; and

(c) When the person to be arrested is a prisoner who has escaped from a penal
establishment or place where he is serving final judgment or temporarily confined
while his case is pending, or has escaped while being transferred from one
confinement to another.

In cases falling under paragraphs (a) and (b) hereof, the person arrested without
a warrant shall be forthwith delivered to the nearest police station or jail, and he
shall be proceeded against in accordance with Rule 112, Section 7.

An arrest without a warrant of arrest, under Section 5 paragraphs (a) and (b) of Rule 113 of the
Rules of Court, as amended, is justified when the person arrested is caught in flagranti
delicto, viz., in the act of committing an offense; or when an offense has just been committed
and the person making the arrest has personal knowledge of the facts indicating that the person
arrested has committed it. The rationale behind lawful arrests, without warrant, was stated by
this Court in the case of People vs. Kagui Malasugui 1 thus:

To hold that no criminal can, in any case, be arrested and searched for the
evidence and tokens of his crime without a warrant, would be to leave society, to
a large extent, at the mercy of the shrewdest, the most expert, and the most
depraved of criminals, facilitating their escape in many instances.

The record of the instant cases would show that the persons in whose behalf these petitions
for habeas corpus have been filed, had freshly committed or were actually committing an
offense, when apprehended, so that their arrests without a warrant were clearly justified, and
that they are, further, detained by virtue of valid informations filed against them in court.

A brief narration of the facts and events surrounding each of the eight (8) petitions is in order.

In G.R. No. 81567 (Umil vs. Ramos), the record shows that, on 1 February 1988, the Regional
Intelligence Operations Unit of the Capital Command (RIOU-CAPCOM) received confidential
information about a member of the NPA Sparrow Unit (liquidation squad) being treated for a
gunshot wound at the St. Agnes Hospital in Roosevelt Avenue, Quezon City. Upon verification,
it was found that the wounded person, who was listed in the hospital records as Ronnie Javelon,
is actually Rolando Dural, a member of the NPA liquidation squad, responsible for the killing of
two (2) CAPCOM soldiers the day before, or on 31 January 1988, in Macanining Street, Bagong
Barrio, Caloocan City. In view of this verification, Rolando Dural was transferred to the Regional
Medical Services of the CAPCOM, for security reasons. While confined thereat, or on 4
February 1988, Rolando Dural was positively identified by eyewitnesses as the gunman who
went on top of the hood of the CAPCOM mobile patrol car, and fired at the two (2) CAPCOM
soldiers seated inside the car identified as T/Sgt. Carlos Pabon and CIC Renato Manligot.

As a consequence of this positive identification, Rolando Dural was referred to the Caloocan
City Fiscal who conducted an inquest and thereafter filed with the Regional Trial Court of
Caloocan City an information charging Rolando Dural alias Ronnie Javelon with the crime of
"Double Murder with Assault Upon Agents of Persons in Authority." The case was docketed
therein as Criminal Case No. C-30112 and no bail was recommended. On 15 February 1988,
the information was amended to include, as defendant, Bernardo Itucal, Jr. who, at the filing of
the original information, was still unidentified.

Meanwhile, on 6 February 1988, a petition for habeas corpus was filed with this Court on behalf
of Roberto Umil, Rolando Dural, and Renato Villanueva. The Court issued the writ of habeas
corpus on 9 February 1988 and the respondents filed a Return of the Writ on 12 February 1988.
Thereafter, the parties were heard on 15 February 1988.

On 26 February 1988, however, Roberto Umil and Renato Villanueva posted bail before the
Regional Trial Court of Pasay City where charges for violation of the Anti-Subversion Act had
been filed against them, and they were accordingly released. The petition for habeas corpus,
insofar as Umil and Villanueva are concerned, is now moot and academic and is accordingly
dismissed, since the writ of habeas corpus does not lie in favor of an accused in a criminal case
who has been released on bail. 2

As to Rolando Dural, it clearly appears that he was not arrested while in the act of shooting the
two (2) CAPCOM soldiers aforementioned. Nor was he arrested just after the commission of the
said offense for his arrest came a day after the said shooting incident. Seemingly, his arrest
without warrant is unjustified.
However, Rolando Dural was arrested for being a member of the New Peoples Army (NPA), an
outlawed subversive organization. Subversion being a continuing offense, the arrest of Rolando
Dural without warrant is justified as it can be said that he was committing an offense when
arrested. The crimes of rebellion, subversion, conspiracy or proposal to commit such crimes,
and crimes or offenses committed in furtherance thereof or in connection therewith constitute
direct assaults against the State and are in the nature of continuing crimes. As stated by the
Court in an earlier case:

From the facts as above-narrated, the claim of the petitioners that they were
initially arrested illegally is, therefore, without basis in law and in fact. The crimes
of insurrection or rebellion, subversion, conspiracy or proposal to commit such
crimes, and other crimes and offenses committed in the furtherance, on the
occasion thereof, or incident thereto, or in connection therewith under
Presidential Proclamation No. 2045, are all in the nature of continuing offenses
which set them apart from the common offenses, aside from their essentially
involving a massive conspiracy of nationwide magnitude. Clearly then, the arrest
of the herein detainees was well within the bounds of the law and existing
jurisprudence in our jurisdiction.

2. The arrest of persons involved in the rebellion whether as its fighting armed
elements, or for committing non-violent acts but in furtherance of the rebellion, is
more an act of capturing them in the course of an armed conflict, to quell the
rebellion, than for the purpose of immediately prosecuting them in court for a
statutory offense. The arrest, therefore, need not follow the usual procedure in
the prosecution of offenses which requires the determination by a judge of the
existence of probable cause before the issuance of a judicial warrant of arrest
and the granting of bail if the offense is bailable. Obviously, the absence of a
judicial warrant is no legal impediment to arresting or capturing persons
committing overt acts of violence against government forces, or any other milder
acts but equally in pursuance of the rebellious movement. The arrest or capture
is thus impelled by the exigencies of the situation that involves the very survival
of society and its government and duly constituted authorities. If killing and other
acts of violence against the rebels find justification in the exigencies of armed
hostilities which is of the essence of waging a rebellion or insurrection, most
assuredly so in case of invasion, merely seizing their persons and detaining them
while any of these contingencies continues cannot be less justified. . . . 3

The record, moreover, shows that the criminal case filed against Rolando Dural and Bernardo
Itucal, Jr. for "Double Murder, etc." was tried in the court below and at the conclusion thereof, or
on 17 August 1988, Rolando Dural and Bernardo Itucal, Jr. were found guilty of the charge and
sentenced accordingly. Rolando Dural is now serving the sentence imposed upon him by the
trial court. Thus, the writ of habeas corpus is no longer available to him. For, as held in the early
case of U.S. vs. Wilson: 4

In this case, whatever may be said about the manner of his arrest, the fact
remains that the defendant was actually in court in the custody of the law on
March 29, when a complaint sufficient in form and substance was read to him. To
this he pleaded not guilty. The trial followed, in which, and in the judgment of
guilty pronounced by the court, we find no error. Whether, if there were
irregularities in bringing him personally before the court, he could have been
released on a writ of habeas corpus or now has a civil action for damages
against the person who arrested him we need not inquire. It is enough to say that
such irregularities are not sufficient to set aside a valid judgment rendered upon
a sufficient complaint and after a trial free from error.

II

In G.R. Nos. 84581-82 (Roque vs. De Villa), the arrest of Amelia Roque and Wilfredo
Buenaobra, without warrant, is also justified. When apprehended at the house of Renato
Constantino in Marikina Heights, Marikina, Metro Manila, Wilfredo Buenaobra admitted that he
was an NPA courier and he had with him letters to Renato Constantino and other members of
the rebel group. Amelia Roque, upon the other hand, was a member of the National United
Front Commission, in charge of finance, and admitted ownership of subversive documents
found in the house of her sister in Caloocan City. She was also in possession of ammunition
and a fragmentation grenade for which she had no permit or authority to possess.

The record of these two (2) cases shows that on 27 June 1988, one Rogelio Ramos y Ibanes, a
member of the NPA, who had surrendered to the military authorities, told military agents about
the operations of the Communist Party of the Philippines (CPP) and the New Peoples Army
(NPA) in Metro Manila. He identified some of his former comrades as "Ka Mong", a staff
member of the Communications and Transportation Bureau; "Ka Nelia", a staff member in
charge of finance; "Ka Miller", an NPA courier from Sorsogon and Lopez, Quezon; "Ka Ted",
and "Ka Totoy". He also pointed to a certain house occupied by Renato Constantino located in
the Villaluz Compound, Molave St., Marikina Heights, Marikina, Metro Manila, which is used as
a safehouse of the National United Front Commission (NUFC) of the CPP-NPA.

In view of these revelations, the Constantino house was placed under military surveillance and
on 12 August 1988, pursuant to a search warrant issued by Judge Eutropio Migrino of the
Regional Trial Court of Pasig, a search of the house was conducted at about 5:00 o'clock in the
afternoon, by a combined team of the Criminal Investigation Service, National Capital District
(CIS-NCD) and the Constabulary Security Group (CSG). In the course of the search, the
following articles were found and taken under proper receipt:

a) One (1) Colt M16A1 long rifle with defaced serial number;

b) One (1) Cal. .380 ACT/9mm Model PPK/8 SN: 260577 & 2605778;

c) Two (2) fragmentation hand grenades;

d) Fifty-six (56) live ammunition for Cal. 5.56 mm;

e) Five (5) live ammunition for Cal. .380;

f) One (1) ICOM VHF FM Radio Transciever SN: 14903

g) One (1) Regulated power supply 220V AC;

h) One (1) Antennae (adjustable);


i) One (1) Speaker with cord ALEXAR;

j) Voluminous Subversive documents.

When confronted, Renato Constatino could not produce any permit or authority to possess the
firearms, ammunition, radio and other communications equipment. Hence, he was brought to
the CIS Headquarters for investigation. When questioned, he refused to give a written
statement, although he admitted that he was a staff member of the executive committee of the
NUFC and a ranking member of the International Department of the Communist Party of the
Philippines (CPP).

At about 8:00 o'clock in the evening of the same day (12 August 1988), Wilfredo Buenaobra
arrived at the house of Renato Constantino in the Villaluz Compound. When accosted, he
readily admitted to the military agents that he is a regular member of the CPP/NPA and that he
went to the place to deliver letters to "Ka Mong", referring to Renato Constatino, and other
members of the rebel group. On further questioning, he also admitted that he is known as "Ka
Miller" and that he was from Barangay San Pedro, Lopez, Quezon. Among the items taken from
him were the following:

(1) Handwritten letter addressed to "Ka Bing & Co. from A & Co." dated August
11, 1988;

(2) Handwritten letter addressed to "ROD from VIC (Schell datre)" dated August
11, 1988;

(3) Handwritten letter addressed to "Suzie" from "Vic", dated August 11, 1988.

Also found Buenaobra's possession was a piece of paper containing a written but jumbled
telephone number of Florida M. Roque, sister of Amelia Roque alias "Ka Nelia", at 69 Geronimo
St., Caloocan City. Acting on the lead provided as to the whereabouts of Amelia Roque, the
military agents went to the given address the next day (13 August 1988). They arrived at the
place at about 11:00 o'clock in the morning. After identifying themselves as military agents and
after seeking permission to search the place, which was granted, the military agents conducted
a search in the presence of the occupants of the house and the barangay captain of the place,
one Jesus D. Olba.

The military agents found the place to be another safehouse of the NUFC/CPP. They found
ledgers, journals, vouchers, bank deposit books, folders, computer diskettes, and subversive
documents as well as live ammunition for a .38 SPL Winchester, 11 rounds of live ammunition
for a cal. .45, 19 rounds of live ammunition for an M16 Rifle, and a fragmentation grenade. As a
result, Amelia Roque and the other occupants of the house were brought to the PC-CIS
Headquarters at Camp Crame, Quezon City, for investigation. Amelia Roque admitted to the
investigators that the voluminous documents belonged to her and that the other occupants of
the house had no knowledge of them. As a result, the said other occupants of the house were
released from custody.

On 15 August 1988, Amelia Roque was brought to the Caloocan City Fiscal for inquest after
which an information charging her with violation of PD 1866 was filed with the Regional Trial
Court of Caloocan City. The case is docketed therein as Criminal Case No. C-1196. Another
information for violation of the Anti-Subversion Act was filed against Amelia Roque before the
Metropolitan Trial Court of Caloocan City, which is docketed therein as Criminal Case No. C-
150458.

An information for violation of the Anti-Subversion Act was filed against Wilfredo
Buenaobra before the Metropolitan Trial Court of Marikina, Metro Manila. The case is docketed
therein as Criminal Case No. 23715. Bail was set at P4,000.00.

On 24 August 1988, a petition for habeas corpus was filed before this Court on behalf of Amelia
Roque and Wilfredo Buenaobra. At the hearing of the case, however, Wilfredo Buenaobra
manifested his desire to stay in the PC-INP Stockade at Camp Crame, Quezon City. According,
the petition for habeas corpus filed on his behalf is now moot and academic. Only the petition of
Amelia Roque remains for resolution.

The contention of respondents that petitioners Roque and Buenaobra are officers and/or
members of the National United Front Commission (NUFC) of the CPP was not controverted or
traversed by said petitioners. The contention must be deemed admitted. 5 As officers and/or
members of the NUFC-CPP, their arrest, without warrant, was justified for the same reasons
earlier stated vis-a-vis Rolando Dural. The arrest without warrant of Roque was additionally
justified as she was, at the time of apprehension, in possession of ammunitions without license
to possess them.

III

In G.R. Nos. 84583-84 (Anonuevo vs. Ramos), the arrest of Domingo Anonuevo and Ramon
Casiple, without warrant, is also justified under the rules. Both are admittedly members of the
standing committee of the NUFC and, when apprehended in the house of Renato Constatino,
they had a bag containing subversive materials, and both carried firearms and ammunition for
which they had no license to possess or carry.

The record of these two (2) cases shows that at about 7:30 o'clock in the evening of 13 August
1988, Domingo T. Anonuevo and Ramon Casiple arrived at the house of Renato Constatino at
Marikina Heights, Marikina, which was still under surveillance by military agents. The military
agents noticed bulging objects on their waist lines. When frisked, the agents found them to be
loaded guns. Anonuevo and Casiple were asked to show their permit or license to possess or
carry firearms and ammunition, but they could not produce any. Hence, they were brought to PC
Headquarters for investigation. Found in their possession were the following articles:

a) Voluminous subversive documents

b) One (1) Cal. 7.65 MOD 83 2C Pistol SN: 001412 with one (1) magazine for
Cal. 7.65 containing ten (10) live ammunition of same caliber;

c) One (1) Cal. 7.65 Pietro Barreta SN; A18868 last digit tampered with one (1)
magazine containing five (5) live ammunition of same caliber.

At the PC Stockade, Domingo Anonuevo was identified as "Ka Ted", and Ramon Casiple as "Ka
Totoy" of the CPP, by their comrades who had previously surrendered to the military.

On 15 August 1988, the record of the investigation and other documentary evidence were
forwarded to the Provincial Fiscal at Pasig, Metro Manila, who conducted an inquest, after which
Domingo Anonuevo and Ramon Casiple were charged with violation of Presidential Decree No.
1866 before the Regional Trial Court of Pasig, Metro Manila. The cases are docketed therein as
Criminal Cases Nos. 74386 ad 74387, respectively. No bail was recommended.

On 24 August 1988, a petition for habeas corpus was filed with this Court on behalf of Domingo
Anonuevo and Ramon Casiple, alleging that the said Anonuevo and Casiple were unlawfully
arrested without a warrant and that the informations filed against them are null and void for
having been filed without prior hearing and preliminary investigation. On 30 August 1988, the
Court issued the writ of habeas corpus, and after the respondents had filed a Return of the Writ,
the parties were heard.

The petitioners' (Anonuevo and Casiple) claim that they were unlawfully arrested because there
was no previous warrant of arrest, is without merit The record shows that Domingo Anonuevo
and Ramon Casiple were carrying unlicensed firearms and ammunition in their person when
they were apprehended.

There is also no merit in the contention that the informations filed against them are null and void
for want of a preliminary investigation. The filing of an information, without a preliminary
investigation having been first conducted, is sanctioned by the Rules. Sec. 7, Rule 112 of the
Rules of Court, as amended, reads:

Sec. 7. When accused lawfully arrested without a warrant. — When a person is


lawfully arrested without a warrant for an offense cognizable by the Regional
Trial Court the complaint or information may be filed by the offended party, peace
officer or fiscal without a preliminary investigation having been first conducted, on
the basis of the affidavit of the offended party or arresting officer or person.

However, before the filing of such complaint or information, the person arrested
may ask for a preliminary investigation by a proper officer in accordance with this
Rule, but he must sign a waiver of the provisions of Article 125 of the Revised
Penal Code, as amended, with the assistance of a lawyer and in case of non-
availability of a lawyer, a responsible person of his choice. Notwithstanding such
waiver, he may apply for bail as provided in the corresponding rule and the
investigation must be terminated within fifteen (15) days from its inception.

If the case has been filed in court without a preliminary investigation having been
first conducted, the accused may within five (5) days from the time he learns of
the filing of the information, ask for a preliminary investigation with the same right
to adduced evidence in his favor in the manner prescribed in this Rule.

The petitioners Domingo Anonuevo and Ramon Casiple, however, refused to sign a waiver of
the provisions of Article 125 of the Revised Penal Code, as amended. In the informations filed
against them, the prosecutor made identical certifications, as follows:

This is to certify that the accused has been charged in accordance with Sec. 7,
Rule 112 of the 1985 Rules on Criminal Procedure, that no preliminary
investigation was conducted because the accused has not made and signed a
waiver of the provisions of Art. 125 of the Revised Penal Code, as amended; that
based on the evidence presented, there is reasonable ground to believe that the
crime has been committed, and that the accused is probably guilty thereof.
Nor did petitioners ask for a preliminary investigation after the informations had been filed
against them in court. Petitioners cannot now claim that they have been deprived of their
constitutional right to due process.

IV

In G.R. No. 83162 (Ocaya vs. Aguirre), the arrest without warrant, of Vicky Ocaya is justified
under the Rules, since she had with her unlicensed ammunition when she was arrested. The
record of this case shows that on 12 May 1988, agents of the PC Intelligence and Investigation
of the Rizal PC-INP Command, armed with a search warrant issued by Judge Eutropio Migrino
of the Regional Trial Court of Pasig, Metro Manila, conducted a search of a house located at
Block 19, Phase II, Marikina Green Heights, Marikina, Metro Manila, believed to be occupied by
Benito Tiamson, head of the CPP-NPA. In the course of the search, Vicky Ocaya arrived in a
car driven by Danny Rivera. Subversive documents and several rounds of ammunition for a .45
cal. pistol were found in the car of Vicky Ocaya. As a result, Vicky Ocaya and Danny Rivera
were brought to the PC Headquarters for investigation. When Vicky Ocaya could not produce
any permit or authorization to possess the ammunition, an information charging her with
violation of PD 1866 was filed with the Regional Trial Court of Pasig, Metro Manila. The case is
docketed therein as Criminal Case No. 73447. Danny Rivera, on the other hand, was released
from custody.

On 17 May 1988, a petition for habeas corpus was filed, with this Court on behalf of Vicky
Ocaya and Danny Rivera. It was alleged therein that Vicky Ocaya was illegally arrested and
detained, and denied the right to a preliminary investigation.

It would appear, however, that Vicky Ocaya was arrested in flagranti delicto so that her arrest
without a warrant is justified. No preliminary investigation was conducted because she was
arrested without a warrant and she refused to waive the provisions of Article 125 of the Revised
Penal Code, pursuant to Sec. 7, Rule 112 of the Rule of Court, as amended.

The petitioners Vicky Ocaya, Domingo Anonuevo, Ramon Casiple, and Amelia Roque claim that
the firearms, ammunition and subversive documents alleged to have been found in their
possession when they were arrested, did not belong to them, but were "planted" by the military
agents to justify their illegal arrest.

The petitioners, however, have not introduced any evidence to support their aforesaid claim. On
the other hand, no evil motive or ill-will on the part of the arresting officers that would cause the
said arresting officers in these cases to accuse the petitioners falsely, has been shown.
Besides, the arresting officers in these cases do not appear to be seekers of glory and bounty
hunters for, as counsel for the petitioners Anonuevo and Casiple say, "there is absolutely
nothing in the evidence submitted during the inquest that petitioners are on the 'AFP Order of
Battle with a reward of P150,000.00 each on their heads.'" 6 On the other hand, as pointed out
by the Solicitor General, the arrest of the petitioners is not a product of a witch hunt or a fishing
expedition, but the result of an in-depth surveillance of NPA safehouses pointed to by no less
than former comrades of the petitioners in the rebel movement.

The Solicitor General, in his Consolidated Memorandum, aptly observes:


. . . . To reiterate, the focal point in the case of petitioners Roque, Buenaobra,
Anonuevo and Casiple, was the lawful search and seizure conducted by the
military at the residence of Renato Constantino at Villaluz Compound, Molave
St., Marikina Heights, Marikina, Metro Manila. The raid at Constantino's
residence, was not a witch hunting or fishing expedition on the part of the
military. It was a result of an in-depth military surveillance coupled with the leads
provided by former members of the underground subversive organizations. That
raid produced positive results. to date, nobody has disputed the fact that the
residence of Constantino when raided yielded communication equipment,
firearms and ammunitions, as well as subversive documents.

The military agents working on the information provided by Constantino that


other members of his group were coming to his place, reasonably conducted a
"stake-out" operation whereby some members of the raiding team were left
behind the place. True enough, barely two hours after the raid and Constantino's
arrest, petitioner Buenaobra arrived at Constantino's residence. He acted
suspiciously and when frisked and searched by the military authorities, found in
his person were letters. They are no ordinary letters, as even a cursory reading
would show. Not only that, Buenaobra admitted that he is a NPA courier and was
there to deliver the letters to Constantino.

Subsequently, less than twenty four hours after the arrest of Constantino and
Buenaobra, petitioners Anonuevo and Casiple arrived at Constantino's place.
Would it be unreasonable for the military agents to believe that petitioners
Anonuevo and Casiple are among those expected to visit Constantino's
residence considering that Constatino's information was true, in that Buenaobra
did come to that place? Was it unreasonable under the circumstances, on the
part of the military agents, not to frisk and search anyone who should visit the
residence of Constantino, such as petitioners Anonuevo and Casiple? Must this
Honorable Court yield to Anonuevo and Casiple's flimsy and bare assertion that
they went to visit Constantino, who was to leave for Saudi Arabia on the day they
were arrested thereat?

As to petitioner Roque, was it unreasonable for the military authorities to effect


her arrest without warrant considering that it was Buenaobra who provided the
leads on her identity? It cannot be denied that Buenaobra had connection with
Roque. Because the former has the phone number of the latter. Why the
necessity of jumbling Roque's telephone number as written on a piece of paper
taken from Buenaobra's possession? Petitioners Roque and Buenaobra have not
offered any plausible reason so far.

In all the above incidents, respondents maintain that they acted reasonably,
under the time, place and circumstances of the events in question, especially
considering that at the time of petitioner's arrest, incriminatory evidence, i.e,
firearms, ammunitions and/or subversive documents were found in their
possession.

Petitioners, when arrested, were neither taking their snacks nor innocently
visiting a camp, but were arrested in such time, place and circumstances, from
which one can reasonably conclude tat they were up to a sinister plot, involving
utmost secrecy and comprehensive conspiracy.

IV

In. G.R. No. 85727 (Espiritu vs. Lim), the release on habeas corpus of the petitioner Deogracias
Espiritu, who is detained by virtue of an Information for Violation of Article 142 of the Revised
Penal Code (Inciting to Sedition) filed with the Regional Trial Court of Manila, is similarly not
warranted.

The record of the case shows that the said petitioner is the General Secretary of the
Pinagkaisahang Samahan ng Tsuper at Operators Nationwide (PISTON), an association of
drivers and operators of public service vehicles in the Philippines, organized for their mutual aid
and protection.

Petitioner claims that at about 5:00 o'clock in the morning of 23 November 1988, while he was
sleeping in his home located at 363 Valencia St., Sta. Mesa, Manila, he was awakened by his
sister Maria Paz Lalic who told him that a group of persons wanted to hire his jeepney. When he
went down to talk to them, he was immediately put under arrest. When he asked for the warrant
of arrest, the men, headed by Col. Ricardo Reyes, bodily lifted him and placed him in their
owner-type jeepney. He demanded that his sister, Maria Paz Lalic, be allowed to accompany
him, but the men did not accede to his request and hurriedly sped away.

He was brought to Police Station No. 8 of the Western Police District at Blumentritt, Manila
where he was interrogated and detained. Then, at about 9:00 o'clock of the same morning, he
was brought before the respondent Lim and, there and then, the said respondent ordered his
arrest and detention. He was thereafter brought to the General Assignment Section,
Investigation Division of the Western Police District under Police Capt. Cresenciano A. Cabasal
where he was detained, restrained and deprived of his liberty. 7

The respondents claim however, that the detention of the petitioner is justified in view of the
Information filed against him before the Regional Trial Court of Manila, docketed therein as
Criminal Case No. 88-683-85, charging him with violation of Art. 142 of the Revised Penal Code
(Inciting to Sedition).

The respondents also claim that the petitioner was lawfully arrested without a judicial warrant of
arrest since petitioner when arrested had in fact just committed an offense in that in the
afternoon of 22 November 1988, during a press conference at the National Press Club.

Deogracias Espiritu through tri-media was heard urging all drivers and operators
to go on nationwide strike on November 23, 1988, to force the government to
give into their demands to lower the prices of spare parts, commodities, water
and the immediate release from detention of the president of the PISTON (Pinag-
isang Samahan ng Tsuper Operators Nationwide). Further, we heard Deogracias
Espiritu taking the place of PISTON president Medardo Roda and also
announced the formation of the Alliance Drivers Association to go on nationwide
strike on November 23, 1988. 8

Policemen waited for petitioner outside the National Pres Club in order to investigate him, but he
gave the lawmen the slip. 9 He was next seen at about 5:00 o'clock that afternoon at a gathering
of drivers and symphatizers at the corner of Magsaysay Blvd. and Valencia Street, Sta. Mesa,
Manila where he was heard to say:

Bukas tuloy ang welga natin, sumagot na ang Cebu at Bicol na kasali sila, at
hindi tayo titigil hanggang hindi binibigay ng gobyerno ni Cory ang gusto nating
pagbaba ng halaga ng spare parts, bilihin at and pagpapalaya sa ating pinuno na
si Ka Roda hanggang sa magkagulo na. 10 (emphasis supplied)

The police finally caught up with the petitioner on 23 November 1988. He was invited for
questioning and brought to police headquarters after which an Information for violation of Art.
142 of the Revised Penal Code was filed against him before the Regional Trial Court of
Manila. 11

Since the arrest of the petitioner without a warrant was in accordance with the provisions of
Rule 113, Sec. 5(b) of the Rules of Court and that the petitioner is detained by virtue of a valid
information filed with the competent court, he may not be released on habeas corpus. He may,
however be released upon posting bail as recommended. However, we find the amount of the
recommended bail (P60,000.00) excessive and we reduce it to P10,000.00 only.

VII

In G.R. No. 86332 (Nazareno vs. Station Commander), we also find no merit in the submission
of Narciso Nazareno that he was illegally arrested and is unlawfully detained. The record of this
case shows that at about 8:30 o'clock in the morning of 14 December 1988, one Romulo Bunye
II was killed by a group of men near the corner of T. Molina and Mendiola Streets in Alabang,
Muntinglupa, Metro Manila. One of the suspects in the killing was Ramil Regal who was
arrested by the police on 28 December 1988. Upon questioning, Regal pointed to Narciso
Nazareno as on of his companions in the killing of the said Romulo Bunye II. In view thereof, the
police officers, without warrant, picked up Narciso Nazareno and brought him to the police
headquarters for questioning. Obviously, the evidence of petitioner's guilt is strong because on
3 January 1989, an information charging Narciso Nazareno, Ramil Regala, and two (2) others,
with the killing of Romulo Bunye II was filed with the Regional Trial Court of Makati, Metro
Manila. The case is docketed therein as Criminal Case No. 731.

On 7 January 1989, Narciso Nazareno filed a motion to post bail, but the motion was denied by
the trial court in an order dated 10 January 1989, even as the motion to post bail, earlier filed by
his co-accused, Manuel Laureaga, was granted by the same trial court.

On 13 January 1989, a petition for habeas corpus was filed with this Court on behalf of Narciso
Nazareno and on 13 January 1989, the Court issued the writ of habeas corpus, returnable to the
Presiding Judge of the Regional Trial Court of Biñan, Laguna, Branch 24, ordering said court to
hear the case on 30 January 1989 and thereafter resolve the petition.

At the conclusion of the hearing, or on 1 February 1989, the Presiding Judge of the Regional
Trial Court of Biñan, Laguna issued a resolution denying the petition for habeas corpus, it
appearing that the said Narciso Nazareno is in the custody of the respondents by reason of an
information filed against him with the Regional Trial Court of Makati, Metro Manila which had
taken cognizance of said case and had, in fact, denied the motion for bail filed by said Narciso
Nazareno (presumably because of the strength of the evidence against him).
The findings of the Presiding Judge of the Regional Trial Court of Biñan, Laguna are based
upon the facts and the law. Consequently, we will not disturb the same. Evidently, the arrest of
Nazareno was effected by the police without warrant pursuant to Sec. 5(b), Rule 113, Rules of
Court after he was positively implicated by his co-accused Ramil Regala in the killing of Romulo
Bunye
II; and after investigation by the police authorities. As held in People vs. Ancheta: 12

The obligation of an agent of authority to make an arrest by reason of a crime,


does not presuppose as a necessary requisite for the fulfillment thereof, the
indubitable existence of a crime. For the detention to be perfectly legal, it is
sufficient that the agent or person in authority making the arrest has reasonably
sufficient grounds to believe the existence of an act having the characteristics of
a crime and that the same grounds exist to believe that the person sought to be
detained participated therein.

VIII

It is to be noted that, in all the petitions here considered, criminal charges have been filed in the
proper courts against the petitioners. The rule is, that if a person alleged to be restrained of his
liberty is in the custody of an officer under process issued by a court judge, and that the court or
judge had jurisdiction to issue the process or make the order, of if such person is charged
before any court, the writ of habeas corpus will not be allowed. Section 4, Rule 102, Rules of
Court, as amended is quite explicit in providing that:

Sec. 4. When writ is allowed or discharge authorized. — If it appears that the


person alleged to be restrained of his liberty is in the custody of an officer under
process issued by a court or judge or by virtue of a judgment or order of a court
of record, and that the court or judge had jurisdiction to issue the process, render
the judgment, or make the order, the writ shall not be allowed; or if the jurisdiction
appears after the writ is allowed, the person shall not be discharged by reason of
any informality or defect in the process, judgment, or order. Nor shall anything in
this rule be held to authorize the discharge of a person charged with a convicted
of an offense in the Philippines or of a person suffering imprisonment under
lawful judgment. (emphasis supplied)

At this point, we refer to petitioner's plea for the Court of re-examine and, thereafter, abandon its
pronouncement in Ilagan vs. Enrile, 13 that a writ of habeas corpus is no longer available after
an information is filed against the person detained and a warrant of arrest or an order of
commitment, is issued by the court where said information has been filed. 14 The petitioners
claim that the said ruling, which was handed down during the past dictatorial regime to enforce
and strengthen said regime, has no place under the present democratic dispensation and
collides with the basic, fundamental, and constitutional rights of the people. Petitioners point out
that the said doctrine makes possible the arrest and detention of innocent persons despite lack
of evidence against them, and, most often, it is only after a petition for habeas corpus is filed
before the court that the military authorities file the criminal information in the courts of law to be
able to hide behind the protective mantle of the said doctrine. This, petitioners assert, stands as
an obstacle to the freedom and liberty of the people and permits lawless and arbitrary State
action.
We find, however, no compelling reason to abandon the said doctrine. It is based upon express
provision of the Rules of Court and the exigencies served by the law. The fears expressed by
the petitioners are not really unremediable. As the Court sees it, re-examination or reappraisal,
with a view to its abandonment, of the Ilagan case doctrine is not the answer. The answer and
the better practice would be, not to limit the function of the habeas corpus to a mere inquiry as
to whether or not the court which issued the process, judgment or order of commitment or
before whom the detained person is charged, had jurisdiction or not to issue the process,
judgment or order or to take cognizance of the case, but rather, as the Court itself states
in Morales, Jr. vs. Enrile, 15 "in all petitions for habeas corpus the court must inquire into every
phase and aspect of petitioner's detention-from the moment petition was taken into custody up
to the moment the court passes upon the merits of the petition;" and "only after such a scrutiny
can the court satisfy itself that the due process clause of our Constitution has in fact been
satisfied." This is exactly what the Court has done in the petitions at bar. This is what should
henceforth be done in all future cases of habeas corpus. In Short, all cases involving deprivation
of individual liberty should be promptly brought to the courts for their immediate scrutiny and
disposition.

WHEREFORE, the petitions are hereby DISMISSED, except that in G.R. No. 85727 (Espiritu vs.
Lim), the bail bond for petitioner's provisional liberty is hereby ordered reduced from P60,000.00
to P10,000.00. No costs.

SO ORDERED.

G.R. No. 93239 March 18, 1991

PEOPLE OF THE PHILIPPINES, plaintiff-appellee,


vs.
EDISON SUCRO, accused-appellant.

The Solicitor General for plaintiff-appellee.


Fidencio S. Raz for accused-appellant.

GUTIERREZ, JR., J.:

Edison Sucro was charged with and convicted of violation of Section 4, Article II of the
Dangerous Drugs Act, under an Information which reads:

That on or about the 21st day of March, 1989, in the evening, in the Poblacion,
Municipality of Kalibo, Province of Aklan, Republic of the Philippines, and within the
jurisdiction of this Honorable Court, the above-named accused, acting as a pusher or
broker in the business of selling, administering, delivery, giving away to another and/or
distributing prohibited drugs, did then and there wilfully, unlawfully and feloniously and
without authority of law have in his possession and control nineteen (19) pieces of
marijuana cigarette sticks and four (4) tea bags of dried marijuana leaves which were
confiscated from him by the police authorities of Kalibo, Aklan, shortly after having sold
one tea bag of dried marijuana leaves to a customer. (Rollo, p. 9)

Upon arraignment, the accused-appellant, assisted by counsel, entered a plea of "not guilty" to
the offense charged. Trial ensued and a judgment of conviction was rendered, the pertinent
portion of which reads:
WHEREFORE, judgment is rendered finding the accused Edison Sucro guilty of the sale
of prohibited drug under Section 4, Article II of the Dangerous Drug Act, as amended,
and sentencing him to suffer the penalty of life imprisonment, and pay a fine of P20,000,
and costs. He shall be entitled to full credit in the service of his sentence with the period
for which he has undergone preventive imprisonment to the date of promulgation of this
judgment. All the items of marijuana confiscated in this case are declared forfeited in
favor of the State. (Rollo, p. 41)

From the foregoing judgment of conviction, accused-appellant interposes this appeal, assigning
the following as errors allegedly committed by the court a quo, to wit:

THE LOWER COURT ERRED IN ADMITTING AS EVIDENCE FOR THE


PROSECUTION EXHIBITS "E"-"E-4", TEA BAGS OF ALLEGED MARIJUANA, TO BE
THE CORPUS DELICTI; FURTHERMORE, THAT THE SAME WERE TAKEN
WITHOUT THE REQUIRED WARRANT OF SEARCH AND ARREST SINCE THE
ACCUSED WAS NOT IN THE ACT OF COMMITTING ANY OFFENSE AT THE TIME
OF HIS ARREST.

II

THE LOWER COURT ERRED IN FINDING THE ACCUSED EDISON SUCRO GUILTY
OF THE SALE OF PROHIBITED DRUGS UNDER SECTION 4, ARTICLE II, OF THE
DANGEROUS DRUGS ACT AND SENTENCING HIM TO SUFFER A PENALTY OF
LIFE IMPRISONMENT AND TO PAY A FINE OF P 20,000.00. (Appellant's Brief, p. 1)

The antecedent facts of the case as summarized by the Solicitor General are as follows:

On March 21, 1989, Pat. Roy Fulgencio, a member of the INP, Kalibo, Aklan, was
instructed by P/Lt. Vicente Seraspi, Jr. (Station Commander of the INP Kalibo, Aklan) to
monitor the activities of appellant Edison Sucro, because of information gathered by
Seraspi that Sucro was selling marijuana. (p. 6, TSN, May 2,1989).

As planned, at about 5:00 P.M. on said date, Pat. Fulgencio Positioned himself under
the house of a certain Arlie Regalado at C. Quimpo Street. Adjacent to the house of
Regalado, about 2 meters away, was a chapel. Thereafter, Pat. Fulgencio saw appellant
enter the chapel, taking something which turned out later to be marijuana from the
compartment of a cart found inside the chapel, and then return to the street where he
handed the same to a buyer, Aldie Borromeo. After a while appellant went back to the
chapel and again came out with marijuana which he gave to a group of persons. (pp. 6-
8, 15-18, Ibid). It was at this instance that Pat. Fulgencio radioed P/Lt. Seraspi and
reported the activity going on. P/Lt. Seraspi instructed Pat. Fulgencio to continue
monitoring developments. At about 6:30 P.M., Pat. Fulgencio again called up Seraspi to
report that a third buyer later Identified as Ronnie Macabante, was transacting with
appellant. (pp. 18-19, Ibid)

At that point, the team of P/Lt. Seraspi proceeded to the area and while the police
officers were at the Youth Hostel at Maagma St., Pat. Fulgencio told P/Lt. Seraspi to
intercept Macabante and appellant. P/Lt. Seraspi and his team caught up with
Macabante at the crossing of Mabini and Maagma Sts. in front of the Aklan Medical
Center. Upon seeing the police, Macabante threw something to the ground which turned
out to be a tea bag of marijuana. (pp. 6-8, TSN, June 19, 1989) When confronted,
Macabante readily admitted that he bought the same from appellant (Edison Sucro) in
front of the chapel. (p. 6, TSN, May 24, 1989) The police team was able to overtake and
arrest appellant at the corner of C. Quimpo and Veterans Sts. The police recovered 19
sticks and 4 teabags of marijuana from the cart inside the chapel and another teabag
from Macabante, The teabags of marijuana were sent to the PC-INP Crime Laboratory
Service, at Camp Delgado, Iloilo City for analysis. The specimens (Exhibits "G" to "G-
18", Exhibits "E" to "E-4") were all found positive of marijuana. (pp. 47, TSN, Sept. 4,
1989)" (Appellee's Brief, pp. 3-6)

As can be seen from the facts, the issue hinges mainly on whether or not the arrest without
warrant of the accused is lawful and consequently, whether or not the evidence resulting from
such arrest is admissible.

We rule in the affirmative.

The accused-appellant contends that his arrest was illegal, being a violation of his rights granted
under Section 2, Article III of the 1987 Constitution. He stresses that there was sufficient time for
the police officers to apply for a search and arrest warrants considering that Fulgencio informed
his Station Commander of the activities of the accused two days before March 21, 1989, the
date of his arrest.

This contention is without merit.

Section 5, Rule 113 of the Rules on Criminal Procedure provides for the instances where arrest
without warrant is considered lawful. The rule states:

Arrest without warrant, when lawful. — A peace officer or private person may, without
warrant, arrest a person:

(a) When in his presence, the person to be arrested has committed, is actually
committing, or is attempting to commit an offense;

(b) When an offense has in fact just been committed, and he has personal knowledge of
facts indicating that the person to be arrested has committed it; (Emphasis supplied)

An offense is committed in the presence or within the view of an officer, within the meaning of
the rule authorizing an arrest without a warrant, when the officer sees the offense, although at a
distance, or hears the disturbances created thereby and proceeds at once to the scene thereof.
(U.S. v. Fortaleza, 12 Phil. 472 [1909]; and U.S. v. Samonte, 16 Phil. 516 [1910])

The records show that Fulgencio went to Arlie Regalado's house at C. Quimpo Street to monitor
the activities of the accused who was earlier reported to be selling marijuana at a chapel two (2)
meters away from Regalado's house.

Fulgencio, within a distance of two meters saw Sucro conduct his nefarious activity. He saw
Sucro talk to some persons, go inside the chapel, and return to them and exchange some
things. These, Sucro did three times during the time that he was being monitored. Fulgencio
would then relay the on-going transaction to P/Lt. Seraspi.

Anent the second requirement, the fact that Macabante, when intercepted by the police, was
caught throwing the marijuana stick and when confronted, readily admitted that he bought the
same from accused-appellant clearly indicates that Sucro had just sold the marijuana stick to
Macabante, and therefore, had just committed an illegal act of which the police officers had
personal knowledge, being members of the team which monitored Sucro's nefarious activity.

The court earlier indicated in the case of People v. Bati (G.R. No. 87429, August 27, 1990) that
police officers have personal knowledge of the actual commission of the crime when it had
earlier conducted surveillance activities of the accused. Thus, it stated:

When Luciano and Caraan reached the place where the alleged transaction would take
place and while positioned at a street comer, they saw appellant Regalado Bati and
Warner Marquez by the side of the street about forty to fifty meters away from them (the
public officers). They saw Marquez giving something to Bati, who, thereafter handed a
wrapped object to Marquez who then inserted the object inside the front of his pants in
front of his abdomen while Bati, on his part, placed the thing given to him inside his
pocket. (p. 2)

xxx xxx xxx

. . . Both Patrolman Luciano and Caraan actually witnessed the same and their
testimonies were based on their actual and personal knowledge of the events that took
place leading to appellant's arrest. They may not have been within hearing distance,
specially since conversation would expectedly be carried on in hushed tones, but they
were certainly near enough to observe the movements of the appellant and the buyer.
Moreover, these prosecution witnesses are all law enforcers and are, therefore,
presumed to have regularly performed their duties in the absence of proof to the contrary
(People v. Bati, supra citing People v. Agapito, G.R. No. 73786, October 12, 1987)

The accused questions the failure of the police officers to secure a warrant considering that
Fulgencio himself knew of Sucro's activities even prior to the former's joining the police force.
Fulgencio reported Sucro's activities only three days before the incident.

As the records reveal, Fulgencio and Sucro had known each other since their childhood years
and that after Fulgencio joined the police force, he told the accused-appellant not to sell drugs in
their locality. Hence, it is possible that because of this friendship, Fulgencio hesitated to report
his childhood friend and merely advised him not to engage in such activity. However, because
of reliable information given by some informants that selling was going on everyday, he was
constrained to report the matter to the Station Commander.

On the other hand, the failure of the police officers to secure a warrant stems from the fact that
their knowledge acquired from the surveillance was insufficient to fulfill the requirements for the
issuance of a search warrant. What is paramount is that probable cause existed. Thus, it has
been held in the case of People v. Lo Ho Wing, et al. (G.R. No. 88017, January 21, 1991):

In the instant case, it was firmly established from the factual findings of the trial court that
the authorities had reasonable ground to believe that appellant would attempt to bring in
contraband and transport it within the country. The belief was based on intelligence
reports gathered from surveillance activities on the suspected syndicate, of which
appellant was touted to be a member. Aside from this, they were also certain as to the
expected date and time of arrival of the accused from China. But such knowledge was
clearly insufficient to enable them to fulfill the requirements for the issuance of a search
warrant. Still and all, the important thing is that there was probable cause to conduct the
warrantless search, which must still be present in such a case.

As the Solicitor General has pointed out:

There are several instances when a warrantless search and seizure can be effected
without necessarily being preceded by an arrest provided the same is effected on the
basis of probable cause (e.g. stop and search without warrant at checkpoints). Between
warrantless searches and seizures at checkpoints and in the case at bar the latter is
more reasonable considering that unlike in the former, it was effected on the basis of
probable cause. Under the circumstances (monitoring of transactions) there existed
probable cause for the arresting officers, to arrest appellant who was in fact selling
marijuana and to seize the contraband.

That searches and seizures must be supported by a valid warrant is not an absolute rule
(Manipon, Jr. v. Sandiganbayan, 143 SCRA 267 [1986]). Among the exceptions granted by law
is a search incidental to a lawful arrest under Sec. 12, Rule 126 of the Rules on Criminal
Procedure, which provides that a person lawfully arrested may be searched for dangerous
weapons or anything which may be used as proof of the commission of an offense, without a
search warrant. (People v. Castiller, G.R. No. 87783, August 6, 1990)

The accused-appellant claims that the arrest having been done without warrant, it follows that
the evidence obtained therefrom is inadmissible.

As earlier discussed, there is nothing unlawful about the arrest considering its compliance with
the requirements of a warrantless arrest. Ergo, the fruits obtained from such lawful arrest are
admissible in evidence.

Edison Sucro assails the trial court's reliance on the statement of Macabante whose reason for
testifying could be merely to escape prosecution.

We quote the trial court's finding as to the testimony of Macabante:

The non-filing of a complaint against him for possession of marijuana may have been the
reason of (sic) his willingness to testify in court against the accused. But this does not
necessarily taint the evidence that proceeds from his lips. As explained by Lt. Seraspi,
the best sources of information against drug pushers are usually their customers,
especially if as in this case, there is no other direct evidence of the selling except the
testimony of the buyer. We accept this observation as a realistic appraisal of a situation
in which drug users are, and should be employed by law enforcement authorities to
bolster the drive against pushers who are the real felons in our society. We have
observed the demeanor of the witness in court, and found him to be straightforward,
unhesitating, and spontaneous in his declarations, so that we are satisfied as to his
intention and disposition to tell the truth (Rollo, p. 40)
Time and again it has been held that the findings of the trial court are entitled to great weight
and should not be disturbed on appeal unless it is shown that the trial court had overlooked
certain facts of weight and importance, it being acknowledged. that the court below, having seen
and heard the witnesses during the trial, is in a better position to evaluate their testimonies
(People v. Umali, et al., G.R. No. 84450, February 4, 1991 citing People v. Alvarez, 163 SCRA
745 [1988]; People v. Dorado, 30 SCRA 53 [1969]; and People v. Espejo, 36 SCRA 400 [1970]).

Furthermore, the testimony of Macabante was corroborated on material points by public officers
Fulgencio and Seraspi.

There is nothing in the record to suggest that the police officers were compelled by any motive
than to accomplish their mission to capture a drug pusher in the execution of the crime, the
presumption being that police officers perform their duties regularly in the absence of any
evidence to the contrary (Rule 131, Sec. 3(m), Revised Rules on Evidence; People v.
Castiller, supra citing People v. Natipravat, 145 SCRA 483 [1986]).

The prosecution evidence was further bolstered by the findings of the Forensic Chemist that the
items seized were all positive for marijuana.

In contrast to the evidence presented by the prosecution, accused-appellant's defense is alibi


which is unavailing considering that he was positively identified by Macabante to be the person
from whom he bought marijuana.

Sucro alleges that he could not have committed the crime since he was with his uncle and
cousin distributing handbills for his Auntie's candidacy. The fact, however, remains that it does
not preclude the possibility that he was present in the vicinity as established by his admission
that he moved a lot and even had the occasion to meet Macabante on the street.

It is well-settled that mere denials cannot prevail against the positive identification of the
appellant as the seller of the prohibited substances. (People v. Khan, 161 SCRA 406 [1988];
and People v. Paco, 170 SCRA 681 [1989])

Premises considered, this Court is convinced that appellant Edison Sucro had indeed
committed the offense charged. The trial court's decision must be upheld.

WHEREFORE, the decision appealed from is hereby AFFIRMED.

SO ORDERED.

G.R. No. 95902 February 4, 1992

PEOPLE OF THE PHILIPPINES, plaintiff-appellee,


vs.
DON RODRIGUEZA, accused-appellant.

The Solicitor General for plaintiff-appellee.

Public Attorney's Office for accused-appellant.


REGALADO, J.:

On appeal before us is the decision of the Regional Trial Court of Legaspi City, Branch 10,
finding accused-appellant Don Rodrigueza guilty beyond reasonable doubt of violating Section
4, Article II of the Dangerous Drugs Act of 1972 (Republic Act No. 6425, as amended) and
sentencing him to suffer the penalty of life imprisonment and to pay a fine of P20,000.00 and
costs.1

However, the Solicitor General, deviating from his conventional stance in the prosecution of
criminal cases, recommends the acquittal of appellant for the reasons stated in his
Manifestation for Acquittal (In Lieu of Appellee's Brief) filed with the Court. We have reviewed
and analyzed the testimonial and documentary evidence in this case and we find said
recommendation to be well taken.

The information, dated July 10, 1987, charges Don Rodrigueza and his co-accused, Samuel
Segovia and Antonio Lonceras, with allegedly having in their custody and possession 100
grams of marijuana leaves and for selling, in a buy-bust operation, said 100 grams of dried
marijuana leaves for a consideration of P200.00. 2

During the arraignment, all the accused pleaded not guilty to the charge against them. At the
trial, the prosecution and the defense presented several witnesses after which the court a
quo rendered judgment acquitting Samuel Segovia and Antonio Lonceras but convicting and
penalizing herein appellant as hereinbefore stated.

The following facts are culled from the decision of the trial court and the evidence presented by
the prosecution.

At around 5:00 o'clock in the afternoon of July 1, 1987, CIC Ciriaco Taduran was in their
headquarters at the Office of the Narcotics Regional Unit at Camp Bagong Ibalon, Legaspi City,
together with S/Sgt. Elpidio Molinawe, CIC Leonardo B. Galutan and their commanding officer,
Major Crisostomo M. Zeidem, when a confidential informer arrived and told them that there was
an ongoing illegal traffic of prohibited drugs in Tagas, Daraga, Albay. Major Zeidem formed a
team to conduct a buy-bust operation, which team was given P200.00 in different
denominations to buy marijuana. These bills were treated with ultraviolet powder at the
Philippine Constabulary Crime Laboratory (PCCL). Sgt. Molinawe gave the money to Taduran
who acted as the poseur buyer. He was told to look for a certain Don, the alleged seller of
prohibited drugs. Taduran went to Tagas alone and, while along the road, he met Samuel
Segovia. He asked Segovia where be could find Don and where he could buy marijuana.
Segovia left for a while and when be returned, he was accompanied by a man who was later on
introduced to him as Don, herein appellant. 3

After agreeing on the price of P200.00 for 100 grams of marijuana, Don halted a passing tricycle
driven by Antonio Lonceras. He boarded it and left Taduran and Segovia. When he came back,
Don gave Taduran "a certain object wrapped in a plastic" which was later identified as
marijuana, and received payment therefor. Thereafter, Taduran returned to the headquarters
and made a report regarding his said purchase of marijuana. 4
Based on that information, Major Zeidem ordered a team to conduct an operation to apprehend
the suspects. In the evening of the same date, CIC Galutan and S/Sgt. Molinawe proceeded to
Regidor Street, Daraga, Albay and arrested appellant, Antonio Lonceras and Samuel Segovia.
The constables were not, however, armed with a warrant of arrest when they apprehended the
three accused. The arrestees were brought to the headquarters for investigation. 5

Thereafter, agents of the Narcotics Command (NARCOM) conducted a raid in the house of
Jovencio Rodrigueza, father of appellant. Taduran did not go with them. During the raid, they
were able to confiscate dried marijuana leaves and a plastic syringe, among others. The search,
however, was not authorized by any search warrant. 6

The next day, July 2, 1987, Jovencio Rodrigueza was released from detention but appellant was
detained. An affidavit, allegedly taken from and executed by him, was sworn to by him before
the assistant city prosecutor. Appellant had no counsel when his sworn statement was taken
during that custodial investigation. The arrestees were also examined by personnel of the PCCL
and were found positive for ultraviolet powder. 7

The three accused presented different versions of their alleged participations.

Samuel Segovia testified that he was in their house in the evening of July 1, 1987 listening to
the radio. Later, he ate his merienda and then went out to buy cigarettes from the store. While
he was at the store, a jeep stopped behind him. Several armed men alighted therefrom and
ordered him to get inside the jeep. He refused but he was forced to board the vehicle. He was
even hit by the butt of a gun. 8

He was thereafter brought to Camp Bagong Ibalon where he was investigated and was
repeatedly asked regarding the whereabouts of Rodrigueza. He was manhandled by the
NARCOM agents and was detained while inside the camp. He was then made to hold a P10.00
bill treated with ultraviolet powder. When he was taken to the PCCL and examined he was
found positive of the ultraviolet powder. He was also made to sign some papers but he did not
know what they were all about. 9

Appellant, on the other hand, testified that on said date he was in the house of his aunt in San
Roque, Legaspi City. He stayed there overnight and did not leave the place until the next day
when his brother arrived and told him that their father was taken by some military men the
preceding night. Appellant went to Camp Bagong Ibalon and arrived there at around 8:00
o'clock in the morning of July 2, 1987. When he arrived, he was asked if he knew anything
about the marijuana incident, to which question he answered in the negative. Like Segovia, he
was made to hold a P10.00 bill and was brought to the crime laboratory for examination. From
that time on, he was not allowed to go home and was detained inside the camp. He was also
tortured in order to make him admit his complicity in the alleged sale of marijuana. 10

In the assignment of errors in his brief, appellant contends that the trial court erred in (1)
admitting in evidence the sworn statement of appellant which was obtained in violation of his
constitutional rights; (2) convicting appellant of the crime charged despite the fact that the 100
grams of dried marijuana leaves allegedly bought from him were not properly identified; (3)
convicting appellant of the crime charged despite the fact that the evidence for the prosecution
is weak and not convincing; and (4) finding appellant guilty beyond reasonable doubt of selling
or at least acting as broker in the sale of the 100 grams of marijuana to CIC Taduran late in the
afternoon of July 1, 1987, despite the failure of the prosecution to prove his guilt beyond
reasonable doubt. 11

We rule for the appellant and approve the recommendation for his acquittal. In disposing of this
case, however, we feel that the issues raised by appellant should properly be
discussed seriatim.

1. A buy-bust operation is a form of entrapment employed by peace officers to trap and catch a
malefactor in flagrante delicto. 12 Applied to the case at bar, the term in flagrante delicto requires
that the suspected drug dealer must be caught redhanded in the act of selling marijuana or any
prohibited drug to a person acting or posing as a buyer.

In the instant case, however, the procedure adopted by the NARCOM agents failed to meet this
qualification. Based on the very evidence of the prosecution, after the alleged consummation of
the sale of dried marijuana leaves, CIC Taduran immediately released appellant Rodrigueza
instead of arresting and taking him into his custody. This act of CIC Taduran,
assuming arguendo that the supposed sale of marijuana did take place, is decidedly contrary to
the natural course of things and inconsistent with the aforestated purpose of a buy-bust
operation. It is rather absurd on his part to let appellant escape without having been subjected
to the sanctions imposed by law. It is, in fact, a dereliction of duty by an agent of the law.

2. The admissibility of the sworn statement allegedly executed by appellant was squarely placed
in issue and, as correctly pointed out by the defense, said sworn statement is inadmissible in
evidence against appellant.

We have once again to reiterate and emphasize that Article III of the 1987 Constitution provides:

Sec. 12 (1). Any person under investigation for the commission of an offense
shall have the right to be informed of his right to remain silent and to have a
competent and independent counsel preferably of his own choice. If the person
cannot afford the services of counsel, he must be provided with one. These rights
cannot be waived except in writing and in the presence of counsel.

xxx xxx xxx

(3) Any confession or admission obtained in violation of this or section 17 hereof


shall be inadmissible in evidence against him.

An examination of said sworn statement shows that appellant was informed of his constitutional
right to remain silent and to be assisted by counsel during custodial examination. He was also
asked if he was waiving his right to be assisted by counsel and he answered in the affirmative.
However, while the rights of a person under custodial investigation may be waived, such waiver
must be made not only voluntarily, knowingly and intelligently but also in the presence and with
the assistance of counsel. 13 In the present case, the waiver made by appellant being without
the assistance of counsel, this omission alone is sufficient to invalidate said sworn statement. 14

3. Corollary to this, we take cognizance of the error of the trial court in admitting in evidence
against appellant the articles allegedly confiscated during the raid conducted in the house of
Jovencio Rodrigueza.
As provided in the present Constitution, a search, to be valid, must generally be authorized by a
search warrant duly issued by the proper government authority. 15 True, in some instances, this
Court has allowed government authorities to conduct searches and seizures even without a
search warrant. Thus, when the owner of the premises waives his right against such
incursion; 16 when the search is incidental to a lawful arrest; 17 when it is made on vessels and
aircraft for violation of customs laws; 18 when it is made on automobiles for the purpose of
preventing violations of smuggling or immigration laws; 19 when it involves prohibited articles in
plain view; 20 or in cases of inspection of buildings and other premises for the enforcement of
fire, sanitary and building regulations, 21 a search may be validly made even without a search
warrant.

In the case at bar, however, the raid conducted by the NARCOM agents in the house of
Jovencio Rodrigueza was not authorized by any search warrant. It does not appear, either, that
the situation falls under any of the aforementioned cases. Hence, appellant's right against
unreasonable search and seizure was clearly violated. The NARCOM agents could not have
justified their act by invoking the urgency and necessity of the situation because the testimonies
of the prosecution witnesses reveal that the place had already been put under surveillance for
quite some time. Had it been their intention to conduct the raid, then they should, because they
easily could, have first secured a search warrant during that time.

4. The Court further notes the confusion and ambiguity in the identification of the confiscated
marijuana leaves and other prohibited drug paraphernalia presented as evidence against
appellant.

CIC Taduran, who acted as the poseur buyer, testified that appellant sold him 100 grams of
dried marijuana leaves wrapped in a plastic bag. Surprisingly, and no plausible explanation has
been advanced therefor, what were submitted to and examined by the PCCL and thereafter
utilized as evidence against the appellant were the following items:

One (1) red and white colored plastic bag containing the following:

Exh. "A"—Thirty (30) grams of suspected dried marijuana fruiting tops contained
inside a transparent plastic bag.

Exh. "B"— Fifty (50) grams of suspected dried marijuana leaves and seeds
contained inside a white colored plastic labelled "Robertson".

Exh. "C"— Four (4) aluminum foils each containing suspected dried marijuana
fruiting tops having a total weight of seven grams then further wrapped with a
piece of aluminum foil.

Exh. "D"— Five (5) small transparent plastic bags each containing suspected
dried marijuana fruiting tops having a total weight of seventeen grams.

Exh. "E"— One plastic syringe. 22

Evidently, these prohibited articles were among those confiscated during the so-called follow-up
raid in the house of Jovencio Rodrigueza. The unanswered question then arises as to the
identity of the marijuana leaves that became the basis of appellant's conviction. 23 In People
vs. Rubio, 24 this Court had the occasion to rule that the plastic bag and the dried marijuana
leaves contained therein constitute the corpus delicti of the crime. As such, the existence
thereof must be proved with certainty and conclusiveness. Failure to do so would be fatal to the
cause of the prosecution.

5. It is accepted that, as a rule, minor inconsistencies in the testimony of a witness will not affect
his credibility. It even enhances such credibility because it only shows that he has not been
rehearsed. 25 However, when the inconsistencies pertain to material and crucial points, the
same detract from his overall credibility.

The exception, rather than the rule, applies in the case at bar. As correctly pointed out by the
Solicitor General, the testimonies of the prosecution witnesses are tainted with serious flaws
and material inconsistencies rendering the same incredible. 26

CIC Taduran, in his testimony, said that they had already been conducting surveillance of the
place where the buy-bust operation was to take place. It turned out, however, that he did not
even know the exact place and the identity of the person from whom he was to buy marijuana
leaves. Thus:

FISCAL TOLOSA

Q What place in Tagas were you able to go (to)?

WITNESS

A I am not actually familiar in (sic) that place, in Tagas, although


we occasionally passed there.

Q Now, upon your arrival in Tagas, what did you do that


afternoon?

A I waited for the suspect because previously, we have already


been conducted (sic) surveylance (sic) in the vicinity.

Q Upon arrival in Tagas, were you able to see the suspect?

A By the road, sir.

Q Who was the first person did you see (sic) when you arrived at
Tagas?

A The first person whom I saw is Samuel Segovia.

Q Were you able to talk with this Samuel Segovia?

A According to him, we could get some. 27

The same findings go for the testimony of witness Galutan. In his direct examination, he
declared that they arrested the three accused all at the same time on the fateful night of July 1,
1987. But, in his cross-examination and as corroborated by the Joint Affidavit of
Arrest 28 submitted by him and Molinawe, it appeared that Lonceras and Segovia were arrested
on different times and that appellant Don Rodrigueza was not among those who were arrested.
Instead, it was Jovencio Rodrigueza, Don's father, who was picked up at a much later time.

With said inconsistencies in sharp focus, we are constrained to give more credibility to the
testimony of appellant Rodrigueza. While it is true that appellant's defense amounts to an alibi,
and as such is the weakest defense in a criminal prosecution, there are, nonetheless, some
evidentiary aspects pointing to the truth in his testimony. Firstly, the Joint Affidavit of Arrest
corroborates his testimony that he was not among those who were arrested on the night of July
1, 1987. His co-accused Segovia also testified that appellant Rodrigueza was not with them
when they were apprehended by the NARCOM agents.

Secondly, the apparent motive of the NARCOM agents in prosecuting the accused was also
revealed during the trial of the case. Rebuttal witnesses Gracita Bahillo, sister of appellant, and
Hospicio Segovia, father of Samuel Segovia, testified that Sgt. Molinawe, who has since been
reportedly dismissed from the service, asked for P10,000.00 from each of them in exchange for
the liberty of the accused. 29 This allegation was never refuted by the prosecution. Hence, the
rule laid down by this Court that the statements of prosecution witnesses are entitled to full faith
and credit 30 has no application in the case at bar.

Finally, the Court has repeatedly ruled that to sustain the conviction of the accused, the
prosecution must rely on the strength of its own evidence and not on the weakness of the
defense. 31 As clearly shown by the evidence, the prosecution has failed to establish its cause. It
has not overcome the presumption of innocence accorded to appellant. This being the case,
appellant should not be allowed to suffer for unwarranted and imaginary imputations against
him.

WHEREFORE, the judgment of conviction of the court below is hereby REVERSED and SET
ASIDE and accused-appellant Don Rodrigueza is hereby ACQUITTED of the crime charged. It
is hereby ordered that he be immediately released from custody unless he is otherwise detained
for some other lawful cause.

SO ORDERED.

G.R. No. 101837 February 11, 1992

ROLITO GO y TAMBUNTING, petitioner,


vs.
THE COURT OF APPEALS, THE HON. BENJAMIN V. PELAYO, Presiding Judge, Branch
168, Regional Trial Court, NCJR Pasig, M.M., and PEOPLE OF THE
PHILIPPINES, respondents.

FELICIANO, J.:

According to the findings of the San Juan Police in their Investigation Report, 1 on 2 July 1991,
Eldon Maguan was driving his car along Wilson St., San Juan, Metro Manila, heading towards
P. Guevarra St. Petitioner entered Wilson St., where it is a one-way street and started travelling
in the opposite or "wrong" direction. At the corner of Wilson and J. Abad Santos Sts., petitioner's
and Maguan's cars nearly bumped each other. Petitioner alighted from his car, walked over and
shot Maguan inside his car. Petitioner then boarded his car and left the scene. A security guard
at a nearby restaurant was able to take down petitioner's car plate number. The police arrived
shortly thereafter at the scene of the shooting and there retrieved an empty shell and one round
of live ammunition for a 9 mm caliber pistol. Verification at the Land Transportation Office
showed that the car was registered to one Elsa Ang Go.

The following day, the police returned to the scene of the shooting to find out where the suspect
had come from; they were informed that petitioner had dined at Cravings Bake Shop shortly
before the shooting. The police obtained a facsimile or impression of the credit card used by
petitioner from the cashier of the bake shop. The security guard of the bake shop was shown a
picture of petitioner and he positively identified him as the same person who had shot Maguan.
Having established that the assailant was probably the petitioner, the police launched a
manhunt for petitioner.

On 8 July 1991, petitioner presented himself before the San Juan Police Station to verify news
reports that he was being hunted by the police; he was accompanied by two (2) lawyers. The
police forthwith detained him. An eyewitness to the shooting, who was at the police station at
that time, positively identified petitioner as the gunman. That same day, the police promptly filed
a complaint for frustrated homicide 2 against petitioner with the Office of the Provincial
Prosecutor of Rizal. First Assistant Provincial Prosecutor Dennis Villa Ignacio ("Prosecutor")
informed petitioner, in the presence of his lawyers, that he could avail himself of his right to
preliminary investigation but that he must first sign a waiver of the provisions of Article 125 of
the Revised Penal Code. Petitioner refused to execute any such waiver.

On 9 July 1991, while the complaint was still with the Prosecutor, and before an information
could be filed in court, the victim, Eldon Maguan, died of his gunshot wound(s).

Accordingly, on 11 July 1991, the Prosecutor, instead of filing an information for frustrated
homicide, filed an information for murder 3 before the Regional Trial Court. No bail was
recommended. At the bottom of the information, the Prosecutor certified that no preliminary
investigation had been conducted because the accused did not execute and sign a waiver of the
provisions of Article 125 of the Revised Penal Code.

In the afternoon of the same day, 11 July 1991, counsel for petitioner filed with the Prosecutor
an omnibus motion for immediate release and proper preliminary investigation,4 alleging that the
warrantless arrest of petitioner was unlawful and that no preliminary investigation had been
conducted before the information was filed. Petitioner also prayed that he be released on
recognizance or on bail. Provincial Prosecutor Mauro Castro, acting on the omnibus motion,
wrote on the last page of the motion itself that he interposed no objection to petitioner being
granted provisional liberty on a cash bond of P100,000.00.

On 12 July 1991, petitioner filed an urgent ex-parte motion for special raffle 5 in order to
expedite action on the Prosecutor's bail recommendation. The case was raffled to the sala of
respondent Judge, who, on the same date, approved the cash bond 6 posted by petitioner and
ordered his release. 7 Petitioner was in fact released that same day.

On 16 July 1991, the Prosecutor filed with the Regional Trial Court a motion for leave to conduct
preliminary investigation8 and prayed that in the meantime all proceedings in the court be
suspended. He stated that petitioner had filed before the Office of the Provincial Prosecutor of
Rizal an omnibus motion for immediate release and preliminary investigation, which motion had
been granted by Provincial Prosecutor Mauro Castro, who also agreed to recommend cash bail
of P100,000.00. The Prosecutor attached to the motion for leave a copy of petitioner's omnibus
motion of 11 July 1991.

Also on 16 July 1991, the trial court issued an Order 9 granting leave to conduct preliminary
investigation and cancelling the arraignment set for 15 August 1991 until after the prosecution
shall have concluded its preliminary investigation.

On 17 July 1991, however, respondent Judge motu proprio issued an Order, 10 embodying the
following: (1) the 12 July 1991 Order which granted bail was recalled; petitioner was given 48
hours from receipt of the Order to surrender himself; (2) the 16 July 1991 Order which granted
leave to the prosecutor to conduct preliminary investigation was recalled and cancelled; (3)
petitioner's omnibus motion for immediate release and preliminary investigation dated 11 July
1991 was treated as a petition for bail and set for hearing on 23 July 1991.

On 19 July 1991, petitioner filed a petition for certiorari, prohibition and mandamus before the
Supreme Court assailing the 17 July 1991 Order, contending that the information was null and
void because no preliminary investigation had been previously conducted, in violation of his
right to due process. Petitioner also moved for suspension of all proceedings in the case
pending resolution by the Supreme Court of his petition; this motion was, however, denied by
respondent Judge.

On 23 July 1991, petitioner surrendered to the police.

By a Resolution dated 24 July 1991, this Court remanded the petition for certiorari, prohibition
and mandamus to the Court of Appeals.

On 16 August 1991, respondent Judge issued an order in open court setting the arraignment of
petitioner on 23 August 1991.

On 19 August 1991, petitioner filed with the Court of Appeals a motion to restrain his
arraignment.

On 23 August 1991, respondent judge issued a Commitment Order directing the Provincial
Warden of Rizal to admit petitioner into his custody at the Rizal Provincial Jail. On the same
date, petitioner was arraigned. In view, however, of his refusal to enter a plea, the trial court
entered for him a plea of not guilty. The Trial court then set the criminal case for continuous
hearings on 19, 24 and 26 September; on 2, 3, 11 and 17 October; and on 7, 8, 14, 15, 21 and
22 November 1991. 11

On 27 August 1991, petitioner filed a petition for habeas corpus 12 in the Court of Appeals. He
alleged that in view of public respondent's failure to join issues in the petition for certiorari earlier
filed by him, after the lapse of more than a month, thus prolonging his detention, he was entitled
to be released on habeas corpus.

On 30 August 1991, the Court of Appeals issued the writ of habeas corpus. 13 The petition
for certiorari, prohibition and mandamus, on the one hand, and the petition for habeas
corpus, upon the other, were subsequently consolidated in the Court of Appeals.
The Court of Appeals, on 2 September 1991, issued a resolution denying petitioner's motion to
restrain his arraignment on the ground that that motion had become moot and academic.

On 19 September 1991, trial of the criminal case commenced and the prosecution presented its
first witness.

On 23 September 1991, the Court of Appeals rendered a consolidated decision 14 dismissing


the two (2) petitions, on the following grounds:

a. Petitioner's warrantless arrest was valid because the offense for which he was
arrested and charged had been "freshly committed." His identity had been
established through investigation. At the time he showed up at the police station,
there had been an existing manhunt for him. During the confrontation at the San
Juan Police Station, one witness positively identified petitioner as the culprit.

b. Petitioner's act of posting bail constituted waiver of any irregularity attending


his arrest. He waived his right to preliminary investigation by not invoking it
properly and seasonably under the Rules.

c. The trial court did not abuse its discretion when it issued the 17 July 1991
Order because the trial court had the inherent power to amend and control its
processes so as to make them conformable to law and justice.

d. Since there was a valid information for murder against petitioner and a valid
commitment order (issued by the trial judge after petitioner surrendered to the
authorities whereby petitioner was given to the custody of the Provincial
Warden), the petition for habeas corpus could not be granted.

On 3 October 1991, the prosecution presented three (3) more witnesses at the trial. Counsel for
petitioner also filed a "Withdrawal of Appearance" 15 with the trial court, with petitioner's
conformity.

On 4 October 1991, the present Petition for Review on Certiorari was filed. On 14 October 1991,
the Court issued a Resolution directing respondent Judge to hold in abeyance the hearing of the
criminal case below until further orders from this Court.

In this Petition for Review, two (2) principal issues need to be addressed: first, whether or not a
lawful warrantless arrest had been effected by the San Juan Police in respect of petitioner Go;
and second, whether petitioner had effectively waived his right to preliminary investigation. We
consider these issues seriatim.

In respect of the first issue, the Solicitor General argues that under the facts of the case,
petitioner had been validly arrested without warrant. Since petitioner's identity as the gunman
who had shot Eldon Maguan on 2 July 1991 had been sufficiently established by police work,
petitioner was validly arrested six (6) days later at the San Juan Police Station. The Solicitor
General invokes Nazareno v. Station Commander, etc., et al., 16 one of the seven (7) cases
consolidated with In the Matter of the Petition for Habeas Corpus of Roberto Umil, etc.,
v. Ramos, et al. 17 where a majority of the Court upheld a warrantees arrest as valid although
effected fourteen (14) days after the killing in connection with which Nazareno had been
arrested. Accordingly, in the view of the Solicitor General, the provisions of Section 7, Rule 112
of the Rules of Court were applicable and because petitioner had declined to waive the
provisions of Article 125 of the Revised Penal Code, the Prosecutor was legally justified in filing
the information for murder even without preliminary investigation.

On the other hand, petitioner argues that he was not lawfully arrested without warrant because
he went to the police station six (6) days after the shooting which he had allegedly perpetrated.
Thus, petitioner argues, the crime had not been "just committed" at the time that he was
arrested. Moreover, none of the police officers who arrested him had been an eyewitness to the
shooting of Maguan and accordingly none had the "personal knowledge" required for the
lawfulness of a warrantees arrest. Since there had been no lawful warrantless arrest. Section 7,
Rule 112 of the Rules of Court which establishes the only exception to the right to preliminary
investigation, could not apply in respect of petitioner.

The reliance of both petitioner and the Solicitor General upon Umil v. Ramos is, in the
circumstances of this case, misplaced. In Umil v. Ramos, by an eight-to-six vote, the Court
sustained the legality of the warrantless arrests of petitioners made from one (1) to fourteen
days after the actual commission of the offenses, upon the ground that such offenses
constituted "continuing crimes." Those offenses were subversion, membership in an outlawed
organization like the New People's Army, etc. In the instant case, the offense for which
petitioner was arrested was murder, an offense which was obviously commenced and
completed at one definite location in time and space. No one had pretended that the fatal
shooting of Maguan was a "continuing crime."

Secondly, we do not believe that the warrantees "arrest" or detention of petitioner in the instant
case falls within the terms of Section 5 of Rule 113 of the 1985 Rules on Criminal Procedure
which provides as follows:

Sec. 5 Arrest without warrant; when lawful. — A peace officer or a private person
may, without warrant, arrest a person:

(a) When, in his presence, the person to be arrested has committed, is actually
committing, or is attempting to commit an offense;

(b) When an offense has in fact just been committed, and he has personal
knowledge of facts indicating that the person to be arrested has committed it; and

(c) When the person to be arrested is a prisoner who has escaped from a penal
establishment or place where he is serving final judgment or temporarily confined
while his case is pending, or has escaped while being transferred from one
confinement to another.

In cases falling under paragraphs (a) and (b) hereof, the person arrested without
a warrant shall be forthwith delivered to the nearest police station or jail, and he
shall be proceed against in accordance with Rule 112, Section 7.

Petitioner's "arrest" took place six (6) days after the shooting of Maguan. The "arresting" officers
obviously were not present, within the meaning of Section 5(a), at the time petitioner had
allegedly shot Maguan. Neither could the "arrest" effected six (6) days after the shooting be
reasonably regarded as effected "when [the shooting had] in fact just been committed" within
the meaning of Section 5(b). Moreover, none of the "arresting" officers had any "personal
knowledge" of facts indicating that petitioner was the gunman who had shot Maguan. The
information upon which the police acted had been derived from statements made by alleged
eyewitnesses to the shooting — one stated that petitioner was the gunman; another was able to
take down the alleged gunman's car's plate number which turned out to be registered in
petitioner's wife's name. That information did not, however, constitute "personal knowledge." 18

It is thus clear to the Court that there was no lawful warrantless arrest of petitioner within the
meaning of Section 5 of Rule 113. It is clear too that Section 7 of Rule 112, which provides:

Sec. 7 When accused lawfully arrested without warrant. — When a person is


lawfully arrested without a warrant for an offense cognizable by the Regional
Trial Court the complaint or information may be filed by the offended party, peace
officer or fiscal without a preliminary investigation having been first conducted, on
the basis of the affidavit of the offended party or arresting office or person

However, before the filing of such complaint or information, the person arrested
may ask for a preliminary investigation by a proper officer in accordance with this
Rule, but he must sign a waiver of the provisions of Article 125 of the Revised
Penal Code, as amended, with the assistance of a lawyer and in case of non-
availability of a lawyer, a responsible person of his choice. Notwithstanding such
waiver, he may apply for bail as provided in the corresponding rule and the
investigation must be terminated within fifteen (15) days from its inception.

If the case has been filed in court without a preliminary investigation having been
first conducted, the accused may within five (5) days from the time he learns of
the filing of the information, ask for a preliminary investigation with the same right
to adduce evidence in his favor in the manner prescribed in this Rule. (Emphasis
supplied)

is also not applicable. Indeed, petitioner was not arrested at all. When he walked into San Juan
Police Station, accompanied by two (2) lawyers, he in fact placed himself at the disposal of the
police authorities. He did not state that he was "surrendering" himself, in all probability to avoid
the implication he was admitting that he had slain Eldon Maguan or that he was otherwise guilty
of a crime. When the police filed a complaint for frustrated homicide with the Prosecutor, the
latter should have immediately scheduled a preliminary investigation to determine whether there
was probable cause for charging petitioner in court for the killing of Eldon Maguan. Instead, as
noted earlier, the Prosecutor proceed under the erroneous supposition that Section 7 of Rule
112 was applicable and required petitioner to waive the provisions of Article 125 of the Revised
Penal Code as a condition for carrying out a preliminary investigation. This was substantive
error, for petitioner was entitled to a preliminary investigation and that right should have been
accorded him without any conditions. Moreover, since petitioner had not been arrested, with or
without a warrant, he was also entitled to be released forthwith subject only to his appearing at
the preliminary investigation.

Turning to the second issue of whether or not petitioner had waived his right to preliminary
investigation, we note that petitioner had from the very beginning demanded that a preliminary
investigation be conducted. As earlier pointed out, on the same day that the information for
murder was filed with the Regional Trial Court, petitioner filed with the Prosecutor an omnibus
motion for immediate release and preliminary investigation. The Solicitor General contends that
that omnibus motion should have been filed with the trial court and not with the Prosecutor, and
that the petitioner should accordingly be held to have waived his right to preliminary
investigation. We do not believe that waiver of petitioner's statutory right to preliminary
investigation may be predicated on such a slim basis. The preliminary investigation was to be
conducted by the Prosecutor, not by the Regional Trial Court. It is true that at the time of filing of
petitioner's omnibus motion, the information for murder had already been filed with the Regional
Trial Court: it is not clear from the record whether petitioner was aware of this fact at the time his
omnibus motion was actually filed with the Prosecutor. In Crespo v. Mogul, 19 this Court held:

The preliminary investigation conducted by the fiscal for the purpose of


determining whether a prima facie case exists to warranting the prosecution of
the accused is terminated upon the filing of the information in the proper court. In
turn, as above stated, the filing of said information sets in motion the criminal
action against the accused in Court. Should the fiscal find it proper to conduct a
reinvestigation of the case, at such stage, the permission of the Court must be
secured. After such reinvestigation the finding and recommendations of the fiscal
should be submitted to the Court for appropriate action. While it is true that the
fiscal has the quasi-judicial discretion to determine whether or not a criminal case
should be filed in court or not, once the case had already been brought to Court
whatever disposition the fiscal may feel should be proper in the case thereafter
should be addressed for the consideration of the Court. The only qualification is
that the action of the Court must not impair the substantial rights of the accused.,
or the right of the People to due process of law.

xxx xxx xxx

The rule therefore in this jurisdiction is that once a complaint or information is


filed in Court any disposition of the case [such] as its dismissal or the conviction
or acquittal of the accused rests in the sound discretion of the Court. Although
the fiscal retains the direction and control of the prosecution of criminal cases
even while the case is already in Court he cannot impose his opinion on the trial
court. The Court is the best and sole judge on what to do with the case before it. .
. . 20 (Citations omitted; emphasis supplied)

Nonetheless, since petitioner in his omnibus motion was asking for preliminary
investigation and not for a re-investigation (Crespo v. Mogul involved a re-
investigation), and since the Prosecutor himself did file with the trial court, on the 5th day
after filing the information for murder, a motion for leave to conduct preliminary
investigation (attaching to his motion a copy of petitioner's omnibus motion), we
conclude that petitioner's omnibus motion was in effect filed with the trial court. What
was crystal clear was that petitioner did ask for a preliminary investigation on the very
day that the information was filed without such preliminary investigation, and that the trial
court was five (5) days later apprised of the desire of the petitioner for such preliminary
investigation. Finally, the trial court did in fact grant the Prosecutor's prayer for leave to
conduct preliminary investigation. Thus, even on the (mistaken) supposition apparently
made by the Prosecutor that Section 7 of Rule 112 of the Revised Court was applicable,
the 5-day reglementary period in Section 7, Rule 112 must be held to have been
substantially complied with.

We believe and so hold that petitioner did not waive his right to a preliminary investigation.
While that right is statutory rather than constitutional in its fundament, since it has in fact been
established by statute, it is a component part of due process in criminal justice. 21 The right to
have a preliminary investigation conducted before being bound over to trial for a criminal
offense and hence formally at risk of incarceration or some other penalty, is not a mere formal
or technical right; it is a substantive right. The accused in a criminal trial is inevitably exposed to
prolonged anxiety, aggravation, humiliation, not to speak of expense; the right to an opportunity
to avoid a process painful to any one save, perhaps, to hardened criminals, is a valuable right.
To deny petitioner's claim to a preliminary investigation would be to deprive him the full measure
of his right to due process.

The question may be raised whether petitioner still retains his right to a preliminary investigation
in the instant case considering that he was already arraigned on 23 August 1991. The rule is
that the right to preliminary investigation is waived when the accused fails to invoke
it before or at the time of entering a plea at arraignment. 22 In the instant case, petitioner Go had
vigorously insisted on his right to preliminary investigation before his arraignment. At the time of
his arraignment, petitioner was already before the Court of Appeals on certiorari, prohibition
and mandamus precisely asking for a preliminary investigation before being forced to stand trial.

Again, in the circumstances of this case, we do not believe that by posting bail petitioner had
waived his right to preliminary investigation. In People v. Selfaison, 23 we did hold that
appellants there had waived their right to preliminary investigation because immediately after
their arrest, they filed bail and proceeded to trial "without previously claiming that they did not
have the benefit of a preliminary investigation." 24 In the instant case, petitioner Go asked for
release on recognizance or on bail and for preliminary investigation in one omnibus motion. He
had thus claimed his right to preliminary investigation before respondent Judge approved the
cash bond posted by petitioner and ordered his release on 12 July 1991. Accordingly, we
cannot reasonably imply waiver of preliminary investigation on the part of petitioner. In fact,
when the Prosecutor filed a motion in court asking for leave to conduct preliminary investigation,
he clearly if impliedly recognized that petitioner's claim to preliminary investigation was a
legitimate one.

We would clarify, however, that contrary to petitioner's contention the failure to accord
preliminary investigation, while constituting a denial of the appropriate and full measure of the
statutory process of criminal justice, did not impair the validity of the information for murder nor
affect the jurisdiction of the trial court. 25

It must also be recalled that the Prosecutor had actually agreed that petitioner was entitled to
bail. This was equivalent to an acknowledgment on the part of the Prosecutor that the evidence
of guilt then in his hands was not strong. Accordingly, we consider that the 17 July 1991 order of
respondent Judge recalling his own order granting bail and requiring petitioner to surrender
himself within forty-eight (48) hours from notice, was plainly arbitrary considering that no
evidence at all — and certainly no new or additional evidence — had been submitted to
respondent Judge that could have justified the recall of his order issued just five (5) days before.
It follows that petitioner was entitled to be released on bail as a matter of right.

The final question which the Court must face is this: how does the fact that, in the instant case,
trial on the merits has already commenced, the Prosecutor having already presented four (4)
witnesses, impact upon, firstly, petitioner's right to a preliminary investigation and, secondly,
petitioner's right to be released on bail? Does he continue to be entitled to have a preliminary
investigation conducted in respect of the charge against him? Does petitioner remain entitled to
be released on bail?
Turning first to the matter of preliminary investigation, we consider that petitioner remains
entitled to a preliminary investigation although trial on the merits has already began. Trial on the
merits should be suspended or held in abeyance and a preliminary investigation forthwith
accorded to petitioner. 26 It is true that the Prosecutor might, in view of the evidence that he may
at this time have on hand, conclude that probable cause exists; upon the other hand, the
Prosecutor conceivably could reach the conclusion that the evidence on hand does not warrant
a finding of probable cause. In any event, the constitutional point is that petitioner
was not accorded what he was entitled to by way of procedural due process. 27 Petitioner was
forced to undergo arraignment and literally pushed to trial without preliminary investigation, with
extraordinary haste, to the applause from the audience that filled the courtroom. If he submitted
to arraignment at trial, petitioner did so "kicking and screaming," in a manner of speaking .
During the proceedings held before the trial court on 23 August 1991, the date set for
arraignment of petitioner, and just before arraignment, counsel made very clear petitioner's
vigorous protest and objection to the arraignment precisely because of the denial of preliminary
investigation. 28 So energetic and determined were petitioner's counsel's protests and objections
that an obviously angered court and prosecutor dared him to withdraw or walkout, promising to
replace him with counsel de oficio. During the trial, before the prosecution called its first witness,
petitioner through counsel once again reiterated his objection to going to trial without preliminary
investigation: petitioner's counsel made of record his "continuing objection." 29 Petitioner had
promptly gone to the appellate court on certiorari and prohibition to challenge the lawfulness of
the procedure he was being forced to undergo and the lawfulness of his detention.30 If he did
not walk out on the trial, and if he cross-examined the prosecution's witnesses, it was because
he was extremely loath to be represented by counsel de oficio selected by the trial judge, and to
run the risk of being held to have waived also his right to use what is frequently the only test of
truth in the judicial process.

In respect of the matter of bail, we similarly believe and so hold that petitioner remains entitled
to be released on bail as a matter of right. Should the evidence already of record concerning
petitioner's guilt be, in the reasonable belief of the Prosecutor, strong, the Prosecutor may move
in the trial court for cancellation of petitioner's bail. It would then be up to the trial court, after a
careful and objective assessment of the evidence on record, to grant or deny the motion for
cancellation of bail.

To reach any other conclusions here, that is, to hold that petitioner's rights to a preliminary
investigation and to bail were effectively obliterated by evidence subsequently admitted into the
record would be to legitimize the deprivation of due process and to permit the Government to
benefit from its own wrong or culpable omission and effectively to dilute important rights of
accused persons well-nigh to the vanishing point. It may be that to require the State to accord
petitioner his rights to a preliminary investigation and to bail at this point, could turn out
ultimately to be largely a ceremonial exercise. But the Court is not compelled to speculate. And,
in any case, it would not be idle ceremony; rather, it would be a celebration by the State of the
rights and liberties of its own people and a re-affirmation of its obligation and determination to
respect those rights and liberties.

ACCORDINGLY, the Court resolved to GRANT the Petition for Review on Certiorari. The Order
of the trial court dated 17 July 1991 is hereby SET ASIDE and NULLIFIED, and the Decision of
the Court of Appeals dated 23 September 1991 hereby REVERSED.

The Office of the Provincial Prosecutor is hereby ORDERED to conduct forthwith a preliminary
investigation of the charge of murder against petitioner Go, and to complete such preliminary
investigation within a period of fifteen (15) days from commencement thereof. The trial on the
merits of the criminal case in the Regional Trial Court shall be SUSPENDED to await the
conclusion of the preliminary investigation.

Meantime, petitioner is hereby ORDERED released forthwith upon posting of a cash bail bond
of One Hundred Thousand Pesos (P100,000.00). This release shall be without prejudice to any
lawful order that the trial court may issue, should the Office of the Provincial Prosecutor move
for cancellation of bail at the conclusion of the preliminary investigation.

No pronouncement as to costs. This Decision is immediately executory.

SO ORDERED.

G.R. No. 89139 August 2, 1990

ROMEO POSADAS y ZAMORA, petitioner,


vs.
THE HONORABLE COURT OF APPEALS and THE PEOPLE OF THE
PHILIPPINES, respondents.

Rudy G. Agravate for petitioner.

GANCAYCO, J.:

The validity of a warrantless search on the person of petitioner is put into issue in this case.

On October 16, 1986 at about 10:00 o'clock in the morning Pat. Ursicio Ungab and Pat. Umbra
Umpar, both members of the Integrated National Police (INP) of the Davao Metrodiscom
assigned with the Intelligence Task Force, were conducting a surveillance along Magallanes
Street, Davao City. While they were within the premises of the Rizal Memorial Colleges they
spotted petitioner carrying a "buri" bag and they noticed him to be acting suspiciously.

They approached the petitioner and identified themselves as members of the INP. Petitioner
attempted to flee but his attempt to get away was thwarted by the two notwithstanding his
resistance.

They then checked the "buri" bag of the petitioner where they found one (1) caliber .38 Smith &
Wesson revolver with Serial No. 770196 1 two (2) rounds of live ammunition for a .38 caliber
gun 2 a smoke (tear gas) grenade,3 and two (2) live ammunitions for a .22 caliber gun. 4 They
brought the petitioner to the police station for further investigation. In the course of the same,
the petitioner was asked to show the necessary license or authority to possess firearms and
ammunitions found in his possession but he failed to do so. He was then taken to the Davao
Metrodiscom office and the prohibited articles recovered from him were indorsed to M/Sgt.
Didoy the officer then on duty. He was prosecuted for illegal possession of firearms and
ammunitions in the Regional Trial Court of Davao City wherein after a plea of not guilty and trial
on the merits a decision was rendered on October 8, 1987 finding petitioner guilty of the offense
charged as follows:
WHEREFORE, in view of all the foregoing, this Court , finds the accused guilty
beyond reasonable doubt of the offense charged.

It appearing that the accuse d was below eighteen (18) years old at the time of
the commission of the offense (Art. 68, par. 2), he is hereby sentenced to an
indeterminate penalty ranging from TEN (10) YEARS and ONE (1) DAY
of prision mayor to TWELVE (12) Years, FIVE (5) months and Eleven (11) days
of Reclusion Temporal, and to pay the costs.

The firearm, ammunitions and smoke grenade are forfeited in favor of the
government and the Branch Clerk of Court is hereby directed to turn over said
items to the Chief, Davao Metrodiscom, Davao City. 5

Not satisfied therewith the petitioner interposed an appeal to the Court of Appeals wherein in
due course a decision was rendered on February 23, 1989 affirming in toto the appealed
decision with costs against the petitioner. 6

Hence, the herein petition for review, the main thrust of which is that there being no lawful arrest
or search and seizure, the items which were confiscated from the possession of the petitioner
are inadmissible in evidence against him.

The Solicitor General, in justifying the warrantless search of the buri bag then carried by the
petitioner, argues that under Section 12, Rule 136 of the Rules of Court a person lawfully
arrested may be searched for dangerous weapons or anything used as proof of a commission of
an offense without a search warrant. It is further alleged that the arrest without a warrant of the
petitioner was lawful under the circumstances.

Section 5, Rule 113 of the 1985 Rules on Criminal Procedure provides as follows:

SEC. 5. Arrest without warrant; when lawful — A peace officer or a private


person may, without a warrant, arrest a person:

(a) When in his presence, the person to be arrested has committed is actually
committing, or is attempting to commit an offense;

(b) When an offense has in fact just been committed, and he has personal
knowledge of facts indicating that the person to be arrested has committed it; and

(c) When the person to be arrested is a prisoner who has escaped from a penal
establishment or place where he is serving final judgment or temporarily confined
while his case is pending, or has escaped while being transferred from one
confinement to another.

In cases falling under paragraphs (a) and (b) hereof, the person arrested without
a warrant shall be forthwith delivered to the nearest police station or jail, and he
shall be proceeded against in accordance with Rule 112, Section 7. (6a, 17a)

From the foregoing provision of law it is clear that an arrest without a warrant may be effected
by a peace officer or private person, among others, when in his presence the person to be
arrested has committed, is actually committing, or is attempting to commit an offense; or when
an offense has in fact just been committed, and he has personal knowledge of the facts
indicating that the person arrested has committed it.

The Solicitor General argues that when the two policemen approached the petitioner, he was
actually committing or had just committed the offense of illegal possession of firearms and
ammunitions in the presence of the police officers and consequently the search and seizure of
the contraband was incidental to the lawful arrest in accordance with Section 12, Rule 126 of
the 1985 Rules on Criminal Procedure. We disagree.

At the time the peace officers in this case identified themselves and apprehended the petitioner
as he attempted to flee they did not know that he had committed, or was actually committing the
offense of illegal possession of firearms and ammunitions. They just suspected that he was
hiding something in the buri bag. They did now know what its contents were. The said
circumstances did not justify an arrest without a warrant.

However, there are many instances where a warrant and seizure can be effected without
necessarily being preceded by an arrest, foremost of which is the "stop and search" without a
search warrant at military or police checkpoints, the constitutionality or validity of which has
been upheld by this Court in Valmonte vs. de Villa, 7 as follows:

Petitioner Valmonte's general allegation to the effect that he had been stopped
and searched without a search warrant by the military manning the checkpoints,
without more, i.e., without stating the details of the incidents which amount to a
violation of his light against unlawful search and seizure, is not sufficient to
enable the Court to determine whether there was a violation of Valmonte's right
against unlawful search and seizure. Not all searches and seizures are
prohibited. Those which are reasonable are not forbidden. A reasonable search
is not to be determined by any fixed formula but is to be resolved according to
the facts of each case.

Where, for example, the officer merely draws aside the curtain of a vacant
vehicle which is parked on the public fair grounds, or simply looks into a vehicle
or flashes a light therein, these do not constitute unreasonable search.

The setting up of the questioned checkpoints in Valenzuela (and probably in


other areas) may be considered as a security measure to enable the NCRDC to
pursue its mission of establishing effective territorial defense and maintaining
peace and order for the benefit of the public. Checkpoints may also be regarded
as measures to thwart plots to destabilize the government in the interest of public
security. In this connection, the Court may take judicial notice of the shift to urban
centers and their suburbs of the insurgency movement, so clearly reflected in the
increased killings in cities of police and military men by NPA "sparrow units," not
to mention the abundance of unlicensed firearms and the alarming rise in
lawlessness and violence in such urban centers, not all of which are reported in
media, most likely brought about by deteriorating economic conditions — which
all sum up to what one can rightly consider, at the very least, as abnormal
times. Between the inherent right of the state to protect its existence and promote
public welfare and an individual's right against a warrantless search which is
however reasonably conducted, the former should prevail.
True, the manning of checkpoints by the military is susceptible of abuse by the
men in uniform in the same manner that all governmental power is susceptible of
abuse. But, at the cost of occasional inconvenience, discomfort and even
irritation to the citizen, the checkpoints during these abnormal times, when
conducted within reasonable limits, are part of the price we pay for an orderly
society and a peaceful community. (Emphasis supplied).

Thus, as between a warrantless search and seizure conducted at military or police checkpoints
and the search thereat in the case at bar, there is no question that, indeed, the latter is more
reasonable considering that unlike in the former, it was effected on the basis of a probable
cause. The probable cause is that when the petitioner acted suspiciously and attempted to flee
with the buri bag there was a probable cause that he was concealing something illegal in the
bag and it was the right and duty of the police officers to inspect the same.

It is too much indeed to require the police officers to search the bag in the possession of the
petitioner only after they shall have obtained a search warrant for the purpose. Such an exercise
may prove to be useless, futile and much too late.

In People vs. CFI of Rizal, 8 this Court held as follows:

. . . In the ordinary cases where warrant is indispensably necessary, the


mechanics prescribed by the Constitution and reiterated in the Rules of Court
must be followed and satisfied. But We need not argue that there are exceptions.
Thus in the extraordinary events where warrant is not necessary to effect a valid
search or seizure, or when the latter cannot be performed except without
warrant, what constitutes a reasonable or unreasonable search or seizure
becomes purely a judicial question, determinable from the uniqueness of the
circumstances involved, including the purpose of the search or seizure, the
presence or absence of probable cause, the manner in which the search and
seizure was made, the place or thing searched and the character of the articles
procured.

The Court reproduces with approval the following disquisition of the Solicitor General:

The assailed search and seizure may still be justified as akin to a "stop and frisk"
situation whose object is either to determine the identity of a suspicious individual
or to maintain the status quo momentarily while the police officer seeks to obtain
more information. This is illustrated in the case of Terry vs. Ohio, 392 U.S. 1
(1968). In this case, two men repeatedly walked past a store window and
returned to a spot where they apparently conferred with a third man. This
aroused the suspicion of a police officer. To the experienced officer, the
behaviour of the men indicated that they were sizing up the store for an armed
robbery. When the police officer approached the men and asked them for their
names, they mumbled a reply. Whereupon, the officer grabbed one of them,
spun him around and frisked him. Finding a concealed weapon in one, he did the
same to the other two and found another weapon. In the prosecution for the
offense of carrying a concealed weapon, the defense of illegal search and
seizure was put up. The United States Supreme Court held that "a police officer
may in appropriate circumstances and in an appropriate manner approach a
person for the purpose of investigating possible criminal behaviour even though
there is no probable cause to make an arrest." In such a situation, it is
reasonable for an officer rather than simply to shrug his shoulder and allow a
crime to occur, to stop a suspicious individual briefly in order to determine his
identity or maintain the status quo while obtaining more information. . . .

Clearly, the search in the case at bar can be sustained under the exceptions heretofore
discussed, and hence, the constitutional guarantee against unreasonable searches and
seizures has not been violated. 9

WHEREFORE, the petition is DENIED with costs against petitioner.

SO ORDERED.

G.R. No. 87059 June 22, 1992

THE PEOPLE OF THE PHILIPPINES, plaintiff-appellee,


vs.
ROGELIO MENGOTE y TEJAS, accused-appellant.

CRUZ, J.:

Accused-appellant Rogelio Mengote was convicted of illegal possession of firearms on the


strength mainly of the stolen pistol found on his person at the moment of his warrantless arrest.
In this appeal, he pleads that the weapon was not admissible as evidence against him because
it had been illegally seized and was therefore the fruit of the poisonous tree. The Government
disagrees. It insists that the revolver was validly received in evidence by the trial judge because
its seizure was incidental to an arrest that was doubtless lawful even if admittedly without
warrant.

The incident occurred shortly before noon of August 8, 1987, after the Western Police District
received a telephone call from an informer that there were three suspicious-looking persons at
the corner of Juan Luna and North Bay Boulevard in Tondo, Manila. A surveillance team of
plainclothesmen was forthwith dispatched to the place. As later narrated at the trial by
Patrolmen Rolando Mercado and Alberto Juan, 1 they there saw two men "looking from side to
side," one of whom was holding his abdomen. They approached these persons and identified
themselves as policemen, whereupon the two tried to run away but were unable to escape
because the other lawmen had surrounded them. The suspects were then searched. One of
them, who turned out to be the accused-appellant, was found with a .38 caliber Smith and
Wesson revolver with six live bullets in the chamber. His companion, later identified as Nicanor
Morellos, had a fan knife secreted in his front right pants pocket. The weapons were taken from
them. Mengote and Morellos were then turned over to police headquarters for investigation by
the Intelligence Division.

On August 11, 1987, the following information was filed against the accused-appellant before
the Regional Trial Court of Manila:
The undersigned accuses ROGELIO MENGOTE y TEJAS of a violation of
Presidential Decree No. 1866, committed as follows:

That on or about August 8, 1987, in the City of Manila, Philippines, the said
accused did then and there wilfully, unlawfully and knowingly have in his
possession and under his custody and control a firearm, to wit:

one (1) cal. 38 "S & W" bearing


Serial No. 8720-T

without first having secured the necessary license or permit therefor from the
proper authorities.

Besides the police officers, one other witness presented by the prosecution was Rigoberto
Danganan, who identified the subject weapon as among the articles stolen from him during the
robbery in his house in Malabon on June 13, 1987. He pointed to Mengote as one of the
robbers. He had duly reported the robbery to the police, indicating the articles stolen from him,
including the revolver. 2 For his part, Mengote made no effort to prove that he owned the firearm
or that he was licensed to possess it and claimed instead that the weapon had been "Planted"
on him at the time of his arrest. 3

The gun, together with the live bullets and its holster, were offered as Exhibits A, B, and C and
admitted over the objection of the defense. As previously stated, the weapon was the principal
evidence that led to Mengote's conviction for violation of P.D. 1866. He was sentenced
to reclusion
perpetua. 4

It is submitted in the Appellant's Brief that the revolver should not have been admitted in
evidence because of its illegal seizure. no warrant therefor having been previously obtained.
Neither could it have been seized as an incident of a lawful arrest because the arrest of
Mengote was itself unlawful, having been also effected without a warrant. The defense also
contends that the testimony regarding the alleged robbery in Danganan's house was irrelevant
and should also have been disregarded by the trial court.

The following are the pertinent provision of the Bill of Rights:

Sec. 2. The right of the people to be secure in their persons, houses, papers, and
effects against unreasonable searches and seizures of whatever nature and for
any purpose shall be inviolable, and no search warrant or warrant of arrest shall
issue except upon probable cause to be determined personally by the judge after
examination under oath or affirmation of the complainant and the witnesses he
may produce, and particularly describing the place to be searched and the
persons or things to be seized.

Sec. 3 (1). The privacy of communication and correspondence shall be inviolable


except upon lawful order of the court, or when public safety or order requires
otherwise as prescribed by law.

(2) Any evidence obtained in violation of this or the preceding section shall be
inadmissible for any purpose in any proceeding.
There is no question that evidence obtained as a result of an illegal search or seizure is
inadmissible in any proceeding for any purpose. That is the absolute prohibition of Article III,
Section 3(2), of the Constitution. This is the celebrated exclusionary rule based on the
justification given by Judge Learned Hand that "only in case the prosecution, which itself
controls the seizing officials, knows that it cannot profit by their wrong will the wrong be
repressed." The Solicitor General, while conceding the rule, maintains that it is not applicable in
the case at bar. His reason is that the arrest and search of Mengote and the seizure of the
revolver from him were lawful under Rule 113, Section 5, of the Rules of Court reading as
follows:

Sec. 5. Arrest without warrant when lawful. — A peace officer or private person
may, without a warrant, arrest a person;

(a) When, in his presence, the person to be arrested has committed, is actually
committing, or is attempting to commit an offense;

(b) When an offense has in fact just been committed, and he has personal
knowledge of facts indicating that the person to be arrested has committed it; and

(c) When the person to be arrested is a prisoner who has escaped from a penal
establishment or place where he is serving final judgment or temporarily confined
while his case is pending, or has escaped while being transferred from one
confinement to another.

In cases failing under paragraphs (a) and (b) hereof, the person arrested without
a warrant shall be forthwith delivered to the nearest police station or jail, and he
shall be proceeded against in accordance with Rule 112, Section 7.

We have carefully examined the wording of this Rule and cannot see how we can agree with the
prosecution.

Par. (c) of Section 5 is obviously inapplicable as Mengote was not an escapee from a penal
institution when he was arrested. We therefore confine ourselves to determining the lawfulness
of his arrest under either Par. (a) or Par. (b) of this section.

Par. (a) requires that the person be arrested (1) after he has committed or while he is actually
committing or is at least attempting to commit an offense, (2) in the presence of the arresting
officer.

These requirements have not been established in the case at bar. At the time of the arrest in
question, the accused-appellant was merely "looking from side to side" and "holding his
abdomen," according to the arresting officers themselves. There was apparently no offense that
had just been committed or was being actually committed or at least being attempted by
Mengote in their presence.

The Solicitor General submits that the actual existence of an offense was not necessary as long
as Mengote's acts "created a reasonable suspicion on the part of the arresting officers and
induced in them the belief that an offense had been committed and that the accused-appellant
had committed it." The question is, What offense? What offense could possibly have been
suggested by a person "looking from side to side" and "holding his abdomen" and in a place not
exactly forsaken?

These are certainly not sinister acts. And the setting of the arrest made them less so, if at all. It
might have been different if Mengote bad been apprehended at an ungodly hour and in a place
where he had no reason to be, like a darkened alley at 3 o'clock in the morning. But he was
arrested at 11:30 in the morning and in a crowded street shortly after alighting from a passenger
jeep with I his companion. He was not skulking in the shadows but walking in the clear light of
day. There was nothing clandestine about his being on that street at that busy hour in the blaze
of the noonday sun.

On the other hand, there could have been a number of reasons, all of them innocent, why his
eyes were darting from side to side and be was holding his abdomen. If they excited suspicion
in the minds of the arresting officers, as the prosecution suggests, it has nevertheless not been
shown what their suspicion was all about. In fact, the policemen themselves testified that they
were dispatched to that place only because of the telephone call from the informer that there
were "suspicious-looking" persons in that vicinity who were about to commit a robbery at North
Bay Boulevard. The caller did not explain why he thought the men looked suspicious nor did he
elaborate on the impending crime.

In the recent case of People v. Malmstedt, 5 the Court sustained the warrantless arrest of the
accused because there was a bulge in his waist that excited the suspicion of the arresting
officer and, upon inspection, turned out to be a pouch containing hashish. In People v.
Claudio, 6 the accused boarded a bus and placed the buri bag she was carrying behind the seat
of the arresting officer while she herself sat in the seat before him. His suspicion aroused, be
surreptitiously examined the bag, which he found to contain marijuana. He then and there made
the warrantless arrest and seizure that we subsequently upheld on the ground that probable
cause had been sufficiently established.

The case before us is different because there was nothing to support the arresting officers'
suspicion other than Mengote's darting eyes and his hand on his abdomen. By no stretch of the
imagination could it have been inferred from these acts that an offense had just been
committed, or was actually being committed, or was at least being attempted in their presence.

This case is similar to People v. Aminnudin, 7 where the Court held that the warrantless arrest of
the accused was unconstitutional. This was effected while be was coming down a vessel, to all
appearances no less innocent than the other disembarking passengers. He had not committed
nor was be actually committing or attempting to commit an offense in the presence of the
arresting officers. He was not even acting suspiciously. In short, there was no probable cause
that, as the prosecution incorrectly suggested, dispensed with the constitutional requirement of
a warrant.

Par. (b) is no less applicable because its no less stringent requirements have also not been
satisfied. The prosecution has not shown that at the time of Mengote's arrest an offense had in
fact just been committed and that the arresting officers had personal knowledge of facts
indicating that Mengote had committed it. All they had was hearsay information from the
telephone caller, and about a crime that had yet to be committed.

The truth is that they did not know then what offense, if at all, had been committed and neither
were they aware of the participation therein of the accused-appellant. It was only later, after
Danganan had appeared at the Police headquarters, that they learned of the robbery in his
house and of Mengote's supposed involvement therein. 8 As for the illegal possession of the
firearm found on Mengote's person, the policemen discovered this only after he had been
searched and the investigation conducted later revealed that he was not its owners nor was he
licensed to possess it.

Before these events, the Peace officers had no knowledge even of Mengote' identity, let alone
the fact (or suspicion) that he was unlawfully carrying a firearm or that he was involved in the
robbery of Danganan's house.

In the landmark case of People v. Burgos, 9 this Court declared:

Under Section 6(a) of Rule 113, the officer arresting a person who has just
committed, is committing, or is about to commit an offense must have personal
knowledge of the fact. The offense must also be committed in his presence or
within his view. (Sayo v. Chief of Police, 80 Phil. 859). (Emphasis supplied)

xxx xxx xxx

In arrests without a warrant under Section 6(b), however, it is not enough that
there is reasonable ground to believe that the person to be arrested has
committed a crime. A crime must in fact or actually have been committed first.
That a crime has actually been committed is an essential precondition. It is not
enough to suspect that a crime may have been committed. The fact of the
commission of the offense must be undisputed. The test of reasonable ground
applies only to the identity of the perpetrator. (Emphasis supplied)

This doctrine was affirmed in Alih v. Castro, 10 thus:

If the arrest was made under Rule 113, Section 5, of the Rules of Court in
connection with a crime about to be committed, being committed, or just
committed, what was that crime? There is no allegation in the record of such a
falsification. Parenthetically, it may be observed that under the Revised Rule 113,
Section 5(b), the officer making the arrest must have personal knowledge of the
ground therefor as stressed in the recent case of People v. Burgos. (Emphasis
supplied)

It would be a sad day, indeed, if any person could be summarily arrested and searched just
because he is holding his abdomen, even if it be possibly because of a stomach-ache, or if a
peace officer could clamp handcuffs on any person with a shifty look on suspicion that he may
have committed a criminal act or is actually committing or attempting it. This simply cannot be
done in a free society. This is not a police state where order is exalted over liberty or, worse,
personal malice on the part of the arresting officer may be justified in the name of security.

There is no need to discuss the other issues raised by the accused-appellant as the ruling we
here make is sufficient to sustain his exoneration. Without the evidence of the firearm taken
from him at the time of his illegal arrest, the prosecution has lost its most important exhibit and
must therefore fail. The testimonial evidence against Mengote (which is based on the said
firearm) is not sufficient to prove his guilt beyond reasonable doubt of the crime imputed to him.
We commend Atty. Violeta Calvo-Drilon for her able and spirited defense of the accused-
appellant not only in the brief but also in the reply brief, which she did not have to file but did so
just the same to stress the constitutional rights of her client. The fact that she was acting only as
a counsel de oficio with no expectation of material reward makes her representation even more
commendable.

The Court feels that if the peace officers had been more mindful of the provisions of the Bill of
Rights, the prosecution of the accused-appellant might have succeeded. As it happened, they
allowed their over-zealousness to get the better of them, resulting in their disregard of the
requirements of a valid search and seizure that rendered inadmissible the vital evidence they
had invalidly seized.

This should be a lesson to other peace officers. Their impulsiveness may be the very cause of
the acquittal of persons who deserve to be convicted, escaping the clutches of the law because,
ironically enough, it has not been observed by those who are supposed to enforce it.

WHEREFORE, the appealed decision is REVERSED and SET ASIDE. The accused-appellant
is ACQUITTED and ordered released immediately unless he is validly detained for other
offenses. No costs.

SO ORDERED.

[G.R. No. 123595. December 12, 1997]

SAMMY MALACAT y MANDAR, Petitioner, v. COURT OF APPEALS, and PEOPLE OF THE


PHILIPPINES, Respondents.

DECISION

DAVIDE, JR., J.:

In an Information1 filed on 30 August 1990, in Criminal Case No. 90-86748 before the Regional
Trial Court (RTC) of Manila, Branch 5, petitioner Sammy Malacat y Mandar was charged with
violating Section 3 of Presidential Decree No. 1866,2 as follows:

That on or about August 27, 1990, in the City of Manila, Philippines, the said accused did then
and there willfully, unlawfully and knowingly keep, possess and/or acquire a hand grenade,
without first securing the necessary license and/or permit therefor from the proper authorities.

At arraignment3 on 9 October 1990, Petitioner, assisted by counsel de oficio, entered a plea of


not guilty.

At pre-trial on 11 March 1991, petitioner admitted the existence of Exhibits A, A-1, and A-
2,4 while the prosecution admitted that the police authorities were not armed with a search
warrant nor warrant of arrest at the time they arrested petitioner.5cräläwvirtualibräry

At trial on the merits, the prosecution presented the following police officers as its witnesses:
Rodolfo Yu, the arresting officer; Josefino G. Serapio, the investigating officer; and Orlando
Ramilo, who examined the grenade.
Rodolfo Yu of the Western Police District, Metropolitan Police Force of the Integrated National
Police, Police Station No. 3, Quiapo, Manila, testified that on 27 August 1990, at about 6:30
p.m., in response to bomb threats reported seven days earlier, he was on foot patrol with three
other police officers (all of them in uniform) along Quezon Boulevard, Quiapo, Manila, near the
Mercury Drug store at Plaza Miranda. They chanced upon two groups of Muslim-looking men,
with each group, comprised of three to four men, posted at opposite sides of the corner of
Quezon Boulevard near the Mercury Drug Store. These men were acting suspiciously with
[t]heir eyes moving very fast.6cräläwvirtualibräry

Yu and his companions positioned themselves at strategic points and observed both groups for
about thirty minutes. The police officers then approached one group of men, who then fled in
different directions. As the policemen gave chase, Yu caught up with and apprehended
petitioner. Upon searching petitioner, Yu found a fragmentation grenade tucked inside
petitioners front waist line.7 Yus companion, police officer Rogelio Malibiran, apprehended Abdul
Casan from whom a.38 caliber revolver was recovered. Petitioner and Casan were then brought
to Police Station No. 3 where Yu placed an X mark at the bottom of the grenade and thereafter
gave it to his commander.8cräläwvirtualibräry

On cross-examination, Yu declared that they conducted the foot patrol due to a report that a
group of Muslims was going to explode a grenade somewhere in the vicinity of Plaza Miranda.
Yu recognized petitioner as the previous Saturday, 25 August 1990, likewise at Plaza Miranda,
Yu saw petitioner and 2 others attempt to detonate a grenade. The attempt was aborted when
Yu and other policemen chased petitioner and his companions; however, the former were
unable to catch any of the latter. Yu further admitted that petitioner and Casan were merely
standing on the corner of Quezon Boulevard when Yu saw them on 27 August 1990. Although
they were not creating a commotion, since they were supposedly acting suspiciously, Yu and
his companions approached them. Yu did not issue any receipt for the grenade he allegedly
recovered from petitioner.9cräläwvirtualibräry

Josefino G. Serapio declared that at about 9:00 a.m. of 28 August 1990, petitioner and a certain
Abdul Casan were brought in by Sgt. Saquilla10 for investigation. Forthwith, Serapio conducted
the inquest of the two suspects, informing them of their rights to remain silent and to be assisted
by competent and independent counsel. Despite Serapios advice, petitioner and Casan
manifested their willingness to answer questions even without the assistance of a lawyer.
Serapio then took petitioners uncounselled confession (Exh. E), there being no PAO lawyer
available, wherein petitioner admitted possession of the grenade. Thereafter, Serapio prepared
the affidavit of arrest and booking sheet of petitioner and Casan. Later, Serapio turned over the
grenade to the Intelligence and Special Action Division (ISAD) of the Explosive Ordnance
Disposal Unit for examination.11chanroblesvirtuallawlibrary

On cross-examination, Serapio admitted that he took petitioners confession knowing it was


inadmissible in evidence.12cräläwvirtualibräry

Orlando Ramilo, a member of the Bomb Disposal Unit, whose principal duties included, among
other things, the examination of explosive devices, testified that on 22 March 1991, he received
a request dated 19 March 1991 from Lt. Eduardo Cabrera and PO Diosdado Diotoy for
examination of a grenade. Ramilo then affixed an orange tag on the subject grenade detailing
his name, the date and time he received the specimen. During the preliminary examination of
the grenade, he [f]ound that [the] major components consisting of [a] high filler and fuse
assembly [were] all present, and concluded that the grenade was [l]ive and capable of
exploding. On even date, he issued a certification stating his findings, a copy of which he
forwarded to Diotoy on 11 August 1991.13cräläwvirtualibräry

Petitioner was the lone defense witness. He declared that he arrived in Manila on 22 July 1990
and resided at the Muslim Center in Quiapo, Manila. At around 6:30 in the evening of 27 August
1990, he went to Plaza Miranda to catch a breath of fresh air. Shortly after, several policemen
arrived and ordered all males to stand aside. The policemen searched petitioner and two other
men, but found nothing in their possession. However, he was arrested with two others, brought
to and detained at Precinct No. 3, where he was accused of having shot a police officer. The
officer showed the gunshot wounds he allegedly sustained and shouted at petitioner [i]to ang
tama mo sa akin. This officer then inserted the muzzle of his gun into petitioners mouth and
said, [y]ou are the one who shot me. Petitioner denied the charges and explained that he only
recently arrived in Manila. However, several other police officers mauled him, hitting him with
benches and guns. Petitioner was once again searched, but nothing was found on him. He saw
the grenade only in court when it was presented.14cräläwvirtualibräry

The trial court ruled that the warrantless search and seizure of petitioner was akin to a stop and
frisk, where a warrant and seizure can be effected without necessarily being preceded by an
arrest and whose object is either to maintain the status quo momentarily while the police officer
seeks to obtain more information.15 Probable cause was not required as it was not certain that a
crime had been committed, however, the situation called for an investigation, hence to require
probable cause would have been premature.16 The RTC emphasized that Yu and his
companions were [c]onfronted with an emergency, in which the delay necessary to obtain a
warrant, threatens the destruction of evidence17 and the officers [h]ad to act in haste, as
petitioner and his companions were acting suspiciously, considering the time, place and
reported cases of bombing. Further, petitioners group suddenly ran away in different directions
as they saw the arresting officers approach, thus [i]t is reasonable for an officer to conduct a
limited search, the purpose of which is not necessarily to discover evidence of a crime, but to
allow the officer to pursue his investigation without fear of violence.18cräläwvirtualibräry

The trial court then ruled that the seizure of the grenade from petitioner was incidental to a
lawful arrest, and since petitioner [l]ater voluntarily admitted such fact to the police investigator
for the purpose of bombing the Mercury Drug Store, concluded that sufficient evidence existed
to establish petitioners guilt beyond reasonable doubt.

In its decision19 dated 10 February 1994 but promulgated on 15 February 1994, the trial court
thus found petitioner guilty of the crime of illegal possession of explosives under Section 3 of
P.D. No. 1866, and sentenced him to suffer:

[T]he penalty of not less than SEVENTEEN (17) YEARS, FOUR (4) MONTHS AND ONE (1)
DAY OF RECLUSION TEMPORAL, as minimum, and not more than THIRTY (30) YEARS
OF RECLUSION PERPETUA, as maximum.

On 18 February 1994, petitioner filed a notice of appeal20 indicating that he was appealing to
this Court. However, the record of the case was forwarded to the Court of Appeals which
docketed it as CA-G.R. CR No. 15988 and issued a notice to file briefs.21cräläwvirtualibräry

In his Appellants Brief 22 filed with the Court of Appeals, petitioner asserted that:
1. THE LOWER COURT ERRED IN HOLDING THAT THE SEARCH UPON THE PERSON OF
ACCUSED-APPELLANT AND THE SEIZURE OF THE ALLEGED HANDGRENADE FROM HIM
WAS AN APPROPRIATE INCIDENT TO HIS ARREST.

2. THE LOWER COURT ERRED IN ADMITTING AS EVIDENCE AGAINST ACCUSED-


APPELLANT THE HANDGRENADE ALLEGEDLY SEIZED FROM HIM AS IT WAS A
PRODUCT OF AN UNREASONABLE AND ILLEGAL SEARCH.

In sum, petitioner argued that the warrantless arrest was invalid due to absence of any of the
conditions provided for in Section 5 of Rule 113 of the Rules of Court, citing People vs.
Mengote.23 As such, the search was illegal, and the hand grenade seized, inadmissible in
evidence.

In its Brief for the Appellee, the Office of the Solicitor General agreed with the trial court and
prayed that its decision be affirmed in toto.24cräläwvirtualibräry

In its decision of 24 January 1996,25 the Court of Appeals affirmed the trial court, noting, first,
that petitioner abandoned his original theory before the court a quo that the grenade was
planted by the police officers; and second, the factual finding of the trial court that the grenade
was seized from petitioners possession was not raised as an issue. Further, respondent court
focused on the admissibility in evidence of Exhibit D, the hand grenade seized from petitioner.
Meeting the issue squarely, the Court of Appeals ruled that the arrest was lawful on the ground
that there was probable cause for the arrest as petitioner was attempting to commit an offense,
thus:

We are at a loss to understand how a man, who was in possession of a live grenade and in the
company of other suspicious character[s] with unlicensed firearm[s] lurking in Plaza Miranda at
a time when political tension ha[d] been enkindling a series of terroristic activities, [can] claim
that he was not attempting to commit an offense. We need not mention that Plaza Miranda is
historically notorious for being a favorite bomb site especially during times of political upheaval.
As the mere possession of an unlicensed grenade is by itself an offense, Malacats posture is
simply too preposterous to inspire belief.

In so doing, the Court of Appeals took into account petitioners failure to rebut the testimony of
the prosecution witnesses that they received intelligence reports of a bomb threat at Plaza
Miranda; the fact that PO Yu chased petitioner two days prior to the latters arrest, or on 27
August 1990; and that petitioner and his companions acted suspiciously, the accumulation of
which was more than sufficient to convince a reasonable man that an offense was about to be
committed. Moreover, the Court of Appeals observed:

The police officers in such a volatile situation would be guilty of gross negligence and dereliction
of duty, not to mention of gross incompetence, if they [would] first wait for Malacat to hurl the
grenade, and kill several innocent persons while maiming numerous others, before arriving at
what would then be an assured but moot conclusion that there was indeed probable cause for
an arrest. We are in agreement with the lower court in saying that the probable cause in such a
situation should not be the kind of proof necessary to convict, but rather the practical
considerations of everyday life on which a reasonable and prudent mind, and not legal
technicians, will ordinarily act.
Finally, the Court of Appeals held that the rule laid down in People v. Mengote,26 which
petitioner relied upon, was inapplicable in light of [c]rucial differences, to wit:

[In Mengote] the police officers never received any intelligence report that someone [at] the
corner of a busy street [would] be in possession of a prohibited article. Here the police officers
were responding to a [sic] public clamor to put a check on the series of terroristic bombings in
the Metropolis, and, after receiving intelligence reports about a bomb threat aimed at the vicinity
of the historically notorious Plaza Miranda, they conducted foot patrols for about seven days to
observe suspicious movements in the area. Furthermore, in Mengote, the police officers [had]
no personal knowledge that the person arrested has committed, is actually committing, or is
attempting to commit an offense. Here, PO3 Yu [had] personal knowledge of the fact that he
chased Malacat in Plaza Miranda two days before he finally succeeded in apprehending him.

Unable to accept his conviction, petitioner forthwith filed the instant petition and assigns the
following errors:

1. THE RESPONDENT COURT ERRED IN AFFIRMING THE FINDING OF THE TRIAL


COURT THAT THE WARRANTLESS ARREST OF PETITIONER WAS VALID AND LEGAL.

2. THE RESPONDENT COURT ERRED IN HOLDING THAT THE RULING IN PEOPLE VS.
MENGOTE DOES NOT FIND APPLICATION IN THE INSTANT CASE.

In support thereof, petitioner merely restates his arguments below regarding the validity of the
warrantless arrest and search, then disagrees with the finding of the Court of Appeals that he
was attempting to commit a crime, as the evidence for the prosecution merely disclosed that he
was standing at the corner of Plaza Miranda and Quezon Boulevard with his eyes moving very
fast and looking at every person that come (sic) nearer (sic) to them. Finally, petitioner points
out the factual similarities between his case and that of People v. Mengote to demonstrate that
the Court of Appeals miscomprehended the latter.

In its Comment, the Office of the Solicitor General prays that we affirm the challenged decision.

For being impressed with merit, we resolved to give due course to the petition.

The challenged decision must immediately fall on jurisdictional grounds. To repeat, the penalty
imposed by the trial court was:

[N]ot less than SEVENTEEN (17) YEARS, FOUR (4) MONTHS AND ONE (1) DAY
OF RECLUSION TEMPORAL, as minimum, and not more than THIRTY (30) YEARS
OF RECLUSION PERPETUA, as maximum.

The penalty provided by Section 3 of P.D. No. 1866 upon any person who shall unlawfully
possess grenades is reclusion temporal in its maximum period to reclusion perpetua.

For purposes of determining appellate jurisdiction in criminal cases, the maximum of the
penalty, and not the minimum, is taken into account. Since the maximum of the penalty
is reclusion perpetua, the appeal therefrom should have been to us, and not the Court of
Appeals, pursuant to Section 9(3) of the Judiciary Reorganization Act of 1980 (B.P. Blg.
129),27 in relation to Section 17 of the Judiciary Act of 1948,28 Section 5(2) of Article VIII of the
Constitution29 and Section 3(c) of Rule 122 of the Rules of Court.30 The term life imprisonment
as used in Section 9 of B.P. Blg. 129, the Judiciary Act of 1948, and Section 3 of Rule 122 must
be deemed to include reclusion perpetua in view of Section 5(2) of Article VIII of the
Constitution.

Petitioners Notice of Appeal indicated that he was appealing from the trial courts decision to this
Court, yet the trial court transmitted the record to the Court of Appeals and the latter proceeded
to resolve the appeal.

We then set aside the decision of the Court of Appeals for having been rendered without
jurisdiction, and consider the appeal as having been directly brought to us, with the petition for
review as petitioners Brief for the Appellant, the comment thereon by the Office of the Solicitor
General as the Brief for the Appellee and the memoranda of the parties as their Supplemental
Briefs.

Deliberating on the foregoing pleadings, we find ourselves convinced that the prosecution failed
to establish petitioners guilt with moral certainty.

First, serious doubt surrounds the story of police officer Yu that a grenade was found in and
seized from petitioners possession. Notably, Yu did not identify, in court, the grenade he
allegedly seized. According to him, he turned it over to his commander after putting an X mark
at its bottom; however, the commander was not presented to corroborate this claim. On the
other hand, the grenade presented in court and identified by police officer Ramilo referred to
what the latter received from Lt. Eduardo Cabrera and police officer Diotoy not immediately after
petitioners arrest, but nearly seven (7) months later, or on 19 March 1991; further, there was no
evidence whatsoever that what Ramilo received was the very same grenade seized from
petitioner. In his testimony, Yu never declared that the grenade passed on to Ramilo was the
grenade the former confiscated from petitioner. Yu did not, and was not made to, identify the
grenade examined by Ramilo, and the latter did not claim that the grenade he examined was
that seized from petitioner. Plainly, the law enforcement authorities failed to safeguard and
preserve the chain of evidence so crucial in cases such as these.

Second, if indeed petitioner had a grenade with him, and that two days earlier he was with a
group about to detonate an explosive at Plaza Miranda, and Yu and his fellow officers chased,
but failed to arrest them, then considering that Yu and his three fellow officers were in uniform
and therefore easily cognizable as police officers, it was then unnatural and against common
experience that petitioner simply stood there in proximity to the police officers. Note that Yu
observed petitioner for thirty minutes and must have been close enough to petitioner in order to
discern petitioners eyes moving very fast.

Finally, even assuming that petitioner admitted possession of the grenade during his custodial
investigation by police officer Serapio, such admission was inadmissible in evidence for it was
taken in palpable violation of Section 12(1) and (3) of Article III of the Constitution, which
provide as follows:

SEC. 12 (1). Any person under investigation for the commission of an offense shall have the
right to be informed of his right to remain silent and to have competent and independent counsel
preferably of his own choice. If the person cannot afford the services of counsel, he must be
provided with one. These rights cannot be waived except in writing and in the presence of
counsel.
xxx

(3) Any confession or admission obtained in violation of this or Section 17 hereof shall be
inadmissible in evidence against him.

Serapio conducted the custodial investigation on petitioner the day following his arrest. No
lawyer was present and Serapio could not have requested a lawyer to assist petitioner as no
PAO lawyer was then available. Thus, even if petitioner consented to the investigation and
waived his rights to remain silent and to counsel, the waiver was invalid as it was not in writing,
neither was it executed in the presence of counsel.

Even granting ex gratia that petitioner was in possession of a grenade, the arrest and search of
petitioner were invalid, as will be discussed below.

The general rule as regards arrests, searches and seizures is that a warrant is needed in order
to validly effect the same.31 The Constitutional prohibition against unreasonable arrests,
searches and seizures refers to those effected without a validly issued warrant,32 subject to
certain exceptions. As regards valid warrantless arrests, these are found in Section 5, Rule 113
of the Rules of Court, which reads, in part:

Sec. 5. -- Arrest, without warrant; when lawful -- A peace officer or a private person may, without
a warrant, arrest a person:

(a) When, in his presence, the person to be arrested has committed, is actually committing, or is
attempting to commit an offense;

(b) When an offense has in fact just been committed, and he has personal knowledge of facts
indicating that the person to be arrested has committed it; and

(c) When the person to be arrested is a prisoner who has escaped ***

A warrantless arrest under the circumstances contemplated under Section 5(a) has been
denominated as one "in flagrante delicto," while that under Section 5(b) has been described as
a "hot pursuit" arrest.

Turning to valid warrantless searches, they are limited to the following: (1) customs searches;
(2) search of moving vehicles; (3) seizure of evidence in plain view; (4) consent searches; 33 (5)
a search incidental to a lawful arrest;34 and (6) a "stop and frisk."35chanroblesvirtuallawlibrary

In the instant petition, the trial court validated the warrantless search as a stop and frisk with the
seizure of the grenade from the accused [as] an appropriate incident to his arrest, hence
necessitating a brief discussion on the nature of these exceptions to the warrant requirement.

At the outset, we note that the trial court confused the concepts of a "stop-and-frisk" and of a
search incidental to a lawful arrest. These two types of warrantless searches differ in terms of
the requisite quantum of proof before they may be validly effected and in their allowable scope.

In a search incidental to a lawful arrest, as the precedent arrest determines the validity of the
incidental search, the legality of the arrest is questioned in a large majority of these cases, e.g.,
whether an arrest was merely used as a pretext for conducting a search.36 In this instance, the
law requires that there first be a lawful arrest before a search can be made -- the process
cannot be reversed.37 At bottom, assuming a valid arrest, the arresting officer may search the
person of the arrestee and the area within which the latter may reach for a weapon or for
evidence to destroy, and seize any money or property found which was used in the commission
of the crime, or the fruit of the crime, or that which may be used as evidence, or which might
furnish the arrestee with the means of escaping or committing violence.38cräläwvirtualibräry

Here, there could have been no valid in flagrante delicto or hot pursuit arrest preceding the
search in light of the lack of personal knowledge on the part of Yu, the arresting officer, or an
overt physical act, on the part of petitioner, indicating that a crime had just been committed, was
being committed or was going to be committed.

Having thus shown the invalidity of the warrantless arrest in this case, plainly, the search
conducted on petitioner could not have been one incidental to a lawful arrest.

We now proceed to the justification for and allowable scope of a "stop-and-frisk" as a "limited
protective search of outer clothing for weapons," as laid down in Terry, thus:

We merely hold today that where a police officer observes unusual conduct which leads him
reasonably to conclude in light of his experience that criminal activity may be afoot and that the
persons with whom he is dealing may be armed and presently dangerous, where in the course
of investigating this behavior he identifies himself as a policeman and makes reasonable
inquiries, and where nothing in the initial stages of the encounter serves to dispel his reasonable
fear for his own or others' safety, he is entitled for the protection of himself and others in the
area to conduct a carefully limited search of the outer clothing of such persons in an attempt to
discover weapons which might be used to assault him. Such a search is a reasonable search
under the Fourth Amendment ***39cräläwvirtualibräry

Other notable points of Terry are that while probable cause is not required to conduct a "stop
and frisk,"40 it nevertheless holds that mere suspicion or a hunch will not validate a "stop and
frisk." A genuine reason must exist, in light of the police officer's experience and surrounding
conditions, to warrant the belief that the person detained has weapons concealed about
him.41 Finally, a "stop-and-frisk" serves a two-fold interest: (1) the general interest of effective
crime prevention and detection, which underlies the recognition that a police officer may, under
appropriate circumstances and in an appropriate manner, approach a person for purposes of
investigating possible criminal behavior even without probable cause; and (2) the more pressing
interest of safety and self-preservation which permit the police officer to take steps to assure
himself that the person with whom he deals is not armed with a deadly weapon that could
unexpectedly and fatally be used against the police officer.

Here, here are at least three (3) reasons why the stop-and-frisk was invalid:

First, we harbor grave doubts as to Yus claim that petitioner was a member of the group which
attempted to bomb Plaza Miranda two days earlier. This claim is neither supported by any police
report or record nor corroborated by any other police officer who allegedly chased that group.
Aside from impairing Yu's credibility as a witness, this likewise diminishes the probability that a
genuine reason existed so as to arrest and search petitioner. If only to further tarnish the
credibility of Yu's testimony, contrary to his claim that petitioner and his companions had to be
chased before being apprehended, the affidavit of arrest (Exh. "A") expressly declares
otherwise, i.e., upon arrival of five (5) other police officers, petitioner and his companions were
"immediately collared."

Second, there was nothing in petitioners behavior or conduct which could have reasonably
elicited even mere suspicion other than that his eyes were moving very fast an observation
which leaves us incredulous since Yu and his teammates were nowhere near petitioner and it
was already 6:30 p.m., thus presumably dusk. Petitioner and his companions were merely
standing at the corner and were not creating any commotion or trouble, as Yu explicitly declared
on cross-examination:

Q And what were they doing?

A They were merely standing.

Q You are sure of that?

A Yes, sir.

Q And when you saw them standing, there were nothing or they did not create any commotion?

A None, sir.

Q Neither did you see them create commotion?

A None, sir.42cräläwvirtualibräry

Third, there was at all no ground, probable or otherwise, to believe that petitioner was armed
with a deadly weapon. None was visible to Yu, for as he admitted, the alleged grenade was
discovered inside the front waistline of petitioner, and from all indications as to the distance
between Yu and petitioner, any telltale bulge, assuming that petitioner was indeed hiding a
grenade, could not have been visible to Yu. In fact, as noted by the trial court:

When the policemen approached the accused and his companions, they were not yet aware
that a handgrenade was tucked inside his waistline. They did not see any bulging object in [sic]
his person.43chanroblesvirtuallawlibrary

What is unequivocal then in this case are blatant violations of petitioners rights solemnly
guaranteed in Sections 2 and 12(1) of Article III of the Constitution.

WHEREFORE, the challenged decision of the Seventeenth Division of the Court of Appeals in
CA-G.R. CR No. 15988 is SET ASIDE for lack of jurisdiction on the part of said Court and, on
ground of reasonable doubt, the decision of 10 February 1994 of Branch 5 of the Regional Trial
Court of Manila is REVERSED and petitioner SAMMY MALACAT y MANDAR is hereby
ACQUITTED and ORDERED immediately released from detention, unless his further detention
is justified for any other lawful cause.

Costs de oficio.

SO ORDERED.
G.R.No. 74869 July 6, 1988

PEOPLE OF THE PHILIPPINES, plaintiff-appellee,


vs.
IDEL AMINNUDIN y AHNI, defendant-appellant.

The Solicitor General for plaintiff-appellee.

Herminio T. Llariza counsel de-officio for defendant-appellant.

CRUZ, J.:

The accused-appellant claimed his business was selling watches but he was nonetheless
arrested, tried and found guilty of illegally transporting marijuana. The trial court, disbelieving
him, held it was high time to put him away and sentenced him to life imprisonment plus a fine of
P20,000.00. 1

Idel Aminnudin was arrested on June 25, 1984, shortly after disembarking from the M/V Wilcon
9 at about 8:30 in the evening, in Iloilo City. The PC officers who were in fact waiting for him
simply accosted him, inspected his bag and finding what looked liked marijuana leaves took him
to their headquarters for investigation. The two bundles of suspect articles were confiscated
from him and later taken to the NBI laboratory for examination. When they were verified as
marijuana leaves, an information for violation of the Dangerous Drugs Act was filed against
him. 2 Later, the information was amended to include Farida Ali y Hassen, who had also been
arrested with him that same evening and likewise investigated. 3 Both were arraigned and
pleaded not guilty. 4 Subsequently, the fiscal filed a motion to dismiss the charge against Ali on
the basis of a sworn statement of the arresting officers absolving her after a 'thorough
investigation." 5 The motion was granted, and trial proceeded only against the accused-
appellant, who was eventually convicted .6

According to the prosecution, the PC officers had earlier received a tip from one of their
informers that the accused-appellant was on board a vessel bound for Iloilo City and was
carrying marijuana. 7 He was Identified by name. 8 Acting on this tip, they waited for him in the
evening of June 25, 1984, and approached him as he descended from the gangplank after the
informer had pointed to him. 9 They detained him and inspected the bag he was carrying. It was
found to contain three kilos of what were later analyzed as marijuana leaves by an NBI forensic
examiner, 10 who testified that she conducted microscopic, chemical and chromatographic tests
on them. On the basis of this finding, the corresponding charge was then filed against
Aminnudin.

In his defense, Aminnudin disclaimed the marijuana, averring that all he had in his bag was his
clothing consisting of a jacket, two shirts and two pairs of pants. 11 He alleged that he was
arbitrarily arrested and immediately handcuffed. His bag was confiscated without a search
warrant. At the PC headquarters, he was manhandled to force him to admit he was carrying the
marijuana, the investigator hitting him with a piece of wood in the chest and arms even as he
parried the blows while he was still handcuffed. 12 He insisted he did not even know what
marijuana looked like and that his business was selling watches and sometimes
cigarettes. 13 He also argued that the marijuana he was alleged to have been carrying was not
properly Identified and could have been any of several bundles kept in the stock room of the PC
headquarters. 14

The trial court was unconvinced, noting from its own examination of the accused that he claimed
to have come to Iloilo City to sell watches but carried only two watches at the time, traveling
from Jolo for that purpose and spending P107.00 for fare, not to mention his other
expenses. 15 Aminnudin testified that he kept the two watches in a secret pocket below his belt
but, strangely, they were not discovered when he was bodily searched by the arresting officers
nor were they damaged as a result of his manhandling. 16 He also said he sold one of the
watches for P400.00 and gave away the other, although the watches belonged not to him but to
his cousin, 17 to a friend whose full name he said did not even know. 18 The trial court also
rejected his allegations of maltreatment, observing that he had not sufficiently proved the
injuries sustained by him. 19

There is no justification to reverse these factual findings, considering that it was the trial judge
who had immediate access to the testimony of the witnesses and had the opportunity to weigh
their credibility on the stand. Nuances of tone or voice, meaningful pauses and hesitation, flush
of face and dart of eyes, which may reveal the truth or expose the lie, are not described in the
impersonal record. But the trial judge sees all of this, discovering for himself the truant fact
amidst the falsities.

The only exception we may make in this case is the trial court's conclusion that the accused-
appellant was not really beaten up because he did not complain about it later nor did he submit
to a medical examination. That is hardly fair or realistic. It is possible Aminnudin never had that
opportunity as he was at that time under detention by the PC authorities and in fact has never
been set free since he was arrested in 1984 and up to the present. No bail has been allowed for
his release.

There is one point that deserves closer examination, however, and it is Aminnudin's claim that
he was arrested and searched without warrant, making the marijuana allegedly found in his
possession inadmissible in evidence against him under the Bill of Rights. The decision did not
even discuss this point. For his part, the Solicitor General dismissed this after an all-too-short
argument that the arrest of Aminnudin was valid because it came under Rule 113, Section 6(b)
of the Rules of Court on warrantless arrests. This made the search also valid as incidental to a
lawful arrest.

It is not disputed, and in fact it is admitted by the PC officers who testified for the prosecution,
that they had no warrant when they arrested Aminnudin and seized the bag he was carrying.
Their only justification was the tip they had earlier received from a reliable and regular informer
who reported to them that Aminnudin was arriving in Iloilo by boat with marijuana. Their
testimony varies as to the time they received the tip, one saying it was two days before the
arrest, 20 another two weeks 21 and a third "weeks before June 25." 22 On this matter, we may
prefer the declaration of the chief of the arresting team, Lt. Cipriano Querol, Jr., who testified as
follows:

Q You mentioned an intelligence report, you mean with respect to


the coming of Idel Aminnudin on June 25, 1984?

A Yes, sir.
Q When did you receive this intelligence report?

A Two days before June 25, 1984 and it was supported by reliable
sources.

Q Were you informed of the coming of the Wilcon 9 and the


possible trafficking of marijuana leaves on that date?

A Yes, sir, two days before June 25, 1984 when we received this
information from that particular informer, prior to June 25, 1984 we
have already reports of the particular operation which was being
participated by Idel Aminnudin.

Q You said you received an intelligence report two days before


June 25, 1984 with respect to the coming of Wilcon 9?

A Yes, sir.

Q Did you receive any other report aside from this intelligence
report?

A Well, I have received also other reports but not pertaining to the
coming of Wilcon 9. For instance, report of illegal gambling
operation.

COURT:

Q Previous to that particular information which you said two days


before June 25, 1984, did you also receive daily report regarding
the activities of Idel Aminnudin

A Previous to June 25, 1984 we received reports on the activities


of Idel Aminnudin.

Q What were those activities?

A Purely marijuana trafficking.

Q From whom did you get that information?

A It came to my hand which was written in a required sheet of


information, maybe for security reason and we cannot Identify the
person.

Q But you received it from your regular informer?

A Yes, sir.

ATTY. LLARIZA:
Q Previous to June 25, 1984, you were more or less sure that Idel
Aminnudin is coming with drugs?

A Marijuana, sir.

Q And this information respecting Idel Aminnudin's coming to Iloilo


with marijuana was received by you many days before you
received the intelligence report in writing?

A Not a report of the particular coming of Aminnudin but his


activities.

Q You only knew that he was coming on June 25,1984 two days
before?

A Yes, sir.

Q You mean that before June 23, 1984 you did not know that
minnudin was coming?

A Before June 23,1984, I, in my capacity, did not know that he


was coming but on June 23, 1984 that was the time when I
received the information that he was coming. Regarding the
reports on his activities, we have reports that he was already
consummated the act of selling and shipping marijuana stuff.

COURT:

Q And as a result of that report, you put him under surveillance?

A Yes, sir.

Q In the intelligence report, only the name of Idel Aminnudin was


mentioned?

A Yes, sir.

Q Are you sure of that?

A On the 23rd he will be coming with the woman.

Q So that even before you received the official report on June 23,
1984, you had already gathered information to the effect that Idel
Aminnudin was coming to Iloilo on June 25, 1984?

A Only on the 23rd of June.

Q You did not try to secure a search warrant for the seizure or
search of the subject mentioned in your intelligence report?
A No, more.

Q Why not?

A Because we were very very sure that our operation will yield
positive result.

Q Is that your procedure that whenever it will yield positive result


you do not need a search warrant anymore?

A Search warrant is not necessary. 23

That last answer is a cavalier pronouncement, especially as it comes from a mere lieutenant of
the PC. The Supreme Court cannot countenance such a statement. This is still a government of
laws and not of men.

The mandate of the Bill of Rights is clear:

Sec. 2. The right of the people to be secure in their persons, houses, papers and
effects against unreasonable searches and seizures of whatever nature and for
any purpose shall be inviolable, and no search warrant or warrant of arrest shall
issue except upon probable cause to be determined personally by the judge after
examination under oath or affirmation of the complainant and the witnesses he
may produce, and particularly describing the place to be searched and the
persons or things to be seized.

In the case at bar, there was no warrant of arrest or search warrant issued by a judge after
personal determination by him of the existence of probable cause. Contrary to the averments of
the government, the accused-appellant was not caught in flagrante nor was a crime about to be
committed or had just been committed to justify the warrantless arrest allowed under Rule 113
of the Rules of Court. Even expediency could not be invoked to dispense with the obtention of
the warrant as in the case of Roldan v. Arca, 24 for example. Here it was held that vessels and
aircraft are subject to warrantless searches and seizures for violation of the customs law
because these vehicles may be quickly moved out of the locality or jurisdiction before the
warrant can be secured.

The present case presented no such urgency. From the conflicting declarations of the PC
witnesses, it is clear that they had at least two days within which they could have obtained a
warrant to arrest and search Aminnudin who was coming to Iloilo on the M/V Wilcon 9. His
name was known. The vehicle was Identified. The date of its arrival was certain. And from the
information they had received, they could have persuaded a judge that there was probable
cause, indeed, to justify the issuance of a warrant. Yet they did nothing. No effort was made to
comply with the law. The Bill of Rights was ignored altogether because the PC lieutenant who
was the head of the arresting team, had determined on his own authority that a "search warrant
was not necessary."

In the many cases where this Court has sustained the warrantless arrest of violators of the
Dangerous Drugs Act, it has always been shown that they were caught red-handed, as a result
of what are popularly called "buy-bust" operations of the narcotics agents. 25 Rule 113 was
clearly applicable because at the precise time of arrest the accused was in the act of selling the
prohibited drug.

In the case at bar, the accused-appellant was not, at the moment of his arrest, committing a
crime nor was it shown that he was about to do so or that he had just done so. What he was
doing was descending the gangplank of the M/V Wilcon 9 and there was no outward indication
that called for his arrest. To all appearances, he was like any of the other passengers innocently
disembarking from the vessel. It was only when the informer pointed to him as the carrier of the
marijuana that he suddenly became suspect and so subject to apprehension. It was the furtive
finger that triggered his arrest. The Identification by the informer was the probable cause as
determined by the officers (and not a judge) that authorized them to pounce upon Aminnudin
and immediately arrest him.

Now that we have succeeded in restoring democracy in our country after fourteen years of the
despised dictatorship, when any one could be picked up at will, detained without charges and
punished without trial, we will have only ourselves to blame if that kind of arbitrariness is allowed
to return, to once more flaunt its disdain of the Constitution and the individual liberties its Bill of
Rights guarantees.

While this is not to say that the accused-appellant is innocent, for indeed his very own words
suggest that he is lying, that fact alone does not justify a finding that he is guilty. The
constitutional presumption is that he is innocent, and he will be so declared even if his defense
is weak as long as the prosecution is not strong enough to convict him.

Without the evidence of the marijuana allegedly seized from Aminnudin, the case of the
prosecution must fall. That evidence cannot be admitted, and should never have been
considered by the trial court for the simple fact is that the marijuana was seized illegally. It is the
fruit of the poisonous tree, to use Justice Holmes' felicitous phrase. The search was not an
incident of a lawful arrest because there was no warrant of arrest and the warrantless arrest did
not come under the exceptions allowed by the Rules of Court. Hence, the warrantless search
was also illegal and the evidence obtained thereby was inadmissible.

The Court strongly supports the campaign of the government against drug addiction and
commends the efforts of our law-enforcement officers against those who would inflict this
malediction upon our people, especially the susceptible youth. But as demanding as this
campaign may be, it cannot be more so than the compulsions of the Bill of Rights for the
protection of the liberty of every individual in the realm, including the basest of criminals. The
Constitution covers with the mantle of its protection the innocent and the guilty alike against any
manner of high- handedness from the authorities, however praiseworthy their intentions.

Those who are supposed to enforce the law are not justified in disregarding the rights of the
individual in the name of order. Order is too high a price for the loss of liberty. As Justice
Holmes, again, said, "I think it a less evil that some criminals should escape than that the
government should play an ignoble part." It is simply not allowed in the free society to violate a
law to enforce another, especially if the law violated is the Constitution itself.

We find that with the exclusion of the illegally seized marijuana as evidence against the
accused-appellant, his guilt has not been proved beyond reasonable doubt and he must
therefore be discharged on the presumption that he is innocent.
ACCORDINGLY, the decision of the trial court is REVERSED and the accused-appellant is
ACQUITTED. It is so ordered.

G.R. No. 91107 June 19, 1991

THE PEOPLE OF THE PHILIPPINES, plaintiff-appellee,


vs.
MIKAEL MALMSTEDT, *defendant-appellant.

The Solicitor General for plaintiff-appellee.


Romulo, Mabanta, Buenaventura, Sayoc & De los Angeles for defendant-appellant.

PADILLA, J.:

In an information dated 15 June 1989, accused-appellant Mikael Malmstedt (hereinafter referred


to as the accused) was charged before the Regional Trial Court (RTC) of La Trinidad, Benguet,
Branch 10, in Criminal Case No. 89-CR-0663, for violation of Section 4, Art. II of Republic Act
6425, as amended, otherwise known as the Dangerous Drugs Act of 1972, as amended. The
factual background of the case is as follows:

Accused Mikael Malmstedt, a Swedish national, entered the Philippines for the third time in
December 1988 as a tourist. He had visited the country sometime in 1982 and 1985.

In the evening of 7 May 1989, accused left for Baguio City. Upon his arrival thereat in the
morning of the following day, he took a bus to Sagada and stayed in that place for two (2) days.

At around 7:00 o'clock in the morning of 11 May 1989, accused went to the Nangonogan bus
stop in Sagada to catch the first available trip to Baguio City. From Baguio City, accused
planned to take a late afternoon trip to Angeles City, then proceed to Manila to catch his flight
out of the country, scheduled on 13 May 1989. From Sagada, accused took a Skyline bus with
body number 8005 and Plate number AVC 902.1

At about 8: 00 o'clock in the morning of that same day (11 May 1989), Captain Alen Vasco, the
Commanding Officer of the First Regional Command (NARCOM) stationed at Camp Dangwa,
ordered his men to set up a temporary checkpoint at Kilometer 14, Acop, Tublay, Mountain
Province, for the purpose of checking all vehicles coming from the Cordillera Region. The order
to establish a checkpoint in the said area was prompted by persistent reports that vehicles
coming from Sagada were transporting marijuana and other prohibited drugs. Moreover,
information was received by the Commanding Officer of NARCOM, that same morning, that a
Caucasian coming from Sagada had in his possession prohibited drugs.2

The group composed of seven (7) NARCOM officers, in coordination with Tublay Police Station,
set up a checkpoint at the designated area at about 10:00 o'clock in the morning and inspected
all vehicles coming from the Cordillera Region.
At about 1:30 o'clock in the afternoon, the bus where accused was riding was stopped. Sgt.
Fider and CIC Galutan boarded the bus and announced that they were members of the
NARCOM and that they would conduct an inspection. The two (2) NARCOM officers started
their inspection from the front going towards the rear of the bus. Accused who was the sole
foreigner riding the bus was seated at the rear thereof.

During the inspection, CIC Galutan noticed a bulge on accused's waist. Suspecting the bulge on
accused's waist to be a gun, the officer asked for accused's passport and other identification
papers. When accused failed to comply, the officer required him to bring out whatever it was
that was bulging on his waist. The bulging object turned out to be a pouch bag and when
accused opened the same bag, as ordered, the officer noticed four (4) suspicious-looking
objects wrapped in brown packing tape, prompting the officer to open one of the wrapped
objects. The wrapped objects turned out to contain hashish, a derivative of marijuana.

Thereafter, accused was invited outside the bus for questioning. But before he alighted from the
bus, accused stopped to get two (2) travelling bags from the luggage carrier.

Upon stepping out of the bus, the officers got the bags and opened them. A teddy bear was
found in each bag. Feeling the teddy bears, the officer noticed that there were bulges inside the
same which did not feel like foam stuffing. It was only after the officers had opened the bags
that accused finally presented his passport.

Accused was then brought to the headquarters of the NARCOM at Camp Dangwa, La Trinidad,
Benguet for further investigation. At the investigation room, the officers opened the teddy bears
and they were found to also contain hashish. Representative samples were taken from the
hashish found among the personal effects of accused and the same were brought to the PC
Crime Laboratory for chemical analysis.

In the chemistry report, it was established that the objects examined were hashish. a prohibited
drug which is a derivative of marijuana. Thus, an information was filed against accused for
violation of the Dangerous Drugs Act.

During the arraignment, accused entered a plea of "not guilty." For his defense, he raised the
issue of illegal search of his personal effects. He also claimed that the hashish was planted by
the NARCOM officers in his pouch bag and that the two (2) travelling bags were not owned by
him, but were merely entrusted to him by an Australian couple whom he met in Sagada. He
further claimed that the Australian couple intended to take the same bus with him but because
there were no more seats available in said bus, they decided to take the next ride and asked
accused to take charge of the bags, and that they would meet each other at the Dangwa
Station.

Likewise, accused alleged that when the NARCOM officers demanded for his passport and
other Identification papers, he handed to one of the officers his pouch bag which was hanging
on his neck containing, among others, his passport, return ticket to Sweden and other papers.
The officer in turn handed it to his companion who brought the bag outside the bus. When said
officer came back, he charged the accused that there was hashish in the bag. He was told to
get off the bus and his picture was taken with the pouch bag placed around his neck. The trial
court did not give credence to accused's defense.
The claim of the accused that the hashish was planted by the NARCOM officers, was belied by
his failure to raise such defense at the earliest opportunity. When accused was investigated at
the Provincial Fiscal's Office, he did not inform the Fiscal or his lawyer that the hashish was
planted by the NARCOM officers in his bag. It was only two (2) months after said investigation
when he told his lawyer about said claim, denying ownership of the two (2) travelling bags as
well as having hashish in his pouch bag.

In a decision dated 12 October 1989, the trial court found accused guilty beyond reasonable
doubt for violation of the Dangerous Drugs Act, specifically Section 4, Art. II of RA 6425, as
amended.3 The dispositive portion of the decision reads as follows:

WHEREFORE, finding the guilt of the accused Mikael Malmstedt established beyond
reasonable doubt, this Court finds him GUILTY of violation of Section 4, Article 11 of
Republic Act 6425, as amended, and hereby sentences him to suffer the penalty of life
imprisonment and to pay a fine of Twenty Thousand Pesos (P20,000.00), with subsidiary
imprisonment in case of insolvency and to pay the costs.

Let the hashish subject of this case be turned over to the First Narcotics Regional Unit at
Camp Bado; Dangwa, La Trinidad Benguet for proper disposition under Section 20,
Article IV of Republic Act 6425, as amended.

SO ORDERED.4

Seeking the reversal of the decision of the trial court finding him guilty of the crime charged,
accused argues that the search of his personal effects was illegal because it was made without
a search warrant and, therefore, the prohibited drugs which were discovered during the illegal
search are not admissible as evidence against him.

The Constitution guarantees the right of the people to be secure in their persons, houses,
papers and effects against unreasonable searches and seizures.5 However, where the search is
made pursuant to a lawful arrest, there is no need to obtain a search warrant. A lawful arrest
without a warrant may be made by a peace officer or a private person under the following
circumstances.6

Sec. 5 Arrest without warrant; when lawful. –– A peace officer or a private person may,
without a warrant, arrest a person:

(a) When, in his presence, the person to be arrested has committed is actually
committing, or is attempting to commit an offense;

(b) When an offense has in fact just been committed, and he has personal knowledge of
facts indicating that the person to be arrested has committed it; and

(c) When the person to be arrested is a prisoner who has escaped from a penal
establishment or place where he is serving final judgment or temporarily confined while
his case is pending, or has escaped while being transferred from one confinement to
another.
In cases falling under paragraphs (a) and (b) hereof, the person arrested without a
warrant shall be forthwith delivered to the nearest police station or jail, and he shall be
proceeded against in accordance with Rule 112, Section 7. (6a 17a).

Accused was searched and arrested while transporting prohibited drugs (hashish). A crime was
actually being committed by the accused and he was caught in flagrante delicto. Thus, the
search made upon his personal effects falls squarely under paragraph (1) of the foregoing
provisions of law, which allow a warrantless search incident to a lawful arrest.7

While it is true that the NARCOM officers were not armed with a search warrant when the
search was made over the personal effects of accused, however, under the circumstances of
the case, there was sufficient probable cause for said officers to believe that accused was then
and there committing a crime.

Probable cause has been defined as such facts and circumstances which could lead a
reasonable, discreet and prudent man to believe that an offense has been committed, and that
the objects sought in connection with the offense are in the place sought to be searched.8 The
required probable cause that will justify a warrantless search and seizure is not determined by
any fixed formula but is resolved according to the facts of each case.9

Warrantless search of the personal effects of an accused has been declared by this Court as
valid, because of existence of probable cause, where the smell of marijuana emanated from a
plastic bag owned by the accused,10 or where the accused was acting suspiciously,11 and
attempted to flee.12

Aside from the persistent reports received by the NARCOM that vehicles coming from Sagada
were transporting marijuana and other prohibited drugs, their Commanding Officer also received
information that a Caucasian coming from Sagada on that particular day had prohibited drugs in
his possession. Said information was received by the Commanding Officer of NARCOM the
very same morning that accused came down by bus from Sagada on his way to Baguio City.

When NARCOM received the information, a few hours before the apprehension of herein
accused, that a Caucasian travelling from Sagada to Baguio City was carrying with him
prohibited drugs, there was no time to obtain a search warrant. In the Tangliben case,13 the
police authorities conducted a surveillance at the Victory Liner Terminal located at Bgy. San
Nicolas, San Fernando Pampanga, against persons engaged in the traffic of dangerous drugs,
based on information supplied by some informers. Accused Tangliben who was acting
suspiciously and pointed out by an informer was apprehended and searched by the police
authorities. It was held that when faced with on-the-spot information, the police officers had to
act quickly and there was no time to secure a search warrant.

It must be observed that, at first, the NARCOM officers merely conducted a routine check of the
bus (where accused was riding) and the passengers therein, and no extensive search was
initially made. It was only when one of the officers noticed a bulge on the waist of accused,
during the course of the inspection, that accused was required to present his passport. The
failure of accused to present his identification papers, when ordered to do so, only managed to
arouse the suspicion of the officer that accused was trying to hide his identity. For is it not a
regular norm for an innocent man, who has nothing to hide from the authorities, to readily
present his identification papers when required to do so?
The receipt of information by NARCOM that a Caucasian coming from Sagada had prohibited
drugs in his possession, plus the suspicious failure of the accused to produce his passport,
taken together as a whole, led the NARCOM officers to reasonably believe that the accused
was trying to hide something illegal from the authorities. From these circumstances arose
a probable cause which justified the warrantless search that was made on the personal effects
of the accused. In other words, the acts of the NARCOM officers in requiring the accused to
open his pouch bag and in opening one of the wrapped objects found inside said bag (which
was discovered to contain hashish) as well as the two (2) travelling bags containing two (2)
teddy bears with hashish stuffed inside them, were prompted by accused's own attempt to hide
his identity by refusing to present his passport, and by the information received by the NARCOM
that a Caucasian coming from Sagada had prohibited drugs in his possession. To deprive the
NARCOM agents of the ability and facility to act accordingly, including, to search even without
warrant, in the light of such circumstances, would be to sanction impotence and ineffectiveness
in law enforcement, to the detriment of society.

WHEREFORE, premises considered, the appealed judgment of conviction by the trial court is
hereby AFFIRMED. Costs against the accused-appellant.

SO ORDERED.

Melencio-Herrera, Paras, Feliciano, Bidin, Griño-Aquino, Medialdea, Regalado and Davide, Jr.,
JJ., concur.
Sarmiento, J., is on leave.

Separate Opinions

NARVASA, J., concurring and dissenting:

The ancient tradition that a man's home is his castle, safe from intrusion even by the king, has
not only found its niche in all our charters, from 1935 to the present; it has also received
unvarying recognition and acceptance in our case law.1 The present Constitution2 declares that

The right of the people to be secure in their persons, houses, papers, and effects against
unreasonable searches and seizures of whatever nature and for any purpose, shall be
inviolable, and no search warrant or warrant of arrest shall issue except upon probable
cause to be determined personally by the judge after examination under oath or
affirmation of the complainant and the witnesses he may produce, and particularly
describing the place to be searched, and the persons or things to be seized.

It further ordains that any evidence obtained in violation of said right, among others, "shall be
inadmissible for any purpose in any proceeding."3

The rule is that no person may be subjected by the police or other government authority to a
search of his body, or his personal effects or belongings, or his residence except by virtue of a
search warrant or on the occasion of a legitimate arrest.4
An arrest is legitimate, of course, if effected by virtue of a warrant of arrest. Even without a
warrant, an arrest may also be lawfully made by a peace officer or a private person: 5

(a) when, in his presence, the person to be arrested has committed is actually
committing, or is attempting to commit an offense;

(b) When an offense has in fact just been committed, and he has personal knowledge of
facts indicating that the person to be arrested has committed it; and

(c) When the person to be arrested is a prisoner who has escaped from a penal
establishment or place where he is serving final judgment or temporarily confined while
his case is pending, or has escaped while being transferred from one confinement to
another.

In cases falling under paragraphs (a) and (b) hereof, the person arrested without a
warrant shall be forthwith delivered to the nearest police station or jail, and he shall be
proceeded against in accordance with Rule 112, Section 7.

In any of these instances of a lawful arrest, the person arrested "may be searched for
dangerous weapons or anything which may be used as proof of the commission of an offense,
without a search warrant."6 And it has been held that the search may extend to the area "within
his immediate control," i.e., the area from which said person arrested might gain possession of a
weapon or destructible evidence.7

Apart from "search incidental to an arrest," a warrantless search has also been held to be
proper in cases of "search of a moving vehicle,8 and "seizure of evidence in plain view."9 This
was the pronouncement in Manipon, Jr. v. Sandiganbayan, 143 SCRA 267, 276, which drew
attention to Moreno v. Ago Chi;10 Alvero v. Dizon,11 Papa v. Mago,12 and an American
precedent, Harris v. U.S.13

If, on the other, a person is searched without a warrant, or under circumstances other than
those justifying an arrest without warrant in accordance with law, supra, merely on suspicion
that he is engaged in some felonious enterprise, and in order to discover if he has indeed
committed a crime, it is not only the arrest which is illegal but also, the search on the occasion
thereof, as being "the fruit of the poisonous tree.14 In that event, any evidence taken, even if
confirmatory of the initial suspicion, is inadmissible "for any purpose in any proceeding."15 But
the right against an unreasonable search and seizure may be waived by the person arrested,
provided he knew of such right and knowingly decided not to invoke it.16

There is unanimity among the members of the Court upon the continuing validity of these
established principles. However, the Court is divided as regards the ultimate conclusions which
may properly be derived from the proven facts and consequently, the manner in which the
principles just cited should apply thereto.

The proofs of the prosecution and those of the defense are diametrically at odds. What is
certain, however, is that the soldiers had no warrant of arrest when they conducted a search of
Malmstedt's person and the things in his possession at the time. Indeed, the Court a
quo acknowledged that the soldiers could "not be expected to be armed with a warrant or arrest
nor a search warrant everytime they establish a temporary checkpoint . . . (and) no judge would
issue them one considering that searching questions have to be asked before a warrant could
be issued." Equally plain is that prior to the search, a warrantless arrest of Malmstedt could not
validly have been in accordance with the norms of the law. For Malmstedt had not committed,
nor was he actually committing or attempting to commit a crime, in the soldiers' presence, nor
did said soldiers have personal and competent knowledge that Malmstedt had in fact just
committed a crime. All they had was a suspicion that Malmstedt might have some prohibited
drug on him or in his bags; all they had was, in the words of the Trial Court, "the hope of
intercepting any dangerous drug being transported," or, as the Office of the Solicitor General
asserts, "information that most of the buses coming . . . (from the Cordillera) were transporting
marijuana and other prohibited drugs."

This case, is remarkably similar to Peo. v. Aminnudin, decided on July 6, 1988 also by the First
Division.17 There, Aminnudin was arrested without a warrant by PC officers as he was
disembarking from an inter-island vessel. The officers were waiting for him because he was,
according to an informer's report, then transporting marijuana. The search of Aminnudin's bag
confirmed the informer's report; the bag indeed contained marijuana. The Court nevertheless
held that since the PC officers had failed to procure a search warrant although they had
sufficient time (two days) to do so and therefore, the case presented no such urgency as to
justify a warrantless search, the search of Aminnudin's person and bag, the seizure of the
marijuana and his subsequent arrest were illegal; and the marijuana was inadmissible in
evidence in the criminal action subsequently instituted against Aminnudin for violating the
Dangerous Drugs Act.

There are, on the other hand, other cases adjudicated by this Court in which apparently different
conclusions were reached. It is needful to devote a few words to them so that the relevant
constitutional and legal propositions are not misunderstood.

In People v. Claudio (decision promulgated on April 15, 1988),18 the accused boarded a "Victory
Liner" passenger bus going to Olongapo from Baguio City. She placed the plastic bag she was
carrying at the back of the seat then occupied by Obiña, an INP member "on Detached Service
with the Anti-Narcotics Unit." This avowedly aroused Obiña's suspicion, and at the first
opportunity, and without Claudio's knowledge, he surreptitiously looked into the plastic bag and
noted that it contained camote tops as well as a package, and that there emanated from the
package the smell of marijuana with which he had become familiar on account of his work. So
when the bus stopped at Sta. Rita, and Claudio alighted, Obiña accosted her, showed her his
ID, identified himself as a policeman, and announced his intention to search her bag which he
said contained marijuana because of the distinctive odor detected by him. Ignoring her plea —
"Please go with me, let us settle this at home" — he brought her to the police headquarters.,
where examination of the package in Claudio's bag confirmed his suspicion that it indeed
contained marijuana. The Court held the warrantless arrest under the circumstances to be
lawful, the search justified, and the evidence thus discovered admissible in evidence against the
accused.

In People v. Tangliben (decision promulgated on April 6, 1990),19 two police officers and
a barangay tanod were conducting a "surveillance mission" at the Victory Liner Terminal at San
Nicolas, San Fernando, Pampanga, "aimed not only against persons who may commit
misdemeanors . . . (there) but also on persons who may be engaging in the traffic of dangerous
drugs based on information supplied by informers; . . . they noticed a person carrying a red
travelling bag . . who was acting suspiciously;" they asked him to open the bag; the person did
so only after they identified themselves as peace officers; found in the bag were marijuana
leaves wrapped in plastic weighing one kilogram, more or less; the person was then taken to the
police headquarters at San Fernando, Pampanga, where he was investigated; and an
information was thereafter filed against that person, Tangliben, charging him with a violation of
the Dangerous Drugs Act of 1972 (RA 6425), as amended. Upon these facts it was
ruled, citing Claudio, supra, that there was a valid warrantless arrest and a proper warrantless
search incident thereto.

The facts in Tangliben were pronounced to be different from those in People v.


Aminnudin, supra. "In contrast" to Aminnudin where the Court perceived no urgency as to
preclude the application for and obtention of a search warrant, it was declared that
the Tangliben case —

. . . presented urgency. . . (The evidence revealed) that there was an informer who
pointed to the accused-appellant as carrying marijuana . . . Faced with such on-the-spot
information, the police officers had to act quickly. There was not enough time to secure a
search warrant . . . To require search warrants during on-the-spot apprehensions of drug
pushers, illegal possessors of firearms, jueteng collectors, smugglers of contraband
goods, robber, etc. would make it extremely difficult, if not impossible to contain the
crimes with which these persons are associated.

In Tangliben, therefore, there was in the Court's view sufficient evidence on hand to enable the
PC officers to secure a search warrant, had there been time. But because there was actually no
time to get the warrant, and there were "on-the-spot" indications that Tangliben was then
actually committing a crime, the search of his person and his effects was considered valid.

Two other decisions presented substantially similar circumstance instances: Posadas v. C.A., et
al., decided on August 2, 1990,20 and People v. Moises Maspil, Jr., et al., decided on August 20,
1990.21

In the first case, Posadas was seen to be acting suspiciously by two members of the INP,
Davao Metrodiscom, and when he was accosted by the two, who identified themselves as
police officers, he suddenly fled. He was pursued, overtaken and, notwithstanding his
resistance, placed in custody. The buri bag Posadas was then carrying was found to contain a
revolver, for which he could produce no license or authority to possess, four rounds of live
ammunition, and a tear gas grenade. He was prosecuted for illegal possession of firearms and
ammunition and convicted after trial. This Court affirmed Posadas' conviction, holding that there
was, in the premises, probable cause for a search without warrant, i.e., the appellant was acting
suspiciously and attempted to flee with the buri bag he had with him at the time. The Court cited
with approval the ruling of the U.S. Federal Supreme Court in John W. Terry v. State of
Ohio,22 a 1968 case, which the Solicitor General had invoked to justify the search.

In the case of Maspil, et al., a checkpoint was set up by elements of the First Narcotics Regional
Unit of the Narcotics Command at Sayangan, Atok, Benguet, to monitor, inspect and scrutinize
vehicles on the highway going towards Baguio City. This was done because of a confidential
report by informers that Maspil and another person, Bagking, would be transporting a large
quantity of marijuana to Baguio City. In fact, the informers were with the policemen manning the
checkpoint. As expected, at about 2 o'clock in the early morning of November 1, 1986, a
jeepney approached the checkpoint, driven by Maspil, with Bagking as passenger. The officers
stopped the vehicle and saw that on it were loaded 2 plastic sacks, a jute sack, and 3 big round
tin cans. When opened, the sacks and cans were seen to contain what appeared to be
marijuana leaves. The policemen thereupon placed Maspil and Bagking under arrest, and
confiscated the leaves which, upon scientific examination, were verified to be marijuana leaves.
The Court upheld the validity of the search thus conducted, as being incidental to a lawful
warrantless arrest,23 and declared that, as in Tangliben, supra, Maspil and Bagking had been
caught in flagrante delicto transporting prohibited drugs at the time of their arrest. Again, the
Court took occasion to distinguish the case from Aminnudin24 in which, as aforestated, it
appeared that the police officers were aware of Aminnudin's identity, his projected criminal
enterprise and the vessel on which he would be arriving, and, equally as importantly, had
sufficient time and opportunity to obtain a search warrant. In the case of Maspil and Bagking,
the Court found that the officers concerned had no exact description of the vehicle the former
would be using to transport marijuana, and no inkling of the definite time of the suspects' arrival,
and pointed out that a jeepney on the road is not the same as a passenger boat on the high
seas whose route and time of arrival are more or less certain, and which ordinarily cannot
deviate from or otherwise alter its course, or select another destination.25

The most recent decision treating of warrantless search and seizure appears to be People v. Lo
Ho Wing; et al., G.R. No. 88017, decided on January 21, 1991 (per Gancayco, J.). In that case,
an undercover or "deep penetration" agent, Tia, managed somehow to gain acceptance into a
group of suspected drug smugglers, which included Peter Lo and Lim Ching Huat. Tia
accompanied Peter Lo to Guangzhou, China, where he saw him and other person empty the
contents of six (6) tins of tea and replace them with white powder. On their return to Manila with
the cans of substituted "tea," they were met at the airport by Lim. As they were leaving the
airport in separate vehicles, they were intercepted by officers and operatives of the Narcotics
Command (NARCOM), who had earlier been tipped off by Tia, and placed under arrest. As
search of the luggage brought in by Tia and Peter Lo, loaded on the group's vehicles, quickly
disclosed the six (6) tin cans containing fifty-six (56) bags of white crystalline powder which,
upon analysis, was identified as metamphetamine. Tia, Lo and Lim were indicted for violation of
the Dangerous Drugs Act of 1972. Tia was discharged as state witness. Lo and Lim were
subsequently convicted and sentenced to life imprisonment. One of the questions raised by
them in this Court on appeal was whether the warrantless search of their vehicles and personal
effects was legal. The Court, citing Manipon, Jr. v. Sandiganbayan, 143 SCRA 267
(1986),26 held legal the search of the appellants' moving vehicles and the seizure therefrom of
the dangerous drug, considering that there was intelligence information, including clandestine
reports by a planted spy actually participating in the activity, that the appellants were bringing
prohibited drugs into the country; that the requirement of obtaining a search warrant "borders on
the impossible in the case of smuggling effected by the use of a moving vehicle that can
transport contraband from one place to another with impunity," and "it is not practicable to
secure a warrant because the vehicle can be quickly moved out of the locality or jurisdiction in
which the warrant must be sought.27

In all five cases, Claudio, Tangliben, Posadas, Maspil, and Lo Ho Wing, facts existed which
were found by the Court as justifying warantless arrests. In Claudio, the arresting officer had
secretly ascertained that the woman he was arresting was in fact in possession of marijuana; he
had personally seen that her bag contained not only vegetables but also a package emitting the
odor of marijuana. In Tangliben, the person arrested and searched was acting suspiciously, and
had been positively pointed to as carrying marijuana. And in both cases, the accused were
about to board passenger buses, making it urgent for the police officers concerned to take quick
and decisive action. In Posadas, the person arrested and searched was acting suspiciously, too,
and when accosted had attempted to flee from the police officers. And in Maspil and Lo Ho
Wing, there was definite information of the precise identity of the persons engaged in
transporting prohibited drugs at a particular time and place.
Now, as regards the precise issue at hand, whether or not the facts in the case at bar make out
a legitimate instance of a warrantless search and seizure, there is, as earlier pointed out, a
regrettable divergence of views among the members of the Court.

Contrary to the conclusion reached by the majority, I believe that the appellant should be
absolved on reasonable doubt. There was in this case no confidential report from, or positive
identification by an informer; no attempt to flee; no bag or package emitting tell-tale odors; no
other reasonably persuasive indications that Malmstedt was at the time in process of
perpetrating the offense for which he was subsequently prosecuted. Hence, when the soldiers
searched Malmstedt's pouch and the bags in his possession, they were simply "fishing" for
evidence. It matters not that the search disclosed that the bags contained prohibited
substances, confirming their initial information and suspicion. The search was not made by
virtue of a warrant or as an incident of a lawful warrantless arrest, i.e., under circumstances
sufficient to engender a reasonable belief that some crime was being or about to be committed,
or adjust been committed. There was no intelligent and intentional waiver of the right against
unreasonable searches and seizure. The search was therefore illegal, since the law requires
that there first be a lawful arrest of an individual before a search of his body and his belongings
may licitly be made. The process cannot be reversed, i.e., a search be first undertaken, and
then an arrest effected, on the strength of the evidence yielded by the search. An arrest made in
that case would be unlawful, and the search undertaken as an incident of such an unlawful
arrest, also unlawful.

The fact that when investigated at the headquarters of the Narcotic Command at Camp
Dangwa, La Trinidad, Malmstedt had, it is said, willingly admitted that there were was hashish
inside the "teddy bears" in the luggage found in his possession — an admission subsequently
confirmed by laboratory examination — does not help the cause of the prosecution one bit.
Nothing in the record even remotely suggests that Malmstedt was accorded the rights
guaranteed by the Constitution to all persons under custodial investigation.28 He was not
informed, prior to being interrogated, that he had the "right to remain silent and to have
competent and independent counsel preferably of his own choice," and that if he could not
afford the services of counsel, he would be provided with one; not does it appear at all that he
waived those rights "in writing and in the presence of counsel." The soldiers and the police
officers simply went ahead with the investigation of Malmstedt, without counsel. The admissions
elicited from Malmstedt under these circumstances, as the Constitution clearly states, are
"inadmissible in evidence against him.29

The prohibited drugs supposedly discovered in Malmstedt's bags, having been taken in violation
of the constitutional right against unreasonable searches and seizures, are inadmissible against
him "for any purpose in any proceeding." Also pronounced as incompetent evidence against him
are the admissions supposedly made by him without his first being accorded the constitutional
rights of persons under custodial investigation. Without such object evidence and admissions,
nothing remains of the case against Malmstedt.

It may be conceded that, as the Trial Court points out, the evidence presented by Malmstedt in
his defense is feeble, unworthy of credence. This is beside the point; for conformably to the
familiar axiom, the State must rely on the strength of its evidence and not on the weakness of
the defense. The unfortunate fact is that although the existence of the hashish is an objective
physical reality that cannot but be conceded, there is in law no evidence to demonstrate with
any degree of persuasion, much less beyond reasonable doubt, that Malmstedt was engaged in
a criminal activity. This is the paradox created by the disregard of the applicable constitutional
safeguards. The tangible benefit is that the hashish in question has been correctly confiscated
and thus effectively withdrawn from private use.

What is here said should not by any means be taken as a disapproval or a disparagement of the
efforts of the police and military authorities to deter and detect offenses, whether they be
possession of and traffic in prohibited drugs, or some other. Those efforts obviously merit the
support and commendation of the Courts and indeed of every responsible citizen. But those
efforts must take account of the basic rights granted by the Constitution and the law to persons
who may fall under suspicion of engaging in criminal acts. Disregard of those rights may not be
justified by the objective of ferreting out and punishing crime, no matter how eminently desirable
attainment of that objective might be. Disregard of those rights, as this Court has earlier
stressed, may result in the escape of the guilty, and all because the "constable has blundered,"
rendering the evidence inadmissible even if truthful or otherwise credible.30

I therefore vote to reverse the Trial Court's judgment of October 12, 1989 and to acquit the
appellant on reasonable doubt.

CRUZ, J., dissenting:

I join Mr. Justice Andres R. Narvasa in his dissent, which I believe represents the correct
application to the facts of this case of the provisions of the Bill of Rights and the Rules of Court
on searches and seizures. It is consistent with my ponencia in People v. Aminnudin, 163 SCRA
402, and also with Alih v. Castro, 151 SCRA 279, the latter being a unanimous decision of the
Court en banc, and my dissents in Umil v. Ramos (on warrantless arrests, 187 SCRA 311,
Valmonte v. De Villa (on checkpoints), 178, SCRA 211, 185 SCRA 665, and Guazon v. De Villa
(on "zonas"), 181 SCRA 623.

I write this separate opinion merely to remark on an observation made during the deliberation on
this case that some members of the Court seem to be coddling criminals instead of extending its
protection to society, which deserves our higher concern. The inference is that because of our
wrong priorities, criminals are being imprudently let free, to violate our laws again; and it is all
our fault.

Believing myself to be among those alluded to, I will say without apology that I do not consider a
person a criminal, until he is convicted by final judgment after a fair trial by a competent and
impartial court. Until then, the Constitution bids us to presume him innocent. He may seem
boorish or speak crudely or sport tattoos or dress weirdly or otherwise fall short of our own
standards of propriety and decorum. None of these makes him a criminal although he
may look like a criminal.

It is so easy to condemn a person on the basis of his appearance but it is also so wrong.

On the question before us, it seems to be the inclination of some judges to wink at an illegal
search and seizure as long as the suspect has been actually found in possession of a prohibited
article That fact will retroactively validate the violation of the Bill of Rights for after all, as they
would rationalize, the suspect is a criminal. What matters to them is the fact of illegal
possession, not the fact of illegal search and seizure.
This kind of thinking takes us back to the intolerant days of Moncado v. People's Court, 80 Phil.
1, which was discredited in Stonehill v. Diokno, 20 SCRA 383, even before it was definitely
rejected by an express provision in the 1973 Constitution. That provision, which has been
retained in the present Constitution, again explicitly declares that any evidence illegally obtained
"shall be inadmissible for any purpose in any proceeding."

The fruit of the poisonous tree should not be allowed to poison our system of criminal
justice.1âwphi1 In the case at bar, the search was made at a checkpoint established for the
preposterous reason that the route was being used by marijuana dealers and on an individual
who had something bulging at his waist that excited the soldier's suspicion. Was that probable
cause? The ponencia notes that the military had advance information that a Caucasian was
coming from the Sagada with prohibited drugs in his possession. This is what the military says
now, after the fact, to justify the warrantless search. It is so easy to make such a claim, and I am
surprised that the majority should readily accept it.

The conclusion that there was probable cause may have been influenced by the subsequent
discovery that the accused was carrying a prohibited drug. This is supposed to justify the
soldier's suspicion. In other words, it was the fact of illegal possession
that retroactively established the probable cause that validated the illegal search and seizure. It
was the fruit of the poisonous tree that washed clean the tree itself.

In Olmstead v. U.S., 277 U.S. 438, Justice Holmes said sixty-four years ago:

. . . It is desirable that criminals should be detected, and to that end that all available
evidence should be used.1avvphi1 It is also desirable that the government should not
itself foster and pay for other crimes, when they are the means by which the evidence is
to be obtained. If it pays its officers for having got evidence by crime, I do not see why it
may not as well pay them for getting it in the same way, and I can attach no importance
to protestations of disapproval if it knowingly accepts and pays and announces that in
the future it will pay for the fruits. We have to choose, and for my part I think it a less evil
that some criminals should escape than that the government should play an ignoble part.

If by deterring the government from playing "an ignoble part," I am "coddling criminals," I
welcome the accusation and take pride in it. I would rather err in favor of the accused who is
impaled with outlawed evidence than exalt order at the price of liberty.

G.R. No. 197788 February 29, 2012

RODEL LUZ y ONG, Petitioner,


vs.
PEOPLE OF THE PHILIPPINES,1 Respondent.

DECISION

SERENO, J.:

This is a Petition for Review on Certiorari under Rule 45 seeking to set aside the Court of
Appeals (CA) Decision in CA-G.R. CR No. 32516 dated 18 February 20112 and Resolution
dated 8 July 2011.
Statement of the Facts and of the Case

The facts, as found by the Regional Trial Court (RTC), which sustained the version of the
prosecution, are as follows:

PO2 Emmanuel L. Alteza, who was then assigned at the Sub-Station 1 of the Naga City Police
Station as a traffic enforcer, substantially testified that on March 10, 2003 at around 3:00 o’clock
in the morning, he saw the accused, who was coming from the direction of Panganiban Drive
and going to Diversion Road, Naga City, driving a motorcycle without a helmet; that this
prompted him to flag down the accused for violating a municipal ordinance which requires all
motorcycle drivers to wear helmet (sic) while driving said motor vehicle; that he invited the
accused to come inside their sub-station since the place where he flagged down the accused is
almost in front of the said sub-station; that while he and SPO1 Rayford Brillante were issuing a
citation ticket for violation of municipal ordinance, he noticed that the accused was uneasy and
kept on getting something from his jacket; that he was alerted and so, he told the accused to
take out the contents of the pocket of his jacket as the latter may have a weapon inside it; that
the accused obliged and slowly put out the contents of the pocket of his jacket which was a
nickel-like tin or metal container about two (2) to three (3) inches in size, including two (2)
cellphones, one (1) pair of scissors and one (1) Swiss knife; that upon seeing the said container,
he asked the accused to open it; that after the accused opened the container, he noticed a
cartoon cover and something beneath it; and that upon his instruction, the accused spilled out
the contents of the container on the table which turned out to be four (4) plastic sachets, the two
(2) of which were empty while the other two (2) contained suspected shabu.3

Arraigned on 2 July 2003, petitioner, assisted by counsel, entered a plea of "Not guilty" to the
charge of illegal possession of dangerous drugs. Pretrial was terminated on 24 September
2003, after which, trial ensued.

During trial, Police Officer 3 (PO3) Emmanuel Alteza and a forensic chemist testified for the
prosecution. On the other hand, petitioner testified for himself and raised the defense of planting
of evidence and extortion.

In its 19 February 2009 Decision,4 the RTC convicted petitioner of illegal possession of
dangerous drugs5 committed on 10 March 2003. It found the prosecution evidence sufficient to
show that he had been lawfully arrested for a traffic violation and then subjected to a valid
search, which led to the discovery on his person of two plastic sachets later found to contain
shabu. The RTC also found his defense of frame-up and extortion to be weak, self-serving and
unsubstantiated. The dispositive portion of its Decision held:

WHEREFORE, judgment is hereby rendered, finding accused RODEL LUZ y ONG GUILTY
beyond reasonable doubt for the crime of violation of Section 11, Article II of Republic Act No.
9165 and sentencing him to suffer the indeterminate penalty of imprisonment ranging from
twelve (12) years and (1) day, as minimum, to thirteen (13) years, as maximum, and to pay a
fine of Three Hundred Thousand Pesos (₱ 300,000.00).

The subject shabu is hereby confiscated for turn over to the Philippine Drug Enforcement
Agency for its proper disposition and destruction in accordance with law.

SO ORDERED.6
Upon review, the CA affirmed the RTC’s Decision.

On 12 September 2011, petitioner filed under Rule 45 the instant Petition for Review on
Certiorari dated 1 September 2011. In a Resolution dated 12 October 2011, this Court required
respondent to file a comment on the Petition. On 4 January 2012, the latter filed its Comment
dated 3 January 2012.

Petitioner raised the following grounds in support of his Petition:

(i) THE SEARCH AND SEIZURE OF THE ALLEGED SUBJECT SHABU IS INVALID.

(ii) THE PRESUMPTION OF REGULARITY IN THE PERFORMANCE OF DUTY OF


THE POLICE OFFICER CANNOT BE RELIED UPON IN THIS CASE.

(iii) THE INTEGRITY AND EVIDENTIARY VALUE OF THE ALLEGED SUBJECT


SPECIMEN HAS BEEN COMPROMISED.

(iv) THE GUILT OF THE ACCUSED-PETITIONER WAS NOT PROVEN BEYOND THE
REASONABLE DOUBT (sic).7

Petitioner claims that there was no lawful search and seizure, because there was no lawful
arrest. He claims that the finding that there was a lawful arrest was erroneous, since he was not
even issued a citation ticket or charged with violation of the city ordinance. Even assuming there
was a valid arrest, he claims that he had never consented to the search conducted upon him.

On the other hand, finding that petitioner had been lawfully arrested, the RTC held thus:

It is beyond dispute that the accused was flagged down and apprehended in this case by Police
Officers Alteza and Brillante for violation of City Ordinance No. 98-012, an ordinance requiring
the use of crash helmet by motorcycle drivers and riders thereon in the City of Naga and
prescribing penalties for violation thereof. The accused himself admitted that he was not
wearing a helmet at the time when he was flagged down by the said police officers, albeit he
had a helmet in his possession. Obviously, there is legal basis on the part of the apprehending
officers to flag down and arrest the accused because the latter was actually committing a crime
in their presence, that is, a violation of City Ordinance No. 98-012. In other words, the accused,
being caught in flagrante delicto violating the said Ordinance, he could therefore be lawfully
stopped or arrested by the apprehending officers. x x x.8

We find the Petition to be impressed with merit, but not for the particular reasons alleged. In
criminal cases, an appeal throws the entire case wide open for review and the reviewing tribunal
can correct errors, though unassigned in the appealed judgment, or even reverse the trial
court’s decision based on grounds other than those that the parties raised as errors.9

First, there was no valid arrest of petitioner. When he was flagged down for committing a traffic
violation, he was not, ipso facto and solely for this reason, arrested.

Arrest is the taking of a person into custody in order that he or she may be bound to answer for
the commission of an offense.10 It is effected by an actual restraint of the person to be arrested
or by that person’s voluntary submission to the custody of the one making the arrest. Neither the
application of actual force, manual touching of the body, or physical restraint, nor a formal
declaration of arrest, is required. It is enough that there be an intention on the part of one of the
parties to arrest the other, and that there be an intent on the part of the other to submit, under
the belief and impression that submission is necessary.11

Under R.A. 4136, or the Land Transportation and Traffic Code, the general procedure for
dealing with a traffic violation is not the arrest of the offender, but the confiscation of the driver’s
license of the latter:

SECTION 29. Confiscation of Driver's License. — Law enforcement and peace officers of other
agencies duly deputized by the Director shall, in apprehending a driver for any violation of this
Act or any regulations issued pursuant thereto, or of local traffic rules and regulations not
contrary to any provisions of this Act, confiscate the license of the driver concerned and issue a
receipt prescribed and issued by the Bureau therefor which shall authorize the driver to operate
a motor vehicle for a period not exceeding seventy-two hours from the time and date of issue of
said receipt. The period so fixed in the receipt shall not be extended, and shall become invalid
thereafter. Failure of the driver to settle his case within fifteen days from the date of
apprehension will be a ground for the suspension and/or revocation of his license.

Similarly, the Philippine National Police (PNP) Operations Manual12 provides the following
procedure for flagging down vehicles during the conduct of checkpoints:

SECTION 7. Procedure in Flagging Down or Accosting Vehicles While in Mobile Car. This rule
is a general concept and will not apply in hot pursuit operations. The mobile car crew shall
undertake the following, when applicable: x x x

m. If it concerns traffic violations, immediately issue a Traffic Citation Ticket (TCT) or Traffic
Violation Report (TVR). Never indulge in prolonged, unnecessary conversation or argument with
the driver or any of the vehicle’s occupants;

At the time that he was waiting for PO3 Alteza to write his citation ticket, petitioner could not be
said to have been "under arrest." There was no intention on the part of PO3 Alteza to arrest
him, deprive him of his liberty, or take him into custody. Prior to the issuance of the ticket, the
period during which petitioner was at the police station may be characterized merely as waiting
time. In fact, as found by the trial court, PO3 Alteza himself testified that the only reason they
went to the police sub-station was that petitioner had been flagged down "almost in front" of that
place. Hence, it was only for the sake of convenience that they were waiting there. There was
no intention to take petitioner into custody.

In Berkemer v. McCarty,13 the United States (U.S.) Supreme Court discussed at length whether
the roadside questioning of a motorist detained pursuant to a routine traffic stop should be
considered custodial interrogation. The Court held that, such questioning does not fall under
custodial interrogation, nor can it be considered a formal arrest, by virtue of the nature of the
questioning, the expectations of the motorist and the officer, and the length of time the
procedure is conducted. It ruled as follows:

It must be acknowledged at the outset that a traffic stop significantly curtails the "freedom of
action" of the driver and the passengers, if any, of the detained vehicle. Under the law of most
States, it is a crime either to ignore a policeman’s signal to stop one’s car or, once having
stopped, to drive away without permission. x x x
However, we decline to accord talismanic power to the phrase in the Miranda opinion
emphasized by respondent. Fidelity to the doctrine announced in Miranda requires that it be
enforced strictly, but only in those types of situations in which the concerns that powered the
decision are implicated. Thus, we must decide whether a traffic stop exerts upon a detained
person pressures that sufficiently impair his free exercise of his privilege against self-
incrimination to require that he be warned of his constitutional rights.

Two features of an ordinary traffic stop mitigate the danger that a person questioned will be
induced "to speak where he would not otherwise do so freely," Miranda v. Arizona, 384 U. S., at
467. First, detention of a motorist pursuant to a traffic stop is presumptively temporary and brief.
The vast majority of roadside detentions last only a few minutes. A motorist’s expectations,
when he sees a policeman’s light flashing behind him, are that he will be obliged to spend a
short period of time answering questions and waiting while the officer checks his license and
registration, that he may then be given a citation, but that in the end he most likely will be
allowed to continue on his way. In this respect, questioning incident to an ordinary traffic stop is
quite different from stationhouse interrogation, which frequently is prolonged, and in which the
detainee often is aware that questioning will continue until he provides his interrogators the
answers they seek. See id., at 451.

Second, circumstances associated with the typical traffic stop are not such that the motorist
feels completely at the mercy of the police. To be sure, the aura of authority surrounding an
armed, uniformed officer and the knowledge that the officer has some discretion in deciding
whether to issue a citation, in combination, exert some pressure on the detainee to respond to
questions. But other aspects of the situation substantially offset these forces. Perhaps most
importantly, the typical traffic stop is public, at least to some degree. x x x

In both of these respects, the usual traffic stop is more analogous to a so-called "Terry stop,"
see Terry v. Ohio, 392 U. S. 1 (1968), than to a formal arrest. x x x The comparatively
nonthreatening character of detentions of this sort explains the absence of any suggestion in
our opinions that Terry stops are subject to the dictates of Miranda. The similarly noncoercive
aspect of ordinary traffic stops prompts us to hold that persons temporarily detained pursuant to
such stops are not "in custody" for the purposes of Miranda.

xxx xxx xxx

We are confident that the state of affairs projected by respondent will not come to pass. It is
settled that the safeguards prescribed by Miranda become applicable as soon as a suspect’s
freedom of action is curtailed to a "degree associated with formal arrest." California v. Beheler,
463 U. S. 1121, 1125 (1983) (per curiam). If a motorist who has been detained pursuant to a
traffic stop thereafter is subjected to treatment that renders him "in custody" for practical
purposes, he will be entitled to the full panoply of protections prescribed by Miranda. See
Oregon v. Mathiason, 429 U. S. 492, 495 (1977) (per curiam). (Emphasis supplied.)

The U.S. Court in Berkemer thus ruled that, since the motorist therein was only subjected to
modest questions while still at the scene of the traffic stop, he was not at that moment placed
under custody (such that he should have been apprised of his Miranda rights), and neither can
treatment of this sort be fairly characterized as the functional equivalent of a formal arrest.
Similarly, neither can petitioner here be considered "under arrest" at the time that his traffic
citation was being made.
It also appears that, according to City Ordinance No. 98-012, which was violated by petitioner,
the failure to wear a crash helmet while riding a motorcycle is penalized by a fine only. Under
the Rules of Court, a warrant of arrest need not be issued if the information or charge was filed
for an offense penalized by a fine only. It may be stated as a corollary that neither can a
warrantless arrest be made for such an offense.

This ruling does not imply that there can be no arrest for a traffic violation. Certainly, when there
is an intent on the part of the police officer to deprive the motorist of liberty, or to take the latter
into custody, the former may be deemed to have arrested the motorist. In this case, however,
the officer’s issuance (or intent to issue) a traffic citation ticket negates the possibility of an
arrest for the same violation.

Even if one were to work under the assumption that petitioner was deemed "arrested" upon
being flagged down for a traffic violation and while awaiting the issuance of his ticket, then the
requirements for a valid arrest were not complied with.

This Court has held that at the time a person is arrested, it shall be the duty of the arresting
officer to inform the latter of the reason for the arrest and must show that person the warrant of
arrest, if any. Persons shall be informed of their constitutional rights to remain silent and to
counsel, and that any statement they might make could be used against them. 14 It may also be
noted that in this case, these constitutional requirements were complied with by the police
officers only after petitioner had been arrested for illegal possession of dangerous drugs.

In Berkemer, the U.S. Court also noted that the Miranda warnings must also be given to a
person apprehended due to a traffic violation:

The purposes of the safeguards prescribed by Miranda are to ensure that the police do not
coerce or trick captive suspects into confessing, to relieve the "inherently compelling pressures"
"generated by the custodial setting itself," "which work to undermine the individual’s will to
resist," and as much as possible to free courts from the task of scrutinizing individual cases to
try to determine, after the fact, whether particular confessions were voluntary. Those purposes
are implicated as much by in-custody questioning of persons suspected of misdemeanors as
they are by questioning of persons suspected of felonies.

If it were true that petitioner was already deemed "arrested" when he was flagged down for a
traffic violation and while he waiting for his ticket, then there would have been no need for him to
be arrested for a second time—after the police officers allegedly discovered the drugs—as he
was already in their custody.

Second, there being no valid arrest, the warrantless search that resulted from it was likewise
illegal.

The following are the instances when a warrantless search is allowed: (i) a warrantless search
incidental to a lawful arrest; (ii) search of evidence in "plain view;" (iii) search of a moving
vehicle; (iv) consented warrantless search; (v) customs search; (vi) a "stop and frisk" search;
and (vii) exigent and emergency circumstances.15 None of the above-mentioned instances,
especially a search incident to a lawful arrest, are applicable to this case.
It must be noted that the evidence seized, although alleged to be inadvertently discovered, was
not in "plain view." It was actually concealed inside a metal container inside petitioner’s pocket.
Clearly, the evidence was not immediately apparent.16

Neither was there a consented warrantless search. Consent to a search is not to be lightly
inferred, but shown by clear and convincing evidence.17 It must be voluntary in order to validate
an otherwise illegal search; that is, the consent must be unequivocal, specific, intelligently given
and uncontaminated by any duress or coercion. While the prosecution claims that petitioner
acceded to the instruction of PO3 Alteza, this alleged accession does not suffice to prove valid
and intelligent consent. In fact, the RTC found that petitioner was merely "told" to take out the
contents of his pocket.18

Whether consent to the search was in fact voluntary is a question of fact to be determined from
the totality of all the circumstances. Relevant to this determination are the following
characteristics of the person giving consent and the environment in which consent is given: (1)
the age of the defendant; (2) whether the defendant was in a public or a secluded location; (3)
whether the defendant objected to the search or passively looked on; (4) the education and
intelligence of the defendant; (5) the presence of coercive police procedures; (6) the defendant’s
belief that no incriminating evidence would be found; (7) the nature of the police questioning; (8)
the environment in which the questioning took place; and (9) the possibly vulnerable subjective
state of the person consenting. It is the State that has the burden of proving, by clear and
positive testimony, that the necessary consent was obtained, and was freely and voluntarily
given.19 In this case, all that was alleged was that petitioner was alone at the police station at
three in the morning, accompanied by several police officers. These circumstances weigh
heavily against a finding of valid consent to a warrantless search.

Neither does the search qualify under the "stop and frisk" rule. While the rule normally applies
when a police officer observes suspicious or unusual conduct, which may lead him to believe
that a criminal act may be afoot, the stop and frisk is merely a limited protective search of outer
clothing for weapons.20

In Knowles v. Iowa,21 the U.S. Supreme Court held that when a police officer stops a person for
speeding and correspondingly issues a citation instead of arresting the latter, this procedure
does not authorize the officer to conduct a full search of the car. The Court therein held that
there was no justification for a full-blown search when the officer does not arrest the motorist.
Instead, police officers may only conduct minimal intrusions, such as ordering the motorist to
alight from the car or doing a patdown:

In Robinson, supra, we noted the two historical rationales for the "search incident to arrest"
exception: (1) the need to disarm the suspect in order to take him into custody, and (2) the need
to preserve evidence for later use at trial. x x x But neither of these underlying rationales for the
search incident to arrest exception is sufficient to justify the search in the present case.

We have recognized that the first rationale—officer safety—is "‘both legitimate and weighty,’" x x
x The threat to officer safety from issuing a traffic citation, however, is a good deal less than in
the case of a custodial arrest. In Robinson, we stated that a custodial arrest involves "danger to
an officer" because of "the extended exposure which follows the taking of a suspect into custody
and transporting him to the police station." 414 U. S., at 234-235. We recognized that "[t]he
danger to the police officer flows from the fact of the arrest, and its attendant proximity, stress,
and uncertainty, and not from the grounds for arrest." Id., at 234, n. 5. A routine traffic stop, on
the other hand, is a relatively brief encounter and "is more analogous to a so-called ‘Terry stop’ .
. . than to a formal arrest." Berkemer v. McCarty, 468 U. S. 420, 439 (1984). See also Cupp v.
Murphy, 412 U. S. 291, 296 (1973) ("Where there is no formal arrest . . . a person might well be
less hostile to the police and less likely to take conspicuous, immediate steps to destroy
incriminating evidence").

This is not to say that the concern for officer safety is absent in the case of a routine traffic
stop.1âwphi1 It plainly is not. See Mimms, supra, at 110; Wilson, supra, at 413-414. But while
the concern for officer safety in this context may justify the "minimal" additional intrusion of
ordering a driver and passengers out of the car, it does not by itself justify the often considerably
greater intrusion attending a full fieldtype search. Even without the search authority Iowa urges,
officers have other, independent bases to search for weapons and protect themselves from
danger. For example, they may order out of a vehicle both the driver, Mimms, supra, at 111, and
any passengers, Wilson, supra, at 414; perform a "patdown" of a driver and any passengers
upon reasonable suspicion that they may be armed and dangerous, Terry v. Ohio, 392 U. S. 1
(1968); conduct a "Terry patdown" of the passenger compartment of a vehicle upon reasonable
suspicion that an occupant is dangerous and may gain immediate control of a weapon,
Michigan v. Long, 463 U. S. 1032, 1049 (1983); and even conduct a full search of the
passenger compartment, including any containers therein, pursuant to a custodial arrest, New
York v. Belton, 453 U. S. 454, 460 (1981).

Nor has Iowa shown the second justification for the authority to search incident to arrest—the
need to discover and preserve evidence. Once Knowles was stopped for speeding and issued a
citation, all the evidence necessary to prosecute that offense had been obtained. No further
evidence of excessive speed was going to be found either on the person of the offender or in
the passenger compartment of the car. (Emphasis supplied.)

The foregoing considered, petitioner must be acquitted. While he may have failed to object to
the illegality of his arrest at the earliest opportunity, a waiver of an illegal warrantless arrest does
not, however, mean a waiver of the inadmissibility of evidence seized during the illegal
warrantless arrest.22

The Constitution guarantees the right of the people to be secure in their persons, houses,
papers and effects against unreasonable searches and seizures.23 Any evidence obtained in
violation of said right shall be inadmissible for any purpose in any proceeding. While the power
to search and seize may at times be necessary to the public welfare, still it must be exercised
and the law implemented without contravening the constitutional rights of citizens, for the
enforcement of no statute is of sufficient importance to justify indifference to the basic principles
of government.24

The subject items seized during the illegal arrest are inadmissible.25 The drugs are the very
corpus delicti of the crime of illegal possession of dangerous drugs. Thus, their inadmissibility
precludes conviction and calls for the acquittal of the accused.26

WHEREFORE, the Petition is GRANTED. The 18 February 2011 Decision of the Court of
Appeals in CA-G.R. CR No. 32516 affirming the judgment of conviction dated 19 February 2009
of the Regional Trial Court, 5th Judicial Region, Naga City, Branch 21, in Criminal Case No.
RTC 2003-0087, is hereby REVERSED and SET ASIDE. Petitioner Rodel Luz y Ong is hereby
ACQUITTED and ordered immediately released from detention, unless his continued
confinement is warranted by some other cause or ground.
SO ORDERED.

G.R. No. 120431 April 1, 1998

RODOLFO ESPANO, accused-Petitioner, v. COURT OF APPEALS and PEOPLE OF THE


PHILIPPINES, Respondents.

ROMERO, J.:

This is a petition for review of the decision of the Court of Appeals in CA-G.R. CR No. 13976
dated January 16, 1995, 1 which affirmed in toto the judgment of the Regional Trial Court of
Manila, Branch 1, convincing petitioner Rodolfo Espano for violation of Article II, Section 8 of
Republic Act No. 6425, as amended, otherwise known as the Dangerous Drugs Act.

Petitioner was charged under the following information:

That on or about July 14, 1991, in the City of Manila, Philippines, the said accused not being
authorized by law to possess or use any prohibited drug, did then and there willfully, unlawfully
and knowingly have in his possession and under his custody and control twelve (12) plastic
cellophane (bags) containing crushed flowering tops, marijuana weighing 5.5 grams which is a
prohibited drug.

Contrary to law. 2

The evidence for the prosecution, based on the testimony of Pat. Romeo Pagilagan, shows that
on July 14, 1991, at about 12:30 a.m., he and other police officers, namely, Pat. Wilfredo
Aquino, Simplicio Rivera, and Erlindo Lumboy of the Western Police District (WPD), Narcotics
Division went to Zamora and Pandacan Streets, Manila to confirm reports of drug pushing in the
area. They saw petitioner selling "something" to another person. After the alleged buyer left,
they approached petitioner, identified themselves as policemen, and frisked him. The search
yielded two plastic cellophane tea bags of marijuana. When asked if he had more marijuana, he
replied that there was more in his house. The policemen went to his residence where they found
ten more cellophane tea bags of marijuana. Petitioner was brought to the police headquarters
where he was charged with possession of prohibited drugs. On July 24, 1991, petitioner posted
bail 3 and the trial court issued his order of release on July 29, 1991. 4

Annabelle Alip, forensic chemist of the WPD Criminal Investigation Laboratory Section, testified
that the articles sent to her by Pat. Wilfredo Aquino regarding the apprehension of a certain
Rodolfo Espano for examination tested positive for marijuana, with a total weight of 5.5 grams.

By way of defense, petitioner testified that on said evening, he was sleeping in his house and
was awakened only when the policemen handcuffed him. He alleged that the policemen were
looking for his brother-in-law Lauro, and when they could not find the latter, he was instead
brought to the police station for investigation and later indicted for possession of prohibited
drugs. His wife Myrna corroborated his story.
The trial court rejected petitioner's, defense as a "mere afterthought" and found the version of
the prosecution "more credible and trustworthy."

Thus, on August 14, 1992, the trial court rendered a decision, convicting petitioner of the crime
charged, the dispositive portion of which reads:

WHEREFORE there being proof beyond reasonable doubt, the court finds the accused Rodolfo
Espano y Valeria guilty of the crime of violation of Section 8, Article II, in relation to Section 2 (e-
L) (I) of Republic Act No. 6425 as amended by Batas Pambansa Blg. 179, and pursuant to law
hereby sentences him to suffer imprisonment of six (6) years and one (1) day to twelve (12)
years and to pay a fine of P6,000.00 with subsidiary imprisonment in case of default plus costs.

The marijuana is declared forfeited in favor of government and shall be turned over to the
Dangerous Drugs Board without delay.

SO ORDERED. 5

Petitioner appealed the decision to the Court of Appeals. The appellate court, however, affirmed
the decision of the trial court in toto.

Hence, this petition.

Petitioner contends that the trial and appellate courts erred in convicting him on the basis of the
following: (a) the pieces of evidence seized were inadmissible; (b) the superiority of his
constitutional right to be presumed innocent over the doctrine of presumption of regularity, (c)
he was denied the constitutional right of confrontation and to compulsory process; and (d) his
conviction was based on evidence which was irrelevant and not properly identified.

After a careful examination of the records of the case, this Court finds no compelling reason
sufficient to reverse the decisions of the trial and appellate courts.

First, it is a well settled doctrine that findings of trial courts on the credibility of witnesses
deserve a high degree of respect. Having observed the deportment of witnesses during the trial,
the trial judge is in a better position to determine the issue of credibility and, thus, his findings
will not be disturbed during appeal in the absence of any clear showing that he had overlooked,
misunderstood or misapplied some facts or circumstances of weight and substance which could
have altered the conviction of the appellants. 6

In this case, the findings of the trial court that the prosecution witnesses were more credible
than those of the defense must stand. Petitioner failed to show that Pat. Pagilagan, in testifying
against him, was motivated by reasons other than his duty to curb drug abuse and had any
intent to falsely impute to him such a serious crime as possession of prohibited drugs. In the
absence of such ill motive, the presumption of regularity in the performance of his official duty
must prevail.

In People v. Velasco, 7 this Court reiterated the doctrine of presumption of regularity in the
performance of official duty which provides:
. . . Appellant failed to establish that Pat. Godoy and the other members of the buy-bust team
are policemen engaged in mulcting or other unscrupulous activities who were motivated either
by the desire to extort money or exact personal vengeance, or by sheer whim and caprice,
when they entrapped her. And in the absence of proof of any intent on the part of the police
authorities to falsely impute such a serious crime against appellant, as in this case, the
presumption of regularity in the performance of official duty, . . . , must prevail over the self-
serving and uncorroborated claim of appellant that she had been framed. 8

Furthermore, the defense set up by petitioner does not deserve any consideration. He simply
contended that he was in his house sleeping at the time of the incident. This Court has
consistently held that alibi is the weakest of all defenses; and for it to prosper, the accused has
the burden of proving that he was not at the scene of the crime at the time of its commission
and that it was physically impossible for him to be there. Moreover, the "claim of a 'frame-up',
like alibi, is a defense that has been invariably viewed by the Court with disfavor for it can just
as easily be concocted but difficult to prove, and is a common and standard line of defense in
most prosecutions arising from violations of the Dangerous Drugs Act." 9 No clear and
convincing evidence was presented by petitioner to prove his defense of alibi.

Second, petitioner contends that the prosecution's failure to present the alleged informant in
court cast a reasonable doubt which warrants his acquittal. This is again without merit, since
failure of the prosecution to produce the informant in court is of no moment especially when he
is not even the best witness to establish the fact that a buy-bust operation had indeed been
conducted. In this case, Pat. Pagilagan, one of the policemen who apprehended petitioner,
testified on the actual incident of July 14, 1991, and identified him as the one they caught in
possession of prohibited drugs. Thus,

We find that the prosecution had satisfactorily proved its case against appellants. There is no
compelling reason for us to overturn the finding of the trial court that the testimony of Sgt.
Gamboa, the lone witness for the prosecution, was straightforward spontaneous and
convincing. The testimony of a sole witness, if credible and positive and satisfies the court
beyond reasonable doubt, is sufficient to convict. 10

Thus on the basis of Pat. Pagilagan's testimony, the prosecution was able to prove that
petitioner indeed committed the crime charged; consequently, the finding of conviction was
proper.

Lastly, the issue on the admissibility of the marijuana seized should likewise be ruled upon. Rule
113 Section 5(a) of the Rules of Court provides:

A peace officer or a private person may, without a warrant, arrest a person:

a. when, in his presence, the person to be arrested has committed, is actually committing, or is
attempting to commit an offense;

xxx xxx xxx

Petitioner's arrest falls squarely under the aforecited rule. He was caught in flagranti as a result
of a buy-bust operation conducted by police officers on the basis of information received
regarding the illegal trade of drugs within the area of Zamora and Pandacan Streets, Manila.
The police officer saw petitioner handing over something to an alleged buyer. After the buyer
left, they searched him and discovered two cellophanes of marijuana. His arrest was, therefore,
lawful and the two cellophane bags of marijuana seized were admissible in evidence, being the
fruits of the crime.

As for the ten cellophane bags of marijuana found at petitioner's residence, however, the same
are inadmissible in evidence.

The 1987 Constitution guarantees freedom against unreasonable searches and seizures under
Article III, Section 2 which provides:

The right of the people to be secure in their persons, houses, papers and effects against
unreasonable searches and seizures of whatever nature and for any purpose shall be inviolable,
and no search warrant or warrant of arrest shall issue except upon probable cause to be
determined personally by the judge after examination under oath or affirmation of the
complainant and the witnesses he may produce, and particularly describing the place to be
searched and the persons or things to be seized.

An exception to the said rule is a warrantless search incidental to a lawful arrest for dangerous
weapons or anything which may be used as proof of the commission of an offense. 11 It may
extend beyond the person of the one arrested to include the premises or surroundings under his
immediate control. In this case, the ten cellophane bags of marijuana seized at petitioner's
house after his arrest at Pandacan and Zamora Streets do not fall under the said exceptions.

In the case of People v. Lua, 12 this Court held:

As regards the brick of marijuana found inside the appellant's house, the trial court correctly
ignored it apparently in view of its inadmissibility. While initially the arrest as well as the body
search was lawful, the warrantless search made inside the appellant's house became unlawful
since the police operatives were not armed with a search warrant. Such search cannot fall
under "search made incidental to a lawful arrest," the same being limited to body search and to
that point within reach or control of the person arrested, or that which may furnish him with the
means of committing violence or of escaping. In the case at bar, appellant was admittedly
outside his house when he was arrested. Hence, it can hardly be said that the inner portion of
his house was within his reach or control.

The articles seized from petitioner during his arrest were valid under the doctrine of search
made incidental to a lawful arrest. The warrantless search made in his house, however, which
yielded ten cellophane bags of marijuana became unlawful since the police officers were not
armed with a search warrant at the time. Moreover, it was beyond the reach and control of
petitioner.

In sum, this Court finds petitioner Rodolfo Espano guilty beyond reasonable doubt of violating
Article II, Section 8, in relation to Section 2 (e-L) (I) of Republic Act No. 6425, as amended.
Under the said provision, the penalty imposed is six years and one day to twelve years and a
fine ranging from six thousand to twelve thousand pesos. With the passage of Republic Act No.
7659, which took effect on December 31, 1993, the imposable penalty shall now depend on the
quantity of drugs recovered. Under the provisions of Republic Act No. 7629, Section 20, and as
interpreted in People v. Simon 13 and People v. Lara, 14 if the quantity of marijuana involved is
less than 750 grams, the imposable penalty ranges from prision correccional to reclusion
temporal. Taking into consideration that petitioner is not a habitual delinquent, the amendatory
provision is favorable to him and the quantity of marijuana involved is less than 750 grams, the
penalty imposed under Republic Act No. 7659 should be applied. There being no mitigating nor
aggravating circumstances, the imposable penalty shall be prision correccional in its medium
period. Applying the Indeterminate Sentence Law, the maximum penalty shall be taken from the
medium period of prision correccional, which is two (2) years, four (4) months and one (1) day to
four (4) years and two (2) months, while the minimum shall be taken from the penalty next lower
in degree, which is one (1) month and one (1) day to six (6) months of arresto mayor.

WHEREFORE, the instant petition is hereby DENIED. The decision of the Court of Appeals in
C.A.-G.R. CR No. 13976 dated January 16, 1995 is AFFIRMED with the MODIFICATION that
petitioner Rodolfo Espano is sentenced to suffer an indeterminate penalty of TWO (2) months
and ONE (1) day of arresto mayor, as minimum to TWO (2) years, FOUR (4) months and ONE
(1) day of prision correccional, as maximum.

SO ORDERED.

G.R. No. 163858 June 28, 2005

UNITED LABORATORIES, INC., petitioner,


vs.
ERNESTO ISIP and/or SHALIMAR PHILIPPINES and/or OCCUPANTS, Shalimar Building,
No. 1571, Aragon Street, Sta. Cruz, Manila, respondents.

DECISION

CALLEJO, SR., J.:

Rolando H. Besarra, Special Investigator III of the National Bureau of Investigation (NBI), filed
an application, in the Regional Trial Court (RTC) of Manila, for the issuance of a search warrant
concerning the first and second floors of the Shalimar Building, located at No. 1571, Aragon
Street (formerly No. 1524, Lacson Avenue, Sta. Cruz, Manila) occupied and/or used by
Shalimar Philippines, owned/operated by Ernesto Isip; and for the seizure of the following for
violation of Section 4(a), in relation to Section 8, of Republic Act (R.A.) No. 8203:

a. Finished or unfinished products of UNITED LABORATORIES (UNILAB), particularly


REVICON multivitamins;

b. Sundry items such as tags, labels, boxes, packages, wrappers, receptacles,


advertisements and other paraphernalia used in the offering for sale, sale and/or
distribution of counterfeit REVICON multivitamins;

c. Sales invoices, delivery receipts, official receipts, ledgers, journals, purchase orders
and all other books of accounts and documents used in recording the manufacture
and/or importation, distribution and/or sales of counterfeit REVICON multivitamins.1

The application was docketed as People v. Ernesto Isip, et al., Respondents, Search Warrant
Case No. 04-4916 and raffled to Branch 24 of the court. Appended thereto were the following:
(1) a sketch2 showing the location of the building to be searched; (2) the affidavit3 of Charlie
Rabe of the Armadillo Protection and Security Agency hired by United Laboratories, Inc.
(UNILAB), who allegedly saw the manufacture, production and/or distribution of fake drug
products such as Revicon by Shalimar Philippines; (3) the letter-request of UNILAB, the duly
licensed and exclusive manufacturer and/or distributor of Revicon and Disudrin, for the
monitoring of the unauthorized production/manufacture of the said drugs and, if warranted, for
their seizure; (4) the letter-complaint4 of UNILAB issued through its Director of the Security and
Safety Group; and (5) the joint affidavit5 of NBI Agents Roberto Divinagracia and Rolando
Besarra containing the following allegations:

2. When learned that an Asset was already placed by ARMADILLO PROTECTIVE AND
SECURITY AGENCY named CHARLIE RABE, who was renting a room since November
2003, at the said premises located at No. 1571 Aragon St., Sta. Cruz, Manila. MR.
RABE averred that the owner of the premises is a certain MR. ERNESTO ISIP and that
the said premises which is known as SHALIMAR PHILIPPINES, Shalimar Building, are
being used to manufacture counterfeit UNILAB products, particularly REVICON
multivitamins, which was already patented by UNILAB since 1985;

3. Upon verification of the report, we found out that the said premises is a six-story
structure, with an additional floor as a penthouse, and colored red-brown. It has a tight
security arrangement wherein non-residents are not allowed to enter or reconnoiter in
the premises;

4. We also learned that its old address is No. 1524 Lacson Avenue, Sta. Cruz, Manila,
and has a new address as 1571 Aragon St., Sta. Cruz, Manila; and that the area of
counterfeiting operations are the first and second floors of Shalimar Building;

5. Since we cannot enter the premises, we instructed the Asset to take pictures of the
area especially the places wherein the clandestine manufacturing operations were being
held. At a peril to his well-being and security, the Asset was able to take photographs
herein incorporated into this Search Warrant Application.6

A representative from UNILAB, Michael Tome, testified during the hearing on the application for
the search warrant. After conducting the requisite searching questions, the court granted the
application and issued Search Warrant No. 04-4916 dated January 27, 2004, directing any
police officer of the law to conduct a search of the first and second floors of the Shalimar
Building located at No. 1571, Aragon Street, Sta. Cruz, Manila. The court also directed the
police to seize the following items:

a. Finished or unfinished products of UNITED LABORATORIES (UNILAB), particularly


REVICON multivitamins;

b. Sundry items such as tags, labels, boxes, packages, wrappers, receptacles,


advertisements and other paraphernalia used in the offering for sale, sale and/or
distribution of counterfeit REVICON multivitamins;

c. Sales invoices, delivery receipts, official receipts, ledgers, journals, purchase orders
and all other books of accounts and documents used in recording the manufacture
and/or importation, distribution and/or sales of counterfeit REVICON multivitamins.7

The court also ordered the delivery of the seized items before it, together with a true inventory
thereof executed under oath.
The search warrant was implemented at 4:30 p.m. on January 27, 2004 by NBI agents Besarra
and Divinagracia, in coordination with UNILAB employees. No fake Revicon multivitamins were
found; instead, there were sealed boxes at the first and second floors of the Shalimar Building
which, when opened by the NBI agents in the presence of respondent Isip, contained the
following:

QUANTITY/UNIT DESCRIPTION

792 Bottles Disudrin 60 ml.

30 Boxes (100 pieces each) Inoflox 200 mg.8

NBI Special Investigator Divinagracia submitted an inventory of the things seized in which he
declared that the search of the first and second floors of the Shalimar Building at No. 1571,
Aragon Street, Sta. Cruz, Manila, the premises described in the warrant, was done in an orderly
and peaceful manner. He also filed a Return of Search Warrant,9 alleging that no other
articles/items other than those mentioned in the warrant and inventory sheet were seized. The
agent prayed that of the items seized, ten boxes of Disudrin 60 ml., and at least one box of
Inoflox be turned over to the custody of the Bureau of Food and Drugs (BFAD) for
examination.10 The court issued an order granting the motion, on the condition that the turn over
be made before the court, in the presence of a representative from the respondents and the
court.11

The respondents filed an "Urgent Motion to Quash the Search Warrant or to Suppress
Evidence."12 They contended that the implementing officers of the NBI conducted their search at
the first, second, third and fourth floors of the building at No. 1524-A, Lacson Avenue, Sta. Cruz,
Manila, where items in "open display" were allegedly found. They pointed out, however, that
such premises was different from the address described in the search warrant, the first and
second floors of the Shalimar Building located at No. 1571, Aragon Street, Sta. Cruz, Manila.
The respondents, likewise, asserted that the NBI officers seized Disudrin and Inoflox products
which were not included in the list of properties to be seized in the search warrant.

UNILAB, in collaboration with the NBI, opposed the motion, insisting that the search was limited
to the first and second floors of the Shalimar building located at the corner of Aragon Street and
Lacson Avenue, Sta. Cruz, Manila. They averred that, based on the sketch appended to the
search warrant application, Rabe’s affidavit, as well as the joint affidavit of Besarra and
Divinagracia, the building where the search was conducted was located at No. 1571, Aragon
Street corner Lacson Avenue, Sta. Cruz, Manila. They pointed out that No. 1524 Lacson
Avenue, Sta. Cruz, Manila was the old address, and the new address was No. 1571, Aragon
Street, Sta. Cruz, Manila. They maintained that the warrant was not implemented in any other
place.13

In reply, the respondents insisted that the items seized were different from those listed in the
search warrant. They also claimed that the seizure took place in the building located at No.
1524-A which was not depicted in the sketch of the premises which the applicant submitted to
the trial court.14 In accordance with the ruling of this Court in People v. Court of Appeals,15 the
respondents served a copy of their pleading on UNILAB.16
On March 11, 2004, the trial court issued an Order17 granting the motion of the respondents, on
the ground that the things seized, namely, Disudrin and Inoflox, were not those described in the
search warrant. On March 16, 2004, the trial court issued an advisory18 that the seized articles
could no longer be admitted in evidence against the respondents in any proceedings, as the
search warrant had already been quashed.

UNILAB, through the Ureta Law Office, filed a motion, in collaboration with the NBI agents, for
the reconsideration of the order, contending that the ground used by the court in quashing the
warrant was not that invoked by the respondents, and that the seizure of the items was justified
by the plain view doctrine. The respondents objected to the appearance of the counsel of
UNILAB, contending that the latter could not appear for the People of the Philippines. The
respondents moved that the motion for reconsideration of UNILAB be stricken off the record.
Disputing the claims of UNILAB, they insisted that the items seized were contained in boxes at
the time of the seizure at No. 1524-A, Lacson Avenue corner Aragon Street, Sta. Cruz, Manila,
and were not apparently incriminating on plain view. Moreover, the seized items were not those
described and itemized in the search warrant application, as well as the warrant issued by the
court itself. The respondents emphasized that the Shalimar Laboratories is authorized to
manufacture galenical preparations of the following products:

Products:

- Povidone Iodine

- Chamomile Oil

- Salicylic Acid 10 g.

- Hydrogen Peroxide 3% Topical Solution

- Aceite de Alcamforado

- Aceite de Manzanilla19

In a manifestation and opposition, the respondents assailed the appearance of the counsel of
UNILAB, and insisted that it was not authorized to appear before the court under the Rules of
Court, and to file pleadings. They averred that the BFAD was the authorized government
agency to file an application for a search warrant.

In its counter-manifestation, UNILAB averred that it had the personality to file the motion for
reconsideration because it was the one which sought the filing of the application for a search
warrant; besides, it was not proscribed by Rule 126 of the Revised Rules of Criminal Procedure
from participating in the proceedings and filing pleadings. The only parties to the case were the
NBI and UNILAB and not the State or public prosecutor. UNILAB also argued that the offended
party, or the holder of a license to operate, may intervene through counsel under Section 16 of
Rule 110, in relation to Section 7(e), of the Rules of Criminal Procedure.

UNILAB prayed that an ocular inspection be conducted of the place searched by the NBI
officers.20 In their rejoinder, the respondents manifested that an ocular inspection was the option
to look forward to.21 However, no such ocular inspection of the said premises was conducted.
In the meantime, the BFAD submitted to the court the result of its examination of the Disudrin
and Inoflox samples which the NBI officers seized from the Shalimar Building. On its
examination of the actual component of Inoflox, the BFAD declared that the substance failed the
test.22 The BFAD, likewise, declared that the examined Disudrin syrup failed the test.23 The
BFAD had earlier issued the following report:

PRODUCT NAME Manufacturer L.N. E.D. FINDINGS

1.Phenylpropanolamine Unilab 21021552 3-06 -Registered, however,


(Disudrin) label/physical
12.5 mg./5mL Syrup appearance does not
conform with the BFAD
approved label/
registered specifications.

2.Ofloxacin (Inoflox) Unilab 99017407 3-05 -Registered, however,


200 mg. tablet. label/physical
appearance does not
conform with the BFAD
approved label/
registered
specifications.24

On May 28, 2004, the trial court issued an Order25 denying the motion for reconsideration filed
by UNILAB. The court declared that:

The Search Warrant is crystal clear: The seizing officers were only authorized to take
possession of "finished or unfinished products of United Laboratories (UNILAB), particularly
REVICON Multivitamins, and documents evidencing the counterfeit nature of said products. The
Receipt/Inventory of Property Seized pursuant to the warrant does not, however, include
REVICON but other products. And whether or not these seized products are imitations of
UNILAB items is beside the point. No evidence was shown nor any was given during the
proceedings on the application for search warrant relative to the seized products.

On this score alone, the search suffered from a fatal infirmity and, hence, cannot be sustained.26

UNILAB, thus, filed the present petition for review on certiorari under Rule 45 of the Rules of
Court, where the following issues are raised:

Whether or not the seized 792 bottles of Disudrin 60 ml. and 30 boxes of Inoflox 200 mg. are
INADMISSIBLE as evidence against the respondents because they constitute the "fruit of the
poisonous tree" or, CONVERSELY, whether or not the seizure of the same counterfeit drugs is
justified and lawful under the "plain view" doctrine and, hence, the same are legally admissible
as evidence against the respondents in any and all actions?27

The petitioner avers that it was deprived of its right to a day in court when the trial court
quashed the search warrant for a ground which was not raised by the respondents herein in
their motion to quash the warrant. As such, it argues that the trial court ignored the issue raised
by the respondents. The petitioner insists that by so doing, the RTC deprived it of its right to due
process. The petitioner asserts that the description in the search warrant of the products to be
seized – "finished or unfinished products of UNILAB" – is sufficient to include counterfeit drugs
within the premises of the respondents not covered by any license to operate from the BFAD,
and/or not authorized or licensed to manufacture, or repackage drugs produced or
manufactured by UNILAB. Citing the ruling of this Court in Padilla v. Court of Appeals,28 the
petitioner asserts that the products seized were in plain view of the officers; hence, may be
seized by them. The petitioner posits that the respondents themselves admitted that the seized
articles were in open display; hence, the said articles were in plain view of the implementing
officers.

In their comment on the petition, the respondents aver that the petition should have been filed
before the Court of Appeals (CA) because factual questions are raised. They also assert that
the petitioner has no locus standi to file the petition involving the validity and the implementation
of the search warrant. They argue that the petitioner merely assisted the NBI, the BFAD and the
Department of Justice; hence, it should have impleaded the said government agencies as
parties-petitioners. The petition should have been filed by the Office of the Solicitor General
(OSG) in behalf of the NBI and/or the BFAD, because under the 1987 Revised Administrative
Code, the OSG is mandated to represent the government and its officers charged in their official
capacity in cases before the Supreme Court. The respondents further assert that the trial court
may consider issues not raised by the parties if such consideration would aid the court in the
just determination of the case.

The respondents, likewise, maintain that the raiding team slashed the sealed boxes so fast even
before respondent Isip could object. They argue that the seizure took place at No. 1524-A,
Lacson Avenue, Sta. Cruz, Manila covered by Transfer Certificate of Title (TCT) No. 220778,
and not at No. 1571, Aragon Street, Sta. Cruz, Manila covered by TCT No. 174412 as stated in
the search warrant. They assert that the ruling of the Court in People v. Court of Appeals29 is
applicable in this case. They conclude that the petitioner failed to prove the factual basis for the
application of the plain view doctrine.30

In reply, the petitioner asserts that it has standing and is, in fact, the real party-in-interest to
defend the validity of the search warrant issued by the RTC; after all, it was upon its instance
that the application for a search warrant was filed by the NBI, which the RTC granted. It asserts
that it is not proscribed under R.A. No. 8203 from filing a criminal complaint against the
respondents and requesting the NBI to file an application for a search warrant. The petitioner
points out that the Rules of Criminal Procedure does not specifically prohibit a private
complainant from defending the validity of a search warrant. Neither is the participation of a
state prosecutor provided in Rule 126 of the said Rules. After all, the petitioner insists, the
proceedings for the application and issuance of a search warrant is not a criminal action. The
petitioner asserts that the place sought to be searched was sufficiently described in the warrant
for, after all, there is only one building on the two parcels of land described in two titles where
Shalimar Philippines is located, the place searched by the NBI officers.31 It also asserts that the
building is located at the corner of Aragon Street and Lacson Avenue, Sta. Cruz, Manila.32

The petitioner avers that the plain view doctrine is applicable in this case because the boxes
were found outside the door of the respondents’ laboratory on the garage floor. The boxes
aroused the suspicion of the members of the raiding team – precisely because these were
marked with the distinctive UNILAB logos. The boxes in which the items were contained were
themselves so designated to replicate true and original UNILAB boxes for the same medicine.
Thus, on the left hand corner of one side of some of the boxes33 the letters "ABR" under the
words "60 ml," appeared to describe the condition/quality of the bottles inside (as it is with
genuine UNILAB box of the true medicine of the same brand). The petitioner pointed out that
"ABR" is the acronym for "amber bottle round" describing the bottles in which the true and
original Disudrin (for children) is contained.

The petitioner points out that the same boxes also had their own "license plates" which were
instituted as among its internal control/countermeasures. The license plates indicate that the
items within are, supposedly, "Disudrin." The NBI officers had reasonable ground to believe that
all the boxes have one and the same data appearing on their supposedly distinctive license
plates. The petitioner insists that although some of the boxes marked with the distinctive
UNILAB logo were, indeed, sealed, the tape or seal was also a copy of the original because
these, too, were marked with the distinctive UNILAB logo. The petitioner appended to its
pleading pictures of the Shalimar building and the rooms searched showing respondent
Isip;34 the boxes seized by the police officers containing Disudrin syrup;35 and the boxes
containing Inoflox and its contents.36

The issues for resolution are the following: (1) whether the petitioner is the proper party to file
the petition at bench; (2) whether it was proper for the petitioner to file the present petition in this
Court under Rule 45 of the Rules of Court; and (3) whether the search conducted by the NBI
officers of the first and second floors of the Shalimar building and the seizure of the sealed
boxes which, when opened, contained Disudrin syrup and Inoflox, were valid.

On the first issue, we agree with the petitioner’s contention that a search warrant proceeding is,
in no sense, a criminal action37 or the commencement of a prosecution.38 The proceeding is not
one against any person, but is solely for the discovery and to get possession of personal
property. It is a special and peculiar remedy, drastic in nature, and made necessary because of
public necessity. It resembles in some respect with what is commonly known as John Doe
proceedings.39 While an application for a search warrant is entitled like a criminal action, it does
not make it such an action.

A search warrant is a legal process which has been likened to a writ of discovery employed by
the State to procure relevant evidence of crime.40 It is in the nature of a criminal process,
restricted to cases of public prosecutions.41 A search warrant is a police weapon, issued under
the police power. A search warrant must issue in the name of the State, namely, the People of
the Philippines.42

A search warrant has no relation to a civil process. It is not a process for adjudicating civil rights
or maintaining mere private rights.43 It concerns the public at large as distinguished from the
ordinary civil action involving the rights of private persons.44 It may only be applied for in the
furtherance of public prosecution.45

However, a private individual or a private corporation complaining to the NBI or to a government


agency charged with the enforcement of special penal laws, such as the BFAD, may appear,
participate and file pleadings in the search warrant proceedings to maintain, inter alia, the
validity of the search warrant issued by the court and the admissibility of the properties seized in
anticipation of a criminal case to be filed; such private party may do so in collaboration with the
NBI or such government agency. The party may file an opposition to a motion to quash the
search warrant issued by the court, or a motion for the reconsideration of the court order
granting such motion to quash.46

In this case, UNILAB, in collaboration with the NBI, opposed the respondents’ motion to quash
the search warrant. The respondents served copies of their reply and opposition/comment to
UNILAB, through Modesto Alejandro, Jr.47 The court a quo allowed the appearance of UNILAB
and accepted the pleadings filed by it and its counsel.

The general rule is that the proper party to file a petition in the CA or Supreme Court to assail
any adverse order of the RTC in the search warrant proceedings is the People of the
Philippines, through the OSG. However, in Columbia Pictures Entertainment, Inc. v. Court of
Appeals,48 the Court allowed a private corporation (the complainant in the RTC) to file a petition
for certiorari, and considered the petition as one filed by the OSG. The Court in the said case
even held that the petitioners therein could argue its case in lieu of the OSG:

From the records, it is clear that, as complainants, petitioners were involved in the proceedings
which led to the issuance of Search Warrant No. 23. In People v. Nano, the Court declared that
while the general rule is that it is only the Solicitor General who is authorized to bring or defend
actions on behalf of the People or the Republic of the Philippines once the case is brought
before this Court or the Court of Appeals, if there appears to be grave error committed by the
judge or a lack of due process, the petition will be deemed filed by the private complainants
therein as if it were filed by the Solicitor General. In line with this ruling, the Court gives this
petition due course and will allow petitioners to argue their case against the questioned order in
lieu of the Solicitor General.49

The general rule is that a party is mandated to follow the hierarchy of courts. However, in
exceptional cases, the Court, for compelling reasons or if warranted by the nature of the issues
raised, may take cognizance of petitions filed directly before it.50 In this case, the Court has
opted to take cognizance of the petition, considering the nature of the issues raised by the
parties.

The Court does not agree with the petitioner’s contention that the issue of whether the Disudrin
and Inoflox products were lawfully seized was never raised in the pleadings of the respondents
in the court a quo. Truly, the respondents failed to raise the issue in their motion to quash the
search warrant; in their reply, however, they averred that the seized items were not included in
the subject warrant and, therefore, were not lawfully seized by the raiding team. They also
averred that the said articles were not illegal per se, like explosives and shabu, as to justify their
seizure in the course of unlawful search.51 In their Opposition/Comment filed on March 15,
2004, the respondents even alleged the following:

The jurisdiction of this Honorable Court is limited to the determination of whether there is a legal
basis to quash the search warrant and/or to suppress the seized articles in evidence. Since the
articles allegedly seized during the implementation of the search warrant – Disudrin and Inoflux
products – were not included in the search warrant, they were, therefore, not lawfully seized by
the raiding team; they are not illegal per se, as it were, like an arms cache, subversive materials
or shabu as to justify their seizure in the course of a lawful search, or being in plain view or
some such. No need whatever for some public assay.
The NBI manifestation is a glaring admission that it cannot tell without proper examination or
assay that the Disudrin and Inoflox samples allegedly seized from respondent’s place were
counterfeit. All the relevant presumptions are in favor of legality.52

The Court, therefore, finds no factual basis for the contention of the petitioner that the
respondents never raised in the court a quo the issue of whether the seizure of the Disudrin and
Inoflox products was valid.

In any event, the petitioner filed a motion for the reconsideration of the March 11, 2004 Order of
the court a quo on the following claims:

2.01 The Honorable Court ERRED in ruling on a non-issue or the issue as to the alleged
failure to particularly describe in the search warrant the items to be seized but upon
which NO challenge was then existing and/or NO controversy is raised;

2.02 The Honorable Court ERRED in its ruling that "finished or unfinished products of
UNILAB" cannot stand the test of a particular description for which it then reasons that
the search is, supposedly unreasonable; and,

2.03 The Honorable Court ERRED in finding that the evidence seized is lawfully
inadmissible against respondents.53

The court a quo considered the motion of the petitioner and the issue raised by it before finally
resolving to deny the same. It cannot thus be gainsaid that the petitioner was denied its right to
due process.

On the validity of the seizure of the sealed boxes and its contents of Disudrin and Inoflox, the
Court, likewise, rejects the contention of the petitioner.

A search warrant, to be valid, must particularly describe the place to be searched and the things
to be seized. The officers of the law are to seize only those things particularly described in the
search warrant. A search warrant is not a sweeping authority empowering a raiding party to
undertake a fishing expedition to seize and confiscate any and all kinds of evidence or articles
relating to a crime. The search is limited in scope so as not to be general or explanatory.
Nothing is left to the discretion of the officer executing the warrant.54

Objects, articles or papers not described in the warrant but on plain view of the executing officer
may be seized by him. However, the seizure by the officer of objects/articles/papers not
described in the warrant cannot be presumed as plain view. The State must adduce evidence,
testimonial or documentary, to prove the confluence of the essential requirements for the
doctrine to apply, namely: (a) the executing law enforcement officer has a prior justification for
an initial intrusion or otherwise properly in a position from which he can view a particular order;
(b) the officer must discover incriminating evidence inadvertently; and (c) it must be immediately
apparent to the police that the items they observe may be evidence of a crime, contraband, or
otherwise subject to seizure.55

The doctrine is not an exception to the warrant. It merely serves to supplement the prior
justification – whether it be a warrant for another object, hot pursuit, search as an incident to a
lawful arrest or some other legitimate reason for being present, unconnected with a search
directed against the accused. The doctrine may not be used to extend a general exploratory
search from one object to another until something incriminating at last emerges. It is a
recognition of the fact that when executing police officers comes across immediately
incriminating evidence not covered by the warrant, they should not be required to close their
eyes to it, regardless of whether it is evidence of the crime they are investigating or evidence of
some other crime. It would be needless to require the police to obtain another warrant.56 Under
the doctrine, there is no invasion of a legitimate expectation of privacy and there is no search
within the meaning of the Constitution.

The immediate requirement means that the executing officer can, at the time of discovery of the
object or the facts therein available to him, determine probable cause of the object’s
incriminating evidence.57 In other words, to be immediate, probable cause must be the direct
result of the officer’s instantaneous sensory perception of the object.58 The object is apparent if
the executing officer had probable cause to connect the object to criminal activity. The
incriminating nature of the evidence becomes apparent in the course of the search, without the
benefit of any unlawful search or seizure. It must be apparent at the moment of seizure.59

The requirement of inadvertence, on the other hand, means that the officer must not have
known in advance of the location of the evidence and intend to seize it.60 Discovery is not
anticipated.61

The immediately apparent test does not require an unduly high degree of certainty as to the
incriminating character of evidence. It requires merely that the seizure be presumptively
reasonable assuming that there is probable cause to associate the property with criminal
activity; that a nexus exists between a viewed object and criminal activity.62

Incriminating means the furnishing of evidence as proof of circumstances tending to prove the
guilt of a person.63

Indeed, probable cause is a flexible, common sense standard. It merely requires that the facts
available to the officer would warrant a man of reasonable caution and belief that certain items
may be contrabanded or stolen property or useful as evidence of a crime. It does not require
proof that such belief be correct or more likely than true. A practical, non-traditional probability
that incriminating evidence is involved is all that is required. The evidence thus collected must
be seen and verified as understood by those experienced in the field of law enforcement.64

In this case, Disudrin and/or Inoflox were not listed in the search warrant issued by the court a
quo as among the properties to be seized by the NBI agents. The warrant specifically
authorized the officers only to seize "counterfeit Revicon multivitamins, finished or unfinished,
and the documents used in recording, manufacture and/or importation, distribution and/or sale,
or the offering for sale, sale and/or distribution of the said vitamins." The implementing officers
failed to find any counterfeit Revicon multivitamins, and instead seized sealed boxes which,
when opened at the place where they were found, turned out to contain Inoflox and Disudrin.

It was thus incumbent on the NBI agents and the petitioner to prove their claim that the items
were seized based on the plain view doctrine. It is not enough to prove that the sealed boxes
were in the plain view of the NBI agents; evidence should have been adduced to prove the
existence of all the essential requirements for the application of the doctrine during the hearing
of the respondents’ motion to quash, or at the very least, during the hearing of the NBI and the
petitioner’s motion for reconsideration on April 16, 2004. The immediately apparent aspect, after
all, is central to the plain view exception relied upon by the petitioner and the NBI. There is no
showing that the NBI and the petitioner even attempted to adduce such evidence. In fact, the
petitioner and the NBI failed to present any of the NBI agents who executed the warrant, or any
of the petitioner’s representative who was present at the time of the enforcement of the warrant
to prove that the enforcing officers discovered the sealed boxes inadvertently, and that such
boxes and their contents were incriminating and immediately apparent. It must be stressed that
only the NBI agent/agents who enforced the warrant had personal knowledge whether the
sealed boxes and their contents thereof were incriminating and that they were immediately
apparent.65 There is even no showing that the NBI agents knew the contents of the sealed
boxes before they were opened.

In sum then, the Court finds and so hold that the petitioner and the NBI failed to prove the
essential requirements for the application of the plain view doctrine.

IN LIGHT OF ALL THE FOREGOING, the petition is DENIED for lack of merit. The assailed
orders of the Regional Trial Court are AFFIRMED.

SO ORDERED.

G.R. No. L-27360 February 28, 1968

HON. RICARDO G. PAPA, as Chief of Police of Manila; HON. JUAN PONCE ENRILE, as
Commissioner of Customs; PEDRO PACIS, as Collector of Customs of the Port of Manila;
and MARTIN ALAGAO, as Patrolman of the Manila Police Department, petitioners,
vs.
REMEDIOS MAGO and HILARION U. JARENCIO, as Presiding Judge of Branch 23, Court
of First Instance of Manila, respondents.

Office of the Solicitor General for petitioners.


Juan T. David for respondents.

ZALDIVAR, J.:

This is an original action for prohibition and certiorari, with preliminary injunction filed by
Ricardo Papa, Chief of Police of Manila; Juan once Enrile, Commissioner of Customs; Pedro
Pacis, Collector of Customs of the Port of Manila; and Martin Alagao, a patrolman of the Manila
Police Department, against Remedios Mago and Hon. Hilarion Jarencio, Presiding Judge of
Branch 23 of the Court of First Instance of Manila, praying for the annulment of the order issued
by respondent Judge in Civil Case No. 67496 of the Court of First Instance of Manila under date
of March 7, 1967, which authorized the release under bond of certain goods which were seized
and held by petitioners in connection with the enforcement of the Tariff and Customs Code, but
which were claimed by respondent Remedios Mago, and to prohibit respondent Judge from
further proceeding in any manner whatsoever in said Civil Case No. 67496. Pending the
determination of this case this Court issued a writ of preliminary injunction restraining the
respondent Judge from executing, enforcing and/or implementing the questioned order in Civil
Case No. 67496 and from proceeding with said case.

Petitioner Martin Alagao, head of the counter-intelligence unit of the Manila Police
Department, acting upon a reliable information received on November 3, 1966 to the effect that
a certain shipment of personal effects, allegedly misdeclared and undervalued, would be
released the following day from the customs zone of the port of Manila and loaded on two
trucks, and upon orders of petitioner Ricardo Papa, Chief of Police of Manila and a duly
deputized agent of the Bureau of Customs, conducted surveillance at gate No. 1 of the customs
zone. When the trucks left gate No. 1 at about 4:30 in the afternoon of November 4, 1966,
elements of the counter-intelligence unit went after the trucks and intercepted them at the
Agrifina Circle, Ermita, Manila. The load of the two trucks consisting of nine bales of goods, and
the two trucks, were seized on instructions of the Chief of Police. Upon investigation, a person
claimed ownership of the goods and showed to the policemen a "Statement and Receipts of
Duties Collected in Informal Entry No. 147-5501", issued by the Bureau of Customs in the name
of a certain Bienvenido Naguit.

Claiming to have been prejudiced by the seizure and detention of the two trucks and their
cargo, Remedios Mago and Valentin B. Lanopa filed with the Court of First Instance of Manila a
petition "for mandamus with restraining order or preliminary injunction, docketed as Civil Case
No. 67496, alleging, among others, that Remedios Mago was the owner of the goods seized,
having purchased them from the Sta. Monica Grocery in San Fernando, Pampanga; that she
hired the trucks owned by Valentin Lanopa to transport, the goods from said place to her
residence at 1657 Laon Laan St., Sampaloc, Manila; that the goods were seized by members of
the Manila Police Department without search warrant issued by a competent court; that anila
Chief of Police Ricardo Papa denied the request of counsel for Remedios Mago that the bales
be not opened and the goods contained therein be not examined; that then Customs
Commissioner Jacinto Gavino had illegally assigned appraisers to examine the goods because
the goods were no longer under the control and supervision of the Commissioner of Customs;
that the goods, even assuming them to have been misdeclared and, undervalued, were not
subject to seizure under Section 2531 of the Tariff and Customs Code because Remedios Mago
had bought them from another person without knowledge that they were imported illegally; that
the bales had not yet been opened, although Chief of Police Papa had arranged with the
Commissioner of Customs regarding the disposition of the goods, and that unless restrained
their constitutional rights would be violated and they would truly suffer irreparable injury. Hence,
Remedios Mago and Valentin Lanopa prayed for the issuance of a restraining order, ex parte,
enjoining the above-named police and customs authorities, or their agents, from opening the
bales and examining the goods, and a writ of mandamus for the return of the goods and the
trucks, as well as a judgment for actual, moral and exemplary damages in their favor.

On November 10, 1966, respondent Judge Hilarion Jarencio issued an order ex


parte restraining the respondents in Civil Case No. 67496 — now petitioners in the instant case
before this Court — from opening the nine bales in question, and at the same time set the
hearing of the petition for preliminary injunction on November 16, 1966. However, when the
restraining order was received by herein petitioners, some bales had already been opened by
the examiners of the Bureau of Customs in the presence of officials of the Manila Police
Department, an assistant city fiscal and a representative of herein respondent Remedios Mago.

Under date of November 15, 1966, Remedios Mago filed an amended petition in Civil
Case No. 67496, including as party defendants Collector of Customs Pedro Pacis of the Port of
Manila and Lt. Martin Alagao of the Manila Police Department. Herein petitioners (defendants
below) filed, on November 24, 1966, their "Answer with Opposition to the Issuance of a Writ of
Preliminary Injunction", denying the alleged illegality of the seizure and detention of the goods
and the trucks and of their other actuations, and alleging special and affirmative defenses, to
wit: that the Court of First Instance of Manila had no jurisdiction to try the case; that the case fell
within the exclusive jurisdiction of the Court of Tax Appeals; that, assuming that the court had
jurisdiction over the case, the petition stated no cause of action in view of the failure of
Remedios Mago to exhaust the administrative remedies provided for in the Tariff and Customs
Code; that the Bureau of Customs had not lost jurisdiction over the goods because the full
duties and charges thereon had not been paid; that the members of the Manila Police
Department had the power to make the seizure; that the seizure was not unreasonable; and the
persons deputized under Section 2203 (c) of the Tariff and Customs Code could effect search,
seizures and arrests in inland places in connection with the enforcement of the said Code. In
opposing the issuance of the writ of preliminary injunction, herein petitioners averred in the court
below that the writ could not be granted for the reason that Remedios Mago was not entitled to
the main reliefs she prayed for; that the release of the goods, which were subject to seizure
proceedings under the Tariff and Customs Code, would deprive the Bureau of Customs of the
authority to forfeit them; and that Remedios Mago and Valentin Lanopa would not suffer
irreparable injury. Herein petitioners prayed the court below for the lifting of the restraining
order, for the denial of the issuance of the writ of preliminary injunction, and for the dismissal of
the case.

At the hearing on December 9, 1966, the lower Court, with the conformity of the parties,
ordered that an inventory of the goods be made by its clerk of court in the presence of the
representatives of the claimant of the goods, the Bureau of Customs, and the Anti-Smuggling
Center of the Manila Police Department. On December 13, 1966, the above-named persons
filed a "Compliance" itemizing the contents of the nine bales.

Herein respondent Remedios Mago, on December 23, 1966, filed an ex parte motion to
release the goods, alleging that since the inventory of the goods seized did not show any article
of prohibited importation, the same should be released as per agreement of the patties upon her
posting of the appropriate bond that may be determined by the court. Herein petitioners filed
their opposition to the motion, alleging that the court had no jurisdiction to order the release of
the goods in view of the fact that the court had no jurisdiction over the case, and that most of the
goods, as shown in the inventory, were not declared and were, therefore, subject to forfeiture. A
supplemental opposition was filed by herein petitioners on January 19, 1967, alleging that on
January 12, 1967 seizure proceedings against the goods had been instituted by the Collector of
Customs of the Port of Manila, and the determination of all questions affecting the disposal of
property proceeded against in seizure and forfeiture proceedings should thereby be left to the
Collector of Customs. On January 30, 1967, herein petitioners filed a manifestation that the
estimated duties, taxes and other charges due on the goods amounted to P95,772.00. On
February 2, 1967, herein respondent Remedios Mago filed an urgent manifestation and
reiteration of the motion for the release under bond of the goods.

On March 7, 1967, the respondent Judge issued an order releasing the goods to herein
respondent Remedios Mago upon her filing of a bond in the amount of P40,000.00, and on
March 13, 1967, said respondent filed the corresponding bond.

On March 13, 1967, herein petitioner Ricardo Papa, on his own behalf, filed a motion for
reconsideration of the order of the court releasing the goods under bond, upon the ground that
the Manila Police Department had been directed by the Collector of Customs of the Port of
Manila to hold the goods pending termination of the seizure proceedings.

Without waiting for the court's action on the motion for reconsideration, and alleging that
they had no plain, speedy and adequate remedy in the ordinary course of law, herein petitioners
filed the present action for prohibition and certiorari with preliminary injunction before this Court.
In their petition petitioners alleged, among others, that the respondent Judge acted without
jurisdiction in ordering the release to respondent Remedios Mago of the disputed goods, for the
following reasons: (1) the Court of First Instance of Manila, presided by respondent Judge, had
no jurisdiction over the case; (2) respondent Remedios Mago had no cause of action in Civil
Case No. 67496 of the Court of First Instance of Manila due to her failure to exhaust all
administrative remedies before invoking judicial intervention; (3) the Government was not
estopped by the negligent and/or illegal acts of its agent in not collecting the correct taxes; and
(4) the bond fixed by respondent Judge for the release of the goods was grossly insufficient.

In due time, the respondents filed their answer to the petition for prohibition
and certiorari in this case. In their answer, respondents alleged, among others: (1) that it was
within the jurisdiction of the lower court presided by respondent Judge to hear and decide Civil
Case No. 67496 and to issue the questioned order of March 7, 1967, because said Civil Case
No. 67496 was instituted long before seizure, and identification proceedings against the nine
bales of goods in question were instituted by the Collector of Customs; (2) that petitioners could
no longer go after the goods in question after the corresponding duties and taxes had been paid
and said goods had left the customs premises and were no longer within the control of the
Bureau of Customs; (3) that respondent Remedios Mago was purchaser in good faith of the
goods in question so that those goods can not be the subject of seizure and forfeiture
proceedings; (4) that the seizure of the goods was affected by members of the Manila Police
Department at a place outside control of jurisdiction of the Bureau of Customs and affected
without any search warrant or a warrant of seizure and detention; (5) that the warrant of seizure
and detention subsequently issued by the Collector of Customs is illegal and unconstitutional, it
not being issued by a judge; (6) that the seizing officers have no authority to seize the goods in
question because they are not articles of prohibited importation; (7) that petitioners are
estopped to institute the present action because they had agreed before the respondent Judge
that they would not interpose any objection to the release of the goods under bond to answer for
whatever duties and taxes the said goods may still be liable; and (8) that the bond for the
release of the goods was sufficient.

The principal issue in the instant case is whether or not, the respondent Judge had acted
with jurisdiction in issuing the order of March 7, 1967 releasing the goods in question.

The Bureau of Customs has the duties, powers and jurisdiction, among others, (1) to
assess and collect all lawful revenues from imported articles, and all other dues, fees, charges,
fines and penalties, accruing under the tariff and customs laws; (2) to prevent and suppress
smuggling and other frauds upon the customs; and (3) to enforce tariff and customs laws. 1 The
goods in question were imported from Hongkong, as shown in the "Statement and Receipts of
Duties Collected on Informal Entry". 2 As long as the importation has not been terminated the
imported goods remain under the jurisdiction of the Bureau of customs. Importation is deemed
terminated only upon the payment of the duties, taxes and other charges upon the articles, or
secured to be paid, at the port of entry and the legal permit for withdrawal shall have been
granted. 3 The payment of the duties, taxes, fees and other charges must be in full. 4

The record shows, by comparing the articles and duties stated in the aforesaid
"Statement and Receipts of Duties Collected on Informal Entry" with the manifestation of the
Office of the Solicitor General 5 wherein it is stated that the estimated duties, taxes and other
charges on the goods subject of this case amounted to P95,772.00 as evidenced by the report
of the appraiser of the Bureau of Customs, that the duties, taxes and other charges had not
been paid in full. Furthermore, a comparison of the goods on which duties had been assessed,
as shown in the "Statement and Receipts of Duties Collected on Informal Entry" and the
"compliance" itemizing the articles found in the bales upon examination and inventory, 6 shows
that the quantity of the goods was underdeclared, presumably to avoid the payment of duties
thereon. For example, Annex B (the statement and receipts of duties collected) states that there
were 40 pieces of ladies' sweaters, whereas Annex H (the inventory contained in the
"compliance") states that in bale No. 1 alone there were 42 dozens and 1 piece of ladies'
sweaters of assorted colors; in Annex B, only 100 pieces of watch bands were assessed, but in
Annex H, there were in bale No. 2, 209 dozens and 5 pieces of men's metal watch bands
(white) and 120 dozens of men's metal watch band (gold color), and in bale No. 7, 320 dozens
of men's metal watch bands (gold color); in Annex B, 20 dozens only of men's handkerchief
were declared, but in Annex H it appears that there were 224 dozens of said goods in bale No.
2, 120 dozens in bale No. 6, 380 dozens in bale No. 7, 220 dozens in bale No. 8, and another
200 dozens in bale No. 9. The articles contained in the nine bales in question, were, therefore,
subject to forfeiture under Section 2530, pars. e and m, (1), (3), (4), and (5) of the Tariff and
Customs Code. And this Court has held that merchandise, the importation of which is effected
contrary to law, is subject to forfeiture, 7 and that goods released contrary to law are subject to
seizure and forfeiture. 8

Even if it be granted, arguendo, that after the goods in question had been brought out of
the customs area the Bureau of Customs had lost jurisdiction over the same, nevertheless,
when said goods were intercepted at the Agrifina Circle on November 4, 1966 by members of
the Manila Police Department, acting under directions and orders of their Chief, Ricardo C.
Papa, who had been formally deputized by the Commissioner of Customs, 9 the Bureau of
Customs had regained jurisdiction and custody of the goods. Section 1206 of the Tariff and
Customs Code imposes upon the Collector of Customs the duty to hold possession of all
imported articles upon which duties, taxes, and other charges have not been paid or secured to
be paid, and to dispose of the same according to law. The goods in question, therefore, were
under the custody and at the disposal of the Bureau of Customs at the time the petition
for mandamus, docketed as Civil Case No. 67496, was filed in the Court of First Instance of
Manila on November 9, 1966. The Court of First Instance of Manila, therefore, could not
exercise jurisdiction over said goods even if the warrant of seizure and detention of the goods
for the purposes of the seizure and forfeiture proceedings had not yet been issued by the
Collector of Customs.

The ruling in the case of "Alberto de Joya, et al. v. Hon. Gregorio Lantin, et al.," G.R. No.
L-24037, decided by this Court on April 27, 1967, is squarely applicable to the instant case. In
the De Joya case, it appears that Francindy Commercial of Manila bought from Ernerose
Commercial of Cebu City 90 bales of assorted textiles and rags, valued at P117,731.00, which
had been imported and entered thru the port of Cebu. Ernerose Commercial shipped the goods
to Manila on board an inter-island vessel. When the goods where about to leave the customs
premises in Manila, on October 6, 1964, the customs authorities held them for further
verification, and upon examination the goods were found to be different from the declaration in
the cargo manifest of the carrying vessel. Francindy Commercial subsequently demanded from
the customs authorities the release of the goods, asserting that it is a purchaser in good faith of
those goods; that a local purchaser was involved so the Bureau of Customs had no right to
examine the goods; and that the goods came from a coastwise port. On October 26, 1964,
Francindy Commercial filed in the Court of First Instance of Manila a petition
for mandamus against the Commissioner of Customs and the Collector of Customs of the port
of Manila to compel said customs authorities to release the goods.
Francindy Commercial alleged in its petition for mandamus that the Bureau of Customs
had no jurisdiction over the goods because the same were not imported to the port of Manila;
that it was not liable for duties and taxes because the transaction was not an original
importation; that the goods were not in the hands of the importer nor subject to importer's
control, nor were the goods imported contrary to law with its (Francindy Commercial's)
knowledge; and that the importation had been terminated. On November 12, 1964, the Collector
of Customs of Manila issued a warrant of seizure and identification against the goods. On
December 3, 1964, the Commissioner of Customs and the Collector of Customs, as
respondents in the mandamus case, filed a motion to dismiss the petition on the grounds of lack
of jurisdiction, lack of cause of action, and in view of the pending seizure and forfeiture
proceedings. The Court of First Instance held resolution on the motion to dismiss in abeyance
pending decision on the merits. On December 14, 1964, the Court of First Instance of Manila
issued a preventive and mandatory injunction, on prayer by Francindy Commercial, upon a
bond of P20,000.00. The Commissioner of Customs and the Collector of Customs sought the
lifting of the preliminary and mandatory injunction, and the resolution of their motion to dismiss.
The Court of First Instance of Manila, however, on January 12, 1965, ordered them to comply
with the preliminary and mandatory injunction, upon the filing by Francindy Commercial of an
additional bond of P50,000.00. Said customs authorities thereupon filed with this Court, on
January 14, 1965, a petition for certiorari and prohibition with preliminary injunction. In resolving
the question raised in that case, this Court held:

This petition raises two related issues: first, has the Customs bureau jurisdiction to
seize the goods and institute forfeiture proceedings against them? and (2) has the Court
of First Instance jurisdiction to entertain the petition for mandamus to compel the
Customs authorities to release the goods?

Francindy Commercial contends that since the petition in the Court of first
Instance was filed (on October 26, 1964) ahead of the issuance of the Customs warrant
of seizure and forfeiture (on November 12, 1964),the Customs bureau should yield the
jurisdiction of the said court.

The record shows, however, that the goods in question were actually seized on
October 6, 1964, i.e., before Francindy Commercial sued in court. The purpose of the
seizure by the Customs bureau was to verify whether or not Custom duties and taxes
were paid for their importation. Hence, on December 23, 1964, Customs released 22
bales thereof, for the same were found to have been released regularly from the Cebu
Port (Petition Annex "L"). As to goods imported illegally or released irregularly from
Customs custody, these are subject to seizure under Section 2530 m. of the Tariff and
Customs Code (RA 1957).

The Bureau of Customs has jurisdiction and power, among others to collect
revenues from imported articles, fines and penalties and suppress smuggling and other
frauds on customs; and to enforce tariff and customs laws (Sec. 602, Republic Act
1957).

The goods in question are imported articles entered at the Port of Cebu. Should
they be found to have been released irregularly from Customs custody in Cebu City,
they are subject to seizure and forfeiture, the proceedings for which comes within the
jurisdiction of the Bureau of Customs pursuant to Republic Act 1937.
Said proceeding should be followed; the owner of the goods may set up defenses
therein (Pacis v. Averia, L-22526, Nov. 20, 1966.) From the decision of the
Commissioner of Customs appeal lies to the Court of Tax Appeals, as provided in Sec.
2402 of Republic Act 1937 and Sec. 11 of Republic Act, 1125. To permit recourse to the
Court of First Instance in cases of seizure of imported goods would in effect render
ineffective the power of the Customs authorities under the Tariff and Customs Code and
deprive the Court of Tax Appeals of one of its exclusive appellate jurisdictions. As this
Court has ruled in Pacis v. Averia, supra, Republic Acts 1937 and 1125 vest jurisdiction
over seizure and forfeiture proceedings exclusively upon the Bureau of Customs and the
Court of Tax Appeals. Such law being special in nature, while the Judiciary Act defining
the jurisdiction of Courts of First Instance is a general legislation, not to mention that the
former are later enactments, the Court of First Instance should yield to the jurisdiction of
the Customs authorities.

It is the settled rule, therefore, that the Bureau of Customs acquires exclusive jurisdiction
over imported goods, for the purposes of enforcement of the customs laws, from the moment
the goods are actually in its possession or control, even if no warrant of seizure or detention had
previously been issued by the Collector of Customs in connection with seizure and forfeiture
proceedings. In the present case, the Bureau of Customs actually seized the goods in question
on November 4, 1966, and so from that date the Bureau of Customs acquired jurisdiction over
the goods for the purposes of the enforcement of the tariff and customs laws, to the exclusion of
the regular courts. Much less then would the Court of First Instance of Manila have jurisdiction
over the goods in question after the Collector of Customs had issued the warrant of seizure and
detention on January 12, 1967. 10 And so, it cannot be said, as respondents contend, that the
issuance of said warrant was only an attempt to divest the respondent Judge of jurisdiction over
the subject matter of the case. The court presided by respondent Judge did not acquire
jurisdiction over the goods in question when the petition for mandamus was filed before it, and
so there was no need of divesting it of jurisdiction. Not having acquired jurisdiction over the
goods, it follows that the Court of First Instance of Manila had no jurisdiction to issue the
questioned order of March 7, 1967 releasing said goods.

Respondents also aver that petitioner Martin Alagao, an officer of the Manila Police
Department, could not seize the goods in question without a search warrant. This contention
cannot be sustained. The Chief of the Manila Police Department, Ricardo G. Papa, having been
deputized in writing by the Commissioner of Customs, could, for the purposes of the
enforcement of the customs and tariff laws, effect searches, seizures, and arrests, 11 and it was
his duty to make seizure, among others, of any cargo, articles or other movable property when
the same may be subject to forfeiture or liable for any fine imposed under customs and tariff
laws. 12 He could lawfully open and examine any box, trunk, envelope or other container
wherever found when he had reasonable cause to suspect the presence therein of dutiable
articles introduced into the Philippines contrary to law; and likewise to stop, search and examine
any vehicle, beast or person reasonably suspected of holding or conveying such article as
aforesaid. 13 It cannot be doubted, therefore, that petitioner Ricardo G. Papa, Chief of Police of
Manila, could lawfully effect the search and seizure of the goods in question. The Tariff and
Customs Code authorizes him to demand assistance of any police officer to effect said search
and seizure, and the latter has the legal duty to render said assistance. 14 This was what
happened precisely in the case of Lt. Martin Alagao who, with his unit, made the search and
seizure of the two trucks loaded with the nine bales of goods in question at the Agrifina Circle.
He was given authority by the Chief of Police to make the interception of the cargo. 15
Petitioner Martin Alagao and his companion policemen had authority to effect the seizure
without any search warrant issued by a competent court. The Tariff and Customs Code does not
require said warrant in the instant case. The Code authorizes persons having police authority
under Section 2203 of the Tariff and Customs Code to enter, pass through or search any land,
inclosure, warehouse, store or building, not being a dwelling house; and also to inspect, search
and examine any vessel or aircraft and any trunk, package, or envelope or any person on
board, or to stop and search and examine any vehicle, beast or person suspected of holding or
conveying any dutiable or prohibited article introduced into the Philippines contrary to law,
without mentioning the need of a search warrant in said cases. 16 But in the search of a dwelling
house, the Code provides that said "dwelling house may be entered and searched only upon
warrant issued by a judge or justice of the peace. . . ." 17 It is our considered view, therefor, that
except in the case of the search of a dwelling house, persons exercising police authority under
the customs law may effect search and seizure without a search warrant in the enforcement of
customs laws.

Our conclusion finds support in the case of Carroll v. United States, 39 A.L.R., 790, 799,
wherein the court, considering a legal provision similar to Section 2211 of the Philippine Tariff
and Customs Code, said as follows:

Thus contemporaneously with the adoption of the 4th Amendment, we find in the
first Congress, and in the following second and fourth Congresses, a difference made as
to the necessity for a search warrant between goods subject to forfeiture, when
concealed in a dwelling house of similar place, and like goods in course of transportation
and concealed in a movable vessel, where readily they could be put out of reach of a
search warrant. . . .

Again, by the 2d section of the Act of March 3, 1815 (3 Stat. at L.231, 232, chap.
94), it was made lawful for customs officers not only to board and search vessels within
their own and adjoining districts, but also to stop, search and examine any vehicle, beast
or person on which or whom they should suspect there was merchandise which was
subject to duty, or had been introduced into the United States in any manner contrary to
law, whether by the person in charge of the vehicle or beast or otherwise, and if they
should find any goods, wares, or merchandise thereon, which they had probably cause
to believe had been so unlawfully brought into the country, to seize and secure the
same, and the vehicle or beast as well, for trial and forfeiture. This Act was renewed
April 27, 1816 (3 Sta. at L. 315, chap. 100), for a year and expired. The Act of February
28, 1865, revived § 2 of the Act of 1815, above described, chap. 67, 13 Stat. at L. 441.
The substance of this section was re-enacted in the 3d section of the Act of July 18,
1866, chap. 201, 14 Stat. at L. 178, and was thereafter embodied in the Revised
Statutes as § 3061, Comp. Stat. § 5763, 2 Fed. Stat. Anno. 2d ed. p. 1161. Neither §
3061 nor any of its earlier counterparts has ever been attacked as unconstitutional.
Indeed, that section was referred to and treated as operative by this court in Von
Cotzhausen v. Nazro, 107 U.S. 215, 219, 27 L. ed. 540, 541, 2 Sup. Ct. Rep. 503. . . .

In the instant case, we note that petitioner Martin Alagao and his companion policemen
did not have to make any search before they seized the two trucks and their cargo. In their
original petition, and amended petition, in the court below Remedios Mago and Valentin Lanopa
did not even allege that there was a search. 18 All that they complained of was,
That while the trucks were on their way, they were intercepted without any search
warrant near the Agrifina Circle and taken to the Manila Police Department, where they
were detained.

But even if there was a search, there is still authority to the effect that no search warrant
would be needed under the circumstances obtaining in the instant case. Thus, it has been held
that:

The guaranty of freedom from unreasonable searches and seizures is construed


as recognizing a necessary difference between a search of a dwelling house or other
structure in respect of which a search warrant may readily be obtained and a search of a
ship, motorboat, wagon, or automobile for contraband goods, where it is not practicable
to secure a warrant because the vehicle can be quickly moved out of the locality or
jurisdiction in which the warrant must be sought. (47 Am. Jur., pp. 513-514, citing Carroll
v. United States, 267 U.S. 132, 69 L. ed., 543, 45 S. Ct., 280, 39 A.L.R., 790; People v.
Case, 320 Mich., 379, 190 N.W., 389, 27 A.L.R., 686.)

In the case of People v. Case (320 Mich., 379, 190 N.W., 389, 27 A.L.R., 686), the
question raised by defendant's counsel was whether an automobile truck or an automobile
could be searched without search warrant or other process and the goods therein seized used
afterwards as evidence in a trial for violation of the prohibition laws of the State. Same counsel
contended the negative, urging the constitutional provision forbidding unreasonable searches
and seizures. The Court said:

. . . Neither our state nor the Federal Constitution directly prohibits search and
seizure without a warrant, as is sometimes asserted. Only "unreasonable" search and
seizure is forbidden. . . .

. . . The question whether a seizure or a search is unreasonable in the language of


the Constitution is a judicial and not a legislative question; but in determining whether a
seizure is or is not unreasonable, all of the circumstances under which it is made must
be looked to.

The automobile is a swift and powerful vehicle of recent development, which has
multiplied by quantity production and taken possession of our highways in battalions until
the slower, animal-drawn vehicles, with their easily noted individuality, are rare.
Constructed as covered vehicles to standard form in immense quantities, and with a
capacity for speed rivaling express trains, they furnish for successful commission of
crime a disguising means of silent approach and swift escape unknown in the history of
the world before their advent. The question of their police control and reasonable search
on highways or other public places is a serious question far deeper and broader than
their use in so-called "bootleging" or "rum running," which is itself is no small matter.
While a possession in the sense of private ownership, they are but a vehicle constructed
for travel and transportation on highways. Their active use is not in homes or on private
premises, the privacy of which the law especially guards from search and seizure
without process. The baffling extent to which they are successfully utilized to facilitate
commission of crime of all degrees, from those against morality, chastity, and decency,
to robbery, rape, burglary, and murder, is a matter of common knowledge. Upon that
problem a condition, and not a theory, confronts proper administration of our criminal
laws. Whether search of and seizure from an automobile upon a highway or other public
place without a search warrant is unreasonable is in its final analysis to be determined
as a judicial question in view of all the circumstances under which it is made.

Having declared that the seizure by the members of the Manila Police Department of the
goods in question was in accordance with law and by that seizure the Bureau of Customs had
acquired jurisdiction over the goods for the purpose of the enforcement of the customs and tariff
laws, to the exclusion of the Court of First Instance of Manila, We have thus resolved the
principal and decisive issue in the present case. We do not consider it necessary, for the
purposes of this decision, to discuss the incidental issues raised by the parties in their
pleadings.

WHEREFORE, judgment is hereby rendered, as follows:

(a) Granting the writ of certiorari and prohibition prayed for by petitioners;

(b) Declaring null and void, for having been issued without jurisdiction, the order of
respondent Judge Hilarion U. Jarencio, dated March 7, 1967, in Civil Code No. 67496 of the
Court of First Instance of Manila;

(c) Declaring permanent the preliminary injunction issued by this Court on March 31, 1967
restraining respondent Judge from executing, enforcing and/or implementing his order of March
7, 1967 in Civil Case No. 67496 of the Court of First Instance of Manila, and from proceeding in
any manner in said case;

(d) Ordering the dismissal of Civil Case No. 67496 of the Court of First Instance of Manila;
and1äwphï1.ñët

(e) Ordering the private respondent, Remedios Mago, to pay the costs.

It is so ordered.

G.R. No. 96177 January 27, 1993

PEOPLE OF THE PHILIPPINES, plaintiff-appellee,


vs.
MARI MUSA y HANTATALU, accused-appellant.

The Solicitor General for plaintiff-appellee.

Pablo L. Murillo for accused-appellant.

ROMERO, J.:

The appellant, Mari Musa, seeks, in this appeal, the reversal of the decision, dated August 31,
1990,1 of the Regional Trial Court (RTC) of Zamboanga City, Branch XII, finding him guilty of
selling marijuana in violation of Article II, Section 4 of Republic Act No. 6425, as amended,
otherwise known as the Dangerous Drugs Act of 1972.
The information filed on December 15, 1989 against the appellant reads:

That on or about December 14, 1989, in the City of Zamboanga, Philippines, and
within the jurisdiction of this Honorable Court, the
above-named accused, not being authorized by law, did then and there, wilfully,
unlawfully and feloniously sell to one SGT. AMADO ANI, two (2) wrappers
containing dried marijuana leaves, knowing the same to be a prohibited drug.

CONTRARY TO LAW.2

Upon his arraignment on January 11, 1990, the appellant pleaded not guilty.3

At the trial, the prosecution presented three (3) witnesses, namely: (1) Sgt. Amado Ani, Jr. of
the 9th Narcotics Command (NARCOM) of Zamboanga City, who acted as poseur-buyer in the
buy-bust operation made against the appellant; (2) T/Sgt. Jesus Belarga, also of the 9th
Narcotics Command of Zamboanga City, who was the NARCOM team leader of the buy-bust
operation; and (3) Athena Elisa P. Anderson, the Document Examiner and Forensic Chemist of
PC-INP Crime Laboratory of Regional Command (RECOM) 9. The evidence of the prosecution
was summarized by the trial court as follows:

Prosecution evidence shows that in the morning of December 13, 1989, T/Sgt.
Jesus Belarga, leader of a NARCOTICS COMMAND (NARCOM) team based at
Calarian, Zamboanga City, instructed Sgt. Amado Ani to conduct surveillance
and test buy on a certain Mari Musa of Suterville, Zamboanga City. Information
received from civilian informer was that this Mari Musa was engaged in selling
marijuana in said place. So Sgt. Amado Ani, another NARCOM agent, proceeded
to Suterville, in company with a NARCOM civilian informer, to the house of Mari
Musa to which house the civilian informer had guided him. The same civilian
informer had also described to him the appearance of Mari Musa. Amado Ani
was able to buy one newspaper-wrapped dried marijuana (Exh. "E") for P10.00.
Sgt. Ani returned to the NARCOM office and turned over the newspaper-wrapped
marijuana to T/Sgt. Jesus Belarga. Sgt. Belarga inspected the stuff turned over to
him and found it to be marijuana.

The next day, December 14, 1989, about 1:30 P.M., a buy-bust was planned.
Sgt. Amado Ani was assigned as the poseur buyer for which purpose he was
given P20.00 (with SN GA955883) by Belarga. The
buy-bust money had been taken by T/Sgt. Jesus Belarga from M/Sgt. Noh Sali
Mihasun, Chief of Investigation Section, and for which Belarga signed a receipt
(Exh. "L" & "L-l" ) The team under Sgt. Foncargas was assigned as back-up
security. A pre-arranged signal was arranged consisting of Sgt. Ani's raising his
right hand, after he had succeeded to buy the marijuana. The two NARCOM
teams proceeded to the target site in two civilian vehicles. Belarga's team was
composed of Sgt. Belarga, team leader, Sgt. Amado Ani, poseur buyer, Sgt.
Lego and Sgt. Biong.

Arriving at the target site, Sgt. Ani proceeded to the house of Mari Musa, while
the rest of the NARCOM group positioned themselves at strategic places about
90 to 100 meters from Mari Musa's house. T/Sgt. Belarga could see what went
on between Ani and suspect Mari Musa from where he was. Ani approached
Mari Musa, who came out of his house, and asked Ani what he wanted. Ani said
he wanted some more stuff. Ani gave Mari Musa the P20.00 marked money.
After receiving the money, Mari Musa went back to his house and came back
and gave Amado Ani two newspaper wrappers containing dried marijuana. Ani
opened the two wrappers and inspected the contents. Convinced that the
contents were marijuana, Ani walked back towards his companions and raised
his right hand. The two NARCOM teams, riding the two civilian vehicles, sped
towards Sgt. Ani. Ani joined Belarga's team and returned to the house.

At the time Sgt. Ani first approached Mari Musa, there were four persons inside
his house: Mari Musa, another boy, and two women, one of whom Ani and
Belarga later came to know to be Mari Musa's wife. The second time, Ani with the
NARCOM team returned to Mari Musa's house, the woman, who was later known
as Mari Musa's wife, slipped away from the house. Sgt. Belarga frisked Mari
Musa but could not find the P20.00 marked money with him. Mari Musa was then
asked where the P20.00 was and he told the NARCOM team he has given the
money to his wife (who had slipped away). Sgt. Belarga also found a plastic bag
containing dried marijuana inside it somewhere in the kitchen. Mari Musa was
then placed under arrest and brought to the NARCOM office. At Suterville, Sgt.
Ani turned over to Sgt. Belarga the two newspaper-wrapped marijuana he had
earlier bought from Mari Musa (Exhs. "C" & "D").

In the NARCOM office, Mari Musa first gave his name as Hussin Musa. Later on,
Mari Musa gave his true name — Mari Musa. T/Sgt. Jesus Belarga turned over
the two newspaper-wrapped marijuana (bought at the buy-bust), the one
newspaper-wrapped marijuana (bought at the test-buy) and the plastic bag
containing more marijuana (which had been taken by Sgt. Lego inside the
kitchen of Mari Musa) to the PC Crime Laboratory, Zamboanga City, for
laboratory examination. The turnover of the marijuana specimen to the PC Crime
Laboratory was by way of a letter-request, dated December 14, 1989 (Exh. "B"),
which was stamped "RECEIVED" by the PC Crime Laboratory (Exh. "B-1") on
the same day.

Mrs. Athena Elisa P. Anderson, the Forensic Chemist of the PC Crime


Laboratory, examined the marijuana specimens subjecting the same to her three
tests. All submitted specimens she examined gave positive results for the
presence of marijuana. Mrs. Anderson reported the results of her examination in
her Chemistry Report D-100-89, dated December 14, 1989, (Exh. "J", "J-1", "J-
2", "J-3", "J-4" and "J-5"). Mrs. Anderson identified in court the two newspaper
wrapped marijuana bought at the
buy-bust on December 14, 1989, through her initial and the weight of each
specimen written with red ink on each wrapper (Exhs. "C-1" and "D-1"). She also
identified the one newspaper-wrapped marijuana bought at the test-buy on
December 13, 1989, through her markings (Exh. "E-1"). Mrs. Anderson also
identified her Chemistry Report (Exh. "J" & sub-markings.)

T. Sgt. Belarga identified the two buy-bust newspaper wrapped marijuana


through his initial, the words "buy-bust" and the words "December 14, 1989, 2:45
P.M." (written on Exhs. "C" and "D"). Belarga also identified the receipt of the
P20 marked money (with SN GA955883) (Exh. "L"), dated December 14, 1989,
and his signature thereon (Exh.
"L-1"). He also identified the letter-request, dated December 14, 1989, addressed
to the PC Crime Laboratory (Exh. "B") and his signature thereon (Exh. "B-2") and
the stamp of the PC Crime Laboratory marked "RECEIVED" (Exh. "B-1").4

For the defense, the following testified as witnesses: (1) the accused-appellant Mari H. Musa;
and (2) Ahara R. Musa, his wife. The trial court summarized the version of the defense, thus:

[O]n December 14, 1989, at about 1:30 in the afternoon, Mari Musa was in his
house at Suterville, Zamboanga City. With him were his wife, Ahara Musa,
known as Ara, his one-year old child, a woman manicurist, and a male cousin
named Abdul Musa. About 1:30 that afternoon, while he was being manicured at
one hand, his wife was inside the one room of their house, putting their child to
sleep. Three NARCOM agents, who introduced themselves as NARCOM agents,
dressed in civilian clothes, got inside Mari Musa's house whose door was open.
The NARCOM agents did not ask permission to enter the house but simply
announced that they were NARCOM agents. The NARCOM agents searched
Mari Musa's house and Mari Musa asked them if they had a search warrant. The
NARCOM agents were just silent. The NARCOM agents found a red plastic bag
whose contents, Mari Musa said, he did not know. He also did not know if the
plastic bag belonged to his brother, Faisal, who was living with him, or his father,
who was living in another house about ten arms-length away. Mari Musa, then,
was handcuffed and when Mari Musa asked why, the NARCOM agents told him
for clarification.

Mari Musa was brought in a pick-up, his wife joining him to the NARCOM Office
at Calarian, Zamboanga City. Inside the NARCOM Office, Mari Musa was
investigated by one NARCOM agent which investigation was reduced into
writing. The writing or document was interpreted to Mari Musa in Tagalog. The
document stated that the marijuana belonged to Mari Musa and Mari Musa was
asked to sign it. But Mari Musa refused to sign because the marijuana did not
belong to him. Mari Musa said he was not told that he was entitled to the
assistance of counsel, although he himself told the NARCOM agents he wanted
to be assisted by counsel.

Mari Musa said four bullets were then placed between the fingers of his right
hand and his fingers were pressed which felt very painful. The NARCOM agents
boxed him and Mari Musa lost consciousness. While Mari Musa was maltreated,
he said his wife was outside the NARCOM building. The very day he was
arrested (on cross-examination Mari Musa said it was on the next day), Mari
Musa was brought to the Fiscal's Office by three NARCOM agents. The fiscal
asked him if the marijuana was owned by him and he said "not." After that single
question, Mari Musa was brought to the City Jail. Mari Musa said he did not tell
the fiscal that he had been maltreated by the NARCOM agents because he was
afraid he might be maltreated in the fiscal's office.

Mari Musa denied the NARCOM agents' charge that he had sold two wrappers of
marijuana to them; that he had received from them a P20.00 bill which he had
given to his wife. He did not sell marijuana because he was afraid that was
against the law and that the person selling marijuana was caught by the
authorities; and he had a wife and a very small child to support. Mari Musa said
he had not been arrested for selling marijuana before.5

After trial, the trial court rendered the assailed decision with the following disposition:

WHEREFORE, finding accused Mari Musa y Hantatalu guilty beyond reasonable


doubt of selling marijuana and pursuant to Sec. 4, Art II of Rep. Act No. 6425, he
is sentenced to life imprisonment and to pay the fine of P20,000.00, the latter
imposed without subsidiary imprisonment.6

In this appeal, the appellant contends that his guilt was not proved beyond reasonable doubt
and impugns the credibility of the prosecution witnesses.

The appellant claims that the testimony of Sgt. Ani, the poseur-buyer, is not credible because:
(1) prior to the buy-bust operation, neither Sgt. Ani nor the other NARCOM agents were
personally known by the appellant or vice-versa; and (2) there was no witness to the alleged
giving of the two wrappers of marijuana by the appellant to Sgt. Ani.

Sgt. Ani testified that on December 13, 1989, upon instruction by T/Sgt. Jesus Belarga, he
conducted a test-buy operation on the appellant whereby he bought one wrapper of marijuana
for P15.00 from the latter.7 He reported the successful operation to T/Sgt. Belarga on the same
day.8 Whereupon, T/Sgt. Belarga conducted a conference to organize a buy-bust operation for
the following day.9

On December 14, 1989, at 1:30 p.m., two NARCOM teams in separate vehicles headed by
T/Sgt. Belarga and a certain Sgt. Foncardas went to the place of operation, which was the
appellant's house located in Laquian Compound, Suterville, Zamboanga City. Sgt. Ani was with
the team of T/Sgt. Belarga, whose other members were Sgts. Lego and Biong. 10 Sgt. Ani was
given a marked P20.00 bill by T/Sgt. Belarga, which was to be used in the operation.

Upon reaching the place, the NARCOM agents positioned themselves at strategic places.11 Sgt.
Ani approached the house. Outside the house, the appellant asked Sgt. Ani what he wanted.
Sgt. Ani asked him for some more marijuana.12 Sgt. Ani gave him the marked P20.00 bill and
the appellant went inside the house and brought back two paper wrappers containing marijuana
which he handed to Sgt. Ani.13 From his position, Sgt. Ani could see that there were other
people in the house.14

After the exchange, Sgt. Ani approached the other NARCOM agents and made the pre-
arranged signal of raising his right hand.15 The NARCOM agents, accompanied by Sgt. Ani,
went inside the house and made the arrest. The agents searched the appellant and unable to
find the marked money, they asked him where it was. The appellant said that he gave it to his
wife.16

The Court, after a careful reading of the record, finds the testimony of Sgt. Ani regarding the
buy-bust operation, which resulted in the apprehension, prosecution and subsequent conviction
of the appellant, to be direct, lucid and forthright. Being totally untainted by contradictions in any
of the material points, it deserves credence.

The contention that the appellant could not have transacted with Sgt. Ani because they do not
know each other is without merit. The day before the
buy-bust operation, Sgt. Ani conducted a test-buy and he successfully bought a wrapper of
marijuana from the appellant. Through this previous transaction, Sgt. Ani was able to gain the
appellant's confidence for the latter to sell more marijuana to Sgt. Ani the following day, during
the buy-bust operation. Moreover, the Court has held that what matters is not an existing
familiarity between the buyer and the seller, for quite often, the parties to the transaction may be
strangers, but their agreement and the acts constituting the sale and delivery of the marijuana.17

The appellant, again to cast doubt on the credibility of Sgt. Ani, argues that it was impossible for
the appellant to sell marijuana while his wife, cousin and manicurist were present. But the place
of the commission of the crime of selling prohibited drugs has been held to be not crucial18 and
the presence of other people apart from the buyer and seller will not necessarily prevent the
consummation of the illegal sale. As the Court observed in People v. Paco,19 these factors may
sometimes camouflage the commission of the crime. In the instant case, the fact that the other
people inside the appellant's house are known to the appellant may have given him some
assurance that these people will not report him to the authorities.

The appellant, besides assailing Sgt. Ani's credibility, also questions the credibility of T/Sgt.
Belarga. The appellant submits that since T/Sgt. Belarga admitted that he was about 90 meters
away from Sgt. Ani and the appellant, he could not have possibly witnessed the sale. The
appellant invokes People v.
Ale20 where the Court observed that from a distance of 10-15 meters, a policeman cannot
distinguish between marijuana cigarette from ordinary ones by the type of rolling done on the
cigarette sticks. And since T/Sgt. Belarga allegedly did not see the sale, the appellant contends
that the uncorroborated testimony of Sgt. Ani can not stand as basis for his conviction.

People v. Ale does not apply here because the policeman in that case testified that he and his
companion were certain that the appellant therein handed marijuana cigarettes to the poseur-
buyer based on the appearance of the cigarette sticks. The Court rejected this claim, stating
that:

This Court cannot give full credit to the testimonies of the prosecution witnesses
marked as they are with contradictions and tainted with inaccuracies.

Biñan testified that they were able to tell that the four cigarettes were marijuana
cigarettes because according to him, the rolling of ordinary cigarettes are
different from those of marijuana cigarettes. (tsn, November 13, 1984, p. 10).

It is however, incredible to believe that they could discern the type of rolling done
on those cigarettes from the distance where they were observing the alleged sale
of more or less 10 to 15 meters.21

In the case at bar, however, T/Sgt. Belarga did not positively claim that he saw the appellant
hand over marijuana to Sgt. Ani. What he said was that there was an exchange of certain
articles between the two. The relevant portion of T/Sgt. Belarga's testimony reads:22

Q Now, do you remember whether Sgt. Ani was able to reach the
house of Mari Musa?

A Yes, ma'am.
Q After reaching Mari Musa, did you see what happened (sic)?

A Yes, ma'am.

Q Could you please tell us?

A From our vehicle the stainless owner type jeep where Sgt. Lego,
Sgt. Biong were boarded, I saw that Sgt. Ani proceeded to the
house near the road and he was met by one person and later
known as Mari Musa who was at the time wearing short pants and
later on I saw that Sgt. Ani handed something to him, thereafter
received by Mari Musa and went inside the house and came back
later and handed something to Sgt. Ani.

Contrary to the contention of the appellant, it was not impossible for T/Sgt. Belarga to have
seen, from a distance of 90-100 meters, Sgt. Ani hand to the appellant "something" and for the
latter to give to the former "something."

Notwithstanding the fact that T/Sgt. Belarga could not have been certain that what Sgt. Ani
received from the appellant was marijuana because of the distance, his testimony, nevertheless,
corroborated the direct evidence, which the Court earlier ruled to be convincing, presented by
Sgt. Ani on the following material points: (1) T/Sgt. Belarga instructed Sgt. Ani to conduct a
surveillance and test-buy operation on the appellant at Suterville, Zamboanga City on
December 13, 1989; 23 (2) later that same day, Sgt. Ani went back to their office and reported a
successful operation and turned over to T/Sgt. Belarga one wrapper of marijuana; 24 (3) T/Sgt.
Belarga then organized a team to conduct a buy-bust operation the following day; 25 (4) on
December 14, 1989, T/Sgt. Belarga led a team of NARCOM agents who went to Suterville,
Zamboanga City;26 (5) T/Sgt. Belarga gave a P20.00 marked bill to Sgt. Ani which was to be
used in the buy-bust operation; 27 (6) upon the arrival of the NARCOM agents in Suterville,
Zamboanga City, Sgt. Ani proceeded to the house of the appellant while some agents stayed in
the vehicles and others positioned themselves in strategic places;28 the appellant met Sgt. Ani
and an exchange of articles took place.29

The corroborative testimony of T/Sgt. Belarga strengthens the direct evidence given by Sgt. Ani.
Additionally, the Court has ruled that the fact that the police officers who accompanied the
poseur-buyer were unable to see exactly what the appellant gave the poseur-buyer because of
their distance or position will not be fatal to the prosecution's case30 provided there exists other
evidence, direct or circumstantial, e.g., the testimony of the poseur-buyer, which is sufficient to
prove the consummation of the sale of the prohibited drug

The appellant next assails the seizure and admission as evidence of a plastic bag containing
marijuana which the NARCOM agents found in the appellant's kitchen. It appears that after Sgt.
Ani gave the pre-arranged signal to the other NARCOM agents, the latter moved in and arrested
the appellant inside the house. They searched him to retrieve the marked money but didn't find
it. Upon being questioned, the appellant said that he gave the marked money to his
wife.31 Thereafter, T/Sgt. Belarga and Sgt. Lego went to the kitchen and noticed what T/Sgt.
Belarga described as a "cellophane colored white and stripe hanging at the corner of the
kitchen."32 They asked the appellant about its contents but failing to get a response, they
opened it and found dried marijuana leaves. At the trial, the appellant questioned the
admissibility of the plastic bag and the marijuana it contains but the trial court issued an Order
ruling that these are admissible in evidence.33

Built into the Constitution are guarantees on the freedom of every individual against
unreasonable searches and seizures by providing in Article III, Section 2, the following:

The right of the people to be secure in their persons, houses, papers, and effects
against unreasonable searches and seizures of whatever nature and for any
purpose shall be inviolable, and no search warrant or warrant of arrest shall issue
except upon probable cause to be determined personally by the judge after
examination under oath or affirmation of the complainant and the witness he may
produce, and particularly describing the place to be searched and the persons or
things to be seized.

Furthermore, the Constitution, in conformity with the doctrine laid down in Stonehill v.
Diokno, 34 declares inadmissible, any evidence obtained in violation of the freedom from
unreasonable searches and seizures.35

While a valid search warrant is generally necessary before a search and seizure may be
effected, exceptions to this rule are recognized. Thus, in Alvero v. Dizon,36 the Court stated that.
"[t]he most important exception to the necessity for a search warrant is the right of search and
seizure as an incident to a lawful arrest."37

Rule 126, Section 12 of the Rules of Court expressly authorizes a warrantless search and
seizure incident to a lawful arrest, thus:

Sec. 12. Search incident to lawful arrest. — A person lawfully arrested may be
searched for dangerous weapons or anything which may be used as proof of the
commission of an offense, without a search warrant.

There is no doubt that the warrantless search incidental to a lawful arrest authorizes the
arresting officer to make a search upon the person of the person arrested. As early as 1909, the
Court has ruled that "[a]n officer making an arrest may take from the person arrested any money
or property found upon his person which was used in the commission of the crime or was the
fruit of the crime or which might furnish the prisoner with the means of committing
violence or of escaping, or which may be used as evidence in the trial of the cause . . .
"38 Hence, in a buy-bust operation conducted to entrap a drug-pusher, the law enforcement
agents may seize the marked money found on the person
of the pusher immediately after the arrest even without arrest and search warrants. 39

In the case at bar, the NARCOM agents searched the person of the appellant after arresting him
in his house but found nothing. They then searched the entire house and, in the kitchen, found
and seized a plastic bag hanging in a corner.

The warrantless search and seizure, as an incident to a suspect's lawful arrest, may extend
beyond the person of the one arrested to include the premises or surroundings under his
immediate control.40 Objects in the "plain view" of an officer who has the right to be in the
position to have that view are subject to seizure and may be presented as evidence.41
In Ker v. California42 police officers, without securing a search warrant but having information
that the defendant husband was selling marijuana from his apartment, obtained from the
building manager a passkey to defendants' apartment, and entered it. There they found the
defendant husband in the living room. The defendant wife emerged from the kitchen, and one of
the officers, after identifying himself, observed through the open doorway of the kitchen, a small
scale atop the kitchen sink, upon which lay a brick-shaped package containing green leafy
substance which he recognized as marijuana. The package of marijuana was used as evidence
in prosecuting defendants for violation of the Narcotic Law. The admissibility of the package was
challenged before the U.S. Supreme Court, which held, after observing that it was not
unreasonable for the officer to walk to the doorway of the adjacent kitchen on seeing the
defendant wife emerge therefrom, that "the discovery of the brick of marijuana did not constitute
a search, since the officer merely saw what was placed before him in full view.43 The U.S.
Supreme Court ruled that the warrantless seizure of the marijuana was legal on the basis of the
"plain view" doctrine and upheld the admissibility of the seized drugs as part of the prosecution's
evidence. 44

The "plain view" doctrine may not, however, be used to launch unbridled searches and
indiscriminate seizures nor to extend a general exploratory search made solely to find evidence
of defendant's guilt. The "plain view" doctrine is usually applied where a police officer is not
searching for evidence against the accused, but nonetheless inadvertently comes across an
incriminating object.45 Furthermore, the U.S. Supreme Court stated the following limitations on
the application of the doctrine:

What the "plain view" cases have in common is that the police officer in each of them had a
prior justification for an intrusion in the course of which he came inadvertently across a piece of
evidence incriminating the accused. The doctrine serves to supplement the prior justification —
whether it be a warrant for another object, hot pursuit, search incident to lawful arrest, or some
other legitimate reason for being present unconnected with a search directed against the
accused — and permits the warrantless seizure. Of course, the extension of the original
justification is legitimate only where it is immediately apparent to the police that they have
evidence before them; the "plain view" doctrine may not be used to extend a general exploratory
search from one object to another until something incriminating at last emerges.46

It has also been suggested that even if an object is observed in "plain view," the "plain view"
doctrine will not justify the seizure of the object where the incriminating nature of the object is
not apparent from the "plain view" of the object.47 Stated differently, it must be immediately
apparent to the police that the items that they observe may be evidence of a crime, contraband,
or otherwise subject to seizure.

In the instant case, the appellant was arrested and his person searched in the living room.
Failing to retrieve the marked money which they hoped to find, the NARCOM agents searched
the whole house and found the plastic bag in the kitchen. The plastic bag was, therefore, not
within their "plain view" when they arrested the appellant as to justify its seizure. The NARCOM
agents had to move from one portion of the house to another before they sighted the plastic
bag. Unlike Ker vs. California, where the police officer had reason to walk to the doorway of the
adjacent kitchen and from which position he saw the marijuana, the NARCOM agents in this
case went from room to room with the obvious intention of fishing for more evidence.

Moreover, when the NARCOM agents saw the plastic bag hanging in one corner of the kitchen,
they had no clue as to its contents. They had to ask the appellant what the bag contained.
When the appellant refused to respond, they opened it and found the marijuana. Unlike Ker v.
California, where the marijuana was visible to the police officer's eyes, the NARCOM agents in
this case could not have discovered the inculpatory nature of the contents of the bag had they
not forcibly opened it. Even assuming then, that the NARCOM agents inadvertently came
across the plastic bag because it was within their "plain view," what may be said to be the object
in their "plain view" was just the plastic bag and not the marijuana. The incriminating nature of
the contents of the plastic bag was not immediately apparent from the "plain view" of said
object. It cannot be claimed that the plastic bag clearly betrayed its contents, whether by its
distinctive configuration, its transprarency, or otherwise, that its contents are obvious to an
observer.48

We, therefore, hold that under the circumstances of the case, the "plain view" doctrine does not
apply and the marijuana contained in the plastic bag was seized illegally and cannot be
presented in evidence pursuant to Article III, Section 3(2) of the Constitution.

The exclusion of this particular evidence does not, however, diminish, in any way, the damaging
effect of the other pieces of evidence presented by the prosecution to prove that the appellant
sold marijuana, in violation of Article II, Section 4 of the Dangerous Drugs Act of 1972. We hold
that by virtue of the testimonies of Sgt. Ani and T/Sgt. Belarga and the two wrappings of
marijuana sold by the appellant to Sgt. Ani, among other pieces of evidence, the guilt of the
appellant of the crime charged has been proved beyond reasonable doubt.

WHEREFORE, the appeal is DISMISSED and the judgment of the Regional Trial Court
AFFIRMED.

SO ORDERED.

G.R. No. 145176 March 30, 2004

PEOPLE OF THE PHILIPPINES, appellee,


vs.
SANTIAGO PERALTA y POLIDARIO (at large), ARMANDO DATUIN JR. y GRANADOS (at
large), ULYSSES GARCIA y TUPAS, MIGUELITO DE LEON y LUCIANO, LIBRANDO
FLORES y CRUZ and ANTONIO LOYOLA y SALISI, accused,
ULYSSES GARCIA y TUPAS, MIGUELITO DE LEON y LUCIANO, LIBRANDO FLORES y
CRUZ and ANTONIO LOYOLA y SALISI, appellants.

DECISION

PANGANIBAN, J.:

The right of the accused to counsel demands effective, vigilant and independent representation.
The lawyer’s role cannot be reduced to being that of a mere witness to the signing of an extra-
judicial confession.

The Case

Before the Court is an appeal from the August 21, 2000 Decision1 of the Regional Trial Court
(RTC) of Manila (Branch 18) in Criminal Case No. 92-112322. Appellants Ulysses Garcia y
Tupas, Miguelito de Leon y Luciano, Librando Flores y Cruz and Antonio Loyola y Salisi, as well
as their co-accused -- Santiago Peralta y Polidario and Armando Datuin Jr. y Granados -- were
convicted therein of qualified theft. The dispositive portion of the Decision reads:

"WHEREFORE, the accused, Santiago Peralta y Polidario, Armando Datuin, Jr. y Granados,
Ulysses Garcia y Tupas, Miguelito De Leon y Luciano, Librando Flores y Cruz and Antonio
Loyola y Salisi, are hereby convicted of the crime of qualified theft of P194,190.00 and
sentenced to suffer the penalty of reclusion perpetua with all the accessory penalties provided
by law, and to pay the costs. Moreover, all the accused are ordered to pay the Central Bank of
the Philippines, now Bangko Sentral ng Pilipinas, actual damages in the sum of P194,190.00
with interest thereon at the legal rate from the date of the filing of this action, November 9, 1992,
until fully paid."2

In an Information dated November 9, 1992,3 appellants and their co-accused were charged as
follows:

"That sometime in the year 1990 and including November 4, 1992, in the City of Manila,
Philippines, the said accused, conspiring and confederating with others whose true names,
identities and present whereabouts are still unknown and helping one another, did then and
there wilfully, unlawfully and feloniously, with intent to gain and without the knowledge and
consent of the owner thereof, take, steal and carry away punctured currency notes due for
shredding in the total amount of P194,190.00, belonging to the Central Bank of the Philippines
as represented by Pedro Labita y Cabriga, to the damage and prejudice of the latter in the
aforesaid sum of P194,190.00 Philippine currency;

"That said accused Santiago Peralta y Polidario, Armando Datuin, Jr. y Granados, Ulysses
Garcia y Tupas, Miguelito de Leon y Luciano and Antonio Loyola y Salisi committed said
offense with grave abuse of confidence they being at the time employed as Currency
Reviewers, Driver, Currency Assistant I and Money Counter of the offended party and as such
they had free access to the property stolen."4

Garcia was arrested on November 4, 1992; and his co-accused, on November 9, 1992.
Appellants, however, obtained two Release Orders from RTC Vice Executive Judge Corona
Ibay-Somera on November 9 and 10, 1992, upon their filing of a cash bond to secure their
appearance whenever required by the trial court.5

During their arraignment on May 4, 1993, appellants, assisted by their respective counsels,
pleaded not guilty.6 On September 30, 1998, the trial court declared that Datuin Jr. and Peralta
were at large, because they had failed to appear in court despite notice.7

After trial in due course, they were all found guilty and convicted of qualified theft in the
appealed Decision.

The Facts
Version of the Prosecution

The Office of the Solicitor General (OSG) presents the prosecution’s version of the facts as
follows:

"About 10:00 o’clock in the morning of November 4, 1992, Pedro Labita of Central Bank
of the Philippines (CBP) [now Bangko Sentral ng Pilipinas (BSP)] went to the Theft and
Robbery Section of Western Police District Command (WPDC), and filed a complaint for
Qualified Theft against Santiago Peralta, Armando Datuin, Jr., Ulysses Garcia, Miguelito
de Leon, Librando Flores and Antonio S. Loyola.

"Pedro Labita submitted to SPO4 Cielito Coronel, the investigating officer at WPDC,
punctured currency notes in P100.00 and P500.00 bills with a face value of
Php194,190.00. Said notes were allegedly recovered by the BSP Cash Department
during its cash counting of punctured currency bills submitted by different banks to the
latter. The punctured bills were rejected by the BSP money counter machine and were
later submitted to the investigation staff of the BSP Cash Department. As a result of the
investigation, it was determined that said rejected currency bills were actually punctured
notes already due for shredding. These currency bills were punctured because they
were no longer intended for circulation. Before these notes could be shredded, they
were stolen from the BSP by the above-named accused.

"On the basis of the complaint filed by Pedro Labita, Ulysses Garcia was apprehended in
front of Golden Gate Subdivision, Las Piñas City, while he was waiting for a passenger
bus on his way to the BSP. Garcia was brought to the police station for investigation.

"On November 4, 5 and 6, 1992, while in the custody of the police officers, Garcia gave
three separate statements admitting his guilt and participation in the crime charged. He
also identified the other named accused as his cohorts and accomplices and narrated
the participation of each and everyone of them.

"On the basis of Garcia’s sworn statements, the other named accused were invited for
questioning at the police station and were subsequently charged with qualified theft
together with Garcia."8 (Citations omitted)

Version of the Defense

The defense states its version of the facts in the following manner:

"Accused-appellant Garcia served as a driver of the armored car of the Central Bank
from 1978 to 1994.

"On November 4, 1992, between 7:00 a.m. and 8:00 a.m., a man who had identified
himself as a police officer arrested accused-appellant Garcia while waiting for a
passenger bus in front of the Golden Gate Subdivision, Las Piñas City. He was arrested
without any warrant for his arrest. The police officer who had arrested accused-appellant
Garcia dragged the latter across the street and forced him to ride x x x a car.

"While inside the car, he was blindfolded, his hands were handcuffed behind his back,
and he was made to bend with his chest touching his knees. Somebody from behind hit
him and he heard some of the occupants of the car say that he would be salvaged if he
would not tell the truth. When the occupants of the car mentioned perforated notes, he
told them that he does not know anything about those notes.

"After the car had stopped, he was dragged out of the car and x x x up and down x x x
the stairs. While being dragged out of the car, he felt somebody frisk his pocket.
"At a safe house, somebody mentioned to him the names of his co-accused and he told
them that he does not know his co-accused x x x. Whenever he would deny knowing his
co-accused, somebody would box him on his chest. Somebody poured water on
accused-appellant Garcia’s nose while lying on the bench. He was able to spit out the
water that had been poured on his nose [at first], but somebody covered his mouth. As a
result, he could not breath[e].

"When accused-appellant Garcia realized that he could not bear the torture anymore, he
decided to cooperate with the police, and they stopped the water pouring and allowed
him to sit down.

"Accused-appellant Garcia heard people talking and he heard somebody utter, ‘may
nakikinig.‘ Suddenly his two ears were hit with open palm[s] x x x. As he was being
brought down, he felt somebody return his personal belongings to his pocket. Accused-
appellant Garcia’s personal belongings consisted of [his] driver’s license, important
papers and coin purse.

"He was forced to ride x x x the car still with blindfold. His blindfold and handcuffs were
removed when he was at the office of police officer Dante Dimagmaliw at the Western
Police District, U.N. Avenue, Manila.

"SPO4 Cielito Coronel asked accused-appellant Garcia about the latter’s name, age and
address. The arrival of Mr. Pedro Labita of the Cash Department, Central Bank of the
Philippines, interrupted the interview, and Mr. Labita instructed SPO4 Coronel to get
accused-appellant Garcia’s wallet and examine the contents thereof. SPO4 Coronel
supposedly found three pieces of P100 perforated bill in accused-appellant Garcia’s
wallet and the former insisted that they recovered the said perforated notes from
accused-appellant’s wallet. SPO4 Coronel took down the statement of Mr. Labita.

"It was actually Mr. Labita, and not accused-appellant Garcia, who gave the answers
appearing in accused-appellant Garcia’s alleged three sworn statements dated
November 4, 1992, November 5, 1992 and x x x November 6, 1992.

"At or about 6:00 p.m. on November 5, 1992, accused-appellant Garcia was brought to
the cell of the Theft and Robbery Section of the WPD. At or about 8:00 p.m., he was
brought to the office of Col. Alladin Dimagmaliw where his co-accused were also inside.
He did not identify his co-accused, but he merely placed his hands on the shoulders of
each of his co-accused, upon being requested, and Mr. Labita took x x x pictures while
he was doing the said act.

"Accused-appellant Garcia came to know Atty. Francisco Sanchez of the Public


Attorney’s Office on November 4, 1992, at the office of police officer Dante Dimagmaliw,
when SPO4 Coronel introduced Atty. Sanchez to accused-appellant Garcia and told him
that Atty. Sanchez would be his lawyer. However, accused-appellant Garcia did not
agree to have Atty. Sanchez to be his lawyer. Atty. Sanchez left after talking to SPO4
Coronel, and accused-appellant Garcia had not met Atty. Sanchez anymore since then.
He was not present when Atty. Sanchez allegedly signed x x x the alleged three (3)
sworn statements.
"During the hearing of the case on April 6, 2000, Atty. Sanchez manifested in open court
that he did not assist accused-appellant Garcia when the police investigated accused-
appellant Garcia, and that he signed x x x the three (3) sworn statements only as a
witness thereto.

"Accused-appellant Garcia signed the alleged three sworn statements due to SPO4
Coronel’s warning that if he would not do so, he would again be tortured by water cure.

"SPO[4] Coronel caused the arrest without any warrant of accused appellants De Leon,
Loyola, [Flores] on the basis of the complaint of Mr. Pedro Labita, and which arrest was
effected on November 5, 1992, by SPO1 Alfredo Silva and SPO1 Redelico.

"SPO4 Coronel, in his letter dated November 6, 1992, forwarded the case to the Duty
Inquest Prosecutor assigned at the WPDC Headquarters."9 (Citations omitted)

Ruling of the Trial Court

The trial court found that all the accused used to work for the BSP. Garcia was a driver
assigned to the Security and Transport Department; while Peralta, Datuin Jr., De Leon, Flores
and Loyola were laborers assigned to the Currency Retirement Division. Their main task was to
haul perforated currency notes from the currency retirement vault to the basement of the BSP
building for shredding.

On several occasions, during the period 1990-1992, they handed to Garcia perforated currency
notes placed in a coin sack that he, in turn, loaded in an armored escort van and delivered to
someone waiting outside the premises of the building. The trial court held that the coordinated
acts of all the accused unerringly led to the conclusion that they had conspired to pilfer the
perforated currency notes belonging to the BSP.

The RTC rejected the disclaimer by Garcia of his own confessions, as such disclaimer was "an
eleventh hour concoction to exculpate himself and his co-accused." The trial court found his
allegations of torture and coerced confessions unsupported by evidence. Moreover, it held that
the recovery of three pieces of perforated P100 bills from Garcia’s wallet and the flight of Peralta
and Datuin Jr. were indicative of the guilt of the accused.

Hence, this appeal.10

Issues

In his Brief, Garcia raises the following issues:

"1

The trial court erred in admitting in evidence the alleged three Sworn Statements of Accused-
appellant Garcia and the alleged three pieces of P100 perforated notes

"2

The trial court erred in finding the accused-appellant guilty of qualified theft."11
In their joint Brief, De Leon, Loyola and Flores interpose this additional assignment of errors:

"1

The trial court erred in admitting in evidence the alleged three sworn statements of Accused
Ulysses Garcia (Exhibits ‘I’, ‘J’ and ‘K’) and the alleged three pieces of P100 perforated notes
(Exhibits ‘N’ to ‘N-2’) over the objections of the accused-appellants.

"2

The trial court erred in denying the demurrer to evidence of Accused-appellants De Leon,
Loyola and Flores;

"3

The trial court erred in denying the Motion for Reconsideration of the Order denying the
demurrer to evidence;

"4

The trial court erred when it failed to consider the evidence adduced by the accused-appellants,
consisting of exhibits ‘1’, ‘2’ to ‘2-B’, ‘3’ and ‘4’ and the testimony of their witness, State Auditor
Esmeralda Elli;

"5

The trial court erred in finding the accused-appellants guilty of qualified theft."12

Simplified, the issues are as follows: (1) the sufficiency of the evidence against appellants,
including the admissibility of Garcia’s confessions and of the three perforated P100 currency
notes; and (2) the propriety of the denial of their demurrer to evidence.

The Court’s Ruling

The appeal has merit.

First Issue:
Sufficiency of Evidence

The trial court convicted appellants mainly on the strength of the three confessions given by
Garcia and the three perforated P100 currency notes confiscated from him upon his arrest.
Appellants, however, contend that these pieces of evidence are inadmissible.

Extrajudicial Confessions

Appellants aver that the alleged three Sworn Statements of Garcia were obtained without the
assistance of counsel in violation of his rights under Article III, Section 12 (1) and (2) of the 1987
Constitution, which provides thus:
"Sec. 12. (1) Any person under investigation for the commission of an offense shall have the
right to be informed of his right to remain silent and to have competent and independent
counsel, preferably of his own choice. If the person cannot afford the services of counsel, he
must be provided with one. These rights cannot be waived except in writing and in the presence
of counsel.

"(2) No torture, force, violence, threat, intimidation, or any other means which vitiate the free will
shall be used against him. Secret detention places, solitary, incomunicado, or other similar
forms of detention are prohibited."

On the other hand, the OSG contends that counsel, Atty. Francisco Sanchez III of the Public
Attorney’s Office, duly assisted Garcia during the custodial investigation.

It is clear from a plain reading of the three extrajudicial confessions13 that Garcia was not
assisted by Atty. Sanchez. The signature of the latter on those documents was affixed after the
word "SAKSI." Moreover, he appeared in court and categorically testified that he had not
assisted Garcia when the latter was investigated by the police, and that the former had signed
the Sworn Statement only as a witness.14

The written confessions, however, were still admitted in evidence by the RTC on the ground that
Garcia had expressed in writing his willingness and readiness to give the Sworn Statements
without the assistance of counsel. The lower court’s action is manifest error.

The right to counsel has been written into our Constitution in order to prevent the use of duress
and other undue influence in extracting confessions from a suspect in a crime. The basic law
specifically requires that any waiver of this right must be made in writing and executed in the
presence of a counsel. In such case, counsel must not only ascertain that the confession is
voluntarily made and that the accused understands its nature and consequences, but also
advise and assist the accused continuously from the time the first question is asked by the
investigating officer until the signing of the confession.

Hence, the lawyer’s role cannot be reduced to being that of a mere witness to the signing of a
pre-prepared confession, even if it indicated compliance with the constitutional rights of the
accused.15 The accused is entitled to effective, vigilant and independent counsel.16

A waiver in writing, like that which the trial court relied upon in the present case, is not enough.
Without the assistance of a counsel, the waiver has no evidentiary relevance.17 The Constitution
states that "[a]ny confession or admission obtained in violation of [the aforecited Section 12]
shall be inadmissible in evidence x x x." Hence, the trial court was in error when it admitted in
evidence the uncounseled confessions of Garcia and convicted appellants on the basis thereof.
The question of whether he was tortured becomes moot.

Perforated Currency Notes

Appellants contend that the three P100 perforated currency notes (Exhibits "N" to "N-2")
allegedly confiscated from Garcia after his arrest were "fruits of the poisonous tree" and, hence,
inadmissible in evidence.
The solicitor general evades the issue and argues, instead, that appellants waived the illegality
of their arrest when they entered a plea. He further contends that the exclusion from the
evidence of the three punctured currency bills would not alter the findings of the trial court.

The police arrested Garcia without a warrant, while he had merely been waiting for a passenger
bus after being pointed out by the Cash Department personnel of the BSP. At the time of his
arrest, he had not committed, was not committing, and was not about to commit any crime.
Neither was he acting in a manner that would engender a reasonable ground to suspect that he
was committing a crime. None of the circumstances justifying an arrest without a warrant under
Section 5 of Rule 113 of the Rules of Court was present.

Hence, Garcia was not lawfully arrested. Nonetheless, not having raised the matter before
entering his plea, he is deemed to have waived the illegality of his arrest. Note, however, that
this waiver is limited to the arrest. It does not extend to the search made as an incident thereto
or to the subsequent seizure of evidence allegedly found during the search.

The Constitution proscribes unreasonable searches and seizures18 of whatever nature. Without
a judicial warrant, these are allowed only under the following exceptional circumstances: (1) a
search incident to a lawful arrest, (2) seizure of evidence in plain view, (3) search of a moving
motor vehicle, (4) customs search, (5) stop and frisk situations, and (6) consented search.19

Where the arrest was incipiently illegal, it follows that the subsequent search was similarly
illegal.20 Any evidence obtained in violation of the constitutional provision is legally inadmissible
in evidence under the exclusionary rule.21 In the present case, the perforated P100 currency
notes were obtained as a result of a search made without a warrant subsequent to an unlawful
arrest; hence, they are inadmissible in evidence.

Moreover, untenable is the solicitor general’s argument that Appellants De Leon, Flores and
Loyola waived the illegality of the arrest and seizure when, without raising objections thereto,
they entered a plea of guilty. It was Garcia who was unlawfully arrested and searched, not the
aforementioned three appellants. The legality of an arrest can be contested only by the party
whose rights have been impaired thereby. Objection to an unlawful search and seizure is purely
personal, and third parties cannot avail themselves of it.22

Indeed, the prosecution sufficiently proved the theft of the perforated currency notes for
retirement. It failed, however, to present sufficient admissible evidence pointing to appellants as
the authors of the crime.

The evidence presented by the prosecution shows that there were other people who had similar
access to the shredding machine area and the currency retirement vault.23 Appellants were
pinpointed by Labita because of an anonymous phone call informing his superior of the people
allegedly behind the theft; and of the unexplained increase in their spending, which was
incompatible with their income. Labita, however, did not submit sufficient evidence to support
his allegation.

Without the extrajudicial confession and the perforated currency notes, the remaining evidence
would be utterly inadequate to overturn the constitutional presumption of innocence.

Second Issue:
Demurrer to Evidence
Appellants contend that the trial court seriously erred when it denied the demurrer to evidence
filed by Appellants Loyola, De Leon and Flores. Not one of the documents offered by the
prosecution and admitted in evidence by the RTC established the alleged qualified theft of
perforated notes, and not one of the pieces of evidence showed appellants’ participation in the
commission of the crime.

On the exercise of sound judicial discretion rests the trial judge’s determination of the sufficiency
or the insufficiency of the evidence presented by the prosecution to establish a prima facie case
against the accused. Unless there is a grave abuse of discretion amounting to lack of
jurisdiction, the trial court’s denial of a motion to dismiss may not be disturbed.24

As discussed earlier, the inadmissibility of the confessions of Garcia did not become apparent
until after Atty. Francisco had testified in court. Even if the confiscated perforated notes from the
person of the former were held to be inadmissible, the confessions would still have constituted
prima facie evidence of the guilt of appellants. On that basis, the trial court did not abuse its
discretion in denying their demurrer to evidence.

WHEREFORE, the assailed Decision is REVERSED and SET ASIDE. Appellants are hereby
ACQUITTED and ordered immediately RELEASED, unless they are being detained for any
other lawful cause. The director of the Bureau of Corrections is hereby directed to submit his
report on the release of the appellant or the reason for his continued detention within five (5)
days from notice of this Decision. No costs.

SO ORDERED.

Davide Jr., CJ.,(Chairman) Panganiban, Ynares-Santiago, Carpio, and Azcuna, JJ concur.

CERTIFICATION

Pursuant to Section 13, Article VIII of the Constitution, it is hereby certified that the conclusions
in the above Decision had been reached in consultation before the case was assigned to the
writer of the opinion of the Court’s Division.

HILARIO G. DAVIDE, JR.


Chief Justice

However, this conclusion of the trial court is contradicted by the Solicitor General who quotes
parts from the confessions to show that Garcia was assisted by counsel. The statement dated
November 4, 199225 contains the following exchange:

"05. T: Meron ka bang abogado na nais na gamitin sa pagsisiyasat na ito upang siyang
tumulong sa iyo?

S: Wala pa ho sa ngayon pero si Attorney na lang pansamantala (Referring to Atty.


Francisco Sanchez III)."
It was also made to appear in the November 6, 1992 statement that Garcia was assisted by
Atty. Francisco:26

"03. T: Meron ka bang abogado na napipisil upang siyang tumulong sa iyo sa


pagsisiyasat na ito?

S: Sa ngayon po ay maski na si Atty. Sanchez na lang pansamantala."

The assailed Decision is a thin five-page conviction of the six accused. The first three pages
summarize the testimonies of the witnesses for both sides while the last two pages contained
the finding and ruling of the trial court. In one paragraph, the trial court refused to believe
Garcia’s recantation because 1) he repudiated them only five years after he executed them; 2)
there was no evidence substantiating Garcia’s allegation that he was tortured, not even an
administrative complaint against his alleged torturers; and 3) at the time of his arrest, the police
recovered from his wallet three pieces of P100 perforated bills. Then the trial court made short
shrift of the denials of Garcia’s co-appellants, and held that the flight of Peralta and Datuin Jr.
shored up the evidence against them all.

G.R. No. 83988 September 29, 1989

RICARDO C. VALMONTE AND UNION OF LAWYERS AND ADVOCATES FOR PEOPLE'S


RIGHTS (ULAP), petitioners,
vs.
GEN. RENATO DE VILLA AND NATIONAL CAPITAL REGION DISTRICT
COMMAND, respondents.

Ricardo C. Valmonte for himself and his co-petitioners.

PADILLA, J.:

This is a petition for prohibition with preliminary injunction and/or temporary restraining order,
seeking the declaration of checkpoints in Valenzuela, Metro Manila or elsewhere, as
unconstitutional and the dismantling and banning of the same or, in the alternative, to direct the
respondents to formulate guidelines in the implementation of checkpoints, for the protection of
the people.

Petitioner Ricardo C. Valmonte sues in his capacity as citizen of the Republic, taxpayer,
member of the Integrated Bar of the Philippines (IBP), and resident of Valenzuela, Metro Manila;
while petitioner Union of Lawyers and Advocates for People's Rights (ULAP) sues in its capacity
as an association whose members are all members of the IBP.

The factual background of the case is as follows:

On 20 January 1987, the National Capital Region District Command (NCRDC) was activated
pursuant to Letter of Instruction 02/87 of the Philippine General Headquarters, AFP, with the
mission of conducting security operations within its area of responsibility and peripheral areas,
for the purpose of establishing an effective territorial defense, maintaining peace and order, and
providing an atmosphere conducive to the social, economic and political development of the
National Capital Region.1 As part of its duty to maintain peace and order, the NCRDC installed
checkpoints in various parts of Valenzuela, Metro Manila.

Petitioners aver that, because of the installation of said checkpoints, the residents of Valenzuela
are worried of being harassed and of their safety being placed at the arbitrary, capricious and
whimsical disposition of the military manning the checkpoints, considering that their cars and
vehicles are being subjected to regular searches and check-ups, especially at night or at dawn,
without the benefit of a search warrant and/or court order. Their alleged fear for their safety
increased when, at dawn of 9 July 1988, Benjamin Parpon, a supply officer of the Municipality of
Valenzuela, Bulacan, was gunned down allegedly in cold blood by the members of the NCRDC
manning the checkpoint along McArthur Highway at Malinta, Valenzuela, for ignoring and/or
refusing to submit himself to the checkpoint and for continuing to speed off inspire of warning
shots fired in the air. Petitioner Valmonte also claims that, on several occasions, he had gone
thru these checkpoints where he was stopped and his car subjected to search/check-up without
a court order or search warrant.

Petitioners further contend that the said checkpoints give the respondents a blanket authority to
make searches and/or seizures without search warrant or court order in violation of the
Constitution; 2 and, instances have occurred where a citizen, while not killed, had been
harassed.

Petitioners' concern for their safety and apprehension at being harassed by the military manning
the checkpoints are not sufficient grounds to declare the checkpoints as per se illegal. No proof
has been presented before the Court to show that, in the course of their routine checks, the
military indeed committed specific violations of petitioners' right against unlawful search and
seizure or other rights.

In a case filed by the same petitioner organization, Union of Lawyers and Advocates for
People's Right (ULAP) vs. Integrated National Police, 3 it was held that individual petitioners
who do not allege that any of their rights were violated are not qualified to bring the action, as
real parties in interest.

The constitutional right against unreasonable searches and seizures is a personal right
invocable only by those whose rights have been infringed, 4 or threatened to be infringed. What
constitutes a reasonable or unreasonable search and seizure in any particular case is purely a
judicial question, determinable from a consideration of the circumstances involved. 5

Petitioner Valmonte's general allegation to the effect that he had been stopped and searched
without a search warrant by the military manning the checkpoints, without more, i.e., without
stating the details of the incidents which amount to a violation of his right against unlawful
search and seizure, is not sufficient to enable the Court to determine whether there was a
violation of Valmonte's right against unlawful search and seizure. Not all searches and seizures
are prohibited. Those which are reasonable are not forbidden. A reasonable search is not to be
determined by any fixed formula but is to be resolved according to the facts of each case. 6

Where, for example, the officer merely draws aside the curtain of a vacant vehicle which is
parked on the public fair grounds, 7 or simply looks into a vehicle, 8 or flashes a light
therein, 9 these do not constitute unreasonable search.
The setting up of the questioned checkpoints in Valenzuela (and probably in other areas) may
be considered as a security measure to enable the NCRDC to pursue its mission of establishing
effective territorial defense and maintaining peace and order for the benefit of the public.
Checkpoints may also be regarded as measures to thwart plots to destabilize the government,
in the interest of public security. In this connection, the Court may take judicial notice of the shift
to urban centers and their suburbs of the insurgency movement, so clearly reflected in the
increased killings in cities of police and military men by NPA "sparrow units," not to mention the
abundance of unlicensed firearms and the alarming rise in lawlessness and violence in such
urban centers, not all of which are reported in media, most likely brought about by deteriorating
economic conditions — which all sum up to what one can rightly consider, at the very least, as
abnormal times. Between the inherent right of the state to protect its existence and promote
public welfare and an individual's right against a warrantless search which is
however reasonably conducted, the former should prevail.

True, the manning of checkpoints by the military is susceptible of abuse by the men in uniform,
in the same manner that all governmental power is susceptible of abuse. But, at the cost of
occasional inconvenience, discomfort and even irritation to the citizen, the checkpoints during
these abnormal times, when conducted within reasonable limits, are part of the price we pay for
an orderly society and a peaceful community.

Finally, on 17 July 1988, military and police checkpoints in Metro Manila were temporarily lifted
and a review and refinement of the rules in the conduct of the police and military manning the
checkpoints was ordered by the National Capital Regional Command Chief and the Metropolitan
Police Director. 10

WHEREFORE, the petition is DISMISSED.

SO ORDERED.

Das könnte Ihnen auch gefallen